IASbaba's 60 Day Plan 2017 - Day 01 To Day 36
IASbaba's 60 Day Plan 2017 - Day 01 To Day 36
IASbaba's 60 Day Plan 2017 - Day 01 To Day 36
DAY 1
SUBJECT POLITY
TOPICS:
PRELIMS MCQs:
Provision Source
a) 1 Only
b) 2 Only
c) 2 and 3 Only
d) 1 and 3 Only
Institution of speaker and her role is taken from UK (British Constitution) . The other 2
statements are right.
Government of India Act 1935: Federal Scheme (also from constitution of Canada),
Office of Governor, Judiciary, Public Service Commission, Emergency Provisions,
Administrative Details,
Irish Constitution: Directive principles of state policy {Ireland itself borrowed it from
Spain}.
Australian Constitution: Freedom of trade and commerce within the country and
between the states, Power of the national legislature to make laws for implementing
treaties, even on matters outside normal Federal jurisdiction, Concurrent List.
Q.2) Which of the following is/are the features of Parliamentary form of the government?
1. Bicameralism
2. Leadership of the Chief Minister
3. Collective responsibility of the executive to the Parliament
4. Membership of the ministers in the legislature
a) 1 and 3 only
b) 2 and 4 only
c) 2, 3 and 4 only
d) 1, 2, 3 and 4 only
IASbabas 60 Day Plan Day 1 (Polity) 2017
Even though the Indian Parliamentary System is largely based on the British pattern, there
are some fundamental differences between the two. For example, the Indian Parliament is
not a sovereign body like the British Parliament. Further, the Indian State has an elected
head (republic) while the British State has hereditary head (monarchy).
In a parliamentary system whether in India or Britain, the role of the Prime Minister has
become so significant and crucial that the political scientists like to call it a Prime Ministerial
Government.
Q.3) After the great revolt of 1857, the British Government felt the necessity of seeking
the cooperation of the Indians in the administration of their country. In pursuance of this
policy of association, many Acts were enacted.
Which Act, introduced, for the first time, bicameralism and direct elections in the country?
a) Morley-Minto Reforms
b) Montagu-Chelmsford Reforms
c) Government of India Act, 1935
d) Government of India Act, 1892
On August 20, 1917, the British Government declared, for the first time, that its objective
was the gradual introduction of responsible government in India
The Government of India Act of 1919 was thus enacted, which came into force in 1921. This
Act is also known as Montagu-Chelmsford Reforms (Montagu was the Secretary of State for
India and Lord Chelmsford was the Viceroy of India).
1. It relaxed the central control over the provinces by demarcating and separating the
central and provincial subjects. However, the structure of government continued to be
centralised and unitary.
2. It further divided the provincial subjects into two partstransferred and reserved. The
transferred subjects were to be administered by the governor with the aid of ministers
responsible to the legislative Council. The reserved subjects, on the other hand, were to be
administered by the governor and his executive council without being responsible to the
legislative Council. This dual scheme of governance was known as dyarchy=
3. It introduced, for the first time, bicameralism and direct elections in the country. Thus,
the Indian Legislative Council was replaced by a bicameral legislature consisting of an Upper
House (Council of State) and a Lower House (Legislative Assembly). The majority of
members of both the Houses were chosen by direct election.
4. It required that the three of the six members of the Viceroys executive Council (other
than the commander-in-chief) were to be Indian.
5. It extended the principle of communal representation by providing separate electorates
for Sikhs, Indian Christians, Anglo-Indians and Europeans.
6. It granted franchise to a limited number of people on the basis of property, tax or
education.
7. It created a new office of the High Commissioner for India in London and transferred to
him some of the functions hitherto performed by the Secretary of State for India.
8. It provided for the establishment of a public service commission. Hence, a Central Public
Service Commission was set up in 1926 for recruiting civil servants.
9. It separated, for the first time, provincial budgets from the Central budget and
authorised the provincial legislatures to enact their budgets.
10. It provided for the appointment of a statutory commission to inquire into and report on
its working after ten years of its coming into force.
Think: Do read Government of India Act, 1935 and Government of India Act, 1909
Important from Exam perspective
Q.4) With regard to the Preamble of the Indian Constitution, consider the following
statements
1. In the event of a conflict between the Preamble and a specific provision of the
Constitution the former will prevail.
2. The Preamble is a part of the Constitution.
3. It was held by the Supreme Court as not a part of the Constitution in Berubari Union
Case.
a) 1 and 2 only
b) 2 and 3 only
c) 1 and 3 only
d) All of the above
The Preamble embodies the basic philosophy and fundamental valuespolitical, moral and
religious on which the Constitution is based.
The Preamble cannot override the specific provisions of the Constitution. In case of any
conflict between the two, the latter shall prevail.
In the Kesavananda Bharati case (1973), the Supreme Court rejected the earlier
opinion and held that Preamble is a part of the Constitution. It observed that the
Preamble is of extreme importance and the Constitution should be read and
interpreted in the light of the grand and noble vision expressed inthe Preamble. In
IASbabas 60 Day Plan Day 1 (Polity) 2017
the LIC of India case (1995) also, the Supreme Court again held that the Preamble is
an integral part of the Constitution.
Q.5) In India, who is constitutionally empowered to act as the final interpreter of the
Constitution?
a) President
b) Union Council of Ministers
c) Parliament
d) Supreme Court
Under Article 13, Judiciary has power to void any provisions of laws, actions of the executive
Q.6) Federalism is a key feature of the Indian constitution. Which of the following
statements concerning this is INCORRECT?
a) Both Central and state governments have been established by the constitution.
b) Both governments derive their power from the constitution.
c) Since the State governments are autonomous political units, the Central government
cannot enforce any order on them.
d) Indian citizens are governed by laws and rules made by both Central and state
governments at the same time.
In India, though the State governments are autonomous political units, the Central
government can enforce an order on them. For instance, in times of emergency.
Q.7) Which of the following is not a pillar of the Parliamentary democracy in India?
IASbabas 60 Day Plan Day 1 (Polity) 2017
Separation of Powers deals with ensuring equitable distribution of power between agencies
of state namely Execuitive, Judiciary and Legislature and can be integral to any kind of Polity
and not specifically to Parliamentary democracy
1. It was constituted under the scheme formulated by the Cabinet Mission Plan
2. The Assembly included all important personalities of India at that time, with the
exception of M A Jinnah
3. Its members were directly elected by the people of India
a) 1 Only
b) 1 and 2 Only
c) 3 Only
d) None of the above
The Constituent Assembly was constituted in November 1946 under the scheme formulated
by the Cabinet Mission Plan.
The Constituent Assembly was to be a partly elected and partly nominated body. Moreover,
the members were to be indirectly elected by the members of the provincial assemblies,
who themselves were elected on a limited franchise.
The elections to the Constituent Assembly (for 296 seats allotted to the British Indian
Provinces) were held in JulyAugust 1946. The Indian National Congress won 208 seats, the
Muslim League 73 seats, and the small groups and independents got the remaining 15 seats.
IASbabas 60 Day Plan Day 1 (Polity) 2017
However, the 93 seats allotted to the princely states were not filled as they decided to stay
away from the Constituent Assembly.
Although the Constituent Assembly was not directly elected by the people of India on the
basis of adult franchise, the Assembly comprised representatives of all sections of Indian
SocietyHindus, Muslims, Sikhs, Parsis, AngloIndians, Indian Christians, SCs, STs including
women of all these sections. The Assembly included all important personalities of India at
that time, with the exception of Mahatma Gandhi and M A Jinnah.
1. In India, both a citizen by birth as well as a naturalised citizen are eligible for the
office of President
2. Parliament has the power to make any provision with respect to the acquisition and
termination of citizenship and all other matters relating to citizenship
a) 1 Only
b) 2 Only
c) Both 1 and 2
d) Neither 1 nor 2
In India both a citizen by birth as well as a naturalised citizen are eligible for the office of
President while in USA, only a citizen by birth and not a naturalised citizen is eligible for the
office of President. Parliament shall have the power to make any provision with respect to
the acquisition and termination of citizenship and all other matters relating to citizenship
(Article 11).
Q.10) The federation as prescribed by Government of India Act, 1935 never came into
being because
a) The Muslim League did not support federalism; rather they wanted a strong Centre
to safeguard their interest.
b) It was optional for the Indian States to join the federation
IASbabas 60 Day Plan Day 1 (Polity) 2017
c) The administration of State subjects like agriculture, law and order was under
Governor-General.
d) None of the above
The federation as prescribed by Government of India Act, 1935 never came into being
because it was optional for the Indian States (or the Princely States) to join the federation.
And since the rulers of the Indian States never gave their consent, the Federation as
envisaged by the Act of 1935 never came into being.
Q.11) Suppose the Parliament of India wants to form a new State X, then which of the
following Article should be invoked?
a) Article 1
b) Article 2
c) Article 3
d) Article 4
Article 2 empowers the Parliament to admit into the Union of India, or establish, new states
on such terms and conditions as it thinks fit. Thus, Article 2 grants two powers to the
Parliament: (a) the power to admit into the Union of India new states; and (b) the power
to establish new states. The first refers to the admission of states which are already in
existence while the second refers to the establishment of states which were not in existence
before.
Notably, Article 2 relates to the admission or establishment of new states that are not part
of the Union of India.
Article 3, on the other hand, relates to the formation of or changes in the existing states of
the Union of India. In other words, Article 3 deals with the internal re-adjustment inter se of
the territories of the constituent states of the Union of India.
Assertion (A) : The Parliament can redraw the political map of India according to its will
Reason (R) : The territorial integrity of any state is not guaranteed by the Constitution
It is thus clear that the Constitution authorises the Parliament to form new states or alter
the areas, boundaries or names of the existing states without their consent. In other words,
the Parliament can redraw the political map of India according to its will. Hence, the
territorial integrity or continued existence of any state is not guaranteed by the
Constitution. Therefore, India is rightly described as an indestructible union of destructible
states. The Union government can destroy the states whereas the state governments
cannot destroy the Union. In USA, on the other hand, the territorial integrity or continued
existence of a state is guaranteed by the Constitution. The American Federal government
cannot form new states or alter the borders of existing states without the consent of the
states concerned. That is why the USA is described as an indestructible union of
indestructible states.
a) 1 Only
b) 2 Only
c) Both 1 and 2
d) Neither 1 nor 2
IASbabas 60 Day Plan Day 1 (Polity) 2017
The Preamble is neither a source of power to legislature nor a prohibition upon the
powers of legislature.
It is non-justiciable, that is, its provisions are not enforceable in courts of law.
Like any other part of the Constitution, the Preamble was also enacted by the
Constituent Assembly, but, after the rest of the Constitution was already enacted.
The reason for inserting the Preamble at the end was to ensure that it was in
conformity with the Constitution as adopted by the Constituent Assembly.
Q.14) Which of the following provisions of the Constitution reveal the Secular character of
the Indian State?
1. Article 30
2. Article 14
3. Article 29
4. Article 44
a) 1 and 2 Only
b) 1, 2 and 3
c) 1, 2, 3 and 4
d) 2 Only
The following provisions of the Constitution reveal the secular character of the Indian State:
(a) The term secular was added to the Preamble of the Indian Constitution by the 42 nd
Constitutional Amendment Act of 1976.
(b) The Preamble secures to all citizens of India liberty of belief, faith and worship.
(c) The State shall not deny to any person equality before the law or equal protection of the
laws (Article 14)
(d) The State shall not discriminate against any citizen on the ground of religion (Article 15).
(e) Equality of opportunity for all citizens in matters of public employment (Article 16).
IASbabas 60 Day Plan Day 1 (Polity) 2017
(f) All persons are equally entitled to freedom of conscience and the right to freely profess,
practice and propagate any religion (Article 25).
(g) Every religious denomination or any of its section shall have the right to manage its
religious affairs (Article 26).
(h) No person shall be compelled to pay any taxes for the promotion of a particular religion
(Article 27)
(i) No religious instruction shall be provided in any educational institution maintained by the
State (Article 28).
(j) Any section of the citizens shall have the right to conserve its distinct language, script or
culture (Article 29).
(k) All minorities shall have the right to establish and administer educational institutions of
their choice (Article 30).
(l) The State shall endeavour to secure for all the citizens a Uniform Civil Code (Article 44).
The Western concept of secularism connotes a complete separation between the religion
(the church) and the state (the politics). This negative concept of secularism is inapplicable
in the Indian situation where the society is multireligious. Hence, the Indian Constitution
embodies the positive concept of secularism, i.e., giving equal respect to all religions or
protecting all religions equally.
Moreover, the Constitution has also abolished the old system of communal representation,
that is, reservation of seats in the legislatures on the basis of religion. However, it provides
for the temporary reservation of seats for the scheduled castes and scheduled tribes to
ensure adequate representation to them.
Q.15) Allocation of seats in the Rajya Sabha to the States and the Union territories is
mentioned in
a) 3rd Schedule
b) 4th Schedule
c) 2nd Schedule
d) 8th Schedule
4th Schedule- Allocation of seats in the Rajya Sabha to the States and the Union territories
a) Only 1
b) Only 2
c) Both 1 and 2
d) Neither 1 nor 2
Explanation
Juang tribe belongs to the Munda ethnic group and the people of this tribe are found in
Keonjhar, Keonjhar, Dhenkanal Angul and Jaipur districts of Odisha. Their current estimated
population is 10,000. They live atop Nagda hills and their main occupation is basket
weaving. They speak the Juang language, which is accepted as a branch of the greater
Austroasiatic language family.
Odisha has 62 tribes, the highest number among all States and Union Territories in the
country, accounting for 22.85 per cent of the total population as per 2011 census. As many
as 13 of these tribes have been identified as Particularly Vulnerable Tribal Groups (PVTGs),
living in over 500 habitations of the State but mostly in hamlets inside the forested hills
across Odisha.
It was to bring the Juangs into the mainstream that the Juang Development Agency (JDA)
was established in 1975, with its headquarters in Gonasika Hills in Keonjhar district. Even
after four decades have elapsed, the agency has not been able to go beyond the Juangs of
Keonjhar, operating in 35 villages in six gram panchayats of Banspal block of Keonjhar.
IASbabas 60 Day Plan Day 1 (Polity) 2017
In News
Death of 19 Juang children in 3 months atop the Nagada hills in Odishas Jajpur
district has woken up the government to the problem of malnutrition in the tribe.
The tribe has been deprived of basic facilities such as drinking water, primary health
care, electricity, and primary education due to lack of road connectivity.
Source: http://www.thehindu.com/news/national/The-lost-tribe-of-
Odisha/article14553424.ece
Q.17) The Quantum Experiments at Space Scale/QUESS, the worlds first quantum
satellite, which will help it establish hack-proof communications between space and the
ground was recently launched by
China has launched the world's first satellite dedicated to testing the fundamentals of
quantum communication in space.
It is nicknamed 'Micius', after a 5th century BC Chinese philosopher and scientist who has
been credited as the first one in human history conducting optical experiments.
Source: http://indianexpress.com/article/technology/science/china-launches-first-
quantum-communication-satellite-2978052/
Q.18) Which of the following states is the first State in the country to draft its own
internal security act?
IASbabas 60 Day Plan Day 1 (Polity) 2017
a) Gujarat
b) Maharashtra
c) Andhra Pradesh
d) Chhattisgarh
Proposed setting up of Special Security Zones (SSZ) where movement of arms, explosives
and inflow of unaccounted funds will be prohibited
Defined Critical Infrastructure Sectors (CIS), and bringing nuclear reactors, dams, major
projects, coastal areas under its ambit, with an emphasis on maintaining law and order and
combating terrorism, insurgency, caste-related violence and communalism.
The act has also clearly defined the SSZs as having a separate police infrastructure. The
zones will also have a command and control system, and separate Standard Operating
Procedure (SoP) to be followed by the SSZ police.
Maharashtra is the first State in the country to draft its own internal security act.
The act has defined internal security as a situation posing threat to state within its borders,
either caused or provoked, prompted, or proxied by a hostile foreign power, perpetrated
even by such groups that use a failed, failing or weak hostile foreign power, causing
insurgency, terrorism or any other subversive act targeting innocent citizens, causing
animosity between groups, violence, destroy, or attempt to destroy public and private
establishments.
Source: http://www.thehindu.com/news/national/other-states/Maharashtra-to-have-
own-law-for-internal-security/article14580519.ece
a) Khajuraho Temple
b) Angkor Wat Temple
IASbabas 60 Day Plan Day 1 (Polity) 2017
It is one Hindu temple, which they dedicate towards Lord Shiva. It is one luminous example
of major heights, which Cholas achieves within Tamil architecture.
This temple remains as Indias largest and Indian architectures one of greatest glories. It is
also a part of UNESCO World Heritage Site as Great Living Chola Temples.
This great temple is Indias one of most prized site for its architecture. It stands in middle of
fortified walls added probably in sixteenth century.
There is one temple tower, which people call as Vimana. It is 216 feet or 66 meters
high. It is also one among tallest tower in whole world in its kind. Kumbam,
Chikharam or Kalasha, which is bulbous, or apex structure on top of temple, people
believe that it is made out of a single stone carving.
There is one statue of a sacred bull or Nandi at entrance of this temple measuring
about thirteen feet high and sixteen feet long. It is result of a single rock carving.
Entire structure of temple is made of granite, whose nearest source are near
Tiruchchirapalli, which is nearly 60 kms to Thanjavurs west, where temple is.
Built by Raja Raja Chola I in 1010 AD in Thanjavur, Brihadeeswarar Temple is also popular
by name of Big Temple and it turns 1000 years in 2010.
Source: http://www.thehindu.com/features/friday-review/dance/a-visit-to-thanjavur-
quartets-house/article9031793.ece
a) Only 1
IASbabas 60 Day Plan Day 1 (Polity) 2017
b) Only 2
c) Both 1 and 2
d) None of the above
It addresses the issue of Pakistans alleged support for cross-border terrorism in South Asia
It is one of the six principal organs of the United Nations and the only one in which
all member nations have equal representation.
Decisions on important questions, such as those on peace and security, admission of
new members and budgetary matters, require a two-thirds majority. Decisions on
other questions are by simple majority.
The General Assembly (GA) is the main deliberative, policymaking and representative
organ of the UN. Its powers are to oversee the budget of the United Nations, appoint
the non-permanent members to the Security Council, receive reports from other
parts of the United Nations and make recommendations in the form of General
Assembly Resolutions.
It has also established a wide number of subsidiary organs.
All 193 members states of the United Nations are members of the General
Assembly. Further, the United Nations General Assembly may grant observer status
to an international organization, entity or non-member state, which entitles the
entity to participate in the work of the United Nations General Assembly, though
with limitations.
IASbabas 60 Day Plan Day 1 (Polity) 2017
Source: http://www.thehindu.com/todays-paper/tp-national/india-plans-antiterror-
drive-at-nam/article9043899.ece
Q.21) Consider the following statements about Genetic Engineering Appraisal Committee
a) Only 1
b) Only 2
c) Both 1 and 2
d) Neither 1 nor 2
IASbabas 60 Day Plan Day 1 (Polity) 2017
For approval of activities involving large scale use of hazardous microorganisms and
recom-binants in research and industrial production from the environ-mental angle
The GEAC is also responsible for approval of proposals relating to release of genetically
engineered organisms and products into the environment including experimen-tal field trials
(Biosafety Research Level trial-I and II known as BRL-I and BRL-II)
Source: http://www.thehindu.com/news/national/GM-mustard-moves-closer-to-
approval/article14589131.ece
a) Only 2
b) 1 and 2
c) 2 and 3
d) None of the above
Xian - China
Ashgabat - Turkmenistan
Aleppo - Syria
IASbabas 60 Day Plan Day 2 (Polity) 2017
DAY 2
SUBJECT POLITY
TOPICS:
PRELIMS MCQs:
Q.1) Article 15 provides that the State shall not discriminate against any citizen on grounds
only of religion, race, caste, sex or place of birth.
a) The State shall discriminate on grounds religion, race, caste, sex or place of birth with
legislation passed by the Parliament.
b) Discrimination on other grounds is not prohibited.
c) It protects only against arbitrary action of the state.
d) None of the above.
Explanation:
Article 15 provides that the State shall not discriminate against any citizen on grounds only of
religion, race, caste, sex or place of birth. The two crucial words in this provision are
discrimination and only. The word discrimination means to make an adverse distinction
with regard to or to distinguish unfavorably from others. The use of the word only connotes
that discrimination on other grounds is not prohibited.
Article 15 says that no citizen shall be subjected to any disability, liability, restriction or
condition on grounds only of religion, race, caste, sex, or place of birth with regard to
IASbabas 60 Day Plan Day 2 (Polity) 2017
(a)Access to shops, public restaurants, hotels and places of public entertainment; or
(b) the use of wells, tanks, bathing ghats, road and places of public resort maintained wholly or
partly by State funds or dedicated to the use of general public.
This provision prohibits discrimination both by the State and private individuals, while the
former provision prohibits discrimination only by the State. There are three exceptions to this
general rule of non-discrimination.
Tribal habitation protected from interference of outsiders, from owning property, commercial
activities etc prohibited.
Q.2) Recently the Supreme Court has held reservation in promotion invalid. Consider the
following statements in this regard.
1. The State does not have unbridled powers to provide promotion in reservation to the
backward classes.
2. The State must demonstrate backwardness, inadequacy of representation and
maintenance of efficiency before providing reservation in promotions to Scheduled
Castes and Scheduled Tribes employees.
3. If the State does not justify this policy, it would violate Article 16.
a) 1 and 2 only
b) 2 only
c) 1 only
d) 1, 2 and 3
Explanation:
The State does not have unbridled powers to provide promotion in reservation to the
backward classes, the Supreme Court recently held that an exercise to determine
inadequacy of representation, backwardness and overall efficiency is a must before
granting promotion to Scheduled Castes and Scheduled Tribes employees.
IASbabas 60 Day Plan Day 2 (Polity) 2017
Article 16 provides for equality of opportunity for all citizens in matters of employment or
appointment to any office under the State. No citizen can be discriminated against or be
ineligible for any employment or office under the State on grounds of only religion, race,
caste, sex, descent, place of birth or residence.
The State can provide for reservation of appointments or posts in favor of any backward
class that is not adequately represented in the state services.
In order to nullify the ruling with regard to reservation in promotions, the 77th Amendment
Act was enacted in 1995. It added a new provision in Article 16 that empowers the State to
provide for reservation in promotions of any services under the State in favor of the SCs and
STs that are not adequately represented in the state services.
Again, the 85th Amendment Act of 2001 provides for consequential seniority in the case of
promotion by virtue of rule of reservation for the government servants belonging to the SCs
and STs with retrospective effect from June 1995.
Maintaining that the State does not have unbridled powers to provide promotion in
reservation to the backward classes, the Supreme Court held that an exercise to determine
inadequacy of representation, backwardness and overall efficiency is a must before
granting promotion to Scheduled Castes and Scheduled Tribes employees.
Why in news?
Karnataka quota law for promotion held invalid by the Supreme Court
http://indianexpress.com/article/india/karnataka-quota-law-for-promotion-invalid-says-
supreme-court-backward-class-reservation-4516818/
Q.3) Forcible Religious conversions violate which of the following Fundamental right?
a) Freedom of conscience
b) Right to profess
c) Right to practice
d) Right to propagate
a) 1 and 2 only
b) 1 only
c) 1, 2 and 3
d) 2 only
Explanation:
Article 17 abolishes untouchability and forbids its practice in any form. The enforcement of
any disability arising out of untouchability shall be an offence punishable in accordance with
law. In 1976, the Untouchability (Offences) Act, 1955 has been comprehensively amended
and renamed as the Protection of Civil Rights Act, 1955 to enlarge the scope and make
penal provisions more stringent.
The term untouchability has not been defined either in the Constitution or in the Act.
However, the Mysore High Court held that the subject matter of Article 17 is not
IASbabas 60 Day Plan Day 2 (Polity) 2017
untouchability in its literal or grammatical sense but the practice as it had developed
historically in the country. It refers to the social disabilities imposed on certain classes of
persons by reason of their birth in certain castes. It does not cover social boycott of a few
individuals or their exclusion from religious services, etc.
1. The principle of locus standi is not necessary in case of an order issuing writ of Habeas
corpus
2. Habeas corpus can be issued against both public and private individual.
a) 1 only
b) 2 only
c) Both 1 and 2
d) Neither 1 nor 2
Habeas Corpus
It is a Latin term which literally means to have the body of. It is an order issued by the
court to a person who has detained another person, to produce the body of the latter
before it. The court then examines the cause and legality of detention. It would set the
detained person free, if the detention is found to be illegal. Thus, this writ is a bulwark
of individual liberty against arbitrary detention.
Principle of locus standi means that the writ can be sought by any interested person
and not necessarily by the aggrieved person. Incase of Habeas corpus and Quo
Warranto, Locus standi is not necessary; whereas incase of Certiorari , Prohibition,
Mandamus - Locus Standi is mandatory
The writ of habeas corpus can be issued against both public authorities as well as
private individuals.
IASbabas 60 Day Plan Day 2 (Polity) 2017
It is not issued where the (a) detention is lawful, (b) the proceeding is for contempt of a
legislature or a court, (c) detention is by a competent court, and (d) detention is outside
the jurisdiction of the court.
Unlike the other four writs, this can be sought by any interested person and not
necessarily by the aggrieved person.
1. Article 29 grants protection only to linguistic minorities and not to religious minorities.
2. Under Article 30, all minorities shall have the right to establish and administer
educational institutions of their choice.
a) 1 only
b) 2 only
c) Both 1 and 2
d) Neither 1 nor 2
Q.7) With regard to Directive Principles of State Policy, consider following statements
IASbabas 60 Day Plan Day 2 (Polity) 2017
1. These principles have origin in the Spanish Constitution.
2. They are fundamental to the governance of the country
3. Dr. Ambedkar described them as Novel Features of Indian constitution.
a) 1 and 2 only
b) 1 and 3 only
c) 1, 2 and 3
d) 2 and 3 only
The Directive Principles of State Policy are enumerated in Part IV of the Constitution from
Articles 36 to 51. The framers of the Constitution borrowed this idea from the Irish
Constitution of 1937, which had copied it from the Spanish Constitution. Dr B R Ambedkar
described these principles as novel features of the Indian Constitution.
The Directive Principles along with the Fundamental Rights contain the philosophy of the
Constitution and is the soul of the Constitution.
The Directive Principles are non-justiciable in nature, that is, they are not legally
enforceable by the courts for their violation. Therefore, the government (Central, state and
local) cannot be compelled to implement them. Nevertheless, the Constitution (Article 37)
itself says that these principles are fundamental in the governance of the country and it
shall be the duty of the State to apply these principles in making laws.
Q.8) Which of the following does not belong to the Directive Principles of State Policy?
Q.9) Economic Justice the objective of Indian Constitution has been provided in
Preamble:
WE, THE PEOPLE OF INDIA, having solemnly resolved to constitute India into a SOVEREIGN,
SOCIALIST, SECULAR, DEMOCRATIC REPUBLIC and to secure to all its citizens:
FRATERNITY assuring the dignity of the individual and the unity and integrity of the Nation;
DPSP: Article 38: To promote welfare of people by securing a social order permeated by justice-
Social, economic and political- and to minimize the inequalities of income, status, facilities and
opportunities.
Source: Chapter 8 Directive Principles of State Policy & Chapter4 Preamble-Indian Polity by
M Laxmikanth
IASbabas 60 Day Plan Day 2 (Polity) 2017
Q.10) Consider the following statements
a) 1 and 2 Only
b) 2 only
c) 2 and 3 Only
d) 1 and 3 Only
Q.10) Solution(c)
Some of them are available only to the citizens while others are available to all persons
whether citizens, foreigners or legal persons like corporations or companies.
Most of them are available against the arbitrary action of the State, with a few exceptions
like those against the States action and against the action of private individuals. When the
rights that are available against the States action only are violated by the private
individuals, there are no constitutional remedies but only ordinary legal remedies. Eg.
Abolition of untouchability, punishable offence on individual who practices it.
They can be suspended during the operation of a National Emergency except the rights
guaranteed by Articles 20 and 21. Further, the six rights guaranteed by Article 19 can be
suspended only when emergency is declared on the grounds of war or external aggression
(i.e., external emergency) and not on the ground of armed rebellion (i.e., internal
emergency).
They are not sacrosanct or permanent. The Parliament can curtail or repeal them but only
by a constitutional amendment act and not by an ordinary act. Moreover, this can be done
without affecting the basic structure of the Constitution.
Article 33 empowers the Parliament to restrict or abrogate the fundamental rights of the
members of armed forces, para-military forces, police forces, intelligence agencies and
analogous forces. The objective of this provision is to ensure the proper discharge of their
duties and the maintenance of discipline among them.
IASbabas 60 Day Plan Day 2 (Polity) 2017
The power to make laws under Article 33 is conferred only on Parliament and not on state
legislatures. Any such law made by Parliament cannot be challenged in any court on the
ground of contravention of any of the fundamental rights.
These impose restrictions on their freedom of speech, right to form associations, right to be
members of trade unions or political associations, right to communicate with the press,
right to attend public meetings or demonstrations, etc.
The expressionmembers of the armed forces also covers such employees of the armed
forces as barbers, carpenters, mechanics, cooks, chowkidars, bootmakers, tailors who are
non-combatants. A parliamentary law enacted under Article 33 can also exclude the cour
martials (tribunals established under the military law) from the writ jurisdiction of the
Supreme Court and the high courts, so far as the enforcement of Fundamental Rights is
concerned.
Q.11) Which committee recommended for incorporation of Fundamental Duties into the
constitution?
In 1976, the Congress Party set up the SardarSwaran Singh Committee to make
recommendations about fundamental duties, the need and necessity of which was felt
during the operation of the internal emergency (19751977). The committee
recommended the inclusion of a separate chapter on fundamental duties in the
Constitution.
Justice Shah committee looked into misuse of power during internal emergency.
Justice Verma committee looked into existence of legal provisions to for the
implementation of some of the fundamental duties.
The States Reorganisation Commission (SRC) was a body constituted by the Central
Government of India in 1953 to recommend the reorganisation of state boundaries. In
IASbabas 60 Day Plan Day 2 (Polity) 2017
1955, after nearly 2 years of study, the Commission recommended that India's state
boundaries should be reorganised to form 16 states and 3 union territories.
States Reorganisation Commission consisted of Fazal Ali, K. M. Panikkar and H. N.
Kunzru. Some of its recommendations were implemented in the States Reorganisation
Act of 1956
a) 1 only
b) 2 only
c) Both 1 and 2
d) Neither 1 nor 2
Only some of the fundamental rights are applicable to foreigners except Article
15,16,19,29 and 30.
Fundamental duties are not applicable to foreigners only applicable to citizens of India.
Also they were not part of original constitution, added in 1976 through 42nd
constitutional amendment.
Though the rights and duties of the citizens are correlative and inseparable, the original
constitution contained only the fundamental rights and not the fundamental duties. In
other words, the framers of the Constitution did not feel it necessary to incorporate the
fundamental duties of the citizens in the Constitution. However, they incorporated the
duties of the State in the Constitution in the form of Directive Principles of State Polity.
Later in 1976, the fundamental duties of citizens were added in the Constitution. In
2002, one more Fundamental Duty was added.
a) 1 only
b) 2 only
c) Both 1 and 2
d) Neither 1 nor 2
Source: Chapter 7 Fundamental Rights & Chapter 3 Salient features of Indian Constitution-
Indian Polity by M. Laxmikanth
a) 1, 2 and 3
b) 1 and 3 only
c) 2, 3 and 4
d) 1 Only
to abide by the Constitution and respect its ideals and institutions, the National Flag and
the National Anthem;
to cherish and follow the noble ideals which inspired our national struggle for freedom;
to uphold and protect the sovereignty, unity and integrity of India;
to defend the country and render national service when called upon to do so;
to promote harmony and the spirit of common brotherhood amongst all the people of
India transcending religious, linguistic and regional or sectional diversities; to renounce
practices derogatory to the dignity of women;
to value and preserve the rich heritage of our composite culture;
to protect and improve the natural environment including forests, lakes, rivers and wild
life, and to have compassion for living creatures;
to develop the scientific temper, humanism and the spirit of inquiry and reform
To protect and improve the environment and to safeguard forests and wild life( Article
48 A)
To promote international peace and security and maintain honorable relations with
other nations( Article 51)
To raise the level of nutrition and the standard of living of people and to improve public
health (Article 47).
a) 1 and 2 only
b) 2 only
c) 1 only
d) 1, 2 and 3
IASbabas 60 Day Plan Day 2 (Polity) 2017
Fundamental duties derived from erstwhile USSR constitution; apart from India, Japan
has duties in its constitution.
US ConstitutionFundamental rights, independence of judiciary, judicial review,
impeachment of the president, removal of Supreme Court and high court judges and
post of vice-president.
Irish ConstitutionDirective Principles of State Policy, nomination of members to
RajyaSabha and method of election of president.
Weimar Constitution of GermanySuspension of Fundamental Rights during
Emergency
Source: Chapter 3, Salient features of the Constitution, Fundamental rights , Directive Principles
of State Policy, Fundamental Duties, Indian Polity- Laxmikanth
Q.16) Which of the following is often described as conscience of the Indian Constitution?
The Directive Principles along with the Fundamental Rights contain the philosophy of the
Constitution and is the soul of the Constitution. Granville Austin has described the Directive
Principles and the Fundamental Rights as the Conscience of the Constitution.
Often termed as the conscience of the Constitution, these principles were initially ignored by
the Supreme Court while interpreting various socio-economic rights. It was only after the
landmark verdict in the Keshavananda Bharatis case in 1973 that the top court said that
directive principles must be viewed as the guidelines by which the fundamental rights are
realised.
The Indian Constitution is founded on the bedrock of the balance between Part-III
(Fundamental Rights) and Part-IV (Directive Principles of State Policy). To give absolute primacy
IASbabas 60 Day Plan Day 2 (Polity) 2017
to one over the other is to disturb the harmony of the Constitution. This harmony and balance
is an essential feature of the basic structure of the Constitution, said Justice PN Bhagwati in
the 1980 Minerva Mills case.
http://www.hindustantimes.com/india/directive-principles-of-state-policy-conscience-of-the-
constitution/story-lB67mrmVaSSJhV5NSC1WbO.html
1. They are classified as protected species under Wildlife Protection Act 1972
2. It is found in the states of Karnataka, Kerala and Tamil Nadu and Andhra Pradesh
3. It is listed as Critically Endangered on The IUCN Red List of Threatened Species
a) Only 1
b) 1 and 2
c) 1 and 3
d) 2 and 3
The Lion-tailed Macaque, Macaca silenus, is listed as Endangered on The IUCN Red List of
Threatened Species TM. It is found only in the rainforests of the Western Ghats in southwestern
India, in the states of Karnataka, Kerala and Tamil Nadu. Its range is small and severely
fragmented, made up of 47 isolated subpopulations in seven different overall regions.
The total wild population is estimated to be less than 4,000 individuals. It is mainly an arboreal
primate, preferring the upper canopy of primary tropical evergreen rainforest, and also
monsoon forest in hilly country, ranging from near sea-level up to 1,800 m in the mountains. It
also occurs in disturbed forest, making use of human-planted fruit trees such as jack fruit, guava
and passion fruit. Habitat degradation, due to use of forest products and the expansion of
IASbabas 60 Day Plan Day 2 (Polity) 2017
agriculture, seems to the biggest threat to the conservation of Lion-tailed Macaques through
most of their range. In some places they are also illegally hunted.
The Lion-tailed Macaque occurs in a number of protected areas in the Western Ghats. About
one-quarter of the population occurs in forest on private land where populations can persist in
coffee and cardamom plantations.
Lion-tailed macaques are classified as protected species under Wildlife Protection Act 1972.
Source: http://www.thehindu.com/news/cities/chennai/One-more-lion-tailed-macaque-at-
zoo/article14544506.ece
a) A basket that is traditionally used by the people of Kashmir to keep themselves warm by
keeping burning charcoals in it
b) The fine art of using lentil paste mix to create exquisite designs
c) It is a pictorial narrative painted on a cloth-based scroll
d) It is an embroidery technique using beautiful designs of flowers on shawls, scarves, saris
etc
Goyna Bori
The fine art of using lentil paste mix to create exquisite designs, distinct in West Bengals
Midnapore.
With distinct historical basis dating back several centuries of the Rajbari and also referred by
Rabindranath Tagore in his works, Goyna Bori is believed to have originated in Midnapore.
Apart from being edible, Goyna Bori is actually more about unique pieces of fine art having
great appeal to the eye.
Source: http://indianexpress.com/article/education/iit-k-to-help-communities-get-gi-tags-
for-crafts2948219/
IASbabas 60 Day Plan Day 2 (Polity) 2017
Q.19) Which of the following statements is/are correct?
1. Coal has the maximum weight in the IIP among the eight core sectors
2. The IIP index is computed and published by the Central Statistical Organization (CSO) on
a fortnightly basis considering 2004-05 as base year
a) Only 1
b) Only 2
c) Both 1 and 2
d) Neither 1 nor 2
The Index of Industrial Production (IIP) is an index which shows the growth rates in different
industry groups of the economy in a stipulated period of time. The IIP index is computed and
published by the Central Statistical Organisation (CSO) on a monthly basis.
IIP is a composite indicator that measures the growth rate of industry groups classified under,
The eight Core Industries viz. Fertilizers, Electricity, Refinery Products, Natural Gas, Steel,
Cement, Crude Oil and coal is nearly 38% in the IIP.
So, on the basis of weightage the arrangement of six core industries in decreasing order is as
follows: Electricity> Steel >Refinery Products> Crude Oil >Coal>Cement >Natural Gas>
Fertilizers.
Source: http://www.thehindu.com/business/Industry/Power-cement-coal-drive-core-
sectors-up-5.2/article14545955.ece
Q.20) DRR Dhan 45 also known as IET23832 was in news recently. What is it?
IET 23832 is a biofortified semi-dwarf, medium duration (125 days) variety with non-lodging
plant type and long slender grains for irrigated conditions. It is the first high zinc rice variety
notified at national level with over all mean zinc content of 22.6ppm (24.0ppm in AP,KA&TN) in
polished rice, developed through conventional breeding without compromising yield using the
material from HarvestPlus. Based on high zinc content and yield performance over 5 t/ha, it is
released for the states of Tamil Nadu, Andhra Pradesh and Karnataka. It has good cooking
quality with desirable amylose content (20.7%). It is moderately resistant to blast, sheath rot
and rice tungro virus. The variety is a proof of concept for Biofortfication and can address the
hidden hunger or mineral malnutrition, thus targeting nutritional security of the nation.
Source: http://www.thehindu.com/sci-tech/science/Biofortification-Micronutrient-built-in-
grains/article14572744.ece
a) 1 and 2
b) 2 and 3
c) Only 2
d) All of the above
The Centre is mulling to introduce the vaccine under the universal immunisation programme,
but no rollout has been announced as yet.
Cervical cancer, mainly caused by Human Papillomavirus infection, is one of the leading cancers
in Indian women. Cervical cancer kills roughly 67,477 Indian women annually. The two HPV
vaccines currently available in India are bivalent (Cervarix, Rs 2190 per dose) and quadrivalent
(Gardasil, Rs 3000 per dose.) The Nonavalent, which is considered to be the most effective in
the West, is extremely expensive and currently not available in India as it is still not approved
by Drug Controller General of India (DCGI).
The currently available bivalent (protection against two types of HPV) and quadrivalent vaccines
(-----offer protection against four types of HPV) 70% of the carcinogenic HPV types, while the
nonavalent vaccine protects against 90% of them.
Of more than 100 HPV types, about 15 are found to be high risk. HPV types 16 and 18 are
responsible for 82%of cervical cancers in India.
WHO (World Health Organisation) recommends two doses of the vaccine preferably in the age
of 9-13 years.
Source: www.thehindu.com/sci-tech/health/medicine-and-research/Cervical-cancer-can-
possibly-be-eliminated/article14574446.ece
Q.22) Consider the following statements with respect to National Committee on Trade
Facilitation (NCTF)
a) Only 1
b) 1 and 2
c) 1 and 3
d) 2 and 3
IASbabas 60 Day Plan Day 2 (Polity) 2017
Consequent to Indias ratification of the WTO Agreement on Trade Facilitation (TFA) in April
2016, the National Committee on Trade Facilitation (NCTF) has been constituted. The
establishment of the Committee is part of the mandatory, institutional arrangement of the TFA.
Objective -
To have a national level body that will facilitate domestic co-ordination and
implementation of TFA provisions.
It will play the lead role in developing the pan-India road map for trade facilitation.
It will be instrumental in synergizing the various trade facilitation perspectives across
the country and will also focus on an outreach programme for sensitization of all
stakeholders about TFA.
Housed by - Its Secretariat will be housed within the Central Board of Excise and Customs
(CBEC), in the Directorate General of Export Promotion, New Delhi.
Source: http://www.thehindubusinessline.com/economy/national-committee-on-trade-
facilitation-set-up/article8981813.ece
a) Indian Ocean
b) Pacific Ocean
c) Atlantic Ocean
d) Arctic Ocean
Vanuatu is a South Pacific Ocean nation made up of roughly 80 islands that stretch 1,300
kilometers.
IASbabas 60 Day Plan Day 2 (Polity) 2017
Vanuatu includes 13 larger islands and about 70 smaller ones. Most are mountainous (volcanic
in origin) (some active), and covered in lush rain forests. Many are protected by coral reefs.
The islands are subject to devastating cyclones (hurricanes). The most destructive one ever (in
1987) damaged or ruined most of the local dwellings.
The local economy still revolves around agriculture and fishing, but tourism is a fast-growing
industry and liberal tax laws have made Vanuatu a popular offshore financial center.
Most of the outer islands maintain their traditional Melanesian lifestyle, the impressive city of
Port-Vila is crammed with colonial buildings, expats, duty-free shops, casinos, open-air markets,
upscale restaurants and tourists.
India has been ranked 77th on the World Risk Index, topped by Island state of Vanuatu. The
World Risk Report 2016, published by the UNU Institute for Environment and Human Security
(UNU-EHS) and Bundnis Entwicklung Hilft, analyses the role that infrastructure plays in shaping
a countrys disaster risk.
The Index, calculated by the University of Stuttgart, ranks 171 countries according to their risk
of becoming a victim of a disaster as a result of natural hazards such as floods, cyclones, or
earthquakes. The Island state of Vanuatu has been ranked no 1 on the index.
Source: http://www.thehindu.com/sci-tech/energy-and-environment/India-ranked-77-in-
disaster-risk-index-of-the-world/article14591228.ece
IASbabas 60 Day Plan Day 3 (Geography) 2017
DAY 3
SUBJECT GEOGRAPHY
TOPICS:
Indian Geography
The Himalayas- The Greater Himalayas (Himadri), Lesser Himalayas (Himachal), Sub-
Himalayas (Siwaliks)
Indian Desert
Islands
PRELIMS MCQs:
1. Naga Hills
2. Patkai Hills
3. Dafla Hills
4. Lushai Hills
a) 1, 2 and 3 only
b) 1, 2 and 4 only
c) 1 and 2 only
d) 2 and 4 only
The Purvachal comprises the Patkai hills, the Naga hills, Manipur hills and the Mizo hills.
IASbabas 60 Day Plan Day 3 (Geography) 2017
Purvachal or the Eastern hills and mountains are part of the Himalayan mountain system having
their general alignment from the north to the south direction.
Q.2) Consider the below statements with regard to Western Ghats and Eastern Ghats:
1. The mean height of the Western Ghats is more than that of Eastern Ghats.
2. The Eastern Ghats do not form a continuous chain like the Western Ghats.
3. The Eastern Ghats average width is less than that of Western Ghats.
IASbabas 60 Day Plan Day 3 (Geography) 2017
a) 1 and 2 only
b) 2 and 3 only
c) 1, 2 and 3
d) None
Statement (3) is incorrect as the Western Ghats average width is 50 to 80 km. But Eastern
Ghats width varying from 100 to 200 km.
Western Ghats is continuous and can be crossed through passes only. But Eastern Ghats has
been divided into several parts by large rivers.
Western Ghats average elevation is 900 to 1,100 meters above sea-level. But the average
elevation of Eastern Ghats is about 600 metres above sea level.
a) 3 only
b) 2 and 3 only
c) 1 and 2 only
d) None
All the characteristic features provided in the statements is about Bhangar, hence none of the
statements are correct with regard to Bhabar.
IASbabas 60 Day Plan Day 3 (Geography) 2017
Bhabar is a narrow belt ranging between 8-10 km parallel to the Shiwalik foothills at the break-
up of the slope. As a result of this, the streams and rivers coming from the mountains deposit
heavy materials of rocks and boulders, and at times, disappear in this zone. It comprises of
pebble- studded rocks in the shape of porous beds. Due to porosity of the rocks, the streams
disappear and flow underground. This region is not much suitable for agriculture.
South of the Bhabar is the Tarai belt and south of Tarai is a belt consisting of old and new
alluvial deposits known as the Bhangar and Khadar respectively.
Q.4) Which among the following is/are not the extensions of Peninsular plateau?
1. Rajmahal hills
2. Gir range
3. Karbi-Anglong
4. Cardamom hills
a) 3 only
b) 2 and 3 only
c) 1 and 2 only
d) None
All the given statements are extensions of peninsular plateau. Delhi ridge in the northwest,
(extension of Aravalis), the Rajmahal hills in the east, Gir range in the west and the Cardamom
hills in the south constitute the outer extent of the Peninsular plateau. However, an extension
of this is also seen in the northeast, in the form of Shillong and Karbi-Anglong plateau.
Q.5) Among the following riverine islands, choose the one which recently became India's first
island district?
Majuli, a riverine island in the Bramaputra river in Assam became India's first island district,
which also recently became the world's largest river island. Home to an estimated 160,000
people of different ethnic groups, the island is an assembly constituency reserved for scheduled
tribes.
Majuli replaced Brazil's Marajo to become the world's largest river island in the Guinness Book
of World Records.
Majuli is the nerve centre of neo-Vaishnavite culture and has been vying for World Heritage Site
status. It has been included in the tentative list by Unesco.
Q.6) Match List I with List II and select the correct answer using the code given below the
Lists:
List I List II
(Plant) (Natural vegetation type)
A. Walnut 1. Alpine
B. Birch 2. Himalayan moist
C. Shisham 3. Moist deciduous
D. Ebony 4. Tropical evergreen
Code:
ABCD
a) 2341
b) 2134
c) 1234
d) 1324
Q.7) Why Western Ghats in Karnataka receive more monsoon rainfall than Maharashtra and
Kerala?
a) 1 and 2 only
b) 1 and 3 only
c) 1, 2 and 3
d) None
Western Ghats in Karnataka receive more monsoon rainfall than Maharashtra and Kerala. There
are several reasons for this.
First, the mountain topography in Karnataka is broader than the narrow topography of the
Ghats in Maharashtra. Due to the greater width of the mountains, the rain bearing winds have
to necessarily travel a longer distance and have more time for the drops to coalesce and
precipitate as rainfall, resulting in higher rainfall. In contrast, the narrow width of the Ghats in
Maharashtra allows the rain-bearing wind to cross over to the leeward side rapidly before
precipitation can occur.
As for Kerala, the Ghats there are in the form of isolated mountains, where the rain-bearing
winds can easily cross over to the leeward side through the gaps in between without
precipitation occurring.
Second, the slope of the mountain has a direct bearing on the possibility of precipitation. This is
borne out by the Ghats of Karnataka where the mountains are gently sloping, compared to the
steep slopes of the Ghats in Maharashtra and Kerala.
The air parcel will retain its energy and speed for a longer time when the slope is gradual. This
will provide sufficient vertical motion to cloud droplets to grow by collisioncoalescence
process and hence form precipitation.
IASbabas 60 Day Plan Day 3 (Geography) 2017
Third, the gentle slope provides a greater area for sunlight absorption and heating leading to
greater convection when compared with an abrupt slope i.e. less Ghat area such as that of the
Maharashtra and Kerala Ghats.
Fourth, the continuous mountain range presents a greater barrier to rain-bearing winds than a
range comprising isolated mountains with gaps in between where the winds can easily pass to
the leeward side. Unlike in the case of Kerala, the Ghats in Maharashtra and Karnataka are
continuous.
Q.8) Which of the statements given below is/are not true in regard to western coastal plain?
a) It is a narrow belt.
b) Eastern coastal plain receives comparatively low rainfall but the Western coastal plain
receives heavy rainfall.
c) West Coast Plain is infertile and agriculturally not prosperous except in the Malabar
Coast.
d) It is an example of emergent coastal plain.
Western coastal plain is an example of submerged coastal plain (not emergent). It provides
natural conditions for development of ports.
Q.9) Which of the statements given below is/are correct about Terai region?
1. Terai is an ill-drained, damp (marshy) and thickly forested narrow tract to the south of
Bhabar running parallel to it.
2. It is a swampy lowland with silty soils.
3. Terai soils are rich in phosphate and organic matter but are deficient in nitrogen.
a) 1 only
b) 1 and 2 only
IASbabas 60 Day Plan Day 3 (Geography) 2017
c) 2 and 3 only
d) 1, 2 and 3
Terai is an ill-drained, damp (marshy) and thickly forested narrow tract (15-30 km wide) to the
south of Bhabar running parallel to it.
The underground streams of the Bhabar belt re-emerge in this belt. It is a swampy lowland with
silty soils.
The terai soils are rich in nitrogen and organic matter but are deficient in phosphate.
These soils are generally covered by tall grasses and forests but are suitable for a number of
crops such as wheat, rice, sugarcane, jute etc.. This thickly forested region provides shelter to a
variety of wild life.
Q.10) Consider the following about Laterite Soils and select the incorrect statement:
a) They are formed under conditions of high temperature and heavy rainfall with alternate
wet and dry periods.
b) They are rich in bauxite or ferric oxides.
c) They are fertile and suitable for growing plantation crops like tea, coffee, rubber.
d) They are found on the summits of Western Ghats and Eastern Ghats.
Laterite Soils are formed under conditions of high temperature and heavy rainfall with alternate
wet and dry periods. (especially Monsoonal regions)
Heavy rainfall promotes leaching (nutrients gets washed away by water) of soil whereby lime
and silica are leached away and a soil rich in oxides of iron and aluminium compounds is left
behind. They lack fertility due to intensive leaching.
a) 1 and 3 only
b) 2 and 4 only
c) 2, 3 and 4 only
d) 1, 2, 3 and 4
Soil in tropical rainforests is very poor, they are highly acidic and low in minerals and nutrients.
If the soil is so poor in tropical rain forests, how does such a dense array of shrubs and trees
grow there?
The key to the luxuriant vegetation of these forests lies in the rapid nutrient cycling of the
rainforest.
In the rainforest, most of the carbon and essential nutrients are locked up in living vegetation,
dead wood, and decaying leaves. As organic material decays, it is recycled so quickly that few
nutrients ever reach the soil, leaving it nearly infertile or sterile.
The tropical rainforest is a biome with a constant temperature and a high rainfall. The level of
humidity and density of the vegetation give the ecosystem a unique water and nutrient cycle.
Because of the lack of seasonal differences, due to the geographical location of the forests, and
the high humidity level the vegetation is luxuriant here.
Q.12) Match List I with List II and select the correct answer using the code given below the
Lists:
List I List II
(Region) (Vegetation)
A. Tundra 1. Conifers
B. Selvas 2. Epiphytes
IASbabas 60 Day Plan Day 3 (Geography) 2017
Code:
ABCD
a) 1243
b) 3421
c) 3241
d) 1343
1. Adjutant stork
2. Two horned rhino
3. Rattle snake
IASbabas 60 Day Plan Day 3 (Geography) 2017
a) 1 only
b) 2 and 3 only
c) 1 and 2 only
d) 1 and 3 only
Adjutant stork is one of the most endangered bird species widely distributed in the plains of the
Brahmaputra valley of Assam. The present global population of the greater adjutant stork is
about 1,500 around 900 of them are in Assam.
Adjutant storks were given the staid name by British colonial troops. This was because of their
slow, stiff military gait and their habit of standing motionless for long periods of time quite
like officers standing to attention.
Link: http://www.thehindu.com/todays-paper/tp-features/tp-metroplus/standing-to-
attention/article17525565.ece
Q.14) Sikkim and Darjeeling Himalayas are best suitable for tea plantations due to
1. Moderate slope
2. Thick soil cover with high organic content
3. Well distributed rainfall throughout the year
4. Mild winters
a) 1 and 3 only
b) 2 and 4 only
c) 2, 3 and 4 only
d) 1, 2, 3 and 4
Sikkim and Darjeeling Himalayas physical conditions such as moderate slope, thick soil cover
with high organic content, well distributed rainfall throughout the year and mild winters makes
it very much suitable for tea plantations. The British took advantage of these physical
conditions and introduced tea plantations.
Q.15) Match List I with List II and select the correct answer using the code given below the
Lists:
List I List II
Code:
ABCD
a) 2341
b) 1342
c) 1234
d) 4231
a) 3 only
b) 2 only
c) 3 and 4 only
d) 1 and 2 only
Moist continental : : Warm summers and cold winters with three months below freezing. Very
large annual temperature range.
Mediterranean : : Temperature range is moderate with warm to hot summers and mild winters.
a) Outstanding Indian scientists for their research and contributions in the field of
agriculture and environment
b) Outstanding contribution towards promotion of communal harmony, national
integration and peace
c) Outstanding contribution towards literature
d) Excellence in Science & Technology
Hindustani classical singer Shubha Mudgal has been selected for the Rajiv Gandhi National
Sadbhavana Award for her outstanding contribution towards the promotion of communal
harmony, peace and goodwill.
IASbabas 60 Day Plan Day 3 (Geography) 2017
Source: http://indianexpress.com/article/lifestyle/art-and-culture/singer-shubha-mudgal-to-
get-rajiv-gandhi-sadbhavana-award-2954597/
The recently released Habitat Commitment Index (HCI), which tracks every countrys
performance record as against the commitments made during Habitat II in Istanbaul in1996,
shows that India has fared worse than other South Asian countries such as Pakistan and Nepal.
The review of each countrys score on the previous urban agenda was released by the New York
based Global Urban Futures Project.
The HCI creates an index that tracks the progress made in six broad categories of the Habitat II
agenda namely Infrastructure, Poverty, Employment, Sustainability, Institutional Capacity, and
Gender. Instead of comparing absolute outcomes between countries of varying levels of
economic development, the outcomes are adjusted for resource difference, as measured by per
capita GDP.
Source: http://indianexpress.com/article/india/india-news-india/habitat-commitment-index-
india-fares-worse-than-pakistan-nepal-2989436/
a) Only 1
b) Only 2
c) Both 1 and 2
d) Neither 1 nor 2
Tala-Maddale is an ancient form of dance practiced in Southern India in the coastal Malnad
regions of Karnataka and Kerala.
The art form is popular in Uttara Kannada, Dakshina Kannada, Udupi and Shimoga districts of
Karnataka and Kasaragod district of Kerala. It is a derived form of Yakshaganaa classical dance
or musical form of art from the same region.
A typical Tala-Maddale show consists of veteran artists sitting in a circular fashion along with a
Bhagavata (the singer, with "Tala" or pair of small hand cymbals) and a "Maddale" (a type of
drum) player. Artists play the roles of characters in stories, typically, from Ramayana,
Mahabharata, and other puranas. Some consider the as a good presentation of oratorial skills.
Subjects of Tala Maddale performances normally focus on episodes from the Hindu epics of
Bhagavata and Puranas. However, episodes have also been created on other more current
issues, some of which include the Second World War, the Tashkent Agreement, the Indian
freedom struggle (Swarajya Vijaya) and computerisation (Ganakasura Kalaga).
In August 2016, the story of the Indian Freedom Struggle, Swarajya Vijaya, was enacted in Tala
Maddale in the outer yard of the Sri Ananteshwara Temple at Rajangana, Karnataka. The
episode (prasanga) had been written by M.V. Hegde. A similar Tala Maddale performance
related to the Indian freedom struggle had been held at this very place on 14 August 1947
when India achieved freedom from British Colonial powers.
Source: http://www.thehindu.com/todays-paper/%E2%80%98Swarajya-Vijaya%E2%80%99-
in-Tala-Maddale-on-eve-of-I-Day/article14563168.ece
TARANG (Transmission App for Real Time Monitoring & Growth) Mobile App & Web Portal has
been developed by RECTPCL, under the guidance of Ministry of Power. Introduction of the
electronic platform shall enhance ease, accountability & transparency and would boost
confidence of investors in power transmission sector. Better price discovery shall ultimately
benefit the power consumers in India.
TARANG is a powerful monitoring tool that tracks upcoming transmission projects and monitor
the progress of Inter-State & Intra-State transmission systems in the country, being developed
through Regulated tariff mechanism as well as Tariff Based Competitive Bidding(TBCB) route.
TARANG shall also include status of stalled/delayed transmission systems in country which
would enable the stakeholders viz. Ministry of Power, State Governments, all private sector
transmission developers and PSUs like Power Grid Corporation of India Ltd., for expeditious
completion of such projects.
Green Energy Corridors, an important component of our renewable energy mission, would also
be monitored through TARANG and help to meet our ambitious targets in same on time. The
web portal can be accessed at www.tarang.website.
Source: http://pib.nic.in/newsite/PrintRelease.aspx?relid=148982
Q.21) Operation Euphrates Shield has been launched in Syria by which country?
Source: http://www.thehindu.com/news/international/turkish-tanks-roll-into-
syria/article9027597.ece
The Panglong Agreement was reached in Panglong, Southern Shan State, between the Burmese
government under Aung San and the Shan, Kachin, and Chin peoples on 12 February 1947.
The agreement accepted "Full autonomy in internal administration for the Frontier Areas" in
principle and envisioned the creation of a Kachin State by the Constituent Assembly. It
continued the financial relations established between the Shan states and the Burmese federal
government, and envisioned similar arrangements for the Kachin Hills and the Chin Hills.
Source: http://www.thehindu.com/news/international/myanmar-holds-historic-peace-talks-
with-ethnic-groups/article9054231.ece
IASbabas 60 Day Plan (2017) Day 4 2017
DAY 4
SUBJECT GEOGRAPHY
TOPICS:
Indian Geography
PRELIMS MCQs:
Vegetation:
Tropical deciduous are the monsoon forests found in the large part of India, northern
Australia and in central America.
These forests are the most wide spread forests of India.
IASbabas 60 Day Plan (2017) Day 4 2017
These regions experience seasonal changes. Trees shed their leaves in the dry season to
conserve water.
They spread over areas of annual rainfall ranging between 200 cm and 70 cm.
The hardwood trees found in these forests are sal, teak, neem and shisham. Hardwood
trees are extremely useful for making furniture, transport and constructional materials.
Tigers, lions, elephants, langoors and monkeys are the common animals of these
regions.
Q.2) Match List I with List II and select the correct answer using the code given below the
Lists:
List I List II
(Drainage Patterns) (Description)
Code:
ABCD
a) 1234
b) 3421
c) 3241
d) 1432
2. When the rivers originate from a hill and flow in all directions, the drainage pattern is
known as radial. The rivers originating from the Amarkantak range present a good example
of it.
3. When the primary tributaries of rivers flow parallel to each other and secondary tributaries
join them at right angles, the pattern is known as trellis.
4. When the rivers discharge their waters from all directions in a lake or depression, the
pattern is know as centripetal.
a) 2 only
b) 2 and 3 only
c) 1 and 3 only
d) 1, 2 and 3
a) R. Krishna
IASbabas 60 Day Plan (2017) Day 4 2017
b) R. Godavari
c) R. Narmada
d) R. Kaveri
R. Kaveri or Cauverys major tributaries include - the Shimsha, the Hemavati, the Arkavati,
Honnuhole, Lakshmana Tirtha, Kabini, Bhavani River, the Lokapavani, the Noyyal and the
Amaravati River.
Q.5) Consider the below statements with regard to Himalayan and Peninsular river system:
a) 1 and 2 only
b) 1, 2 and 4 only
c) 1, 3 and 4 only
d) 1, 2, 3 and 4
Q.6) What are the benefits of temperate cyclones from the Mediterranean sea to India?
In northwestern India, some weak temperate cyclones from the Mediterranean sea cause
rainfall in Punjab, Haryana, Delhi and western Uttar Pradesh.
Although the amount is meagre, it is highly beneficial for rabi crops (not kharif). The
precipitation is in the form of snowfall in the lower Himalayas. It is this snow that sustains the
flow of water in the Himalayan rivers during the summer months. The precipitation goes on
decreasing from west to east in the plains and from north to south in the mountains.
Q.7) Which state has maximum different types of climatic regions in India?
a) Tamil Nadu
b) Gujarat
c) Madhya Pradesh
d) Maharashtra
a) 3 and 4 only
b) 1 and 3 only
c) 1, 3 and 4 only
d) All of the above
Gobind Vallabh Pant Sagar is a man-made lake situated in southern region of Sonebhadra
named after Bharat Ratna Pandit Govind Ballabh Pant.
Ranjit Sagar Dam - also known as the Thein Dam, is part of a hydroelectric project constructed
by the Government of Punjab on the Ravi River in the state of Punjab.
The Ranapratap Sagar Dam is built on the Chambal River at Rawatbhata in Rajasthan in India.
Q.10) What are the favourable conditions for the river capture to occur?
1. Higher volume of water so that velocity and discharge may be sufficiently high
2. Steep channel gradient
3. Relatively broad valley
4. More sediment load
a) 1 and 2 only
b) 1 and 3 only
c) 1, 2 and 3 only
d) 1, 2, 3 and 4
Though river capture is a natural process, but it does not take place in all circumstances rather
it requires certain necessary conditions.
One of the condition include that there should be relatively narrow valley so that water may
not spread in the otherwise wide and flat valleys and there should be low sediment so that the
river may resort to active headward erosion etc.
a) R. Narmada
b) R. Tapi
c) R. Chambal
d) R. Mahi
River Chambal rises near Mhow in the Malwa plateau of Madhya Pradesh and flows northwards
through a gorge up wards of Kota in Rajasthan, where the Gandhisagar dam has been
constructed. From Kota, it traverses down to Bundi, Sawai Madhopur and Dholpur, and finally
joins the Yamuna. The Chambal is famous for its badland topography called the Chambal
ravines.
1. Tapti
2. Narmada
3. Krishna
4. Godavari
a) 1234
b) 2143
c) 2134
d) 1243
Q.13) Which of the below given are conditions that favour the formation of deltas?
a) 1 only
b) 3 only
c) 2 and 3 only
d) 1 and 2 only
a) 1 only
b) 2 only
c) Both 1 and 2
d) Neither 1 nor 2
Weathering is the process of gradual destruction of rocks at or near the earths surface through
physical, chemical and biological processes caused by wind, water, climate change etc.
Erosion is the process of gradual transportation of weathered rock materials through natural
agencies like wind, river, streams, glaciers etc.
Winter monsoons do not cause rainfall as they move from land to the sea. It is because firstly,
they have little humidity; and secondly, due to anti cyclonic circulation on land, the possibility
of rainfall from them reduces. So, most parts of India do not have rainfall in the winter season.
a) progressive
b) regressive
c) proportional
d) based on benefit principle
Progressive tax is the taxing mechanism in which the taxing authority charges more taxes as
the income of the taxpayer increases. A higher tax is collected from the taxpayers who earn
more and lower taxes from taxpayers earning less. The government uses a progressive tax
mechanism.
Under progressive taxes, it is believed that people who earn more should pay more. The
income tax is divided into slabs. As the income of the tax payer crosses a benchmark income, a
new rate of tax (higher than before) is charged to him.
Q.17) India ranks a low 39th in terms of fixed broadband adoption among Asia Pacific
countries according to the report State of ICT in Asia and the Pacific 2016: Uncovering the
Widening Broadband Divide. Consider the following statements with respect to the report.
a) Only 1
b) Only 2
c) Both 1 and 2
d) None
It was revealed by the recently released report State of ICT in Asia and the Pacific 2016:
Uncovering the Widening Broadband Divide. It was published by United Nations Economic and
Social Commission for Asia and the Pacific (ESCAP).
India ranks lower than countries such as Sri Lanka, Bhutan and Bangladesh in fixed broadband
subscriptions per 100 inhabitants in 2015.
Hong Kong, New Zealand, Japan, Macao, Australia and Singapore topped the list. In India, just
1.35 of its citizens have subscribed to fixed broadband service in 2015.
The subscriptions per 100 inhabitants in the ESCAP region are behind Latin America and the
Caribbean countries. It is far lower than Europe and North America.
Source: http://www.thehindu.com/business/Industry/India-ranks-39th-in-Asia-Pacific-on-
fixed-broadband/article14578791.ece
Q.18) Consider the following statements with respect to Organisation of Islamic Cooperation
(OIC)
a) Only 2
b) 1 and 2
c) 1 and 3
d) None of the above
The OIC has permanent delegations to the United Nations and the European Union. The official
languages of the OIC are Arabic, English, and French.
56 of which are also member states of the United Nations. Some, especially in West Africa, are
though with large Muslim populations not necessarily Muslim majority countries. A few
countries with significant Muslim populations, such as Russia and Thailand, sit as Observer
States, while others, such as India and Ethiopia, are not members.
India has been blocked by Pakistan from joining the OIC over Kashmir issue. OIC regard parts of
Kashmir as occupied by India. The Organisation of Islamic Cooperation (OIC) has voiced its
concern over alleged excesses of the Indian military and the paramilitary forces in Kashmir.
Bahrain, Kuwait, Oman, Qatar, Saudi Arabia, and the United Arab Emirates are member of both
GCC and OIC.
The Cooperation Council for the Arab States of the Gulf/GCC is a regional intergovernmental
political and economic union consisting of all Arab states of the Persian Gulf, except for Iraq. Its
member states are Bahrain, Kuwait, Oman, Qatar, Saudi Arabia, and the United Arab Emirates.
IASbabas 60 Day Plan (2017) Day 4 2017
The Charter of the Gulf Cooperation Council was signed on 25 May 1981, formally establishing
the institution.
All current member states are monarchies, including three constitutional monarchies (Qatar,
Kuwait, and Bahrain), two absolute monarchies (Saudi Arabia and Oman), and one federal
monarchy (the United Arab Emirates, composed of seven member states, each with its own
emir). There have been discussions regarding the future membership of Jordan, Morocco, and
Yemen.
Source: http://www.thehindu.com/news/international/oic-expresses-concern-over-kashmir-
violence/article9012449.ece
1. The project aims to address the legal needs of the marginalized and vulnerable sections
of society, particularly women, children, and Scheduled Castes and tribal communities,
who do not have the requisite means to ensure that their rights are guaranteed
2. The first phase of the project focused on North Eastern States and Jammu and Kashmir
3. The project is anchored with the Department of Justice, Ministry of Law and Justice
a) 1 and 2
b) 2 and 3
c) 1 and 3
The first phase was implemented in the seven focus states of Bihar, Chhattisgarh, Jharkhand,
Madhya Pradesh, Orissa, Rajasthan and Uttar Pradesh as well as at the national level with key
institutions. The first phase of the project (2009-2012) focused on both the demand as well as
IASbabas 60 Day Plan (2017) Day 4 2017
supply side of justice, and has reached out to two million people, apart from training 7,000
paralegals and young lawyers.
The Access to Justice project is being implemented in the eight North-Eastern States, and
Jammu and Kashmir, at a total cost of Rs.30 crore for five years (2012-17).
The Project is anchored with the Department of Justice (DoJ), Ministry of Law and Justice,
Government of India. The DoJ has designated a National Project Director (NPD), who is
responsible for the overall management, including achievement of planned results, and for the
use of UNDP funds through effective process management and well established programme
review and oversight mechanisms. The DoJ signs a budgeted Annual Work Plan with UNDP on
an annual basis which lays down in detail what the Project seeks to achieve in that year.
Read More -
http://www.in.undp.org/content/india/en/home/operations/projects/democratic_governan
ce/access_to_justice.html
Source: http://www.thehindu.com/news/national/73-of-Law-Ministry-budget-goes-for-poll-
related-work/article14576555.ece
Q.20) Recently Supreme Court said it may refer a plea challenging the validity of the Jammu
and Kashmir (J&K) resettlement Act to a constitution bench if it finds that some issues
needed interpretation of the Constitution. Consider the following statements with respect to
J&K resettlement Act
1. It envisages grant of permit for resettlement of Pakistani nationals who had migrated to
Pakistan from Jammu and Kashmir after Indias partition
2. People of Jammu and Kashmir who migrated to Pakistan from 1947 could be considered
for their return but their descendants could not be
a) Only 1
b) Only 2
c) Both 1 and 2
d) Neither 1 nor 2
IASbabas 60 Day Plan (2017) Day 4 2017
It envisages grant of permit for resettlement of Pakistani nationals who had migrated to
Pakistan from Jammu and Kashmir between 1947 and 1954 after Indias partition
People of Jammu and Kashmir who migrated to Pakistan from 1947 could be considered
for their return but their descendants could not be.
In 1982, the Act was first challenged before SC and then Governor B.K. Nehru had
refused to sign the Bill and sent it back to the Assembly
Later Atal Bihari Vajpayee, the then President of newly constituted BJP, had also filed a
petition before the apex court seeking intervention
The matter was considered by the Constitution Bench of the Supreme Court in 2001 on
a presidential reference
The apex court returned the reference back to President with a three-word
pronouncement- Returned, respectfully, unanswered
Source: http://www.thehindu.com/news/national/JampK-resettlement-law-SC-hints-
referring-it-to-larger-Bench/article14572823.ece
Q.21) Consider the following statements about National Housing Bank (NHB)
1. The National Housing Policy, 1988 envisaged the setting up of NHB as the Apex level
institution for housing
3. Residex, Indias first official residential housing price index was launched by NHB
a) 1 and 2
b) 2 and 3
IASbabas 60 Day Plan (2017) Day 4 2017
c) 1 and 3
National Housing Bank (NHB), a wholly owned subsidiary of Reserve Bank of India (RBI), was set
up on 9 July 1988 under the National Housing Bank Act, 1987. NHB is an apex financial
institution for housing. NHB has been established with an objective to operate as a principal
agency to promote housing finance institutions both at local and regional levels and to provide
financial and other support incidental to such institutions and for matters connected therewith.
NHB registers, regulates and supervises Housing Finance Company (HFCs), keeps surveillance
through On-site & Off-site Mechanisms and co-ordinates with other Regulators.
The National Housing Policy, 1988 envisaged the setting up of NHB as the Apex level institution
for housing.
In pursuance of the above, NHB was set up on 9 July 1988 under the National Housing Bank Act,
1987. NHB is wholly owned by Reserve Bank of India, which contributed the entire paid-up
capital. The general superintendence, direction and management of the affairs and business of
NHB vest, under the Act, in a Board of Directors.The Head Office of NHB is at New Delhi.
Residex
NHB Residex was launched (first official residential housing price index) in 2006-07.
It is also working on new indices such as rental index and building and construction
material index
The new Residex will be having both evaluated value and registered value of a particular
house as well
NHB is also working on two more indices which include rental index and building and
construction material index
Source: http://www.livemint.com/Opinion/fRYayJUap7vpZsPv5788nN/Finally-an-inclusive-
mortgage-market.html
Q.22) Recently, which of the following communities was granted Scheduled Tribe status?
a) Kattunayakan
b) Malalikuravan
c) Kuruman
d) Irular
Puducherry had sent a proposal for declaration of five communities as ST, including Irular
(including Villi and Vettaikaran), Kattunayakan, Malalikuravan, Yerukkula and Kuruman 14 years
back.
The President has given his assent to declaration of Irular community (including Villi and
Vettaikaran) in the Scheduled Tribes list.
Puducherry has got its first Scheduled Tribes. The three communities had got the first approval
from Registrar General of India and National Commission for Scheduled Tribes in May and then
in November.
The other four communities are still awaiting the necessary approvals.
President gave its assent in the month of December, but the issue was in news earlier as well.
Source: http://www.pib.nic.in/newsite/mbErel.aspx?relid=142361
IASbabas 60 Day Plan (2017) Day 4 2017
2. Electricity Distribution Company and Energy Efficiency Services Limited (EESL) are the
implementing agencies
a) 1 and 3
b) 1 and 2
c) 2 and 3
News: Gujarat has become the first state to distribute 2 crore LED bulbs under the Centre's
Unnat Jyoti by Affordable LEDs for all (UJALA) scheme
Aim: To promote efficient lighting, enhance awareness on using efficient equipment which
reduce electricity bills and help preserve environment.
It urges the people to use LED bulbs in place of incandescent bulbs, tube lights and CFL bulbs as
they are more efficient, long lasting and economical in their life cycle duration
Implementing Agency: Electricity Distribution Company and Energy Efficiency Services Limited
(EESL) a public sector body of Government of India
An ordinary bulb is an extremely energy inefficient form of lighting with just 5% of the
electricity input converted to light. Efficient light bulbs like Light-emitting Diode (LEDs)
IASbabas 60 Day Plan (2017) Day 4 2017
consumes only one-tenth of energy used by ordinary bulb to provide the same or better light
output.
Source: http://www.moneycontrol.com/news/current-affairs/gujarat-becomes-first-state-to-
distribute-2-cr-led-bulbs_7372921.html
IASbabas 60 Day Plan (2017) Day 5 2017
DAY 5
SUBJECT SCIENCE & TECHNOLOGY
TOPICS:
PRELIMS MCQs:
Q.1) A Portable Telemedicine System for Armed Forces (PDF) has been developed. Which
of the following statements are correct about it?
a) 1 only
b) 2 only
c) Both 1 and 2
d) Neither 1 nor 2
The Portable Telemedicine System (PDF) for Armed Forces was developed by the Defence
Bioengineering and Electromedical Laboratory (DEBEL), Bengaluru in early 2016. The system
is a means of providing remote assistance to injured personnel in a field hospital or a ship
out at sea. The rugged and portable system is capable of taking various readings including
blood pressure, temperature, heart rate, and includes an electrocardiogram.
IASbabas 60 Day Plan (2017) Day 5 2017
Image: DRDO/DEBEL
The system can store and transmit annoted data, and can open up a real time live channel
for high quality video conferencing. The data can be transmitted over various
communication channels, and includes the capability to interface with satellites. The
Biomedical Data Acquisition System (BioDAS) on board was developed indigenously. An
archival software on board the system maintains records in a secure environment. The
telemedicine system was successfully demonstrated on board the ships INS Deepak, Gomati,
and Talwar.
Q.2) Armament Research and Development Establishment (ARDE) and the High Energy
Materials Research Laboratory (HEMRL) have developed Penetration cum Blast and
Thermobaric ammunitions. Consider the following statements about these:
1. HEMRL is the research wing of DRDO which researchers in new high energy materials
to be used in warheads and ammunition.
2. The newly developed ammunition uses atmospheric oxygen for the explosions
instead of the oxidiser included in the compound, as is done in conventional
explosives.
3. The blast done by these ammunitions is much weaker than the conventional
explosives.
a) 1 and 2
b) 2 and 3
c) 1 and 3
d) All of the above
Armament Research and Development Establishment (ARDE) and the High Energy Materials
Research Laboratory (HEMRL), research wings of the DRDO based in Pune designed new
ammunition meant specifically for the Arjun tank. The Penetration-cum-Blast (PCB) and
Thermobaric (TB) Ammunition were successfully test fired in Odisha, demonstrating the
devastating power of the ammunition against concrete structures, fortifications and armour
plates.
The HEMRL lab researchers new high energy materials to be used in warheads and
ammunition. The newly developed ammunition uses atmospheric oxygen for the explosions
instead of the oxidiser included in the compound, as is done in conventional explosives. This
means that the resulting mixture is far more explosive for the same amount of weight. The
test firing successfully managed to destroy a derelict tank that had been fitted with various
sensors to measure the shock, blast pressure and temperature of the new ammunition. It
was the first time in India that the effectiveness of the new ammunition was measured in
such detail.
IASbabas 60 Day Plan (2017) Day 5 2017
Q.3) Which of the following processes are endothermic i.e. they absorbs energy?
1. Melting Ice
2. Photosynthesis
3. Respiration
4. Thermal Decomposition
a) 1 and 3
b) 1, 2 and 4
c) 2 and 4
d) All of the above
In the process of respiration, the food we eat reacts with the oxygen we breathe and
releases energy.
Q.4) India has successfully developed its first indigenously built SONAR composite dome.
Which of the following statements is correct about it?
a) The Sonar dome is attached to the bottom of ships, and scans the seas for submarine
threats.
b) It is attached to the nose of fighter planes to navigate properly in bad weather
conditions.
c) It is attached to warheads of fire and forget missile to successfully chase the targets
and destroy them.
d) None of the above
The Defense Minister Manohar Parrikar flagged off the first indigenous composites Sonar
dome during the Defexpo 2016. India joined a select group of countries capable of
manufacturing such structures. The Sonar dome is attached to the bottom of ships, and
scans the seas for submarine threats. The requirements of manufacturing such domes are
technologically demanding, with a need for a solid and robust structure that is also
acoustically transparent.
IASbabas 60 Day Plan (2017) Day 5 2017
Defense Minister Manohar Parrikar flagging off the indigenously made Sonar Dome.
The research and development needed for manufacturing the dome was conducted by a
Pune based DRDO lab known as Research and Development Establishment (Engrs) (RDE(E)).
The dome was manufactured by Kineco Ltd, a composites manufacturing company based in
Pilerne, Goa. The Vacuum Assisted Resin Transfer Molding (VARTM) Process with a process
monitoring capability, to ensure that a quality product was manufactured. Sucessfully
manufacturing the Sonar Dome indigenously means that India now has the capability for
manufacturing more advanced structures, such as entire ship hulls. There are land based
and aerospace applications that stand to benefit from the advancement in manufacturing
capabilities.
a) 1 only
b) 2 only
c) Both 1 and 2
d) Neither 1 nor 2
IASbabas 60 Day Plan (2017) Day 5 2017
Varunastra torpedo
The Defense minister Manohar Parrikar handed over the Varunastra to the Indian Navy at
the end of June. Varunastra is a ship launched heavy weight torpedo, also known as an
underwater missile. The torpedo was developed by Naval Science and Technological
Laboratory (NSTL) in partnership with Bharat Dynamics. The torpedo is manufactured by
using 95 per cent of indigenous parts.
During the handover, Parrikar said In these high technology areas, DRDOs contribution
with 95 per cent of indigenous content is an apt example of Indigenously Designed
Developed and Manufactured category. The torpedo was an important milestone in
pushing India towards self reliance when it comes to underwater defense capabilities. The
Rajput and Delhi class of warships can be equipped with the new torpedos, and future Anti-
Submarine Warfare capable ships will also have the capability. The torpedo is capable of
taking down stealth submarines in deep or shallow waters.
Q.6) Recently Indian Air Force has inducted two Tejas Aircrafts and ordered for twenty
more. Which of the following statements are correct about Tejas?
a) 1 and 2
b) 2 and 3
c) 1 and 3
d) None of the above
The Indian Air Force inducted two Tejas aircraft into its 45th Squadron on July 1. The Light
Combat Aircraft were jointly developed by Hindustan Aeronautics Limited (HAL)
and Aeronautical Development Agency (ADA). The aircraft can take up a variety of roles in
combat, and are supersonic. India has been developing the aircraft since 1980. Prior to the
induction, the aircraft participated in 3,200 sorties to demonstrate their capabilities.
IASbabas 60 Day Plan (2017) Day 5 2017
Image: DRDO/HAL/ADA
Prime minister Modi said Induction of indigenously made Tejas fighter jet into the Air Force
fills our hearts with unparalleled pride and happiness. I laud HAL and ADA on the induction
of Tejas fighter jet. This illustrates our skills and strengths to enhance indigenous defence
manufacturing. The Tejas is capable of carrying four tonnes of weapons. The aircraft are
capable of firing air-to-air missiles, and dropping laser guided bombs. The aircraft includes a
head mounted display for the pilot, and a glass cockpit on which realtime information is
displayed. The Indian Air Force has placed an order for an additional 20 Tejas fighter jets.
Q.7) DRDO successfully carried out the maiden flight of the Rustom-II unmanned aerial
vehicle (UAV). Which of the following statements are correct about Rustom II?
a) 1 only
b) 2 only
c) Both 1 and 2
d) Neither 1 nor 2
IASbabas 60 Day Plan (2017) Day 5 2017
In November, DRDO successfully carried out the maiden flight of the Rustom-II unmanned
aerial vehicle (UAV). The UAV in general is combat capable and can be
equipped with electro optic sensors, radar, electronic intelligence, communication
intelligence and situational awareness payloads. Rustam II was designed as a combat UAV
initially, but in the Current variant it has not been weaponised. So this debate actually gave
a good result Rustom II as of now is not combat capable.
The Rustom-II is a Medium Altitude Long Endurance (MALE) UAV. The maiden flight was
carried out at the Aeronautical Test Range (ATR), Chitradurga, near Bengaluru, a new facility
dedicated to testing UAVs and manned aircraft. The tests proved the capabilities of the
flying platform, including take-off, banking, level flight and landing.
The development of the UAV contributes to the Make in India initiative, as many sub
systems were developed and manufactured entirely in India, with the participation of
private companies. The air frame, landing gear, flight control, and avionics sub systems were
made entirely in India. The UAV is meant for Intelligence, Surveillance and Reconnaissance
(ISR) roles. The UAV is going to undergo further test and trials, for validation of the design
parameters. The next step for the UAV is user validation trials.
Q.8) Which of the following statements are correct about Anti Tank NAG Missile?
IASbabas 60 Day Plan (2017) Day 5 2017
a) 1 and 2
b) 2 and 3
c) 1 and 3
d) All of the above
The Nag anti-tank missile joined the ranks of indigenously developed missiles by
the Integrated Guided Missile Development Programme (IGMDP) a program that was lead
by former president Abdul Kalam. The Nag joins Agni, Prithvi, Trishul and Akash. The Nag is
a lockon-before-launch (LOBL), fire-and-forget, anti-tank guided missile, and was
successfully tested against targets up to a range of four kilometres.
Image: DRDO
The test flights tested the the indigenously developed Thermal Target System and
the Infrared Imaging Seeker on board the missile. Real time image processing algorithms on
IASbabas 60 Day Plan (2017) Day 5 2017
board the missile process the accuracy of the flight in real time. The Nag missile is meant to
tackle modern battle tanks and other heavily armoured tanks. There is a high chance of
neutralising the target with a single missile. The Nag can be fired from land or air based
platforms, and there are amphibious and mobility tests being conducted. The Nag missile is
ready for induction into the Indian Army.
Q.9) Which of the following are the correct differences between Ballistic and Cruise
Missiles?
1. Ballistic missiles follow a projectile motion under gravity while cruise missiles have
maneuvering capabilities.
2. Usually the range of Cruise missiles is much higher than ballistic missiles.
3. The destruction radius of Ballistic missiles is usually much higher than Cruise missiles.
a) 1 only
b) 2 only
c) 1 and 3
d) All of the above
There are some important differences between cruise and ballistic missiles. These are:
Ballistic missiles follow an arc-like trajectory and are launched from the land or sea
They usually carry a nuclear warhead and are very heavy
They rely on Earths gravity to fly down once launched
They have much larger range
Cruise missiles can also be launched from air and fly within Earths atmosphere
They have their own engines and wings to strike the target
They can be supersonic or sub-sonic and are highly accurate
They usually carry conventional warheads although some cruise missiles can also be
equipped with nuclear warheads
IASbabas 60 Day Plan (2017) Day 5 2017
a) 1 and 2
b) 2 only
c) 2 and 3
d) All of the above
Mercury and Bromine are the only two elements which are in liquid state at room
temperature. (Mercury is the only metal.)
Q.11) Magnetic induction stoves are very commonly used these days. Aluminium and
Copper Utensils cannot be directly used over these stoves. What is the reason for that?
In an induction cooker, a coil of copper wire is placed under the cooking pot and an
alternating electric current is passed through it. The resulting oscillating magnetic field
induces a magnetic flux which repeatedly magnetises the pot, treating it like the lossy
magnetic core of a transformer. This produces large eddy currents in the pot, which because
of the resistance of the pot, heats it.
For nearly all models of induction cooktops, a cooking vessel must be made of, or contain, a
ferromagnetic metal such as cast iron or some stainless steels. However, copper, glass, non
magnetic stainless steels, and aluminum vessels can be used if placed on a ferromagnetic
disk which functions as a conventional hotplate.
Induction cooking is quite efficient, which means it puts less waste heat into the kitchen, can
be quickly turned off, and has safety advantages compared to gas hobs (cooktops). Hobs are
also usually easy to clean, because the hob itself does not get very hot.
Q.12) Graphite is an important allotrope of Carbon. Which of the following statements are
correct about Graphite?
1. It is an organic compound.
2. It is a good conductor of electricity unlike other allotropes of carbon.
3. It is used as a lubricating agent because of its layered structure.
a) 1 and 2
b) 2 and 3
c) 1 and 3
d) All of the above
Graphite and diamond are the two mineral forms of carbon. Diamond forms in the mantle
under extreme heat and pressure. Most graphite found near Earth's surface was formed
within the crust at lower temperatures and pressures. Graphite and diamond share the
same composition but have very different structures.
The carbon atoms in graphite are linked in a hexagonal network which forms sheets that are
one atom thick. These sheets are poorly connected and easily cleave or slide over one
another if subjected to a small amount of force. This gives graphite its very low hardness, its
perfect cleavage, and its slippery feel.
Graphite is an allotrope of carbon and it is one of the debatable aspects of chemistry to put
it as organic or inorganic as per its definition. Going by mere definition, it cannot be
considered as organic compound for many reasons.
In contrast, the carbon atoms in diamond are linked into a frameworks structure. Every
carbon atom is linked into a three-dimensional network with four other carbon atoms with
strong covalent bonds. This arrangement holds the atoms firmly in place and makes
diamond an exceptionally hard material.
Q.13) ISRO has recently successfully tested the SCRAMJET engine technology. Which of
the following statements are correct about SCRAMJET?
a) 1 only
b) 2 only
c) Both 1 and 2
d) Neither 1 nor 2
Q.14) There is a growing demand from many states to open more Sainik Schools.
Consider the following statements regarding Sainik Schools:
1. They are established as a Joint Venture between Center and State Government.
2. They are specifically meant to provide education to children of personnel of Indian
Armed Forces only.
3. The aim of Sainik Schools is all round development of a childs personality in order to
prepare him for entry into National Defence Academy and to remove regional
imbalance in the officers cadre of the Armed Forces.
a) 1 and 2
b) 2 and 3
c) 1 and 3
IASbabas 60 Day Plan (2017) Day 5 2017
Sainik Schools
The Sainik Schools are established as a joint venture of the Central and State Governments.
These are under the overall governance of Sainik Schools Society, Ministry of Defence.
At present, there are 25 Sainik Schools located in various parts of the country. There is a
growing demand from many States to open new Sainik Schools. As a result, states of
Andhra Pradesh, Haryana, Bihar and Karnataka have two
Sainik Schools each.
The objectives of Sainik Schools include bringing quality Public School education within the
reach of common man, all round development of a childs personality in order to prepare
him for entry into National Defence Academy and to remove regional imbalance in the
officers cadre of the Armed Forces. The number of cadets joining the National Defence
Academy is on the rise in keeping with the primary aim of establishing of Sainik Schools to
prepare boys academically, physically and mentally for entry into NDA. For the 134th NDA/
INA Course which commenced in July 2015, a total of 101 Cadets from all Sainik Schools
have joined the NDA and Naval Academy.
Sainik Schools admit boys into classes VI and IX. Their age should be 10-11 years for classes
VI and 13-14 years for class IX as on 1st July of the year in which admission is sought.
Admissions are made strictly as per the order of merit on the basis of an All India Sainik
School Entrance
Examination held in January each year.
a) Russia
b) USA
c) Israel
d) France
Parliaments Standing Committee on Defence reveals in a new report released that the
United States has been Indias biggest supplier of weapons platforms over the past three
years.
IASbabas 60 Day Plan (2017) Day 5 2017
Non-official think tanks and non-proliferation groups like the Stockholm International Peace
Research Institute (SIPRI) publish annual assessments of global arms transfers. This,
however, is an authoritative declaration from the government of India.
The committees report number 31 divulges that India concluded contracts worth Rs 28,895
crore ($4.35 billion) with the US during 2013-14, 2014-15 and 2015-16.
Russia was a distant second in this period with contracts worth Rs 8,374 crore ($1.26
billion), Israel third with contracts worth Rs 7,605 crore ($1.14 billion), and France fourth
with contracts worth Rs 1,836 crore. India also signed contracts worth Rs 6,974 crore ($1.05
billion) with others", which include Germany, the UK, Ukraine and Poland.
Q.16) Consider the following statements about Kudankulam Nuclear Power Plant
a) 1 and 2
b) 2 and 3
c) 1 and 3
d) All of the above
The Kudankulam Nuclear Power Plant (KKNPP) is the first in the world to have post-
Fukushima safety enhancement requirements implemented and operated successfully.
There are a number of advanced active and passive safety systems which ensure
unprecedented design-level nuclear and ecological safety of the plant
Double localising and protective containment, passive heat removal system from reactor
plant automatically, core catcher, and closed industrial water intake are some of the safety
features incorporated.
The reactor is protected from the impact of any earthquake, tsunami, tornado and
hurricane.
IASbabas 60 Day Plan (2017) Day 5 2017
Given that a large amount of seawater is drawn in to cool the reactors, measures have been
taken to preserve the biological diversity of the Mannar Bay adjacent to Kudankulam.
Seawater is supplied from the so-called bucket constructed in the sea into the special
facilities and systems which ensure that fish and plankton return to sea.
It is the single largest nuclear power station in India, situated in Koodankulam in the
Tirunelveli district of the southern Indian state of Tamil Nadu.
The main distinguishing features of the VVER compared to other PWRs are:
Source: http://www.thehindu.com/news/national/Kudankulam-plant-safest-in-the-
world-says-Russian-official/article14544107.ece
1. It allows scientists to selectively edit genome parts and replace them with new DNA
stretches
2. It consists of two key molecules that introduce a change into the DNA Cas9 and
Guide RNA (gRNA)
3. CRISPRs do not need to be paired with separate cleaving enzymes as other tools do
a) Only 1
b) 1 and 2
c) 1, 2 and 3
d) 1 and 3
CRISPR-Cas9 is a genome editing tool that is creating a buzz in the science world. It is faster,
cheaper and more accurate than previous techniques of editing DNA and has a wide range
of potential applications.
CRISPR-Cas9 is a unique technology that enables geneticists and medical researchers to edit
parts of the genome by removing, adding or altering sections of the DNA sequence.
The CRISPR-Cas9 system consists of two key molecules that introduce a change into the
DNA. These are:
an enzyme called Cas9. This acts as a pair of molecular scissors that can cut the two
strands of DNA at a specific location in the genome so that bits of DNA can then be
added or removed.
a piece of RNA called guide RNA (gRNA). This consists of a small piece of pre-
designed RNA sequence (about 20 bases long) located within a longer RNA scaffold.
The scaffold part binds to DNA and the pre-designed sequence guides Cas9 to the
right part of the genome. This makes sure that the Cas9 enzyme cuts at the right
point in the genome.
Since the CRISPR-Cas9 system itself is capable of cutting DNA strands, CRISPRs do not need
to be paired with separate cleaving enzymes as other tools do.
Source: http://www.thehindu.com/sci-tech/health/First-human-genetic-editing-trial-in-
China/article14547625.ece
for supporting breastfeeding through health systems. The programme has been named
MAA to signify the support a lactating mother requires from family members and at health
facilities to breastfeed successfully. The chief components of the MAA Programme are
Community awareness generation, Strengthening inter personal communication through
ASHA, Skilled support for breastfeeding at Delivery points in Public health facilities, and
Monitoring and Award/recognition.
Source: http://pib.nic.in/newsite/PrintRelease.aspx?relid=148531
The Reserve Bank of India (RBI) launched a portal to curb illegal collection of money
by companies.
The website will enable public to obtain information regarding entities who accept
deposits, lodge complaints and also share information regarding illegal acceptance of
deposits.
The website would also help enhance coordination among regulators and state
government agencies.
The website also incorporates regulations prescribed by all financial regulators that
one has to follow.
'Sachet' also has a section for closed user group for State Level Coordination
Committee (SLCCs) wherein they could share market intelligence and other
information about their activities as well as agenda and minutes of meetings across
the country in real time.
All states have SLCCs comprising of various regulators, including RBI, Securities and
Exchange Board of India (Sebi), National Housing Bank (NHB), Insurance Regulatory
and Development Authority (IRDA), Registrar of Companies (ROC) and concerned
state government departments, such as, home, finance, law and various police
authorities.
Source: http://www.business-standard.com/article/finance/rbi-launches-sachet-portal-
to-check-illegal-money-collection-116080401392_1.html
Q.20) Which of the following is associated with mobile internet coverage speed?
1. EDGE
2. HSPA
3. NFC
4. GPRS
5. LTE
a) 1, 2, 3 and 4
b) 1, 2, 4 and 5
c) 1, 3, 4 and 5
d) All of the above
Q.21) This tool allows developing countries to raise tariffs temporarily to deal with import
surges or price falls. Identify the tool from the following options:
WTOs Special Safeguard Mechanism (SSM) is a protection measure allowed for developing
countries to take contingency restrictions against agricultural imports that are causing
IASbabas 60 Day Plan (2017) Day 5 2017
injuries to domestic farmers. The contingency measure is imposition of tariff if the import
surge causes welfare loss to the domestic poor farmers. The design and use of the SSM is an
area of conflict under the WTO.
At the Doha Ministerial Conference, the developing countries were given a concession to
adopt a Special Safeguard Mechanism (SSM) besides the existing safeguards (like the Special
Agricultural Safeguard or the SSG). This SSM constituted an important part of the promises
offered to the developing world at Doha (known as Doha Development Agenda) and the
Doha MC became known as a development round.
The SSG was available to all countries- both developing and developed whereas the SSM is
allowable only to the developing countries. It is to be mentioned that the SSG was available
as it was inducted under the GATT agreement; whereas the SSM was the invention of the
Doha MC.
Source: http://pib.nic.in/newsite/PrintRelease.aspx?relid=148541
Q.22) Consider the following statements with respect to Food Safety and Standards
Authority of India (FSSAI).
a) Only 1
b) Only 2
c) Both 1 and 2
d) None of the above
FSSAI
Source: http://www.livemint.com/Politics/8621Grfw5wxzrnPDjPgyqM/FSSAI-wants-to-
regulate-quality-of-tap-water.html
Global Innovation Index (GII) 2016 has been released. It is published by Cornell University,
INSEAD, and the World Intellectual Property Organization, in partnership with other
organizations and institutions.
Source: http://www.livemint.com/Politics/lATexj4C2o8ynCSRyM1VeL/India-climbs-15-
spots-in-innovation-ranking.html
IASbabas Prelims 60 Day Plan Day 6 2017
a) kcal m2
b) gm 2
c) gm2 yr1
d) None of the above
A constant input of solar energy is the basic requirement for any ecosystem to function and
sustain. Primary production is defined as the amount of biomass or organic matter
produced per unit area over a time period by plants during photosynthesis. The rate of
biomass production is called productivity. It is expressed in terms of gm2 yr1 to compare
the productivity of different ecosystems. It can be divided into gross primary productivity
(GPP) and net primary productivity (NPP). Gross primary productivity of an ecosystem is the
rate of production of organic matter during photosynthesis. A considerable amount of GPP
is utilised by plants in respiration. Gross primary productivity minus respiration losses (R), is
the net primary productivity (NPP).
1. It takes into account the same species belonging to two or more trophic levels
2. Saprophytes plays a vital role in ecological pyramids
3. Pyramid of energy is the only pyramid which is always upright
a) 1 Only
IASbabas Prelims 60 Day Plan Day 6 2017
b) 1 and 2
c) 2 only
d) None
The base of each pyramid represents the producers or the first trophic level. Apex
represents tertiary or top level consumers. In general, all pyramids are upright, but there
are few exceptions.
1. Pyramid of Number:
It represents the total number of organisms at each trophic level. It is always upright but in
a tree ecosystem pyramid of number is inverted.
2. Pyramid of Biomass:
It can be:
3. Pyramid of Energy:
It represents total energy of the organisms in each trophic level. Pyramid of energy is always
upright, i.e., it can never be inverted, because when energy is transferred from a particular
trophic level to the next trophic level some energy is always lost as heat at each step.
(i) A given organism may occupy more than one trophic level simultaneously.
(iii) A given species may occupy more than one trophic level in the same ecosystem at the
same time.
For example, a sparrow is primary consumer, when it eats seeds, fruit, peas, etc., and a
secondary consumer when it eats insects and worms.
(iv) In most ecosystems, all the pyramids of number, biomass, energy are upright, i.e.,
producers are more in number and biomass than the herbivores and herbivores are more in
number and biomass than carnivores.
(v) Also energy at lower trophic level is always more at higher trophic level. However, there
are exceptions to this generalisation.
(vi) Pyramid of biomass in sea is inverted because the biomass of fishes far exceeds that of
phytoplankton.
(vii) Each bar in the energy pyramid indicates the amount of energy present at each trophic
level in a given time or annually per unit area.
(i) It never takes into account the same species belonging to two or more trophic levels.
IASbabas Prelims 60 Day Plan Day 6 2017
(ii) It assumes a simple food chain, something that almost never exists in nature.
(iv) Saprophytes are not given any place in ecological pyramids even though; they play an
important role in ecosystem.
a) Only 2
b) 2 and 3
c) 1 and 2
d) Only 1
Unlike carbon cycle, there is no respiratory release of phosphorus into atmosphere: Focus
on basic concept (How can phosphorous be released through respiration?)
Atmospheric inputs of phosphorus through rainfall are much smaller than carbon inputs:
Gaseous exchanges of phosphorus between organism and environment are negligible unlike
carbon
Basic Info:
Nutrients are basically required by all organisms for the overall growth,
Hint development, reproduction and to carry out various life processes. The interesting fact is
that the nutrient amount is fixed in the environment. Thus, the nutrients are naturally
recycled for the constant supply to the organisms. This movement of the nutrients through
various components of the ecosystem is called nutrient cycling. Broadly, we have two types
of nutrient cycling in nature:
Gaseous cycling This kind of cycling takes place for the gases and the atmosphere acts as
the reservoir for these gases i.e Carbon
IASbabas Prelims 60 Day Plan Day 6 2017
Sedimentary cycling This kind of cycling takes place for the nutrients that exist in their
elemental form and the earths crust acts as the reservoir i.e Phosphorous
Q.4) Consider the following statements regarding Flowering and Non-flowering plants-
a) 1 and 2
b) 2 and 3
c) Only 3
d) 1 and 3
Flower is the reproductive organ of the plant. Flowers can be composite or simple. Anyway
using flower as the classification criteria makes this a natural classification.
Angiosperms
Angiosperms are plants that have flowers and their reproduction is done by flowers.
All flowering plants produce seeds and it is covered by a fruit.
Angiosperms have complicated flowers with complicated structure.
IASbabas Prelims 60 Day Plan Day 6 2017
1. Dicotyledonous Plants
Dicotyledonous flowering plants are the widely expanded around the world. The seed of this
type of plants have 2 cotyledons inside it. Also the root system has a long tap root. The
leaves have webbed veins and the flowers have 5 or multiple of 5 petals.
Jak
Mango
Lemon
Apple
Tomato
2. Monocotyledonous Plants
Monocotyledonous flowering plants are very few when compared to dicotyledonous plants.
These plants dont have a tap root system, but a fibrous root system. The seed has only 1
cotyledons and the veins of the leaves are parallel. Number of petals in the flowers are 3 or
multiples of 3.
Coconut
Wheat
Palm
Rice
Garlic
Non-Flowering Plants
Non-flowering plants do have reproductive organs available in the flowers, but these are not
complicated as Dicotyledonous flowers. We can further classify non-flowering plants as with
seeds and without seeds.
Gymnosperms
IASbabas Prelims 60 Day Plan Day 6 2017
Gymnosperms are plant with seeds and the embryo is inside a seed. The seed is not covered
(not enclosed) in a fruit. These are the less advanced type of seed plants.
Yews
Ginkgo
Cycads
Conifers
Pine
Bryophyta
Bryophyta doesnt have seeds. There are so many species under this type and body of the
plant may be a thallus. Sometimes they do have leaves. There are no true roots, but roots
like structures available. These are also known as Rhizoids and grow on moist walls and
ground.
a) 1 and 2
b) 2 and 3
c) Only 2
d) 1, 2 and 3
The Zoological Survey of India (ZSI) was established on 1st July, 1916 to promote survey,
exploration and research leading to the advancement in our knowledge of various aspects
of exceptionally rich life of the erstwhile British Indian Empire . The survey has its genesis
in the establishment of the Zoological Section of the Indian Museum at Calcutta in 1875. By
IASbabas Prelims 60 Day Plan Day 6 2017
gradually strengthening its staff and expanding its research program, the Survey has met the
challenge of the past and is on its way to meet the demands of the future.
Primary objectives
Secondary objectives
Q.6) Consider the following w.r.t Animal Welfare Board of India (AWBI)
a) 1 and 2
b) 1 and 3
IASbabas Prelims 60 Day Plan Day 6 2017
c) 2 and 3
d) 1, 2 and 3
The Animal Welfare Board of India is a statutory advisory body on Animal Welfare Laws and
promotes animal welfare in the country. Established in 1962 under Section 4 of the
Prevention of Cruelty to Animals Act, 1960.
1. This was the first umbrella act which established schedules of protected plant and
animal species.
2. The act has six schedules that extends to whole of India
3. It has given criterions to declare national parks and wildlife sanctuaries
a) 1 and 2
b) 1 and 3
c) 2 and 3
d) 1, 2 and 3
The Wildlife Protection Act, 1972 is an Act of the Parliament of India enacted for protection
of plants and animal species. Before 1972, India only had five designated national parks.
This was the first umbrella act which established schedules of protected plant and animal
species. By this act, hunting or harvesting these species was largely outlawed.
Among other reforms, the Act established schedules of protected plant and animal species;
hunting or harvesting these species was largely outlawed.
The Act provides for the protection of wild animals, birds and plants; and for matters
connected therewith or ancillary or incidental thereto. It extends to the whole of India,
except the State of Jammu and Kashmir which has its own wildlife act. It has six schedules
IASbabas Prelims 60 Day Plan Day 6 2017
which give varying degrees of protection. Schedule I and part II of Schedule II provide
absolute protection - offences under these are prescribed the highest penalties. Species
listed in Schedule III and Schedule IV are also protected, but the penalties are much
lower. Schedule V includes the animals which may be hunted. The plants in Schedule VI are
prohibited from cultivation and planting. The hunting to the Enforcement authorities have
the power to compound offences under this Schedule (i.e. they impose fines on the
offenders). Up to April 2010 there have been 16 convictions under this act relating to the
death of tigers.
a) 1 and 2
b) 2 and 3
c) Only 3
d) 1, 2 and 3
Remember that, the ultimate source of nitrogen to the ecosystem is molecular nitrogen in
the atmosphere which cannot be directly metabolized by plants or animals.
Native rocks do not contain nitrogen hence do not contribute nitrogen to plants or animals.
And nitrogen fixation in terrestrial ecosystem is predominantly carried out by symbiotic
microbes, whereas bulk of fixation in aquatic ecosystems is done by free-living microbes.
a) 1 and 2
IASbabas Prelims 60 Day Plan Day 6 2017
b) Only 2
c) 2 and 3
d) None of the above
Did you know that tropical soils are among the deepest in the world! Many tropical soils
have been under forest cover for millions of years. Over this period, and under high rainfall
conditions, deep tropical soils have been formed from the underlying rock.
Tropical soils are often several metres deep, but the soils are often washed out, or
strongly leached, with large amounts of nutrients and minerals being removed from the
subsoils and considerable thickness of rock broken down to produce soil. Over many
millions of years this leaching has left most of the soils lacking many of the fundamental
nutrients needed by the above ground vegetation.
So how does such a lush vegetation exist if the soils are so depleted of nutrients? The
answer lies in the very thin topsoils, made up mainly of decaying vegetal and animal
remains. An amazing cycle exists between the huge body of vegetation above ground and
this thin topsoil. The rainforest depends for its nutrients on the constant recycling of its
enormous biomass.
Millions of years of weathering and torrential rains have washed most of the nutrients out
of the soil. More recent volcanic soils, however, can be very fertile. Tropical rain forest soils
contain less organic matter than temperate forests and most of the available nutrients are
found in the living plant and animal material. Nutrients in the soil are often in forms that are
not accessible by plants.
1. The respiration cost decreases along the successive higher tropic levels.
2. The length of food chains in an ecosystem is limited because of 10% law.
a) 1 Only
b) 2 Only
c) Both
d) None
IASbabas Prelims 60 Day Plan Day 6 2017
Respiration cost will increase sharply along successive higher tropic levels.
The efficiency with which energy or biomass is transferred from one trophic level to the next
is called the ecological efficiency.
Consumers at each level convert on average only about 10% of the chemical energy in their
food to their own organic tissue (the ten-percent law). For this reason, food chains rarely
extend for more than 5 or 6 levels.
At the lowest trophic level (the bottom of the food chain), plants convert about 1% of the
sunlight they receive into chemical energy. It follows from this that the total energy
originally present in the incident sunlight that is finally embodied in a tertiary consumer is
about 0.001%.
a) 1 Only
b) 2 Only
c) Both
d) None
Ecological succession is the gradual process by which ecosystems change and develop over
time. Nothing remains the same and habitats are constantly changing. There are two main
types of succession, primary and secondary.
Primary succession is the series of community changes which occur on an entirely new
habitat which has never been colonized before. For example, a newly quarried rock face or
sand dunes.
Secondary succession is the series of community changes which take place on a previously
colonized, but disturbed or damaged habitat. For example, after felling trees in a woodland,
land clearance or a fire.
IASbabas Prelims 60 Day Plan Day 6 2017
Simply put, secondary succession is the ecological succession that occurs after the initial
succession has been disrupted and some plants and animals still exist. It is usually faster
than primary succession as:
1. Protozoa
2. Mites
3. Lice
4. Ticks
a) 1, 2 and 3
b) 2, 3 and 4
c) 1, 3 and 4
d) All of the above
Parasitism is a non-mutual relationship between species, where one species, the parasite,
benefits at the expense of the other, the host.
Parasites are classified based on their interactions with their hosts and on their life cycles.
An obligate parasite is totally dependent on the host to complete its life cycle, while
a facultative parasite is not. A direct parasite has only one host while an indirect parasite
has multiple hosts. For indirect parasites, there will always be a definitive host and an
intermediate host.
Parasites that live on the outside of the host, either on the skin or the outgrowths of the
skin, are called ectoparasites (e.g. Ticks, lice, fleas, and some mites).
Those that live inside the host are called endoparasites (including all parasitic worms).
Endoparasites can exist in one of two forms: intercellular parasites (inhabiting spaces in the
host's body) or intracellular parasites (inhabiting cells in the host's body). Intracellular
parasites, such as protozoa, bacteria or viruses, tend to rely on a third organism, which is
generally known as the carrier or vector. The vector does the job of transmitting them to the
IASbabas Prelims 60 Day Plan Day 6 2017
An epiparasite is one that feeds on another parasite. This relationship is also sometimes
referred to as hyperparasitism, exemplified by a protozoan (the hyperparasite) living in the
digestive tract of a flea living on a dog.
Social parasites take advantage of interactions between members of social organisms such
as ants, termites, and bumblebees.
a) Only 1
b) Only 2
c) Both
d) None
Symbiotic nitrogen fixation occurs in plants that harbor nitrogen-fixing bacteria within their
tissues. The best-studied example is the association between legumes and bacteria in the
genus Rhizobium.
Each of these is able to survive independently (soil nitrates must then be available to the
legume), but life together is clearly beneficial to both. Only together can nitrogen fixation
take place. A symbiotic relationship in which both partners benefits is called mutualism.
The fungi provide protection to algae and the algae prepares food for the fungi, thereby
exhibiting mutually beneficial relationship.
1. The interaction where one species is benefitted and the other is neither benefitted
nor harmed is called amensalism
2. In commensalism on the other hand one species is harmed whereas the other is
unaffected.
a) Only 1
b) Only 2
c) Both
d) None
1. The topmost zone near the shore of a lake or pond is the littoral zone.
2. The near-surface open water surrounded by the littoral zone is the profundal zone.
3. The deep-water part of the lake or pond is the limnetic zone.
a) 1 and 2
b) 1 Only
c) 2 and 3
d) 3 Only
The aquatic biome can be broken down into two basic regions, freshwater (i.e, ponds and
rivers) and marine (i.e, oceans and estuaries).
Freshwater Regions
Freshwater is defined as having a low salt concentrationusually less than 1%. Plants and
animals in freshwater regions are adjusted to the low salt content and would not be able to
survive in areas of high salt concentration (i.e, ocean). There are different types of
freshwater regions: ponds and lakes, streams and rivers, and wetlands. The following
sections describe the characteristics of these three freshwater zones.
IASbabas Prelims 60 Day Plan Day 6 2017
These regions range in size from just a few square meters to thousands of square
kilometers. Scattered throughout the earth, several are remnants from the Pleistocene
glaciation. Many ponds are seasonal, lasting just a couple of months (such as sessile pools)
while lakes may exist for hundreds of years or more. Ponds and lakes may have limited
species diversity since they are often isolated from one another and from other water
sources like rivers and oceans. Lakes and ponds are divided into three different zones
which are usually determined by depth and distance from the shoreline.
The topmost zone near the shore of a lake or pond is the littoral zone. This zone is the
warmest since it is shallow and can absorb more of the Suns heat. It sustains a fairly diverse
community, which can include several species of algae (like diatoms), rooted and floating
aquatic plants, grazing snails, clams, insects, crustaceans, fishes, and amphibians. In the case
of the insects, such as dragonflies and midges, only the egg and larvae stages are found in
this zone. The vegetation and animals living in the littoral zone are food for other creatures
such as turtles, snakes, and ducks.
The near-surface open water surrounded by the littoral zone is the limnetic zone. The
limnetic zone is well-lighted (like the littoral zone) and is dominated by plankton, both
phytoplankton and zooplankton. Plankton are small organisms that play a crucial role in the
food chain. Without aquatic plankton, there would be few living organisms in the world, and
certainly no humans. A variety of freshwater fish also occupy this zone.
Plankton have short life spanswhen they die, they fall into the deep-water part of the
lake/pond, the profundal zone. This zone is much colder and denser than the other two.
Little light penetrates all the way through the limnetic zone into the profundal zone. The
fauna are heterotrophs, meaning that they eat dead organisms and use oxygen for cellular
respiration.
Temperature varies in ponds and lakes seasonally. During the summer, the temperature can
range from 4 C near the bottom to 22 C at the top. During the winter, the temperature at
the bottom can be 4 C while the top is 0 C (ice). In between the two layers, there is a
narrow zone called the thermocline where the temperature of the water changes rapidly.
During the spring and fall seasons, there is a mixing of the top and bottom layers, usually
due to winds, which results in a uniform water temperature of around 4 C. This mixing also
circulates oxygen throughout the lake. Of course there are many lakes and ponds that do
not freeze during the winter, thus the top layer would be a little warmer.
a) Bhil Uprising
IASbabas Prelims 60 Day Plan Day 6 2017
b) Khasi Uprising
c) Ahoms Revolt
d) Chuar Uprising
The Khasi uprising took place in 1833 in the regions between the Khasi hills and Jaintia Hills,
against a planned British Road in that area. The leader of this uprising was Tirot Sing.
Tirot Sings contribution happened years before even before the first war of Independence.
He was also hanged a century before Bhagat Singh and others were hanged. This is an
indication of the fact that Indias freedom cry began from Meghalaya long before Gandhi
arrived on the scene.
U Tirot Sing, U Kiang Nangbah and Pa Togan Sangma were among the three prominent
freedom fighters in Meghalaya and whose legendary battles against the British do not find a
mention in history text books.
Source: http://www.thehindu.com/todays-paper/tp-in-school/Honouring-the-
martyrs/article14569990.ece
a) 2, 3 and 4
b) 1, 3 and 4
c) 1, 2 and 3
d) All of the above
Croatia is located in Central and Southeast Europe, bordering Hungary to the northeast,
Serbia to the east, Bosnia and Herzegovina to the southeast, Montenegro to the southeast,
the Adriatic Sea to the southwest and Slovenia to the northwest.
The SFR Yugoslavia was a conglomeration of eight federated entities, roughly divided along
ethnic lines, including six republicsSlovenia, Croatia, Bosnia and Herzegovina, the Republic
of Macedonia, Montenegro and Serbiaand two autonomous provinces within Serbia,
Vojvodina and Kosovo.
Croatia is an Eastern European country with a long coastline on the Adriatic Sea.
Encompassing more than a thousand islands, it's also crossed by the Dinaric Alps. Its inland
capital, Zagreb, is distinguished by its medieval Gornji Grad (Upper Town) and diverse
museums. The major coastal city Dubrovnik has massive 16th-century walls encircling an Old
Town with Gothic and Renaissance buildings.
Source: http://pib.nic.in/newsite/PrintRelease.aspx?relid=148705
IASbabas Prelims 60 Day Plan Day 6 2017
a) 1 and 3
b) 1 and 2
c) 2 and 3
d) All of the above
Maglev is a transport method that uses magnetic levitation to move vehicles without
touching the ground.
A vehicle can travel along a guideway using magnets to create both lift and propulsion,
thereby reducing friction by a great extent and allowing high speeds.
Magnets on the track push and lift the train up in the air by about 1 inch to 6 inches.
These "track" magnets are controlled by computers which keep shifting forward the
magnetic force of the "track" magnets so that the vehicle is pulled forward.
Meissner effect - the expulsion of magnetic flux when a material becomes superconducting
in a magnetic field.
Source: http://www.business-standard.com/article/economy-policy/indian-railways-to-
run-magnetic-trains-soon-116081500795_1.html
Q.19) Hairatan was in news recently. It is located in which of the following countries?
a) Uzbekistan
b) Afghanistan
c) Mongolia
d) Turkmenistan
IASbabas Prelims 60 Day Plan Day 6 2017
It was in news wrt. Chinas first cargo train service to Afghanistan as part of OBOR.
Source: http://www.thehindu.com/news/international/Silk-Road%E2%80%99-train-to-
reach-Afghanistan-on-Sept.-9/article14594743.ece
1. BARC has recently developed a simple, user friendly, quick and cost effective kit for
onsite determination of this 'element' and to check its water-contamination
capacity.
2. The element is widely used in various industries like leather, steel, chrome plating,
paint manufacturing, wood preservation etc.
3. One of its types is toxic and the World Health Organization has classified it as
carcinogenic and has mentioned that it can cause stomach ulcers and cancers, and
severe damage to kidneys and liver.
a) Cadmium
b) Iron
c) Chromium
d) Beryllium
Chromium
Chromium is widely used in various industries like leather, steel, chrome plating,
paint manufacturing, wood preservation etc.
Untreated effluents from these industries cause widespread contamination of water
as been reported in several parts of the country.
Chromium in the environment primarily exists as Trivalent Chromium Cr (III) and
Hexavalent Chromium Cr(VI). The later is toxic and the World Health Organization
has classified it as carcinogenic that also causes stomach ulcers, and cancers and
severe damage to kidneys and liver.
IASbabas Prelims 60 Day Plan Day 6 2017
As per Indian standard IS10500 for drinking water, the maximum permissible
concentration of Cr (VI) in drinking water is 50 microgram per litre. The US
Environmental Protection Agency (EPA) recommends a still lower permissible
concentration of 10 microgram per litre.
Detection of Cr (VI) -
At such low levels it is not only technically challenging but also expensive and time
consuming since it involves collection of water samples from affected areas,
transportation to laboratory, storage and finally analysis. The method can be used
for limited water samples with errors due to conversion of Cr (III) to Cr(VI) and vice
versa during transport and storage.
BARC -
BARC has developed a simple, user friendly, quick and cost effective kit for onsite
determination of Cr (VI), which meets IS10500 as well as EPA criterion.
It provides the much needed solution to measure the level of Chromium
contamination in drinking water and tap water, lakes, rivers as well as ground water.
Procedure involves adding a specified amount of specific reagents to the water
sample and identifying the developed colour.
Colour chart -
Colour develops within 5 minutes and the distinction can be made with naked eye.
For ease of comparison a colour chart is provided with the kit.
Water samples can be immediately categorized as being safe or toxic for drinking
from Chromium (VI) point of view.
Kit provides several advantages including onsite detection and instantaneous results,
elimination of use of sophisticated instruments for analysis, low investment on
infrastructure for production of the kit, easy availability of raw materials and very
good accuracy for the intended purpose.
Source: http://pib.nic.in/newsite/PrintRelease.aspx?relid=148984
a) 1 and 2
b) 2 and 3
c) 1 and 3
d) All of the above
Ethanol blending is the practice of blending petrol with ethanol. Many countries, including
India, have adopted ethanol blending in petrol in order to reduce vehicle exhaust emissions
and also to reduce the import burden on account of crude petroleum from which petrol is
produced.
OMCs sell ethanol blended petrol with percentage of ethanol upto 10%. In 2015, the
Government has asked OMCs to target 10 % blending of ethanol in as many States as
possible.
Ethanol blending first found mention in the Auto fuel policy of 2003. It suggested developing
technologies for producing ethanol/ bio fuels from renewable energy sources and
introducing vehicles to utilise these bio fuels.
Source: http://pib.nic.in/newsite/PrintRelease.aspx?relid=148302
Q.22) Which of the following statements about Totaliser machines is/are correct?
a) Only 1
b) Only 2
c) Both 1 and 2
d) None of the above
IASbabas Prelims 60 Day Plan Day 6 2017
Current Counting
Votes from each EVM are counted separately and thus it reveals the voting trends in
each polling station.
Voters in that vicinity open to harassment, intimidation and post-election
victimisation
Totaliser Machines
Source:
http://www.thehindu.com/news/national/centre-moves-on-vote-totaliser-
machines/article9046763.ece
http://www.thehindu.com/news/national/election-commission-for-new-machine-to-
enhance-voter-secrecy/article7065217.ece
The All India Radio (AIR) during the Bangladesh Liberation Movement in 1971 had launched
Akashvani Maitree, a Bangla radio service. The channel had played a historic role during the
Bangladesh liberation movement. However, it was discontinued in 2010.
Source: http://indianexpress.com/article/india/india-news-india/president-pranab-
mukherjee-launches-akashvani-maitree-channel-2992012/
List-1 List-2
A-B-C-D
a) 3-4-1-2
b) 1-2-3-4
c) 4-1-3-2
d) 2-3-4-1
Self-explanatory
1. Nastoc
2. Spirogyra
3. Anabaena
4. Aulosira
a) 1, 2 and 3
b) 2, 3 and 4
c) 1, 3 and 4
d) 1, 2, 3 and 4
IASbabas Prelims 60 Day Plan Day 6 2017
Nastoc, Anabaena and Aulosira are the most important nitrogen-fixation cynobacteria.
Kabir was born to a Brahmin widow, and was brought up by a Muslim weaver. He preached
using Dohas (couplets) and sakhi (Poems), which were very popular among the common
people. He believed in pantheism i.e. God is everywhere. He pleaded for Hindu-Muslim
unity and peaceful co-existence. He emphasized unity and Formlessness of god (Nirguna).
He believed that asceticism and book knowledge are not necessary for true knowledge and
salvation.
a) 1 only
b) 2 only
c) 3 only
d) All of the above
Guru Nanak was the founder of Sikhism. He was followed by nine Sikh gurus. Guru Angad
started the Gurumukhi Script. Guru Amardas the third Guru began the institution of Langar
(common kitchen). Guru Ramdas the fourth guru was given Land for Harmandir by Akbar.
Guru Arjun Compiled the Adi Granth and completed the construction of Har-mandir. Guru
Gobind Singh the tenth guru started the Khalsa (organized military sect).
Q.3) Consider the following statements with respect to the tax revenue of Mughal empire.
a) 1 and 3 only
b) 2 and 3 only
c) 2 and 4 only
d) 1 and 4 only
During the reign of Shah Jahan, the Jama was the Stipulated/ assessed land revenue from a
jagir and the Hasil was the actual land revenue collected from the Jagir land.
Source: A New Look at the Modern Indian History by B.L. Grover and Alka Mehta.
Q.4) Consider the following statements with reference to the Jagir System.
a) 1 and 3 only
b) 2 and 3 only
c) 1 and 2 only
d) None of the above.
Jagirdari system was a part of the Umbrella Mansabdari System. Jagir or Tuyul was the Unit
of land given to the soldiers in lieu of salary, the revenue from the Jagir was to be the salary
for the mansabdar and the soldiers.
The land given in lieu for military service was called Tankhwa jagir and the hereditary land
given to mansabdars was called Watan Jagir, they were not hereditary in the sense that the
revenues could be used only as long as they remained in service, the emperor could re-
assign the Jagirs to anyone at any time.
All Jagirdars were mansabdars, but all mansabdars were not Jagirdars as some of them were
paid in cash.
IASbabas Prelims 60 Day Plan Day 7 2017
Q.5) Which of the following events did not take place during the Governorship of William
Bentinck?
Source: A New Look at the Modern Indian History by B.L. Grover and Alka Mehta.
Q.6) Consider the following statements regarding the Regulating Act of 1773.
1. It changed the constitution of Court of Directors and subjected their actions to the
British Government.
2. It appointed the First Governor-General of India.
3. It prohibited the Company servants from receiving gifts and bribes.
a) 1 and 2 only
b) 2 and 3 only
c) 1 and 3 only
d) All of the above.
Changed the constitution of the Court of Directors, eligibility and right to vote and made
their actions accountable to the British Government. The office of Governor General of
Bengal was made with a council of four to help. A Supreme Court was constituted in
IASbabas Prelims 60 Day Plan Day 7 2017
Calcutta with Elijha Impey as the Chief Justice. It prohibited the servants of the company
from receiving any presents, gifts and bribes.
Source: A New Look at the Modern Indian History by B.L. Grover and Alka Mehta.
a) 1 and 2 only
b) 2 and 3 only
c) 1 and 3 only
d) 2 and 4 only
The India Council was constituted under the Government of India Act of 1858. It was headed
by the Secretary of States for India. Since the Court of Directors was dissolved by the act
there were no nominees of the Court of Directors.
Source: A New Look at the Modern Indian History by B.L. Grover and Alka Mehta.
Q.8) Which of the following statements is not true regarding the provisions of Indian
Councils Act 1909?
The Act amended the Indian Councils Act 1861 and the Indian Councils Act 1892:
1. The members of the Legislative Councils, both in the centre and in the provinces, were to
be of four categories: ex officio members (Governor General and the members of their
Executive Councils), nominated official members (those nominated by the Governor General
and were government officials), nominated non-official members (nominated by the
IASbabas Prelims 60 Day Plan Day 7 2017
Governor General but were not government officials) and elected members (elected by
different categories of Indian people).
2. The maximum number of nominated and elected members of the Legislative Council at
the Center was increased from 16 to 69, excluding ex officio members.
3. The maximum number of nominated and elected members of the provincial legislative
councils, under a governor or lieutenant governor, was also increased. It was fixed as 50
in Bengal, Bombay, Madras, United Provinces, and Eastern Bengal and Assam, and 30
in Punjab, Burma, and any lieutenant-governor province created thereafter. Legislative
councils were not created for provinces under a chief commissioner.
5. Official members were to form the majority but in provinces, nonofficial members would
be in majority.
6. The members of the Legislative Councils were permitted to discuss budgets, suggest
amendments and even vote on them except items that were included as non-vote items.
They were also entitled to ask supplementary questions during the legislative proceedings.
7. The Secretary of State for India was empowered to increase the number of the Executive
Councils of Madras and Bombay from two to four.
8. Two Indians were nominated to the Council of the Secretary of State for Indian Affairs.
9. The Governor-General was empowered to nominate one Indian member to his Executive
Council.
Source: Source: A New Look at the Modern Indian History by B.L. Grover and Alka Mehta.
Q.9) Which of the following statements regarding The Charter Act of 1833 is incorrect?
It ended the activities of the British East India Company as a commercial body and became a
purely administrative body. In particular,the company lost its monopoly in China and also
the trade of tea which it enjoyed with Charter act of 1813.
The Governor-General in council was given the power to control, superintend and direct the
civil and military affairs of the Company. Central government was to have complete control
over raising of revenues and expenditure. i.e. All financial and administrative powers were
centralized in the hands of Governor General-in-Council.
The number of the members of the Governor Generals council was again fixed to 4, which
had been reduced by the Pitts India act to 3. However, certain limits were imposed on the
functioning of the 4th member.
The 4th member was NOT entitled to act as a member of the council except for legislative
purposes.
First fourth person to be appointed as the member of the Council was Lord Macaulay.
It deprived the Governors of Bombay and Madras of their legislative powers. The Governor-
General was given exclusive legislative powers for the whole of British India.
The section 87 of the Charter Act of 1833, declared that merit was to be the basis for
employment in Government Services and the religion, birth place, and race of the
candidates were not to be considered in employment.
This policy was not seen in any other previous acts. So the Charter act of 1833 was the first
act which provisioned to freely admit the natives of India to share an administration in the
country.
a) 1 and 2 only
b) 1 and 3 only
c) 2 and 3 only
d) None of the above
Permanent Settlement:
Permanent Settlement was introduced in Bengal and Bihar in 1793 by Lord Cornwallis. It had
the following features:
The reminders and revenue collectors were converted into so many landlords. They were
not only to act as agents of the Government in collecting land revenue from the ryot, but
also to become the owners of the entire land (over which they were collecting revenue).
Their right of ownership was made hereditary and transferable.
On the other hand, the cultivators were reduced to the low status of mere tenants and were
deprived of long-standing rights to the soil and other customary rights.
The use of the pasture and forest lands, irrigation canals, fisheries, and homestead plots and
protection against enhancement of rent were some of the cultivators rights which were
sacrificed.
In fact the tenancy of Bengal was left entirely at the mercy of the zamindars. This was done
so that the zamindars might be able to pay in time the exorbitant land revenue demand of
the Company.
The zamindars were to give 10/11th of the rental they derived from the peasantry to the
state, keeping only 1/11th for themselves. But the sums to be paid by them as land revenue
were fixed in perpetuity.
At the same time, the zamindar had to pay his revenue rigidly on the due date even if the
crop had failed for some reason; otherwise his lands were to be sold.
Ryotwari Settlement:
The establishment of British rule in South and South-Western India brought new problems
of land settlement. The officials believed that in these regions there were no zamindars with
large estates with whom settlement of land revenue could be made and that the
introduction of zamindari system would upset the existing state of affairs.
IASbabas Prelims 60 Day Plan Day 7 2017
Many Madras officials led by Reed and Munro recommended that settlement should
therefore be made directly with the actual cultivators.
The system they proposed, is known as the Ryotwari Settlement, under which the cultivator
was to be recognized as the owner of his plot of land subject to the payment of land
revenue.
The supporters of the Ryotwari Settlement claimed that it was a continuation of the state of
affairs that had existed in the past.
Munro said: "It is the system which has always prevailed in India".
The Ryotwari Settlement was introduced in parts of the Madras and Bombay Presidencies in
the beginning of the 19th century.
The settlement under the Ryotwari system was not made permanent. It was revised
periodically after 20 to 30 years when the revenue demand was usually raised.
Mahalwari System:
A modified version of the zamindari settlement, introduced in the Gangetic valley, the
North-West Provinces, parts of Central India, and Punjab, was known as
the Mahalwari System.
The revenue settlement was to be made village by village or estate (mahal) by estate with
landlords or heads of families who collectively claimed to be the landlords of the village or
the estate.
In Punjab, a modified Mahalwari System known as the village system was introduced.
In Mahalwari areas also, the land revenue was periodically revised.
Both the Zamindari and the Ryotwari systems, departed fundamentally from the traditional
land systems of the country.
The British created a new form of private property in land in such a way that the benefit of
the innovation did not go to the cultivators.
All over the country, the land was now made salable, mortgagable, and alienable. This was
done primarily to protect the Government's revenue.
If land had not been made transferable or salable, the Government would find it very
difficult to realize revenue from a cultivator who had no savings or possessions out of which
to pay it.
The British by making land a commodity which could be freely bought and sold introduced a
fundamental change in the existing land systems of the country. The stability and the
continuity of the Indian villages were shaken, in fact, the entire structure of the rural society
began to break up.
IASbabas Prelims 60 Day Plan Day 7 2017
Q.11) By which act did education come under the control of Indians for the first time?
The Government of India Act of 1919 introduced Dyarchy in the Provinces according to
which the subjects were divided as reserved and transferred subjects. The "transferred list",
were given to a Government of ministers answerable to the Provincial Council. The
'transferred list' included agriculture, supervision of local government, health, and
education
Source: A New Look at the Modern Indian History by B.L. Grover and Alka Mehta.
Q.12) Consider the following statements about the Government of India Act of 1919.
a) 1 and 3 only
b) 2 and 4 only
c) 1 and 2 only
d) 3 and 4 only
In line with the government policy contained in Montagus statement (August 1917), the
Government announced further constitutional reforms in July 1918, known as Montagu-
Chelmsford or Montford Reforms.
Based on these, the Government of India Act, 1919 was enacted. The main features of the
Montford Reforms were as follows.
(i) Dyarchy, i.e., rule of twoexecutive councillors and popular ministerswas introduced.
The governor was to be the executive head in the province.
(ii) Subjects were divided into two lists: reserved which included subjects such as law and
order, finance, land revenue, irrigation, etc., and transferred subjects such as education,
health, local government, industry, agriculture, excise, etc.
The reserved subjects were to be administered by the governor through his executive
council of bureaucrats, and the transferred subjects were to be administered by ministers
nominated from among the elected members of the legislative council.
(iii) The ministers were to be responsible to the legislature and had to resign if a no-
confidence motion was passed against them by the legislature, while the executive
councilors were not to be responsible to the legislature.
(iv) In case of failure of constitutional machinery in the province the governor could take
over the administration of transferred subjects also.
(v) The secretary of state and the governor-general could interfere in respect of reserved
subjects while in respect of the transferred subjects; the scope for their interference was
restricted.
Legislature:
(i) Provincial Legislative Councils were further expanded70% of the members were to be
elected.
(ii) The system of communal and class electorates was further consolidated.
(iv) The Legislative Councils could initiate legislation but the governors assent was required.
The governor could veto bills and issue ordinances.
(v) The Legislative Councils could reject the budget but the governor could restore it, if
necessary.
Executive:
(v) The Governor-General could restore cuts in grants, certify bills rejected by the Central
Legislature and issue ordinances.
Legislature:
(i) A bicameral arrangement was introduced. The lower house or Central Legislative
Assembly would consist of 144 members (41 nominated and 103 elected52 General, 30
Muslims, 2 Sikhs, 20 Special) and the upper house or Council of State would have 60
members (26 nominated and 34 elected20 General, 10 Muslims, 3 Europeans and 1 Sikh).
(ii) The Council of State had tenure of 5 years and had only male members, while the Central
Legislative Assembly had tenure of 3 years.
(iii) The legislators could ask questions and supplementaries pass adjournment motions and
vote a part of the budget, but 75% of the budget was still not votable.
(iv) Some Indians found their way into important committees including finance.
Source: Source: A New Look at the Modern Indian History by B.L. Grover and Alka Mehta.
Organization Founder
a) 1 and 2 only
b) 2 only
c) 3 only
d) 1 and 3 only
M.G. Ranade: In 1887 he founded the Indian National Social Conference, whose meetings
were concurrent with the annual Congress sessions. In 1890 he inaugurated the Industrial
Association of Western India, having come to the conclusion that a constructive solution to
India's problems lay in a vigorous policy of industrial and commercial development.
IASbabas Prelims 60 Day Plan Day 7 2017
G. K. Gokhale: The Servants of India Society was formed in Pune, Maharashtra, on June 12,
1905 by Gopal Krishna Gokhale, who left the Deccan Education Society to form this
association. Along with him were a small group of educated Indians, as Natesh Appaji
Dravid, Gopal Krishna Deodhar and Anant Patwardhan who wanted to promote social and
human development and overthrow the British rule in India. The Society organized many
campaigns to promote education, sanitation, health care and fight the social
evils of untouchability and discrimination, alcoholism, poverty, oppression of women and
domestic abuse. The publication of The Hitavada, the organ of the Society in English
from Nagpur commenced in 1911.
N. M. Joshi: in 1911, N. M. Joshi started the Social service league to work for the welfare of
society, it established many schools, clinics and libraries, He
Q.14) Which of the following statements with reference to the Wardha Education Scheme
is incorrect?
In 1937, Gandhiji in a series of articles in the Harijan discussed his ideas about the problem
of education in India and wanted that education should be given through a profit yielding
vocation, so that it could become self-supporting.
The All India Educational Conference held at Wardha in October, 1937 discussed the ideas of
Gandhiji and passed the following resolutions:
I. Provision of Free and Compulsory Education for seven year on a nationwide scale;
II. Relating education with the manual and productive work and some form of handicraft;
A committee was formed under the Chairmanship of Dr. Zakir Hussain to prepare a scheme
of education keeping in view the suggestions of the resolution. Gandhiji formulated his
scheme of education in the Indian context and it was the outcome of his long experience.
The Zakir Hussain Committee after analyzing each aspect of the new scheme submitted its
report. The Congress Ministry evinced interest for the implementation of the new scheme.
But unfortunately the scheme of education could not be properly worked out, as the
Congress remained in power only for a short period. Owing to the outbreak of the Second
IASbabas Prelims 60 Day Plan Day 7 2017
World War in 1939, and protesting against the Governments policy. Congress ministries
tendered resignations. Thus, the rising tide of educational expansion was temporarily
suspended.
Source: http://www.yourarticlelibrary.com/law/acts/all-india-educational-conference-
wardha-1937/44828/
1. 1 and 2 only
2. 1 and 3 only
3. 2 and 3 only
4. All of the above.
In 1814 Rammohun came and settled in Calcutta and in 1815 founded the Atmiya Sabha - an
association for the dissemination of the religious truth and the promotion of free
discussions of theological subjects. Amongst the rich and influential who gathered around
him at that time were Prince Dwarkanath Tagore of Jorasanko, Prasanna Kumar Tagore of
Pathuriaghata, Kali Nath and Baikuntha Nath Munshi of Taki, Raja Kali Shankar Ghoshal of
Bhukailash etc. But the meetings of the Sabha were not the only means to propagate his
doctrines. Here recital and expounding of Hindu scriptures were done and Govinda Mala
would sing songs composed by Rammohun. In 1819 there took place a celebrated debate
between Rammohun and Subrahmanya Sastri on the subject of idol worship in presence of
the leading citizens of Calcutta including Raja Radhakanta Deb and Rammohun vanquished
his adversary.
Source: http://www.thebrahmosamaj.net/founders/rammohun.html
a) 1 and 2 only
b) 1 and 3 only
c) 2 and 3 only
d) All of the above
Jyotiba Phule was born in low caste and waged a life-long war against Caste system and
Brahmanical supremacy. He founded the Satyashodk Samaj in 1873. He pioneered the
widow remarriage movement in Maharashtra and worked for the education of women.
Jyotibas quest for providing women and girls with right to education was supported by his
wife Savitribai Phule. One of the few literate women of the time, Savitribai was taught to
read and write by her husband Jyotirao. In 1851, Jyotiba established a girls' school and
asked his wife to teach the girls in the school. Later, he opened two more schools for the
girls and an indigenous school for the lower castes, especially for the Mahars and Mangs.
Source: http://www.culturalindia.net/reformers/jyotiba-phule.html
1. It will be launched in the districts having the lowest total fertility rates (TFR) in the
country
2. It will promote family planning by discouraging parents to have more than 2 children
a) Only 1
b) Only 2
c) Both 1 and 2
d) Neither 1 nor 2
Ministry of Health and Family Welfare will soon launch Mission Parivar Vikas in 145 high
focus districts having the highest total fertility rates in the country. These 145 districts are
in the seven high focus, high TFR states of Uttar Pradesh, Bihar, Rajasthan, Madhya
Pradesh, Chhattisgarh, Jharkhand and Assam that constitute 44% of the countrys
population. The main objective of Mission Parivas Vikas will be to accelerate access to high
IASbabas Prelims 60 Day Plan Day 7 2017
quality family planning choices based on information, reliable services and supplies within a
rights-based framework.
These 145 districts have been identified based on total fertility rate and service delivery
(PPIUCD and Sterilization performance) for immediate, special and accelerated efforts to
reach the replacement level fertility goals of 2.1 by 2025.
India has specifically been struggling with these seven States, where health and education
standards lag behind the rest of the country. High population density, coupled with low
levels of education and poor infrastructure have resulted in these States being at the
forefront of a number of health concerns, including high maternal and infant mortality, high
incidents of infectious diseases and others.
The key strategic focus of this initiative will be on improving access to contraceptives
through delivering assured services, dovetailing with new promotional schemes, ensuring
commodity security, building capacity (service providers), creating an enabling environment
along with close monitoring and implementation.
Source: http://www.thehindu.com/news/national/kerala/Mission-on-family-planning-to-
focus-on-145-backward-districts/article14996361.ece
Q.18) Consider the following statements about Higher Education Financing Agency (HEFA)
1. Ministry of Human Resources Development (MHRD) has appointed Canara Bank for
establishing the HEFA
2. It would be a special purpose vehicle
3. It will mobilise CSR funds from PSUs/Corporates, which would in turn be released for
promoting research and innovation in these institutions on grant basis
a) 1 and 2
b) 2 and 3
c) 1 and 3
d) All of the above
To enable institutions of higher education in the country achieve financial autonomy, fund
more research and improve infrastructure, the Union government on Thursday signed an
agreement with Canara Bank to set up a non-banking financial company (NBFC).
IASbabas Prelims 60 Day Plan Day 7 2017
Called the Higher Education Finance Agency (HEFA), the joint venture between the human
resource development (HRD) ministry and the bank will start lending to institutions from
May-June 2017.
Canara Bank was selected through an open bidding system after the HRD ministry invited
bids from financial institutions in November-December to partner it in the setting up of
HEFA.
HEFA will provide an alternative source of funding to institutions to meet their financial
requirements. It will also push institutions to become financially self-sustaining and not
depend on the government for all their growth requirements.
The HEFA would be a special purpose vehicle. It would leverage the equity to raise up to
20,000 crore for funding projects for infrastructure and development of world class Labs in
IITs/IIMs/NITs and such other institutions.
It would also mobilise CSR funds from PSUs/corporates, which would in turn be released for
promoting research and innovation in these institutions on grant basis.
The HEFA would finance the civil and lab infrastructure projects through a 10-year loan. The
principal portion of the loan will be repaid through the internal accruals (earned through
the fee receipts, research earnings etc.,) of the institutions. The government would service
the interest portion through the regular Plan assistance.
Canara Bank is also the nodal bank for administering the Central scheme to provide interest
subsidy during the period of moratorium on educational loans for students belonging to
economically weaker sections (with parental income from all sources up to 4.5 lakh per
annum).
The scheme is effective for all educational loans as per Indian Banks Associations model
education loan scheme, sanctioned to eligible students in respect of courses in technical and
professional streams from recognised institutions in India, disbursed from the academic
year 2009-10.
Source: http://pib.nic.in/newsite/PrintRelease.aspx?relid=149707
Source: http://economictimes.indiatimes.com/news/economy/indicators/india-on-path-
of-becoming-pivot-for-hi-tech-manufacturing-un/articleshow/54043459.cms
a) Manipur
b) Goa
c) Odisha
d) Nagaland
Source: http://www.thehindu.com/sci-tech/energy-and-environment/goa-to-host-
threeday-bird-festival-from-nov-11/article9066370.ece
Q.21) Which one of the following is the best description of INS Trikand that was in the
news recently?
INS Trikand (F51) is a Talwar-class frigate of the Indian Navy. he is the third and final ship of
the second batch of Talwar-class frigates ordered by the Indian Navy. She was built by the
Yantar shipyard in Kaliningrad, Russia. She was commissioned to Indian Navy service on 29
June 2013.
Ins Trikand has been deployed on a goodwill visit to East Africa and Southern Indian Ocean.
The ship was at Antsiranana from 31 Aug to 03 Sep 2016 to further bilateral ties with
Madagascar.
IASbabas Prelims 60 Day Plan Day 7 2017
Source: http://www.thehindu.com/news/national/INS-Trikand-hands-over-relief-
material-to-Madagascar/article14621952.ece
a) Only 1
b) Only 2
c) Both 1 and 2
d) Neither 1 nor 2
Jogi Art is tribal art form that hinges on lines and dots.
It is mostly done in black and whites; however the recent installation in the city of
Jaipur is of vibrant colours.
It is practised by artists from Magriwada in Reodar tehsil of Sirohi district in
Rajasthan.
Interestingly, now this tribal art form is just practised by a nuclear family
Reference - http://rajasthanpost.com/index.php/2015/10/03/open-your-hearts-to-the-
jogi/
Source: http://www.thehindu.com/news/cities/Delhi/exploring-jogi-
art/article8207783.ece
Q.23) Biswa Bidhya Sangraha is concerned with which of the following personalities
From science, history, geography, art, industry and culture to folklores, the scope of the
series was wide. It was christened 'Biswa Bidya Sangraha' and was published in 135
volumes. For decades, the collection -replete with original writings of doyens of that era -
remained out of print. VisvaBharati has now decided to bring all of them back.
Source: http://timesofindia.indiatimes.com/city/kolkata/V-B-to-revive-100-year-old-
mass-education-series-by-Tagore/articleshow/54320420.cms
Q.24) Recently Bratislava Declaration was in news concerning which of the following
groupings?
a) Visegrd Group
b) Western European and Others Group
c) European Union
d) G-20
The Bratislava Summit of 27 Member States has been devoted to diagnose together the
present state of the European Union and discuss our common future.
The summit was held in Bratislava to discuss the issues after exit of Britain from European
Union. Except Britain all other EU members were participated in the summit. Currently EU is
facing existential crisis, and exit of Britain from EU is the latest symptom of the fear and
discontent that have spread across the continent, fuelled by the migrant crisis,
Islamophobia, Eurozone woes and terrorist attacks. The summit was held to diagnose the
situation and to forge a united path forward.
The Bratislava Declaration offers a road map for European Union countries for the next six
months, on the issues like migration, border security, counter-terrorism, defence and
economic and social development etc.
The Bratislava Declaration identifies various areas where much needed action required,
such as funding for strategic investments across the region, establishing a common capital
market across the EU and acquiring advanced traveller information to secure borders.
IASbabas Prelims 60 Day Plan Day 7 2017
Bratislava Declaration - http://www.consilium.europa.eu/en/press/press-
releases/2016/09/16-bratislava-declaration-and-roadmap/
Source: http://www.thehindu.com/opinion/editorial/The-road-from-
Bratislava/article14988719.ece
IASbabas Prelims 60- Day Plan Day 8 2017
Q.1) Consider the following statements with reference to the Famine Commission of 1880.
a) 1 and 2 only
b) 2 and 3 only
c) 1 and 3 only
d) All of the above.
In 1880 the Governor General of India Lord Lytton appointed a Famine commission headed
by Sir Richard Starchey. It recommended providing employment to the able bodied persons,
and to provide relief to the physically infirmed, It suggested that the famine relief should be
primary responsibility of the Provincial governments with the centres help wherever
necessary.
Based on the recommendations of the committee, First Famine Code was formulated in
1883.
The second Famine Commission was appointed by Lord Curzon in 1901, after the draught of
1899-1900.
Source: A New Look At the Modern Indian History by B.L. Grover and Alka Mehta.
a) 1 and 2 only
b) 2 and 3 only
c) 1 and 3 only
d) None of the above
Mayo's Resolution of 1870The Indian Council of Act of 1861 introduced the policy of
legislative devolution and Mayo's Resolution of 1870 on financial decentralisation was its
likely consequence. Administrative suitability and financial severity instigated the Imperial
Government to reassign to the management of provincial governments specific
departments of administration, which, along with others, incorporated education, medical
services and roads. The Indian Council of Act of 1861 introduced the policy of legislative
devolution and Mayo's Resolution of 1870 on financial decentralisation was its likely
consequence. Administrative suitability and financial severity instigated the Imperial
Government to reassign to the management of provincial governments specific
departments of administration, which, along with others, incorporated education, medical
services and roads.
The Royal Commission upon Decentralisation was appointed in 1907 under the
chairmanship of Sir Henry William Primrose. It was a six-member body including the
Chairman, other five members being Frederic Lely, Steyning Edgerley, Romesh Chunder
Dutt, William Meyer and William Hichons.
The Royal Commission upon Decentralization in India was appointed on 12 September 1907
to inquire into the relations now existing for financial and administrative purposes between
the Supreme Government (i.e., the Government of India) and the various Provincial
Governments in India, and between the Provincial Governments and the authorities
subordinate to them and to report whether, by measures of decentralization or otherwise,
those relations can be simplified and improved, and the system of Government better
adapted both to meet the requirements and promote the welfare of the different provinces
and, without improving its strength and unity, to bring the executive power into closer
touch with local conditions. The Commission submitted its report in February 1909.
Source: A New Look At the Modern Indian History by B.L. Grover and Alka Mehta.
Q.3) Consider the following statements about the Deccan Riots of 1875.
1. It was a peasant uprising against the confiscation of tribal lands by the Government.
2. It was a peasant uprising against the Marwari and Gujrati money lenders.
The Deccan riots of 1875 were directed mainly against the Marwari and Gujrati
moneylenders.
A combination of excess land revenue demands by the government and the world cotton
prices slump due to the American Civil War had pushed the peasants into deep
indebtedness. The Moneylenders had confiscated the lands of the local peasants. The
Rioting peasants targeted the Bonds they had signed with the moneylenders, the entire
Poona district was ablaze in June 1875. The government had to call in the army to control
the situation.
Source: A New Look At the Modern Indian History by B.L. Grover and Alka Mehta.
Q.4) Which of the following events are not attributed to Curzons tenure Governor-
General of India?
Police Commission {Sir Andrew Frazer} Lord Curzon set up a Police Commission under Sir
Andrew Frazer in 1902-03. On the basis of recommendation of this commission, a basic
structure of Indian domestic intelligence developed and remained almost same till 1947. He
attached Department of Criminal Intelligence (DCI) to the Government of India while set up
the Criminal Investigation Departments (CIDs) in the provinces of British India. In 1903, the
Thugi and Dakaiti Department was abolished.
Raleigh Commission the Indian Universities and the colleges were slowly becoming cradle of
propaganda against the Government. To bring the universities under control, Lord Curzon
appointed Raleigh Commission under Sir Thomas Raleigh. The Raleigh Commission had only
one Indian member, Syed Hussain Belgrami. When Hindus protested about this, Justice Guru
Das Banerjee was called from the High Court of Calcutta and made a member. This
commission submitted its report in 1902 and this followed introduction of a Bill called
IASbabas Prelims 60- Day Plan Day 8 2017
Raleigh Bill. The Raleigh Bill when became an act, it was called Indian Universities Act 1904.
Indian Universities Act 1904 This Act reorganised the constitution of the Syndicates;
provided for the official inspection of the colleges and placed the final decision concerning
the affiliation and disaffiliation of colleges in the hands of the Government of India. This act
was severely criticized by scholars like Gopal Krishna Gokhle. The first provision of this act
was that the governing bodies of the universities were to be reconstituted and the size of
the Senates was reduced. Now the number in the senate could be minimum 50 and
maximum 100. Each of them would hold the office for 6 years. For the Universities of
Bombay, Calcutta and Madras, the elected fellows were to be 50 and for rest of the
universities, the number was fixed 15. This act allowed the Government to appoint a
majority of the fellows in a university. The Governor General was now empowered to decide
a Universitys territorial limits and also affiliation between the universities and colleges. The
Indian Universities Act made the universities and colleges completely under the
Government control. However, for better education and research a grant of Rs. 5 Lakh per
year for 5 years was also accepted. This was the beginning of university grants in India which
later became a permanent feature in the structure of India education.
Ancient Monuments Preservation Act 1904 one of the most remarkable acts passed during
the times of Lord Curzon was the Ancient Monuments Preservation Act of 1904. This act
made any injury to the protected monuments an offence punishable under the law. This act
also established the Archaeological Department which was to collect the historical
documents and importance, conduct excavations and bring the ancient historical
information into light.
Q.5) Who among the following was/were economic critic/critics of colonialism in India?
1. Dadabhai Naoroji
2. M.G. Ranade
3. R. C. Dutt
a) 1 and 3 only
b) 2 and 3 only
c) 1 and 2 only
d) All of the above.
Dadabhai Naoroji, R. C. Dutt, Ranade, Gokhale, G. Subramania Iyer, were among those who
grounded Indian nationalism firmly on the foundation of anti-imperialism by fashioning the
worlds first economic critique of colonialism, before Hobson and Lenin.
IASbabas Prelims 60- Day Plan Day 8 2017
The drain theory was established by Dadabhai Naoroji. Increased poverty and lower wages
were among the indirect products of colonial rule according to Dutt.
Q.6) Which of the following was/were the main objectives of Queen Victoria's
Proclamation (1858)?
a) 1 and 2 only
b) 2 and 3 only
c) 1 and 3 only
d) All of the above
The Company as a commercial entity ceased to exist after the Proclamation of 1858. Hence
statement (3) is incorrect. The Proclamation intended to create a group of local kingdoms
and provinces which could be used to safeguard the Indian territories if a revolt like 1857
happened again. This was the beginning of Policy of Subordinate Union which lasted till
1935. The Queens proclamation also transferred all the Companys territories and their
administration into the hands of British Government.
Q.7) Which of the following statements regarding The Ilbert Bill is correct?
The 'Ilbert Bill' was a bill introduced in 1883 under the reign of the Viceroy Lord Ripon (who
earned the epithet Ripon-The good) which was written by Sir C.P Ilbert (The law member of
the Viceroy's Council). According to the said Act, Indian judges could try a European
accused.
IASbabas Prelims 60- Day Plan Day 8 2017
This led to organised lobbying and opposition by all Europeans in India. And an amendment
was brought as a compromise. As per amendment- Indian judges could preside over cases
involving Europeans BUT the Europeans got right to demand trial by jury, where atleast half
of the jury would be White Europeans.
This completely defeated the purpose of the original Ilbert Bill. Though the educated Indians
had also protested against the amendments, but they realized that to pressurise the
government, all India organisation and more coordination amongst Indians to press for
equal treatment. This is why Ilbert Bill controversy is seen as an important precursor to the
formation of the INC.
Source: A New Look At the Modern Indian History by B.L. Grover and Alka Mehta.
Q.8) Which of the following Political Associations did not belong to the Bengal Province?
a) Zamindar Association.
b) Indian League.
c) Indian Association.
d) Madras Mahajan Sabha.
The Zamindari Association or Landlords society was founded in1838 to safeguard the
interests of the landlords of Bengalfor stimulating sense of nationalism among the people
Indian Association was founded by Anand Mohan Bose and Surendranath Banerjee.
Madras Mahajan Sabha was founded in 1884 to co-ordinate the local associations.
Source: A New Look At the Modern Indian History by B.L. Grover and Alka Mehta.
Q.9) Consider the following statements with reference to the Shyamji Krishna Varma.
Shyamji Krishna Varma was an Indian revolutionary fighter, lawyer and journalist who
founded the Indian Home Rule Society, India House and The Indian Sociologist in London.
In 1905 he founded the India House and The Indian Sociologist, which rapidly developed as
an organised meeting point for radical nationalists among Indian students in Britain at the
time and one of the most prominent centres for revolutionary Indian nationalism outside
India. Most famous among the members of this organisation was Veer Savarkar. Krishna
Varma moved to Paris in 1907, avoiding prosecution. He died in 1935
Later in 1905, Shyamji attended the United Congress of Democrats held at Holborn Town
Hall as a delegate of the India Home Rule Society. His resolution on India received an
enthusiastic ovation from the entire conference. Shyamjis activities in England aroused the
concern of the British government: He was disbarred from Inner Temple and removed from
the membership list on 30 April 1909 for writing anti-British articles in The Indian
Sociologist. Most of the British press were antiShyamji and carried outrageous allegations
against him and his newspaper. He defended them boldly. The Times referred to him as the
"Notorious Krishnavarma". Many newspapers criticised the British progressives who
supported Shyamji and his view. His movements were closely watched by British Secret
Services, so he decided to shift his headquarters to Paris, leaving India House in charge
of Vir Savarkar. Shyamji left Britain secretly before the government tried to arrest him.
Source: A New Look At the Modern Indian History by B.L. Grover and Alka Mehta.
From Plassey to Partition by Sekhar Bandyopadhyay.
Q.10) "Home Charges" constituted a major thread in the economic critique of the British
government. Which of the following funds constituted "Home Charges"?
a) 1 and 2 only
b) 1 and 3 only
IASbabas Prelims 60- Day Plan Day 8 2017
c) 2 and 3 only
d) All of the above.
It is clear from Economic History of India by RC Datt: " The Indian Tribute whether weighted
in the scales of justice or viewed in the light of our interest, will be found to be at variance
with humanity, with the commonsense and with the received maxims of economical
science. It would be true wisdom then to provide for the future payment of such of the
Home Charges of the Indian Government as really from the tribute out of Indian Exchequer.
These charges would be probably found to be the dividends on East India Stock, interest in
Home debt, the salaries of the officers, establishments of the and building connected with
the Home Department of Indian Government, furlough and retired pay to members of the
Indian Military and Civil Services when at Home, Charges of all descriptions paid in this
country connected with the British troops serving in India and portion of the cost of
transporting the British troops to and from India".
Source: A New Look At the Modern Indian History by B.L. Grover and Alka Mehta.
a) 1 and 2 only
b) 2 and 3 only
c) 1 and 3 only
d) None of the above.
The Santhals of Santhal Pargana raised the banner of revolt in 1856-57 under the leadership
of Kanu and Sidhu. The Santhals were generally peace-loving people who worked under
primitive agricultural conditions. The simple and peaceful life of the Santhals was disturbed
by the effects of the alien civilisation. Besides, the contractors, money-lenders, revenue-
collectors and other agencies exercised a variety of tyrannies upon the timid Santhal
tribesmen.
IASbabas Prelims 60- Day Plan Day 8 2017
Redressal of the grievances through the machinery of law being of no avail the Santhals
themselves took up arms against the oppressors. Under the leadership of Sidhu and his
elder brother Kanu of Bhavnadihi, about ten thousand Santhals armed with primitive
weapons rose in revolt in 1855. The Santhal rebellion for some months swept the British
rule out of existence in the region.
Bengal projected the first strike in the history of the peasant movement in India. The
European planters in Bengal forced the local peasants to resort to indigo cultivation and
earned a good deal of profit. The peasants suffered a lot in 1860.
The peasants of the districts of Pabna and Nadia and Barasat sub-division went on strike and
refused to cultivate indigo. Soon, the news spread and peasants of Dacca, Malda, Jessore,
Khulna, Rajsahi and several other places followed their path.
The British Government was alarmed and issued order to different police stations to take
due caution in protecting the peasants from the clutches of indigo planters. In an Act of
1862, it was decided that the planters can go to the court of law. This law freed the
peasants from the clutches of the planters who left Bengal and ultimately went to Bihar and
U. P
In 1921 the Muslim peasants of the Malabar districts of Kerala known as the Moplahs rose
against their landlords, the Namboodris and Nairs. These upper classes exploited the
peasants. The Moplahs had no security of their tenure.
The renewal of fees, high rents and other extractions by the zamindars, broke the backbone
of the Moplahs. They became united and made armed attacks on the Namboodris, Nairs
and other higher castes. The British Government became active and suppressed them.
Source: A New Look At the Modern Indian History by B.L. Grover and Alka Mehta.
Q.12) Consider the following statements regarding the Subsidiary Alliance System:
1. Indian State was to surrender its External Relations to the East India Company.
2. Indian State was to accept a British Resident at its Headquarters
3. The company had the right to interfere in the Internal Matters of the Indian State.
a) 1 and 2 only
b) 2 and 3 only
c) 1 and 3 only
IASbabas Prelims 60- Day Plan Day 8 2017
d) All of the above
Subsidiary Alliance System was initiated by Warren Hastings under this the Company
entered into an agreement with the Indian State, Under which the Indian State was to
surrender the foreign/external relations to the company. It could not wage war against any
other state without the permission from the Company. A British Resident was to stay in the
headquarters of the Indian State as the representative of the Company.
A small British force was to be stationed in the Headquarters of the Indian State ostensibly
to protect the State. The expenses of the force were to be defrayed from the Indian State.
Asper the agreement the Company could not interfere in the internal affairs of the State,
but in practice the Company interfered in the internal affairs on the flimsiest of reasons.
Source: A New Look At the Modern Indian History by B.L. Grover and Alka Mehta.
Q.13) Which of the following Governor General took action against the Pindaris?
a) Wellesley
b) Hastings
c) Cornwallis
d) Dalhousie
The Pindaris followed the Maratha bands who raided Mughal territory from the late 17th
century. With the collapse of the Mughal Empire in the 18th century, these camp followers
organized themselves into groups, each usually attached to one of the leading Maratha
chiefs. But as those chiefs themselves grew weak at the end of the century, the Pindaris
became largely a law unto themselves and conducted raids from hideouts in central India.
The majority of their leaders were Muslims, but they recruited from all classes.
Lord Hastings after getting permission from the British government waged wars against the
Pindaris which resulted in the Suppression of Pindaris.
Source: A New Look At the Modern Indian History by B.L. Grover and Alka Mehta.
Q.14) Consider the following statements with reference to the Harijan Sevak Sangh
IASbabas Prelims 60- Day Plan Day 8 2017
1. It was started by Mahatma Gandhi.
2. It was headquartered at Bombay.
a) 1 only
b) 2 only
c) Both 1 and 2
d) Neither 1 nor 2
All India Anti-Untouchability league was organized by Mahatma Gandhi in 1932 which was
later renamed as Harijan Sevak Sangh. It was headquartered at Delhi.
Source: A New Look At the Modern Indian History by B.L. Grover and Alka Mehta.
Q.15) Which of the following events is not related to tenure of Lord Lytton as the
Governor General of India?
The First Factory Act was passed in 1881 by lord Ripon, Successor of Lord Lytton. The
Statutory Civil Services was created in 1878-89. Under this the Government could employ
some Indians of good family and standing to the SCC on the recommendation of the
Provincial Governments, subject to the final approval of the Secretary of States for India.
The vernacular press Act was passed in 1878 to control the publications in
oriental/Vernacular languages. Under this a magistrate was empowered to call for any
publisher to furnish a bond undertaking not to publish content considered as degrading the
British Government. He could demand security and forfeit it, and could seize the printing
equipment.
Source: A New Look At the Modern Indian History by B.L. Grover and Alka Mehta.
About
It is the country's first-ever hotline to curb sexual abuse of children through the
Internet and to remove child pornographic content online unveiled.
To eliminate the scourge of online child pornography and further the cause of child
protection in online spaces.
It is a network of organizations and individuals working on child protection in the
country, has collaborated with the U.K.-based Internet Watch Foundation (IWF).
The hotline in India will be hosted on aarambhindia.org and will enable users to
report child sexual abuse images and videos in a safe and anonymous environment.
It is a simple, accessible form (available in Hindi & English) that any informed user
who stumbles across sexually explicit imagery of a child on the public internet can
use to report the content. Latter it will be started in other languages.
Source: http://www.thehindu.com/news/national/Hotline-to-curb-child-
pornography/article14636764.ece
a) 1 and 2
b) 2 and 3
c) 1 and 3
d) All of the above
IASbabas Prelims 60- Day Plan Day 8 2017
The major architectures achievements are the shrine at Tiruvannamalai and the Great
Temple at Madurai. Other significant architectural wonders included that of the Temple at
Rameswaram (famous for its long corridors) and the Subramanya Temple at Thanjavur
District.
Their most famous architectural achievement was in form of Meenakshi Temple of Madurai
or Sundereswara Temple built by Thirumalai Nayak, dedicated to Parvati and Lord Shiva. It
was actually not built by the Nayakas, they only refurbished the temple by building new
larger gopurams and other changes in temple.
It has two shrines one dedicated to Shiva/Sundereswara and other dedicated to his
wife parvati/Meenakshi.
A large water tank in the temple is one of the distinct features of the temple.
Parakramas or roofed ambulatory paths are another distinguishing features of
Meenakshi Temple apart from other Dravidian features.
A thousand pillared Mandapam is another unique feature of this temple.
This temple is also known for its highly impressive gopuram, which is perhaps more
impressive than the temple itself and it totally dwarfs the central shrines. The
Meenakshi temple in Madurai has the tallest gopuram in the world.
For all the innovation of its massive but unsystematic plan including the large gopurams
and huge courtyards, the architecture of Madurai, under the Nayak dynasty represented
only an exaggeration of already established forms in every detail of its structure, rather than
a new development as it is best exemplified by the Meenakshi temple which was a Pandya
temple, later modified by Nayakas.
Source: http://www.thehindu.com/features/metroplus/Voices-and-music-bring-alive-the-
past/article15004000.ece
Q.18) Consider the following statements about Chemical Weapons Convention (CWC)
IASbabas Prelims 60- Day Plan Day 8 2017
1. India is an original state party to the Chemical Weapons Convention and as a
possessor state it has fully completed the destruction of its chemical weapons in
accordance with the Convention.
2. It is administered by the Organisation for the Prohibition of Chemical Weapons
(OPCW)
3. Israel has signed but has yet to ratify the convention.
a) Only 2
b) 1 and 2
c) 2 and 3
d) All of the above
India is an original state party to the Chemical Weapons Convention and as a possessor state
it has fully completed the destruction of its chemical weapons in accordance with the
Convention.
India has offered the services of its experts to help in the destruction of Damascus' chemical
weapons arsenal and related facilities.
India has also decided to offer a contribution of 1 million dollars for use in the destruction of
Syria's chemical weapons and related facilities by the Organisation for the Prohibition of
Chemical Weapons (OPCW).
India has found support from Russia for its participation in an international conference
called Geneva-II which seeks to bring together all warring parties, domestic and
international, to stablilise Syria.
India would also offer the services of its experts to be used by the OPCW in the destruction
verification activity as well as training slots for personnel participating in the UN/OPCW
mission for the destruction of Syria's chemical weapons.
India has welcomed the progress on the time-bound safeguarding and destruction of Syria's
chemical weapons and its accession to the Chemical Weapons Convention.
It is a multilateral treaty that bans chemical weapons and requires their destruction within a
specified period of time. The treaty is of unlimited duration and is far more comprehensive
than the 1925 Geneva Protocol, which outlaws the use but not the possession of chemical
weapons.
IASbabas Prelims 60- Day Plan Day 8 2017
CWC negotiations started in 1980 in the UN Conference on Disarmament. The convention
was opened for signature on January 13, 1993, and entered into force on April 29, 1997.
The CWC is open to all nations and currently has 192 states-parties. Israel has signed but not
ratified the agreement, while three other UN member states (Egypt, North Korea and South
Sudan) have neither signed nor acceded to the treaty. Most recently, Angola deposited its
instrument of accession to the CWC on 16 September 2015.
The CWC is implemented by the Organization for the Prohibition of Chemical Weapons
(OPCW), which is headquartered in the Hague. The OPCW receives States-parties
declarations detailing chemical weapons-related activities or materials and relevant
industrial activities. After receiving declarations, the OPCW inspects and monitors states-
parties facilities and activities that are relevant to the convention, to ensure compliance.
Source; http://www.thehindu.com/todays-paper/tp-international/%E2%80%98Chemical-
weapons%E2%80%99-used-in-Iraq/article14995022.ece
Q.19) Golden Crescent and Golden Triangle are Asias two principal areas of illicit opium
production. Consider the following statements
a) Only 1
b) Only 2
c) Both 1 and 2
d) Neither 1 nor 2
Source: http://indianexpress.com/article/cities/chandigarh/kalgidhar-trust-in-
chandigarh-urges-people-to-join-fight-against-drug-addiction-3038401/
Q.20) Which of the following countries is/are members of South Asia Subregional
Economic Cooperation (SASEC)?
1. Maldives
2. Myanmar
3. Pakistan
4. Bhutan
5. Sri Lanka
a) 1, 2, 4 and 5
b) 2, 3, 4 and 5
c) 1, 2, 3 and 5
d) All of the above
The South Asia Subregional Economic Cooperation (SASEC) Program, set up in 2001, brings
together Bangladesh, Bhutan, India, Maldives, Myanmar, Nepal and Sri Lanka in a project-
based partnership to promote regional prosperity by improving cross-border connectivity,
IASbabas Prelims 60- Day Plan Day 8 2017
boosting trade among member countries, and strengthening regional economic
cooperation.
The Manila, Philippines-based Asian Development Bank (ADB) serves as the Secretariat for
the SASEC member countries.
Source: http://pib.nic.in/newsite/PrintRelease.aspx?relid=151132
1. To become a national party a political party has to become a state party in at least 4
states
2. The status of national party is reviewed every 5 years
a) Only 1
b) Only 2
c) Both 1 and 2
IASbabas Prelims 60- Day Plan Day 8 2017
d) Neither 1 nor 2
Trinamool Congress has satisfied one of the conditions mentioned in the Election Symbols
(Reservation and Allotment) Order, 1968 to be recognised as a national party having
been recognised as a State party in at least four States
It is a recognised State party in West Bengal, Manipur, Tripura and Arunachal Pradesh
Now, India has seven recognised national parties the Congress, the BJP, the BSP, the CPI,
the CPI (M), the NCP and the All India Trinamool Congress
Recognition as a national or a State party ensures that the election symbol of that party is
not used by any other political entity in polls across India. Other registered but unrecognised
political parties have to choose from a pool of free symbols announced by the commission
from time to time
Besides, these parties get land or buildings from the government to set up their party
offices. They can have up to 40 star campaigners during electioneering. Others can have up
to 20 star campaigners
The poll panel had on August 22, 2016 amended a rule whereby it will now review the
national and State party status of political parties every 10 years instead of the five.
Had the rule not been amended, Trinamool Congress would not have been recognised as a
national party as it had not performed well in the Arunachal Pradesh Assembly polls and
would have lost the State party status there.
The amendment in the rule had also come as a major reprieve for the BSP, the NCP and the
CPI as they were facing the prospect of losing their national party status after their dismal
performance in the 2014 Lok Sabha election. The EC had served them notices in 2014 on the
issue.
Source: http://www.thehindu.com/news/national/trinamool-congress-recognised-as-
national-party/article9066266.ece
Project Iceworm was the code name for a top-secret United States Army program during the
Cold War to build a network of mobile nuclear missile launch sites under the Greenland ice
sheet. The ultimate objective of placing medium-range missiles under the ice close
enough to strike targets within the Soviet Union was kept secret from the Danish
government. To study the feasibility of working under the ice, a highly publicized "cover"
project, known as Camp Century, was launched in 1960. However, unsteady ice conditions
within the ice sheet caused the project to be cancelled in 1966.
Source: http://www.thehindu.com/todays-paper/tp-national/melting-greenland-ice-
threatens-to-expose-cold-war-waste/article9151507.ece
IASbabas Prelims 60 Day Plan Day 9 2017
Current Daily Status divides each day into two halves and measures unemployment in each
half of the day, as such it is more comprehensive measure of unemployment, as it not only
measures the scale but also the intensity of unemployment.
In Current Weekly status, a person is considered employed if he has worked for at least 1
hour in the preceding week, now if the person has worked only 1 day in the week, he will
considered employed for the whole week, here lies the problem as the remaining 6 days
that he has not worked are also recorded as employed days.
a) Disguised Unemployment.
b) Cyclical unemployment
c) Under employment
d) Natural Unemployment.
One hundred percent full employment is unattainable in a market economy over the long
run. Such employment is actually undesirable, because a 0% long-run unemployment rate
requires a completely inflexible labor market, where laborers are unable to simply quit their
current job or leave to find a better one.
Under this definition of natural unemployment, it is possible for institutional factors, such as
the minimum wage or high degrees of unionization, to increase the natural rate over the
long run.
Perfect competition is a market structure in which the following five criteria are met: 1)
All firms sell an identical product; 2) All firms are price takers - they cannot control
the market price of their product; 3) All firms have a relatively small market share; 4) Buyers
have complete information about the product being sold and the prices charged by each
firm; and 5) The industry is characterized by freedom of entry and exit. Perfect competition
is sometimes referred to as "pure competition".
Perfect competition is the opposite of a monopoly, in which only a single firm supplies a
particular goods or service, and that firm can charge whatever price it wants because
consumers have no alternatives and it is difficult for would-be competitors to enter the
marketplace. Under perfect competition, there are many buyers and sellers, and prices
reflect supply and demand. Also, consumers have many substitutes if the good or service
they wish to buy becomes too expensive or its quality begins to fall short. New firms can
easily enter the market, generating additional competition. Companies earn just enough
profit to stay in business and no more, because if they were to earn excess profits, other
companies would enter the market and drive profits back down to the bare minimum.
Q.4) The demand for which of the following is likely to be the most price inelastic?
a) Salt.
b) Smart phones.
c) Luxury cars.
d) Airline tickets.
IASbabas Prelims 60 Day Plan Day 9 2017
In case of smartphones, luxury cars and Airline tickets a small drop in prices will lead to
higher demand for these commodities, where as any fluctuations in the price of salt will
have no impact on the consumption of salt.
Q.5) Consider the Following statements with reference to GDP and GNI
a) 1 only
b) 2 only
c) Both 1 and 2
d) Neither 1 nor 2
A perfectly closed economy has contact with the outside world, It is completely self-
sufficient, and has no imports or exports with the other countries. So, all the goods and
services that are produced in the domestic territory (GDP) is produced by the Nationals of
that country (GNI).
GNI may or may not be greater than GDP, if all the residents of the country have migrated
for work outside their country, and the Production is less than the remittances, then GNI
will be greater than GDP,
If there is no outward emigration and only inward immigration, then the income paid to the
foreign nationals will be more, in this case the GDP will be greater than the GNI
Q.6) Consider the following statements about the New National Income series.
a) 1 only
b) 2 only
c) Both 1 and 2.
d) Neither 1 nor 2.
The Ministry of Statistics & Programme Implementation has released the new series of
national accounts, revising the base year from 2004-05 to 2011-12. The base year of
national accounts was last revised in January 2010.
Gross Domestic Product (GDP) at factor cost will no longer be discussed in the press
releases. As is the practice internationally, industry-wise estimates will be presented as
Gross Value Added (GVA) at basic prices, while GDP at market prices will henceforth be
referred to as GDP. Estimates of GVA at factor cost (earlier called GDP at factor cost) can be
compiled by using the estimates of GVA at basic prices and production taxes less subsidies
Q.7) The highest contribution to the Gross Domestic Savings comes from?
As per the data compiled by the CSO, the Household sector is the highest contributor to the
Gross Domestic Savings with a share of 59.4 percent in the year 2013-14. However, the
share has declined from 67.3 percent in 2011-12 and 63.4 percent in 2012-13. This decline
can be attributed to the decline in household savings in physical assets, which has declined
from Rs.13.4 lakh crore in 2011-12 to Rs. 12.1 lakh crore in 2013-14. On the other hand, the
share of Non-Financial Corporations has increased from 29.3 percent in 2011-12 to 34.5
percent in 2013-14. The share of Financial Corporations has been around 9 percent in all
these years, while the dis-saving of General Government has decreased from 5.4 percent in
2011-12 to 3.2 percent in 2013-14
Source: http://pib.nic.in/newsite/PrintRelease.aspx?relid=115084
IASbabas Prelims 60 Day Plan Day 9 2017
Q.8) Which of the following items does not come under the tertiary sector (Service Sector)
in India?
a) Hotels.
b) Transport.
c) Water Supply.
d) Public Administration.
Indian economy is classified in three sectors Agriculture and allied, Industry and Services.
Agriculture sector includes Agriculture (Agriculture proper & Livestock), Forestry & Logging,
Fishing and related activities. Industry includes 'Mining & quarrying', Manufacturing
(Registered & Unregistered), Electricity, Gas, Water supply, and Construction. Services
sector includes 'Trade, hotels, transport, communication and services related to
broadcasting', 'Financial, real estate & professional services', 'Public Administration, defence
and other services'.
Source: http://statisticstimes.com/economy/sectorwise-gdp-contribution-of-india.php
Q.9) The Final value of all the Goods and Services produced by a countrys nationals is
called?
The statement is the definition of Gross National Product, which calculates the final
monetary value of all the goods and services produced by the nationals of a country. The
production in a country is called GDP, here the focus is on the territory and not nationality.
Disguised unemployment exists where part of the labor force is either left without work or
is working in a redundant manner where worker productivity is essentially zero. It is
unemployment that does not affect aggregate output. An economy demonstrates disguised
unemployment when productivity is low and too many workers are filling too few jobs.
Disguised, or hidden, unemployment can refer to any segment of the population not
employed at full capacity, but it is often not counted in official unemployment statistics
within the national economy. This can include those working well below their capabilities,
those whose positions provide little overall value in terms of productivity, or any group that
is not currently looking for work but is able to perform work of value.
Q.11) Consider the following statements with reference to the Multi-Dimensional Poverty
Index.
a) 1 only
b) 1 and 2 only
c) 1 and 3 only
d) None of the above.
Dimension : Indicators
Living Standards : Cooking fuel, Toilet, Water, Electricity, Floor and Assets
Q.12) which of the following statements with respect to Human Development Index
report is/are incorrect?
a) 1 only
b) 1 and 2 only
c) 2 and 3 only
d) None of the above
HDI report is published by the United Nations Development Programme (UNDP), it ranks the
countries based on the achievements in three categories of Health, Education and Standard
of living. From 2010 HDI report onwards the Standard of living is calculated based on the
Gross national Income of the country.
India now belongs to the Medium human development bracket countries with India's
human development index (HDI) value of 0.624. Indias current ranking is 131
The law of demand is a microeconomic law that states, all other factors being equal, as the
price of a good or service increases, consumer demand for the good or service will decrease,
and vice versa.
The chart below depicts the law of demand using a demand curve, which is always
downward sloping. Each point on the curve (A, B, C) reflects a
direct correlation between Quantity demanded (Q) and price (P). So, at point A, the quantity
demanded will be Q1 and the price will be P1, and so on.
IASbabas Prelims 60 Day Plan Day 9 2017
Q.14) If elasticity of demand is very low it shows that the commodity is:
a) A necessity
b) A luxury
c) Has little impact on the household budget.
d) Sensitive to price change.
Commodities which are considered necessity like rice, wheat, salt etc have very low
elasticity of demand i.e. irrespective on change in the price the demand for the commodity
will not change.
On the other hand luxury items have high elasticity of demand, here a little drop in the price
will have significantly higher demand.
The Physical Quality of Life Index (PQLI) is an attempt to measure the quality of life or well-
being of a country. The value is the average of three statistics: basic literacy rate, infant
mortality, and life expectancy at age one, all equally weighted on a 0 to 100 scale.
IASbabas Prelims 60 Day Plan Day 9 2017
It was developed for the Overseas Development Council in the mid-1970s by Morris David
Morris, as one of a number of measures created due to dissatisfaction with the use
of GNP as an indicator of development. PQLI might be regarded as an improvement but
shares the general problems of measuring quality of life in a quantitative way. It has also
been criticized because there is considerable overlap between infant mortality and life
expectancy.
Karamay Declaration is collaboration between Pakistan and China in space technology under
China-Pakistan Economic Corridor (CPEC).
It stressed upon a joint launch of space missions which would consist of astronauts from
both countries.
CPEC is one of the six economic corridors conceived by china under its "Road and Belt
initiative. CPEC is china and Pakistans bilateral project connecting Chinese city Xinjian to
Gwadar port in Pakistan through highways, railways, pipelines etc.
Source: http://www.thehindu.com/news/international/In-China-the-debate-on-a-
corridor-with-Pakistan-hots-up/article14636504.ece
Q.17) Consider the following statements about Heart of Asia - Istanbul Process
1. The 7th Ministerial Conference will be held in Amritsar and India will be co-chairing
the process
2. Iran and Iraq are not members of the process
a) Only 1
b) Only 2
IASbabas Prelims 60 Day Plan Day 9 2017
c) Both 1 and 2
d) Neither 1 nor 2
The Heart of Asia - Istanbul Process was established to provide a platform to discuss
regional issues, particularly encouraging security, political, and economic cooperation
among Afghanistan and its neighbors. This region-led dialogue was launched in November
2011 to expand practical coordination between Afghanistan and its neighbors and regional
partners in facing common threats, including counterterrorism, counternarcotics, poverty,
and extremism. The United States and over 20 other nations and organizations serve as
supporting nations to the process.
Current Members
Afghanistan
Azerbaijan
China
India
Iran
Kazakhstan
Kyrgyzstan
Pakistan
Russia
Saudi Arabia
Tajikistan
Turkey
Turkmenistan
United Arab Emirates
6th Ministerial Conference of Heart of Asia summit was held in Amritsar, India, from
December 3 to December 4, 2016.
The 7th Ministerial Conference will be held in Azerbaijan in the year 2017. Azerbaijan will be
co-chairing the process.
Source: http://www.thehindubusinessline.com/news/india-offers-1-bn-towards-
afghanistans-development/article9107169.ece
1. The Cauvery basin extends over states of Tamil Nadu, Karnataka and Kerala
2. Alluvial soil, black soil and red soil is found in the Cauvery basin
a) Only 1
b) Only 2
c) Both 1 and 2
d) Neither 1 nor 2
The River Cauvery originates at Talakaveri in Coorg District of Karnataka in Brahmagiri Range
of hills in the Western ghats.
The Cauvery basin extends over states of Tamil Nadu, Karnataka, Kerala and Union Territory
of Puducherry draining an area of 81,155 Sq.km. The river drains into the Bay of Bengal.
Physiographically, the basin can be divided into three parts the Westen Ghats, the Plateau
of Mysore and the Delta. The delta area is the most fertile tract in the basin. The principal
soil types found in the basin are black soils, red soils, laterites, alluvial soils, forest soils and
mixed soils. Red soils occupy large areas in the basin. Alluvial soils are found in the delta
areas.
Indias position improved to 39th rank in the World Economic Forums latest Global
Competitiveness Index.
Source: http://indianexpress.com/article/business/economy/global-competitiveness-
index-india-jumps-16-ranks-for-second-time-second-most-competitive-brics-economy/
IASbabas Prelims 60 Day Plan Day 9 2017
Q.20) Consider the following statements about Department of Investment and Public
Asset Management (DIPAM)
a) Only 1
b) Only 2
c) Both 1 and 2
d) Neither 1 nor 2
Source: http://www.thehindu.com/business/NMDC-MOIL%E2%80%99s-Rs-8400-cr-
buyback-offers-to-open-Sep-19/article14639363.ece
Price Stabilisation Fund (PSF) refers to any fund constituted for the purpose of containing
extreme volatility in prices of selected commodities. The amount in the fund is generally
utilised for activities aimed at bringing down/up the high/low prices say for instance,
procurement of such products and distribution of the same as and when required, so that
prices remain in a range.
India first created a price stabilisation fund for some export oriented plantation crops in
2003, and this ceased to exist in 2013. Another fund was created in 2015 for perishable
agricultural and horticultural commodities, but initially limited to support potato and onion
prices only.
Price Stabalisation Fund (PSF) was shifted to Consumer Affairs Ministry from Agriculture
Ministry for effective control of price rise in essential commodities and provide relief to the
consumers.
Read More
http://economictimes.indiatimes.com/news/economy/agriculture/price-stabilisation-
fund-to-be-shifted-to-consumer-affairs-ministry/articleshow/50821750.cms
http://consumeraffairs.nic.in/WriteReadData/userfiles/file/PSF%20Operational%20Guide
llines%20DOCA.pdf
Source: http://economictimes.indiatimes.com/news/economy/policy/govt-to-raise-
buffer-stock-for-pulses-to-20l-tonnes/articleshow/54290783.cms
Q.22) Recently Maria Sharapova was banned for consumption of a drug & the same drug
was banned by World Anti-Doping Agency (WADA). Which is that drug?
a) Acetylsalicylic Acid
b) Paracetamol
c) Creatine
d) Meldonium
Also marketed as mildronate, the website of the drugs Latvian manufacturer Grindeks says
meldonium gives sufferers of heart and circulatory conditions more physical capacity and
mental function and a similar boost to healthy people. Meldonium was banned because it
aids oxygen uptake and endurance.
IASbabas Prelims 60 Day Plan Day 9 2017
Meldonium is most commonly used in Eastern European and ex-Soviet countries as a drug
for people with heart conditions, but its also offered for sale online. There are also signs
that a sizable minority of athletes were using before it was banned.
Source: http://www.thehindu.com/sport/tennis/maria-sharapova-to-find-out-decision-
on-appeal-in-october/article9104918.ece
IASbabas Prelims 60 Day Plan Day 10 2017
Q.1) Consider the following Statements with reference to the Rangarajan Committee
report on Poverty.
1. Along with daily calorie intake, it also includes consumption of protein and fats.
2. Poverty line is estimated as Monthly Per Capita Expenditure of Rs. 1407 in urban
areas and Rs. 972 in rural areas.
a) 1 only
b) 2 only
c) Both 1 and 2
d) Neither 1 nor 2
The poverty line should be based on certain normative levels of adequate nourishment,
clothing, house rent, conveyance and education, and a behaviorally determined level of
other non-food expenses.
The Expert Group (Rangarajan) computed the average requirements of calories, proteins
and fats based on ICMR norms differentiated by age, gender and activity for all-India rural
and urban regions to derive the normative levels of nourishment. Accordingly, the energy
requirement works out to 2,155 kcal per person per day in rural areas and 2,090 kcal per
person per day in urban areas. For reasons elaborated in the text , the Expert Group (
Rangarajan) views the Calorie norm not as a single number but as an average in a band
of +/- 10 per cent of these values and with intakes even at the lower end still
being adequate enough to not adversely affect health and work.
The protein and fat requirements have been estimated on the same lines as for energy.
These requirements are 48 gms and 28 gms per capita per day, respectively, in rural areas;
and 50 gms and 26 gms per capita per day in urban areas.
A food basket that simultaneously meets all the normative requirements of the three
nutrients defines the food component of the poverty line basket proposed by the Expert
Group (Rangarajan). These nutrient norms are met for persons located in the sixth fractile
(25-30%) in rural areas and for those in the fourth fractile (15-20%) in urban areas in 2011-
12.The average monthly per capita consumption expenditure on food in these fractile
classes is Rs.554 in rural areas and Rs.656 in urban areas (NSS 68th Round).
The median fractile (45-50%) values of clothing expenses, rent, conveyance and education
expenses are treated as the normative requirements of the basic non-food expenses of
IASbabas Prelims 60 Day Plan Day 10 2017
clothing, housing, mobility and education of a poverty line basket. This works out to Rs.141
per capita per month in rural areas and Rs.407 in urban areas. The observed expenses of all
other non-food expenses of the fractile classes that meet the nutrition requirements are
considered as part of the poverty line basket. This works out to Rs.277 per capita per month
in rural areas and Rs.344 in urban areas.
The new poverty line thus work out to monthly per capita consumption expenditure of
Rs.972 in rural areas and Rs.1,407 in urban areas in 2011-12. For a family of five, this
translates into a monthly consumption expenditure of Rs.4,860 in rural areas and Rs.7,035
in urban areas.
Source: http://pib.nic.in/newsite/PrintRelease.aspx?relid=108291
Q.2) Consider the following statements with reference to Pradhan Mantri Yuva Yojana.
a) 1 only
b) 2 only
c) Both 1 and 2
d) Neither 1 nor 2.
Marking the 2nd Foundation Day of Ministry of Skill Development and Entrepreneurship,
Minister of State(I/C) for Skill Development and Entrepreneurship Shri Rajiv Pratap Rudy, ,
today launched the Pradhan Mantri YUVA Yojana, MSDEs flagship scheme on
entrepreneurship education and training.
The scheme spans over five years (2016-17 to 2020-21) with a project cost of Rs. 499.94
crore, and will provide entrepreneurship education and training to over 7 lakh students in 5
years through 3050 Institutes. It will also include easy access to information and mentor
network, credit, incubator and accelerator and advocacy to create a pathway for the youth.
Speaking on the occassion Shri Rudy said, with this the government has taken important
strides to scale up entrepreneurship in the country. He said, Pradhan Mantri YUVA Yojana
has national and international best practices of learning in entrepreneurship education.
Ministry, these two institutes are now focusing on mentorship of budding entrepreneurs
across the country. We have seen success so far and we are determined to create more
opportunities of employment for our youth through this initiative, he further added.
The institutes under the PMs YUVA Yojana include 2200 Institutes of Higher Learning
(colleges, universities, and premier institutes), 300 schools, 500 ITIs and 50
Entrepreneurship Development Centres, through Massive Open Online Courses (MOOCs).
Source: http://pib.nic.in/newsite/PrintRelease.aspx?relid=153429
Q.3) Consider the following statements with reference to the National Skill Development
Mission
a) 1 and 2 only
b) 2 and 3 only
c) 1 and 3 only
d) All of the above
The National Skill Development Mission was approved by the Union Cabinet on 01.07.2015,
and officially launched by the Honble Prime Minister on 15.07.2015 on the occasion of
World Youth Skills Day. The Mission has been developed to create convergence across
sectors and States in terms of skill training activities. Further, to achieve the vision of Skilled
India, the National Skill Development Mission would not only consolidate and coordinate
skilling efforts, but also expedite decision making across sectors to achieve skilling at scale
with speed and standards. It will be implemented through a streamlined institutional
mechanism driven by Ministry of Skill Development and Entrepreneurship (MSDE). Key
institutional mechanisms for achieving the objectives of the Mission have been divided into
three tiers, which will consist of a Governing Council for policy guidance at apex level, a
IASbabas Prelims 60 Day Plan Day 10 2017
Steering Committee and a Mission Directorate (along with an Executive Committee) as the
executive arm of the Mission. Mission Directorate will be supported by three other
institutions: National Skill Development Agency (NSDA), National Skill Development
Corporation (NSDC), and Directorate General of Training (DGT) all of which will have
horizontal linkages with Mission Directorate to facilitate smooth functioning of the national
institutional mechanism. Seven sub-missions have been proposed initially to act as building
blocks for achieving overall objectives of the Mission. They are:
(i) Institutional Training, (ii) Infrastructure, (iii) Convergence, (iv) Trainers, (v) Overseas
Employment, (vi) Sustainable Livelihoods, (vii) Leveraging Public Infrastructure.
The mission will cover all the states and UTs, to expedite the skilling process, existing
educational facilities will be upgraded, new ITIs and new Vocational Centres will be opened.
Q.4) Consider the following statements with reference to the Sustainable Development
Goals (SDG).
a) 1 only
b) 2 only
c) Both 1 and 2
d) Neither 1 nor 2
The Sustainable Development Goals (SDGs), officially known as Transforming our world:
the 2030 Agenda for Sustainable Development is a set of 17 "Global Goals" with 169
targets between them.
These included ending poverty and hunger, improving health and education, making cities
more sustainable, combating climate change, and protecting oceans and forests.
Q.5) Which of the following Millennium Development Goals was not achieved by India?
The MDGs are a United Nations initiative that was adopted after the 2000 Millennium
Summit. These represent a core people-oriented development agenda. They mapped out a
plan to tackle, by 2015, many of the important issues facing the world. They are as follows:
India has made notable progress towards reaching the MDGs, but achievement across the
goals varies and helping the weaker states emulate the good performers can improve
performance. More specifically:
India has achieved the target for reducing poverty by half (Goal 1).
Gender parity has been achieved in primary school enrolment (Goal 3) and was close to
being achieved in secondary and tertiary education by end 2015.
The spread of deadly diseases such as HIV/AIDS, malaria and tuberculosis is being controlled
(Goal 6);
Forest cover has increased, although there was a shift towards production of timber from a
more multi-product approach (Goal 7).
The proportion of population without access to clean drinking water has been halved, but
there are still big challenges on sanitation.
IASbabas Prelims 60 Day Plan Day 10 2017
Achieving by 2015 universal primary school enrolment and completion, and achieving
universal youth literacy (Goal 2);
Empowering women through wage employment and political participation (Goal 3);
Improving access to adequate sanitation to eliminate the massive open defecation problem
(Goal 7).
Q.6) Consider the following statements with respect to the Gini Coefficient.
a) 1 only
b) 1 and 2 only
c) 1 and 3 only
d) 2 and 3 only.
Gini Coefficient is a measure of income inequality. It is derived from the Lorenz Curve.
It gives how much of the nations income and wealth is distributed between its population.
Zero represents complete equality in distribution of income. And one represents complete
inequality. However no country has a score of either zero or one, actual figures are
somewhere in between.
A rich country and a poor country can have the same Gini-coefficient, as it does not
measure the absolute income levels but the distribution of wealth.
a) 1 and 2 only
b) 2 and 3 only
c) 1 and 3 only
d) None of the above.
The National Rural Employment Guarantee Act, notified in 200 districts in the first phase on
Feb 2, 2006 was renamed Mahatma Gandhi National Rural Employment Guarantee Act
(MGNREGA) on 02 Oct 2009. 130 districts were notified in 2007 and with the notification of
the remaining districts on 01 April 2008, the entire country has been covered. Exception is
given to district with a hundred percent urban population. The Act seeks to enhance
livelihood security in rural areas by providing at least 100 days of guaranteed wage
employment in a financial year.
Salient Features:
Adult members of a rural household willing to do unskilled manual work are eligible to seek
employment under the act.
Wages are to be paid according to Minimum Wages Act 1948 for agricultural labourers in
the State unless otherwise notified by the centre
Q.8) consider the following statements regarding Pravasi Kaushal Vikas Yojana
1. PKVY will provide training and certify Indians who are seeking overseas employment
in selected sectors.
IASbabas Prelims 60 Day Plan Day 10 2017
a) 1 only
b) 2 only
c) Both 1 and 2
d) Neither 1 nor 2
This scheme was launched by the PM Modi on the inauguration programme of 4th Pravasi
Bhartiya Divas convention which was held in Indias IT hub Bengaluru, Karnataka.
Portuguese Prime Minister Antonio Costa was the Chief Guest of the event.
The Pravasi Kaushal Vikas Yojana (PKVY) is launched with an aim to provide training and
certify Indians who are seeking overseas employment in selected sectors in line with
international standards.
This scheme will be implemented by the National Skill Development Corporation (NSDC)
through its training partners and in consultation with the Union Ministry of External Affairs
and the Union Skill Development Ministry.
This scheme will help for boosting the confidence of the Indian youth who want to work out
of the country of their choice for vocation.
Q.9) Consider the following statements with reference to the Gender Inequality Index.
a) 1 only
b) 2 only
c) Both 1 and 2
d) Neither 1 nor 2
a) Rural Infrastructure.
b) Womens education
c) Digital Literacy.
d) Welfare of Old-age citizens.
PMGDISHA- Pradhan Mantri Gramin Digital Saksharta Abhiyan is the project launched under
"Skill India" scheme. The announcement made by our finance Minister Mr. Arun Jaitley in
the Union Budget 2016-2017 on 1st Feb 2017. Then on 8. Feb 2017 the Union cabinet
chaired by our honorable Prime Minister Mr. Narendra Modi has approved Pradhan Mantri
Digital Saksharta Abhiyan (PMGDISHA). The budget which has been taken for this project is
Rs 2,351.38 crore to use it for digital literacy in rural India till March 2019. This project is
expected to be one of the biggest project of the world in order to make 6 crore Indian
houses digitally literate.
Digital Literacy- means the person would be able to operate digital devices like computer,
tablets, smart phones, a person will be able to do send and receive e-mails, brows internet,
obtained Government services, explore information, do online transactions and use
Information Technology in order to participate in the growth of nation actively.
Target- The overall target is to trained 6 crore students till FY(Financial Year)2019 but 25
lakh candidates are going to trained in the year2016-2017, 275 lakh candidates are going to
trained in year 2017-2018 and 300 candidates are going to be trained in the year 2018-2019.
Prior steps by the government- In order to reach the goal of teaching 6 crore students till
2019 the government has accepted 250,000 Gram Panchayats to register at least 200-300
candidates.
Purpose of the PMGDISHA(Pradhan Mantri Gramin Digital Saksharta Abhiyan): India has the
largest number of youth in the world in order to provide technical and digital training to
these youths specially the youths from the backward or rural area this project is launched.
IASbabas Prelims 60 Day Plan Day 10 2017
To help the rural and the backward household by making them digitally literate by providing
them quality training in both ways theoretical and practical in order to have economic,
social and cultural advantages for them. When these rural household get digital training it
will helpful for them in order to get better opportunities for being employed by companies
in India and abroad and at the same time it will also remove the social gap between the
rural and the urban area.
a) 1 and 2 only
b) 1 and 3 only
c) 2 and 3 only
d) None of the above
Alagh Committee (1979): In 1979, a task force constituted by the Planning Commission for
the purpose of poverty estimation, chaired by YK Alagh, constructed a poverty line for rural
and urban areas on the basis of nutritional requirements.
Tendulkar Committee (2009): In 2005, another expert group to review methodology for
poverty estimation, chaired by Suresh Tendulkar, was constituted by the Planning
Commission to address the following three shortcomings of the previous methods: (i)
consumption patterns were linked to the 1973-74 poverty line baskets (PLBs) of goods and
services, whereas there were significant changes in the consumption patterns of the poor
IASbabas Prelims 60 Day Plan Day 10 2017
since that time, which were not reflected in the poverty estimates; (ii) there were issues
with the adjustment of prices for inflation, both spatially (across regions) and temporally
(across time); and (iii) earlier poverty lines assumed that health and education would be
provided by the State and formulated poverty lines accordingly.
It recommended four major changes: (i) a shift away from calorie consumption based
poverty estimation; (ii) a uniform poverty line basket (PLB) across rural and urban India; (iii)
a change in the price adjustment procedure to correct spatial and temporal issues with price
adjustment; and (iv) Incorporation of private expenditure on health and education while
estimating poverty. The Committee recommended using Mixed Reference Period (MRP)
based estimates, as opposed to Uniform Reference Period (URP) based estimates that were
used in earlier methods for estimating poverty.
It based its calculations on the consumption of the following items: cereal, pulses, milk,
edible oil, non-vegetarian items, vegetables, fresh fruits, dry fruits, sugar, salt & spices,
other food, intoxicants, fuel, clothing, footwear, education, medical (non-institutional and
institutional), entertainment, personal & toilet goods, other goods, other services and
durables.
The Committee computed new poverty lines for rural and urban areas of each state. To do
this, it used data on value and quantity consumed of the items mentioned above by the
population that was classified as poor by the previous urban poverty line. It concluded that
the all India poverty line was Rs 446.68 per capita per month in rural areas and Rs 578.80
per capita per month in urban areas in 2004-05.
Q.12) Consider the following statements regarding Pradhan Mantri Awaas Yojana
(Gramin):
a) 1 and 2 only
b) 1 only
c) 2 and 3 only
d) 1 and 3 only.
Prime Minister today formally launched Housing for All in rural areas under which the
Government proposes to provide an environmentally safe and secure pucca house to every
rural household by 2022. Named the Pradhan Mantri Awaas Yojana (Gramin), in its first
phase the target is to complete one crore houses by March 2019. The unit cost for these
houses has been significantly increased and now through convergence a minimum support
of nearly Rs. 1.5 lakh to Rs. 1.6 lakh to a household is available. There is also a provision of
Bank loan upto Rs. 70,000/-, if the beneficiary so desires. The selection of beneficiaries has
been through a completely transparent process using the Socio Economic Census 2011 data
and validating it through the Gram Sabha
The scheme would enable people in rural areas to construct new houses or add to their
existing pucca houses to improve their dwelling units. The beneficiary who takes a loan
under the scheme would be provided interest subsidy for loan amount upto Rs. 2 Lakhs.
National Housing Bank would implement the scheme. The Government would provide net
present value of the interest subsidy of 3 percent to the National Housing Bank upfront
which will, in turn, pass it to the Primary Lending Institutions (Scheduled Commercial Banks,
NBFCs etc.). As a result the equated monthly installment (EMI) for the beneficiary would be
reduced.
Under the scheme, the Government would also take necessary steps for proper
convergence with PMAY-G including technical support to beneficiary through existing
arrangements. The new scheme is expected to improve housing stock in the rural areas, as
well as create employment opportunities in rural housing sector
Source: http://pib.nic.in/newsite/PrintRelease.aspx?relid=153931
Q.13) Consider the following statements regarding Pradhan Mantri Garib Kalyan Yojana
a) 1 and 2 only
b) 2 and 3 only
c) 1 and 3 only
d) None of the above.
Pradhan Mantri Garib Kalyan Yojana, 2016 (PMGKY) : The declarant under this regime shall
be required to pay tax @ 30% of the undisclosed income, and penalty @10% of the
undisclosed income. Further, a surcharge to be called Pradhan Mantri Garib Kalyan Cess
@33% of tax is also proposed to be levied. In addition to tax, surcharge and penalty (totaling
to approximately 50%), the declarant shall have to deposit 25% of undisclosed income in a
Deposit Scheme to be notified by the RBI under the Pradhan Mantri Garib Kalyan Deposit
Scheme, 2016. This amount is proposed to be utilised for the schemes of irrigation,
housing, toilets, infrastructure, primary education, primary health, livelihood, etc., so that
there is justice and equality.
a) Skill development
b) Self-employment
c) Education for urban poor
d) Urban shelters
The Government today announced an overarching scheme for uplift of urban and rural poor
through enhancement of livelihood opportunities through skill development and other
means. The scheme has been named as Deen Dayal Antyodaya Yojana DAY. The
announcement was made today by Shri M.Venkaiah Naidu, Minister of Housing & Urban
Poverty Alleviation and Shri Nitin Gadkari, Minister of Rural Development at a National
Convention on Skills for Rural and Urban Poor.
The Minister further informed that under the current urban poverty alleviation
programmes, only 790 cities and towns are covered and the government has decided to
extend these measures to all the 4,041 statutory cities and towns, there by covering almost
the entire urban population.
Announcing the details of urban component of DAY, Shri Venkaiah Naidu said, Rs.1,000 cr
has been provisioned for urban poverty alleviation during 2014-15. Out of this, Rs.500 cr will
be spent on skill development of over 5,00,000 urban poor. He said, for realizing the Make
in India objective, skill development is essential. He observed that If India is to emerge as
the manufacturing base to meet global needs, the only certain way is to empower every
youth of the country with the necessary skills. Skill development has multiple outcomes
including enhancing employment opportunities, stimulating economic growth and
promoting self-worth of beneficiaries.
Shri Venkaiah Naidu informed that under the urban component of DAY, focus will be on:
IASbabas Prelims 60 Day Plan Day 10 2017
1. Imparting skills with an expenditure of Rs.15, 000 Rs.18, 000 on each urban poor;
3. Training urban poor to meet the huge demand from urban citizens by imparting market
oriented skills through City Livelihood Centres. Each Centre would be given a capital grant of
Rs.10.00 lakhs.
4. Enabling urban poor form Self-Help Groups for meeting financial and social needs with a
support of Rs.10,000/- per each group who would in turn would be helped with bank
linkages;
6. Construction of permanent shelters for urban homeless and provision of other essential
services.
Q.15) Which of the following currencies are not a part of SDR Basket of IMF?
a) Chinese Renminbi.
b) Indian Rupee.
c) Japanese Yen.
d) British Pound.
The International Monetary Fund (IMF) announced the launch of the new Special Drawing
Right (SDR) valuation basket including the Chinese renminbi (RMB), and the new currency
amounts that will determine the value of the SDR during the new valuation period.
As approved by the Executive Board of the IMF on November 30, 2015, effective October 1,
2016, the RMB is determined to be a freely usable currency and will be included in the SDR
basket as a fifth currency, along with the U.S. dollar, the euro, Japanese yen, and the British
pound. The Board also decided at that time that the weights of each currency would be
41.73 percent for the U.S. dollar, 30.93 percent for the Euro, 10.92 percent for the Chinese
yuan, 8.33 percent for the Japanese yen, and 8.09 percent for the Pound sterling.
a) Pleistocene
IASbabas Prelims 60 Day Plan Day 10 2017
b) Pliocene
c) Anthropocene
d) Oligocene
Source: http://www.thehindu.com/sci-tech/science/Anthropocene-why-this-naming-has-
risen-in-the-earth%E2%80%99s-history/article14640054.ece
Q.17) Under the Scheduled Tribes and Other Traditional Forest Dwellers (Recognition of
Forest Rights) Act, 2006, who shall be the authority to initiate the process for determining
the nature and extent of individual or community forest rights or both?
As per the Forest Rights Act, 2006 (FRA), the Gram Sabha has been assigned substantial role
for implementation of the provisions of the Act.
Under the Forest Rights Act, 2006, the Gram Sabha has been assigned the following roles for
implementing the provisions of the Act:
To initiate the process for determining the nature and extent of individual or community
forest rights or both that may be given to the forest dwelling Scheduled Tribes and other
traditional forest dwellers within the local limits of its jurisdiction under the Act by receiving
claims, consolidating and verifying them and preparing a map delineating the area of each
recommended claim in such manner as may be prescribed for exercise of such rights and
then pass a resolution to that effect and thereafter forward a copy of the same to the Sub-
Divisional Level Committee.
IASbabas Prelims 60 Day Plan Day 10 2017
To protect the wild life, forest , biodiversity, adjoining catchments areas, water sources,
other ecological sensitive areas, preserve the habitat of forest dwelling Scheduled Tribes
and other traditional forest dwellers from any form of destructive practices affecting their
cultural and natural heritage etc. [Section 5];and
To pass a resolution against any higher authority by giving a notice of not less than sixty
days to the State Level Monitoring Committee and the State Level Monitoring Committee
does not proceed against such authority for enabling the Court to take cognizance of any
offence under Section 7 of the Act [Section 8].
Source: http://www.business-standard.com/article/current-affairs/congress-govt-in-
himachal-challenges-tribal-rights-in-sc-116090400413_1.html
Q.18) Consider the following statements about 25th World Conservation Congress
a) Only 1
b) Only 2
c) Both 1 and 2
d) Neither 1 nor 2
The International Union for Conservation of Nature (IUCN) conducted its 25th World
Conservation Congress in Hawaii. The last Congress was held in Jeju, South Korea in 2012.
IASbabas Prelims 60 Day Plan Day 10 2017
Theme: The theme for this years IUCN Congress is Planet at the crossroads
Source: http://www.livemint.com/Politics/GgQngTlTI0emWDmXnXVQiI/10day-IUCN-
World-Conservation-Congress-begins.html
1. It is the largest river island in the world after Marajo Island in Brazil
2. It is the abode of the Assamese neo-Vaisnavite culture
a) Only 1
b) Only 2
c) Both 1 and 2
d) Neither 1 nor 2
Majuli Island, located amidst the river Brahmaputra in Assam, has recently been declared
the largest river island in the world. It toppled Marajo Island in Brazil, according to Guinness
World Records.
The island is formed by the Brahmaputra in the south and the Kherkutia Xuti, an anabranch
of the Brahmaputra, joined by the Subansiri River in the north.
The island was formed due to course changes by the river Brahmaputra and its tributaries,
mainly the Lohit.
The island is inhabited by Mising, Deori and Sonowal Kachri tribes speaking Mising,
Assamese and Deori language.
It has been included in the tentative list of World Heritage Site by UNESCO.
Source: http://www.thehindu.com/todays-paper/tp-in-school/World-heritage-status-for-
Majuli-soon/article14626783.ece
IASbabas Prelims 60 Day Plan Day 10 2017
a) Only 1
b) Only 2
c) Both 1 and 2
d) Neither 1 nor 2
Scrub typhus, also known as bush typhus because the mites (Leptotrombidium deliense,
commonly known as trombiculid mite) that cause it reside in vegetation predominantly
comprising small shrubs, is an infectious disease with symptoms similar to any viral fever.
However, it is not caused by a virus, but by a parasite called Orientia tsutsugamushi, which
is transmitted by the bite of infected mite larvae in soil containing scrub vegetation. The
disease is more common during the wet season when the mites lay eggs.
Himachal Pradesh is one of the endemic areas for the disease it is also endemic in
Pakistan, Afghanistan, Thailand, northern Australia, the Arabian peninsula and Japan. It is an
occupational disease frequently found in people who work in the fields and are in the habit
of gardening.
In Himachal Pradesh, the disease spreads most commonly during grass cutting operations in
the orchards, and is alleged to have taken a serious turn this year due to laxity in controlling
the mite.
Source: http://www.thehindu.com/news/national/other-states/Centre-assures-support-
to-H.P.-to-check-spread-of-scrub-typhus/article15003156.ece
Q.21) Consider the following statements about Indian Seasonal Energy Efficiency Ratio
(ISEER)
a) Only 1
b) Only 2
c) Both 1 and 2
d) Neither 1 nor 2
Indian Seasonal Energy Efficiency Ratio (ISEER) is an evolved rating methodology for air
conditioners that factors in variance in higher temperature in India and rates air
conditioners accordingly. ISEER measures energy efficiency of air conditioners based on a
weighted average of the performance at outside temperatures between 24 and 43 degree C
based on Indian weather data.
As per Indian Weather Data Handbook, 2014, weather profile of 54 major cities shows that
65% of the total number of hours in a year have a temperature above 24 deg C (5778 hours
out of 8760). Air conditioners in India have hitherto been tested under the IS 1391 at a
standard operating conditions of outside temperature of 35 degree C. Star rating is given to
manufacturers based on the test results provided by them as tested on the above standard.
ISEER is the ratio of the total annual amount of heat that the equipment can remove from
the indoor air when operated for cooling in active mode to the total annual amount of
energy consumed by the equipment during the same period.
The method of evaluation of ISEER is based on bin hours of national climatic zone, bin
temperature range of 24 - 43C and 1600 operating hours for cooling per annum.
Source: http://www.thehindubusinessline.com/companies/panasonic-to-upgrade-
refrigerant-but-sees-acs-becoming-costlier/article9100785.ece
Q.22) Consider the following statements about Coalition for Epidemic Preparedness
Innovations (CEPI)
a) 1 and 2
b) 2 and 3
c) 1 and 3
d) All of the above
Source: http://www.thehindu.com/sci-tech/science/India-to-lead-global-coalition-to-
fight-epidemics/article14637887.ece
IASbabas Prelims 60 Day Plan Day 11 2017
a) 1 and 2
b) 2 and 3
c) 1 and 3
d) All of the above
Scientists had used a process known as plasmapheresis that is somewhat like kidney dialysis.
It involved the removal, treatment, and return of blood plasma from circulation. Since the
process occurs outside the body it is extracorporeal therapy. This method has potential to
reduce chronic infections that may help humans in the fight against drug resistant
superbugs.
Antimicrobial resistance happens when microorganisms such as viruses, bacteria, fungi and
parasites change when they are exposed to antimicrobial drugs. These microorganisms are
also termed as superbugs. As a result, the medicines or drugs become ineffective and
infections persist in the body futher increasing the risk of spread to others.
a) 1 and 2
b) 2 and 3
c) 1 and 3
d) All of the above
IASbabas Prelims 60 Day Plan Day 11 2017
Q.3) India along with other countries in the South East Asian Region have signed World
Health Organisations (WHO) Call To end Tuberculosis (TB) by 2030 in the region. Which of
the following statements are correct about TB?
1. It is bacterial disease which generally affects the lungs but can also affect other parts
of the body.
2. It is a communicable disease which generally spreads through physical contact with
the infected person.
3. If the TB strain shows resistance to first line drugs, it is called Multiple Drug Resistant
TB (MDR TB).
a) 1 and 2
b) 2 and 3
c) 1 and 3
d) All of the above
India along with other countries in the South East Asian Region have signed World Health
Organisations (WHO) Call To end Tuberculosis (TB) by 2030 in the region. It was signed by
Health ministers from countries in WHO South-East Asia Region (WHO SEARO) during two-
day ministerial meeting towards ending TB in the region held in New Delhi.
You get TB by inhaling TB bacteria that are in the air. The bacteria spread through the air
from one person to another. Bacteria get released into the air by some person who is
already infected with them. When a person with TB of the lungs or throat coughs, sneezes,
sings or talks, droplets containing the bacteria are released into the air. That is why people
who think they may be infectious, may often hold something over their mouth when they
are near to other people. People working in a health clinic may for this reason sometimes
wear a mask.
a) 1 only
b) 2 only
c) Both 1 and 2
d) Neither 1 nor 2
The top biotech regulator in India is Genetic Engineering Appraisal Committee (GEAC). The
committee functions as a statutory body under the Environment Protection Act 1986 of the
Ministry of Environment & Forests (MoEF). It was earlier known as Genetic Engineering
Approval Committee. Under the EPA 1986 Rules for Manufacture, Use, Import, Export and
Storage of Hazardous Microorganisms/Genetically Engineered Organisms or Cells, GEAC is
responsible for granting permits to conduct experimental and large-scale open field trials
and also grant approval for commercial release of biotech crops.
The country has yet to approve commercial cultivation of a GM food crop. The only
genetically modified cash crop under commercial cultivation in India is cotton.
IASbabas Prelims 60 Day Plan Day 11 2017
1) Bt Cotton For the time being, the only genetically modified crop that is under cultivation
in India is Bt cotton which is grown over 10.8 million hectares. Bt cotton was first used in
India in 2002.
2) Bt Brinjal The GEAC in 2007, recommended the commercial release of Bt Brinjal, which
was developed by Mahyco (Maharashtra Hybrid Seeds Company) in collaboration with the
Dharward University of Agricultural sciences and the Tamil Nadu Agricultural University. But
the initiative was blocked in 2010.
3) GM Mustard GEAC has recently given a go ahead for tests of GM mustard before taking
a decision on commercialization.
1. The mission targets full immunisation of children under 2 years and pregnant
women.
2. Currently mission covers seven vaccine preventable diseases.
a) 1 only
b) 2 only
c) Both 1 and 2
d) Neither 1 nor 2
The ultimate goal of Mission Indradhanush is to ensure full immunization with all available
vaccines for children up to two years and pregnant women.
The mission was launched with an aim to cover all the children who are lack vaccination or
are partially vaccinated against seven vaccine preventable diseases which include
diphtheria, whooping cough, tetanus, polio, tuberculosis, measles and hepatitis B, by the
end of the year 2020.
Last year four new vaccines were added in the mission. The new vaccines includes,
inactivated polio vaccine, adult Japanese Encephalitis vaccine, Rotavirus vaccine and
Measles Rubella vaccine.
IASbabas Prelims 60 Day Plan Day 11 2017
Q.6) Rubella has been recently added in the Universal Immunisation program. Consider
the following statements regarding Rubella:
a) 1 only
b) 2 and 3
c) 1 and 3
d) All of the above
Rubella is a contagious, generally mild viral infection that occurs most often in children and
young adults.
Key facts
Rubella is a contagious, generally mild viral infection that occurs most often in
children and young adults.
Rubella infection in pregnant women may cause fetal death or congenital defects
known as congenital rubella syndrome (CRS).
Worldwide, over 100 000 babies are born with CRS every year.
There is no specific treatment for rubella but the disease is preventable by
vaccination.
The rubella virus is transmitted by airborne droplets when infected people sneeze or cough.
Humans are the only known host.
a) Mitochondria
b) Nucleus
c) Golgi Apparatus
d) All of the above
IASbabas Prelims 60 Day Plan Day 11 2017
cell membrane.
endoplasmic reticulum.
Golgi apparatus.
lysosomes.
mitochondria.
nucleus.
perioxisomes.
microfilaments and microtubules.
Deoxyribonucleic acid DNA is a molecule that carries the genetic instructions used in the
growth, development, functioning and reproduction of all known living organisms and many
viruses. DNA and RNA are nucleic acid (Found in the nucleus of a cell) ; alongside proteins,
lipids and complex carbohydrates (polysaccharides), they are one of the four major types of
macromolecules that are essential for all known forms of life. Most DNA molecules consist
of two biopolymer strands coiled around each other to form a double helix.
IASbabas Prelims 60 Day Plan Day 11 2017
The two DNA strands are termed polynucleotides since they are composed of simpler
monomer units called nucleotides. Each nucleotide is composed of one of four nitrogen-
containing nucleobasescytosine (C), guanine (G), adenine (A), or thymine (T)a sugar
called deoxyribose, and a phosphate group.
a) 1 and 2
b) 2 and 3
c) 1 and 3
d) All of the above
Reproductive cloning generally uses "somatic cell nuclear transfer" (SCNT) to create animals
that are genetically identical. This process entails the transfer of a nucleus from a donor
adult cell (somatic cell) to an egg from which the nucleus has been removed, or to a cell
from a blastocyst from which the nucleus has been removed. If the egg begins to divide
normally it is transferred into the uterus of the surrogate mother. Such clones are not
strictly identical since the somatic cells may contain mutations in their nuclear DNA.
Additionally, the mitochondria in the cytoplasm also contains DNA and during SCNT this
mitochondrial DNA is wholly from the cytoplasmic donor's egg, thus the mitochondrial
genome is not the same as that of the nucleus donor cell from which it was produced.
Dolly was the first mammal to have been successfully cloned from an adult somatic cell.
a) 1 and 2
b) 2 and 3
c) 1 and 3
d) All of the above
Highly contagious rotaviruses are the leading cause of severe diarrheal illnesses among
infants and young children in both developed and resource-limited countries. Each year,
rotavirus-induced diarrheal disease kills roughly 435,000 children younger than 5 years old
and hospitalizes an estimated two million children worldwide, largely in developing
countries. The youngest children those between 6 months and 2 years of age are most
vulnerable.
ROTAVAC is a new rotavirus vaccine that consists of a strain of the virus that was isolated,
manufactured and tested in India. The ROTAVAC trial represents a significant victory for
Indias scientific community. Based on the studys successful findings, infants in India will
gain access to a licensed vaccine and its significant protection against severe rotavirus-
induced gastroenteritis.
The World Health Organization recommends the first dose of vaccine be given right after 6
weeks of age. Two or three doses more than a month apart should be given, depending on
the vaccine administered. Because the majority of cases occur between six months and two
years of age, the vaccine is not recommended for use in children over two years of age
Q.10) A new male contraceptive Vasalgel has been developed by the scientists of
California National Primate Research Center. Which of the following statements are
correct about Vasalgel?
a) 1 and 2
b) 2 and 3
c) 1 and 3
IASbabas Prelims 60 Day Plan Day 11 2017
Scientists from California National Primate Research Centre, US have successfully tested a
new male contraceptive Vasalgel that blocks sperm flow on monkeys.
The polymer forms a hydrogel after injection into the sperm-carrying tube vas deferens,
creating a blockage to the passage of sperm rather than cutting it as in vasectomy, to filter
sperms from the fluid ejaculation.
The contraceptive effect of Vasalgel can also be reversed by flushing the material out with a
simple sodium bicarbonate solution.
Significance: The purpose Vasalgel is to prevent pregnancy, not just by eliminating sperm in
larger animals more anatomically. In over a last century, male contraceptive options have
not changed and currently are limited to vasectomy (meant to be permanent) and condoms
and withdrawal (with high pregnancy rates). Vasagel could be the first long-acting, non-
hormonal, potentially reversible male contraceptive to reach market.
1. It is a condition characterised by a slow heart rate, usually fewer than 60 beats per
minute.
2. In this case heart is unable to pump enough oxygen-rich blood to the body during
normal activity or exercise, causing dizziness, shortness, fatigue of breath or fainting
spells.
3. Pacemakers are the most common way to treat bradycardia.
a) 1 only
b) 1 and 2
c) 2 and 3
d) All of the above
Scientists for the first time have successfully implanted worlds smallest pacemaker dubbed
as Micra Transcatheter Pacing System (TPS) in a patient in United States. The pacemaker
about size of large vitamin capsule is for patients with bradycardia, a condition
characterised by a slow heart rate, usually fewer than 60 beats per minute.
Patients with bradycardia have lower heart rate and their heart is unable to pump enough
oxygen-rich blood to the body during normal activity or exercise, causing dizziness,
shortness, fatigue of breath or fainting spells. Pacemakers are the most common way to
treat bradycardia to help restore the hearts normal rhythm and relieve symptoms by
sending electrical impulses to the heart to increase the heart rate.
Q.12) Potency of a cell is its ability to develop into differentiated cells. In the light of this
statement, consider the following statements:
a) 1 only
b) 2 only
c) Both 1 and 2
d) Neither 1 nor 2
Totipotency is the ability of a single cell to divide and produce all of the differentiated cells
in an organism. Spores and zygotes are examples of totipotent cells. In the spectrum of cell
potency, totipotency represents the cell with the greatest differentiation potential.
Totipotent cells can form all the cell types in a body, plus the extraembryonic, or placental,
cells. Embryonic cells within the first couple of cell divisions after fertilization are the only
cells that are totipotent. Pluripotent cells can give rise to all of the cell types that make up
the body; embryonic stem cells are considered pluripotent. Multipotent cells can develop
into more than one cell type, but are more limited than pluripotent cells; adult stem cells
and cord blood stem cells are considered multipotent.
IASbabas Prelims 60 Day Plan Day 11 2017
Q.13) Zika virus was erupted on a large scale in which more than 1.5 million people were
infected. This lead WHO to declare a global health emergency. Which of the following
statements regarding Zika Virus are correct?
a) 1 only
b) 2 only
c) Both 1 and 2
d) Neither 1 nor 2
Zika virus had erupted on a large scale in mid-2015 in which more than 1.5 million people
were infected, mostly in Brazil and other countries in South America. The virus is
transmitted mainly by mosquitos. It causes mild, flu-like symptoms in most people, pregnant
women run the risk of giving birth to babies with severe brain damage. The World Health
Organization (WHO) had declared a global health emergency in February 2016, and declared
it over in November 2016.
Q.14) Which of the following statements are correct about DNA and RNA?
a) 1 only
b) 2 only
c) Both 1 and 2
d) Neither 1 nor 2
DNA RNA
IASbabas Prelims 60 Day Plan Day 11 2017
Structural
Deoxyribonucleic Acid Ribonucleic Acid
Name:
Transfer the genetic code needed
for the creation of proteins from the
nucleus to the ribosome. This
Medium of long-term storage and
Function: process prevents the DNA from
transmission of genetic information.
having to leave the nucleus, so it
stays safe. Without RNA, proteins
could never be made.
A single-stranded molecule in most
Typically a double- stranded molecule
Structure: of its biological roles and has a
with a long chain of nucleotides.
shorter chain of nucleotides.
Long polymer with a deoxyribose and Shorter polymer with a ribose and
phosphate backbone and four different phosphate backbone and four
Bases/Sugars:
bases: adenine, guanine, cytosine and different bases: adenine, guanine,
thymine. cytosine, and uracil.
A-T (Adenine-Thymine), G-C (Guanine- A-U (Adenine-Uracil), G-C (Guanine-
Base Pairing:
Cytosine) Cytosine)
Deoxyribose sugar in DNA is less Ribose sugar is more reactive
reactive because of C-H bonds. Stable because of C-OH (hydroxyl) bonds.
in alkaline conditions. DNA has smaller Not stable in alkaline conditions.
Stability:
grooves where the damaging enzyme RNA on the other hand has larger
can attach which makes it harder for grooves which makes it easier to be
the enzyme to attack DNA. attacked by enzymes.
The helix geometry of DNA is of B-
The helix geometry of RNA is of A-
Form. DNA is completely protected by
Form. RNA strands are continually
the body i.e. the body destroys
Unique Traits: made, broken down and reused.
enzymes that cleave DNA. DNA can be
RNA is more resistant to damage by
damaged by exposure to Ultra-violet
Ultra-violet rays.
rays.
Q.15) Scientists from Ireland have claimed discovery of a new human organ named the
Mesentery that exists in the digestive system. It will be human bodys 79th organ. Which
of the following is the correct function of this organ?
Scientists from Ireland have claimed discovery of a new human organ named the mesentery
that exists in the digestive system. It will be human bodys 79th organ. Mesentery connects
the intestine to the abdomen. The organ was previously thought to consist of fragmented
and disparate structures.
Medical students and researchers can now investigate what role the mesentery might play
in abdominal diseases, which it is hoped could ultimately lead to new treatments. The organ
is a double fold of peritoneum - the lining of the abdominal cavity - that holds our intestine
to the wall of our abdomen.
Q.16) Recently Margarita Island was in news concerning 17th NAM summit. It is a part of
a) Nicaragua
b) Venezuela
c) Guatemala
d) Ecuador
Honble Vice President Shri M. Hamid Ansari lead the Indian delegation at the 17th Summit
of the Non Aligned Movement (NAM) to be held at Margarita Island in Venezuela.
India is one of the founding members of the Non Aligned Movement and India hosted the
7th NAM Summit in 1983 in New Delhi. The last NAM Summit was hosted by Iran in 2012.
The membership of NAM today comprises 53 countries from Africa, 39 from Asia, 26 from
Latin America and the Caribbean and 2 from Europe (Belarus, Azerbaijan). There are 17
countries and 10 international organizations that are Observers at NAM. The Non Aligned
Movement came into being 55 years ago when leaders of 25 developing countries met at
the 1961 Belgrade Conference.
Source: http://www.thehindu.com/news/national/India-seeks-%E2%80%98concrete-
action%E2%80%99-against-terror-at-NAM-summit/article14986834.ece
IASbabas Prelims 60 Day Plan Day 11 2017
a) New Indirect Tax Network (Systems Integration) of the Central Board of Excise and
Customs (CBEC)
b) Addressing educational & livelihood needs of minorities
c) An effort to counter the illicit diversion and trafficking of methamphetamine and
counterfeit medicines
d) An online mechanism to resolve mismatches in income-tax return through end to
end e-service obviating the need to visit income-tax office by the taxpayer
Project SAKSHAM
A New Indirect Tax Network (Systems Integration) of the Central Board of Excise and
Customs (CBEC).
Source: http://pib.nic.in/newsite/PrintRelease.aspx?relid=151194
Debt service ratio is measured by the proportion of total debt service payments (i.e.
principal repayment plus interest payment) to current receipts (minus official transfers) of
Balance of Payments (BoP). It indicates the claim that servicing of external debt makes on
IASbabas Prelims 60 Day Plan Day 11 2017
current receipts and is, therefore, a measure of strain on BoP due to servicing of debt
service obligations.
Source: http://pib.nic.in/newsite/PrintRelease.aspx?relid=150918
1. It has been set up to reduce its import dependence and to promote indigenous
procurement
2. It will be the interface of the Indian Army for a single point contact for all
stakeholders which would help R&D fraternity in developing indigenous solutions
a) Only 1
b) Only 2
c) Both 1 and 2
d) Neither 1 nor 2
ADB
The Indian Army has set up Army Design Bureau to reduce its import dependence
and to promote indigenous procurement.
The Design Bureau has been established as a part of governments Make in India
programme and will integrate all stakeholders (government, industries, academics)
to enhance indigenization.
ADB will provide a better understanding of the Armys requirements to the
academia, research organisations and the industry for developing high tech defence
products.
It will be the interface of the Indian Army for a single point contact for all
stakeholders which would help R&D fraternity in developing indigenous solutions.
It will help the Indian Army to be constantly involved at every stage from
conceptualisation to design to development trails and production to sustenance for
its modernisation needs.
ADB comes in the wake of rapid changes in technology and consequent changes in
war fighting techniques.
Army Design Bureau (ADB) will be similar to Navys Naval Design Bureau (NDB) in
concept but Naval Design Bureau does not actually design weapons but weapons
IASbabas Prelims 60 Day Plan Day 11 2017
Source: http://www.thehindu.com/todays-paper/tp-national/Army-sets-up-design-
bureau-to-reduce-dependence-on-imports/article14617301.ece
Q.20) Which of the following countries are members of East Asia Summit?
1. Australia
2. United States of America
3. Philippines
4. Vietnam
5. India
a) 3, 4 and 5
b) 3 and 4
c) 1, 3, 4 and 5
d) All of the above
The East Asia Summit is a unique Leaders-led forum of 18 countries of the Asia-Pacific region
formed to further the objectives of regional peace, security and prosperity. It has evolved as
a forum for strategic dialogue and cooperation on political, security and economic issues of
common regional concern and plays an important role in the regional architecture.
Established in 2005, EAS allows the principal players in the Asia-Pacific region to discuss
issues of common interest and concern, in an open and transparent manner, at the highest
level. The membership of EAS consists of ten ASEAN Member States (i.e. Brunei Darussalam,
Cambodia, Indonesia, Lao PDR, Malaysia, Myanmar, Singapore, Thailand, the Philippines and
Vietnam), Australia, China, India, Japan, New Zealand, Republic of Korea, Russian Federation
and the USA. EAS is an initiative of ASEAN and is based on the premise of the centrality of
ASEAN.
Source: http://pib.nic.in/newsite/PrintRelease.aspx?relid=149455
IASbabas Prelims 60 Day Plan Day 11 2017
Sohar port of Oman provides a key gateway to the Gulf and central Asia.
Sohar Port and Freezone is a deep-sea port and free zone situated midway between Dubai
and Muscat. With current investments of $25 billion, it is one of the worlds fastest growing
port and freezone developments. It lies at the centre of the global trade routes between
Europe and Asia.
India is now also eyeing a role at Omans strategically located Duqm Port, which is being
developed by the Sultanate of Oman along with a special economic zone as a regional
economic hub. Given the good political relations between Iran and Oman, India is trying to
establish a link between the Duqm and Chabahar ports to boost connectivity.
Duqm is situated on the southeastern side of Oman and can be accessed from the Indian
Ocean. With India taking an interest in port development in the region, the idea was to
develop a Iran-India-Oman link for sea trade. In February this year Oman and Iran launched
a separate shipping route between the Shahid Rajaee Port and Sohar Port to promote trade.
Source: http://www.business-standard.com/article/economy-policy/oman-s-sohar-port-
comes-looking-for-investments-116091000242_1.html
Source: http://www.hindustantimes.com/business-news/india-jumps-19-places-to-35-in-
world-bank-s-logistics-performance-index/story-rHJV3bGKyEOlfmIytARgEI.html
IASbabas Prelims 60 Day Plan Day 12 2017
Q.1) Scientists have discovered new species belonging to the genus Nyctibatrachus. It is a
genus of?
Scientists have discovered seven new frog species belonging to the genus Nyctibatrachus,
commonly known as Night Frogs. This find is a result of five years of extensive explorations
in the Western Ghats global biodiversity hotspot in India. Four out of seven of the new
species are miniature-sized frogs (12.2-15.4 mm), which can comfortably sit on a coin or a
thumbnail. These are among the smallest known frogs in the world.
Unlike other frogs in the genus that are predominantly stream dwelling, the new miniature
frogs were found under damp forest leaf litter or marsh vegetation.
Listed as Vulnerable under IUCN because its Extent of Occurrence is less than 20,000 km2,
its distribution is severely fragmented, and there is continuing decline in the extent and
quality of its forest habitat.
http://timesofindia.indiatimes.com/home/environment/flora-fauna/indian-scientists-
discover-four-new-species-of-smallest-known-frogs-from-the-western-
ghats/articleshow/57281095.cms
www.iasbaba.com Page 1
IASbabas Prelims 60 Day Plan Day 12 2017
Forests and trees support sustainable agriculture. They stabilize soils and climate, regulate
water flows, give shade and shelter, and provide a habitat for pollinators and the natural
predators of agricultural pests. They also contribute to the food security of hundreds of
millions of people, for whom they are important sources of food, energy and income. Yet,
agriculture remains the major driver of deforestation globally, and agricultural, forestry and
land policies are often at odds.
State of the Worlds Forests (SOFO) 2016 shows that it is possible to increase agricultural
productivity and food security while halting or even reversing deforestation, highlighting the
successful efforts of Costa Rica, Chile, the Gambia, Georgia, Ghana, Tunisia and Viet Nam.
Integrated land-use planning is the key to balancing land uses, underpinned by the right
policy instruments to promote both sustainable forests and agriculture.
The International Day of Forests was established on the 21st day of March, by resolution
of the United Nations General Assembly on November 28, 2012
The World Forestry Congress (WFC) is the largest and most significant gathering of the
world's forestry sector and it has been held every six years since 1926 under the auspices of
the Food and Agriculture Organization of the United Nations (FAO), organized by the
government of the host country. It is a forum for the sharing of knowledge and experience
regarding the conservation, management and use of the world's forests, and covers such
issues as international dialogue, socio-economic and institutional aspects,
and forest policies.
www.iasbaba.com Page 2
IASbabas Prelims 60 Day Plan Day 12 2017
1. BIOFIN, Biodiversity Finance is a global partnership to help government plan and pay
for its actions on biodiversity conservation and sustainable use.
2. The United Nations Environment Programme (UNEP) is implementing a global multi
country project on Biodiversity Finance Initiative (BIOFIN)
3. India has joined BIOFIN looking at the financing options to reverse biodiversity loss
a) 1 and 2
b) 2 and 3
c) 1 and 3
d) 1, 2 and 3
http://envfor.nic.in/sites/default/files/press-releases/Press%20Release_BIOFIN.pdf
Q.4) Recently, our scientists of the Botanical Survey of India (BSI) have found a new
species of Zingiber (commonly referred as Ginger) having medicinal properties. In which
part of the country it was discovered?
Scientists of the Botanical Survey of India (BSI) have found a new species of Zingiber
(commonly referred as Ginger) from the Andaman and Nicobar Islands.
The species Zingiber pseudosquarrosum, new to science, belonging to genus Zingiber, was
already used by the local Particularly Vulnerable Tribal Groups ( PVTGs) of the Andamans for
its medicinal values.
www.iasbaba.com Page 3
IASbabas Prelims 60 Day Plan Day 12 2017
During an expedition to north and middle Andaman, one of local guides, who was a
Nicobarese, shared his traditional knowledge about this species, which was so far hidden
from science.
After collecting and systematically analysing the species, scientists found that the new
species indeed possesses ethno-medicinal uses.
The fresh extract [juice] of fleshy tuberous roots is used to treat abdominal pain and anti-
helminthic troubles by Nicobarese and certain other tribal communities,.
http://www.thehindu.com/sci-tech/energy-and-environment/New-Ginger-species-with-
medicinal-properties-found-in-Andamans/article17009348.ece
a) 1 and 3
b) 2 and 3
c) 1 and 2
d) 1, 2 and 3
Impatiens, also called jewel weeds, are seen in pristine forests where moisture content and
relative humidity are high.
They belong to The Balsaminaceae (commonly known as the Balsam family) are
a family of dicotyledonous plants, comprising two genera: Impatiens, which consists of
1000+ species, and Hydrocera, consisting of 1 species
www.iasbaba.com Page 4
IASbabas Prelims 60 Day Plan Day 12 2017
All Impatiens taste bitter and seem to be slightly toxic upon ingestion, causing intestinal
ailments like vomiting and diarrhea. The toxic compounds have not been identified but are
probably the same as those responsible for the bitter taste, likely might
be glycosides or alkaloids.
http://www.thehindu.com/todays-paper/tp-national/two-endangered-plant-species-
spotted/article17370078.ece
The proposed site for INO is located in the Bodi West Hills region, about 2 km from the
nearest village Pudukottai in Pottipuram Panchayat, Theni District of Tamil Nadu. The
nearest major city is Madurai about 110 km away. It is also the nearest airport and a major
railway station.
a) 1 Only
b) 2 Only
c) Both
d) None
www.iasbaba.com Page 5
IASbabas Prelims 60 Day Plan Day 12 2017
Core Zone
Core zone must contain suitable habitat for numerous plant and animal species, including
higher order predators and may contain centres of endemism. Core areas often conserve
the wild relatives of economic species and also represent important genetic reservoirs
having exceptional scientific interest. A core zone being National Park or
Sanctuary/protected/regulated mostly under the Wildlife (Protection) Act, 1972. Whilst
realizing that perturbation is an ingredient of ecosystem functioning, the core zone is to be
kept free from l human pressures external to the system.
Buffer Zone
The buffer zone, adjoins or surrounds core zone, uses and activities are managed in this area
in the ways that help in protection of core zone in its natural condition. These uses and
activities include restoration, demonstration sites for enhancing value addition to the
resources, limited recreation, tourism, fishing, grazing, etc; which are permitted to reduce
its effect on core zone. Research and educational activities are to be encouraged. Human
activities, if natural within BR, are likely to continue if these do not adversely affect the
ecological diversity.
Transition Zone
The transition area is the outermost part of a biosphere reserve. This is usually not delimited
one and is a zone of cooperation where conservation knowledge and management skills are
applied and uses are managed in harmony with the purpose of the biosphere reserve. This
includes settlements, crop lands, managed forests and area for intensive recreation and
other economic uses characteristics of the region.
a) 1 and 2
b) 2 and 3
c) 1 and 3
d) 1, 2 and 3
www.iasbaba.com Page 6
IASbabas Prelims 60 Day Plan Day 12 2017
In news: The Madurai Bench of the Madras High Court, directed the State government to
bring an Act with Prohibitory and Penal clauses to eradicate seemai karuvelam (Prosopis
Juliflora) trees within two months.
It is native to Mexico, South America and the Caribbean. It has become established as
an invasive weed in Africa, Asia, Australia and elsewhere
Its roots are able to grow to a great depth in search of water: in 1960, they were discovered
at a depth of 53 meters (175 feet) at an open-pit mine near Tucson, Arizona, putting them
among the deepest known roots.
http://www.thehindu.com/news/national/tamil-nadu/a-saviourturnedvillain-whose-
tentacles-spread-far-and-wide/article17379253.ece
a) Only 2
b) 1, 2 and 3
c) Only 3
d) None
Mammals are always thought of giving birth to young ones but sometimes it can be
different too. Can you ever think of mammals laying eggs instead of giving birth to babies?
Yes, there are egg laying mammals and they are known as monotremes.
As suggested by egg laying mammal facts, only five species of monotremes are extant, one
of which is duck billed platypus and remaining four belong to the echidna family. These
echidnas are also called as spiny anteaters because they like to have ants and termites in
their diet. Also, due to their pointing spines, they are called spiny anteaters. They are quite
www.iasbaba.com Page 7
IASbabas Prelims 60 Day Plan Day 12 2017
similar with other mammals in few things, but differ in most of the characteristics.
Sometimes, they behave like reptiles, while at other times; they show the characteristics of
birds.
The existing monotreme species are the platypus and four species of echidnas (Total 5)
Though the echidnas and platypuses share similarities as they both are monotremes, but
there do exist certain differences between them. A female platypus lays eggs in the special
holes or burrows while a female echidna, on the other hand, lays egg in her pouch. Female
platypus can lay two eggs at a time while an echidna can lay only one egg at a time. There
are many other differences in their body structure, feeding habits, diet and so on.
Q.10) Scientists have discovered a new species named Kani maranjandu after the Kani
tribe in Kerala. Identify the species?
a) Lizard
b) Frog
c) Tree-living crab
d) Forest Cat
Scientists have discovered a new species of long legged, tree-dwelling crabs in Western
Ghats of Kerala.
The new species named Kani maranjandu after the Kani tribe in Kerala, are substantially
different from other congeners.
http://www.thehindu.com/news/national/new-tree-living-crab-species-found-in-
kerala/article17809088.ece
1. The rules of Eco- Sensitive Zones are declared under Wildlife Protection Act, 1972
2. Flying over protected areas in an aircraft or hot air balloon is prohibited
3. Felling of trees and Rainwater harvesting are regulated around ESZ
www.iasbaba.com Page 8
IASbabas Prelims 60 Day Plan Day 12 2017
a) Only 3
b) 1 and 3
c) 1, 2 and 3
d) None
The Ministry of Environment and Forests (MoEF) has come out with new guidelines to
create eco-sensitive zones (ESZs) around Protected Areas to prevent ecological damage
caused due to developmental activities around National Parks and Wildlife Sanctuaries.
The guidelines include a broad list of activities that could be allowed, promoted, regulated
or promoted. This is an important checklist for conservationists to keep in mind while
identifying threats in ESZs.
The guidelines said activities, including commercial mining, setting of saw mills and
industries causing pollution, commercial use of firewood and major hydro-power projects,
are prohibited in such areas.
It also prohibits tourism activities like flying over protected areas in an aircraft or hot air
balloon, and discharge of effluents and solid waste in natural water bodies or terrestrial
areas.
Felling of trees, drastic change in agriculture systems and commercial use of natural water
resources, including groundwater harvesting and setting up of hotels and resorts, are the
activities regulated in the areas.
Activities permitted in the areas include ongoing agriculture and horticulture practices by
local communities, rainwater harvesting, organic farming, adoption of green technology and
use of renewable energy sources.
The width of the ESZ and type of regulation may vary from protected area to area. However,
as a general principle, the width of the ESZ could go up to 10 kms around the protected
area. The ministry said all states and union territories were asked to forward site-specific
proposals to set up ESZs. But only few states have forwarded the proposals. This ministry
after careful consideration, has therefore, decided to frame guidelines to facilitate the
state/union territory for declaration of eco-sensitive zones around national parks and wild
life sanctuaries.
www.iasbaba.com Page 9
IASbabas Prelims 60 Day Plan Day 12 2017
http://www.moef.gov.in/sites/default/files/1%20Guidelines%20for%20Eco-
Sensitive%20Zones%20around%20Protected%20Areas.pdf
http://timesofindia.indiatimes.com/city/hyderabad/extinct-birds-to-fly-from-new-
habitat/articleshow/56770355.cms
Q.13) The Montreux Record is a register of wetland sites on the List of Wetlands of
International Importance where changes in ecological character have occurred, are
occurring, or are likely to occur as a result of technological developments, pollution or
other human interference. It is maintained as part of the Ramsar List.
List of Indian wetlands of International importance included in the Montreux Record are:
1. Chilika Lake
2. Loktak Late
3. Keoladeo National Park
4. Wular Lake
a) 1 and 2
b) 2, 3 and 4
c) 2 and 3
d) 1, 2 and 4
www.iasbaba.com Page 10
IASbabas Prelims 60 Day Plan Day 12 2017
The Montreux Record is a register of wetland sites on the List of Wetlands of International
Importance where changes in ecological character have occurred, are occurring, or are likely
to occur as a result of technological developments, pollution or other human interference. It
is maintained as part of the Ramsar List.
List of Indian wetlands of International importance included in the Montreux Record are
Note: Chilika Lake was removed from the register in 2002 in light of the improved conditions
of the lake. In fact, Chilika lake is the first Ramsar site in Asia to be removed from the
Montreux record
http://ramsar.rgis.ch/cda/en/ramsar-documents-montreux-montreux-
record/main/ramsar/1-31-118%5E20972_4000_0__
Q.14) Consider the following about The Slim Ali Centre for Ornithology and Natural
History (SACON)
a) 1 and 3
b) 2 and 3
c) 1 and 2
d) 1, 2 and 3
The necessity for the study and conservation of birds in particular, and wildlife and
biodiversity in general, prompted the Ministry of Environment and Forests, Government of
India to establish the Salim Ali Centre for Ornithology & Natural History as a public - NGO
partnership between the MoEF, and the Bombay Natural History Society (BNHS).
www.iasbaba.com Page 11
IASbabas Prelims 60 Day Plan Day 12 2017
The Salim Ali Centre for Ornithology & Natural History (SACON) is a Society registered
(autonomous body) in 1990 under the Societies registration Act, with the object of
establishing and developing a Centre of Excellence to assist, institute, conduct and promote
scientific research in ornithology, and of species, habitats and ecosystems with and within
which avifauna coexist, and developing scientific solutions to species, habitat and landscape
conservation problems that are sensitive to the socio-economic realities and aspirations of
the people.
The core objectives of SACON, to which all work of the organisation are directed, are:
Design and conduct research in ornithology covering all aspects of biodiversity and
natural History,
Develop and conduct regular courses in ornithology and natural history
for MSc, MPhil and PhD candidates and short-term orientation courses in the above
subjects,
Create a data bank on Indian ornithology and natural history,
Disseminate knowledge relating to ornithology and natural history for the benefit of
the community.
To confer honorary awards and other distinctions to persons who have rendered
outstanding services in the fields of ornithology and natural history.
Q.15) Mangroves for the Future (MFF) is a unique partner-led initiative to promote
investment in coastal ecosystem conservation for sustainable development. It is
developed by
The devastation caused by the Indian Ocean tsunami of December 2004 laid bare the vital
link between coastal ecosystems and human livelihoods. It was United States President Bill
Clintons vision that rebuilding in tsunami-hit areas should improve natural infrastructure
and strengthen resilience against future natural disasters.
In response to this vision, IUCN (International Union for Conservation of Nature) and the
United Nations Development Programme (UNDP) developed Mangroves for the Future in
2006.
www.iasbaba.com Page 12
IASbabas Prelims 60 Day Plan Day 12 2017
Since then, MFF has grown to include eight institutional partners, plus a growing number of
countries. At the launch of the Tsunami Legacy report* at the United Nations in New York in
April 2009, Bill Clinton acclaimed MFF as one of the most positive and forward-looking
developments of the post-tsunami period.
Mangroves for the Future (MFF) is a unique partner-led initiative to promote investment
in coastal ecosystem conservation for sustainable development. Co-chaired by IUCN and
UNDP, MFF provides a platform for collaboration among the many different agencies,
sectors and countries which are addressing challenges to coastal ecosystem and livelihood
issues. The goal is to promote an integrated ocean-wide approach to coastal management
and to building the resilience of ecosystem-dependent coastal communities.
Mangroves are the flagship of the initiative, but MFF is inclusive of all types of coastal
ecosystem, such as coral reefs, estuaries, lagoons, sandy beaches, seagrass and wetlands.
1. They are the largest ethnic group within Chittagong Hill Tracts
2. Supreme Court directed the Union of India and the State of Arunachal Pradesh to
process the citizenship applications of the tribe
3. They follow the Theravada form of Buddhism
a) Tanchangya
b) Chamka
c) Marma
d) Pankho
The Chakmas, also known as the Daingnet people, are an ethnic group concentrated in
Chittagong area of Bangladesh.
Within the Chittagong Hill Tracts, the Chakmas are the largest ethnic group and make up
half of the region's population. The Chakmas are divided into 46 clans or Gozas. They have
their own language, customs and culture, and profess Theravada Buddhism. The community
is headed by the Chakma Raja.
www.iasbaba.com Page 13
IASbabas Prelims 60 Day Plan Day 12 2017
During the period from 1964-68, around 15,000 Chakmas had fled from the Chittagong Hill
tracts of the then East Pakistan to escape the oppression of Pakistani rulers. After migration
to India, they settled in the then Northeastern Front Agency (NEFA), currently Arunachal
Pradesh.
However, the delegation members complained that not a single Chakma migrant in
Arunachal Pradesh has been granted citizenship rights till date despite specific directions of
the Supreme Court of India issued on January 9, 2016 in the case National Human Rights
Commission (NHRC) vs. State of Arunachal Pradesh and others. Later on, following another
writ petition, the Supreme Court in its judgment dated September 17, 2016 directed the
Union of India and the State of Arunachal Pradesh to process the citizenship applications.
China and Russia have carried out joint naval exercises dubbed as Joint Sea-2016 in the
resource-rich South China Sea (SCS) off Chinas southern Guangdong province.
The exercises follow a spike in tensions after Permanent Court of Arbitration in The Hague
rejected Chinas claims in the SCS and slammed it for causing environmental damage there.
Source: http://indianexpress.com/article/world/world-news/china-russia-naval-drill-in-
south-china-sea-to-begin-monday-3025585/
a) Only 3
www.iasbaba.com Page 14
IASbabas Prelims 60 Day Plan Day 12 2017
b) 1 and 3
c) 2 and 3
d) All of the above
Polymetallic nodules (also known as manganese nodules) are potato-shaped, largely porous
nodules found in abundance carpeting the sea floor of world oceans in deep sea. Besides
manganese and iron, they contain nickel, copper, cobalt, lead, molybdenum, cadmium,
vanadium, titanium, of which nickel, cobalt and copper are considered to be of economic
and strategic importance. India signed a 15 year contract for exploration of Polymetallic
Nodules (PMN) in Central Indian Ocean Basin with the International Seabed Authority (ISA)
(an Institution set up under the Convention on Law of the Sea to which India is a Party) on
25th March, 2002 with the approval of Cabinet. India is presently having an area of 75,000
sq.km., located about 2000 km away from her southern tip for exploration of PMN.
Ministry of Earth Sciences is carrying out Survey & Exploration, Environmental Impact
Assessment, Technology Development (Mining and Extractive Metallurgy) under
polymetallic nodules program through various national institutes viz. National Institute of
Oceanography (NIO), Institute of Minerals and Materials Technology (IMMT), National
Metallurgical Laboratory (NML), National Centre for Antarctica and Ocean Research
(NCAOR), National Institute of Ocean Technology (NIOT) etc., in accordance with the
Contract provisions. India is fulfilling all the obligations of the contract.
Source: http://www.business-standard.com/article/news-ani/union-cabinet-approves-
extension-of-contract-between-india-international-seabed-authority-for-poly-metallic-
nodules-exploration-116091200439_1.html
a) Only 1
b) Only 2
c) Both 1 and 2
www.iasbaba.com Page 15
IASbabas Prelims 60 Day Plan Day 12 2017
d) Neither 1 nor 2
As per the governments 2006 rural electrification policy, a village is deemed electrified if
basic infrastructure such as distribution transformer and distribution lines has been set up in
the inhabited locality, including a Dalit basti.
The top five states which have got largest number of unelectrified rural households are
Bihar, Uttar Pradesh, Assam, Jharkhand and Odisha. Under rural electrification scheme, the
households which are below poverty line (BPL) should get free electricity connections from
the government.
The top five states which have got the highest number of unelectrified Below Poverty Line
(BPL) households currently are Uttar Pradesh, Bihar, Jammu and Kashmir, Madhya Pradesh
and Manipur. As per REC data, only 57 per cent of the countrys rural BPL households have
been electrified.
REC is the nodal agency to operate and monitor rural electrification and it receives 0.5 per
cent of the total project cost as its fees.
Source: http://indianexpress.com/article/india/india-news-india/electricity-in-india-
villages-problems-still-no-light-poverty-3030107/
Q.20) Mangalajodi, the second largest hotspot for birds on the banks of
a) Pulicat
b) Wullar
c) Loktak
d) None of the above
Mangalajodi, the second largest hotspot for birds on the banks of Chilika Lake in Odisha, is
set to be developed into an ecotourism destination.
Source: http://www.thehindu.com/todays-paper/tp-
national/MANGALAJODI/article14639089.ece
www.iasbaba.com Page 16
IASbabas Prelims 60 Day Plan Day 12 2017
Q.21) Operation Morthor was in news recently. It is concerned with which of the following
countries?
a) Congo
b) Somalia
c) Sri Lanka
d) Laos
Source: http://www.livemint.com/Opinion/9D5XT497AEYYluFn3moa2H/When-Indian-
troops-entered-Congo-55-years-ago.html (Read through it once)
a) Laos
b) Cambodia
c) Thailand
d) Malaysia
14th ASEAN-India Summit and the 11th East Asia Summit was held in Vientiane, Lao PDR.
It is the capital and largest city of Laos, on the banks of the Mekong River near the border
with Thailand. Vientiane became the capital in 1563 due to fears of a Burmese invasion, but
was later razed and looted to the ground in 1827 by the Siamese (Thai).Vientiane was the
administrative capital during French rule and, due to economic growth in recent times, is
now the economic centre of Laos.
Source: http://pib.nic.in/newsite/PrintRelease.aspx?relid=149455
Q.23) PARAM-ISHAN is a
www.iasbaba.com Page 17
IASbabas Prelims 60 Day Plan Day 12 2017
PARAM-ISHAN have power of 250 Teraflops and three hundred tera bites capacity.
Source: http://pib.nic.in/newsite/PrintRelease.aspx?relid=150905
Q.24) Consider the following statements about National Company Law Tribunal (NCLT)
a) Only 1
b) 1 and 2
c) Only 2
d) 1 and 3
The Central Government has constituted National Company Law Tribunal (NCLT) under
section 408 of the Companies Act, 2013 (18 of 2013) w.e.f. 01st June 2016.
In the first phase the Ministry of Corporate Affairs have set up eleven Benches, one
Principal Bench at New Delhi and one each Regional Benches at New Delhi, Ahmedabad,
Allahabad, Bengaluru, Chandigarh, Chennai, Guwahati, Hyderabad, Kolkata and Mumbai.
These Benches will be headed by the President and 16 Judicial Members and 09 Technical
Members at different locations.
www.iasbaba.com Page 18
IASbabas Prelims 60 Day Plan Day 12 2017
The National Company Law Tribunal has been constituted w.e.f. 1st June,2016 and by virtue
of Section 466(1) of Companies Act,2013, the Company Law Board stands dissolved.
In News - http://www.thehindu.com/business/no-repeat-of-2g-satyam-if-laws-are-
sincerely-implemented-says-nclt-president/article9118738.ece
Q.25) The term Levant is in news. It is concerned with which of the following countries?
1. Jordan
2. North Sudan
3. Egypt
4. Lebanon
5. Syria
a) 1, 2 and 3
b) 1, 3, 4 and 5
c) 4 and 5
d) 1, 4 and 5
The term is employed to refer to modern events, peoples, states or parts of states in the
same region, namely Cyprus, Egypt, Iraq, Israel, Jordan, Lebanon, Palestine, Syria, and
Turkey are sometimes considered Levant countries.
Source: http://www.thehindu.com/opinion/op-ed/the-full-circle-of-
reason/article9111688.ece
www.iasbaba.com Page 19
IASbabas Prelims 60 Day Plan Day 13 2017
TOPICS:
Constitutional Amendment
Union Executive
PRELIMS MCQs:
Q.1) In the Indian Constitution which of the following provisions can be amended by a Simple
Majority?
a) 2 and 3 only
b) 2 and 4 only
c) 2, 3 and 4
d) 1, 2 and 4
Q.2) Basic structure as a new doctrine was laid down by which of the following?
a) Golaknath case
b) Minerva Mills case
c) Kesavananda Bharati case
d) Shankari prasad case
The basic structure doctrine is an Indian judicial principle that the Constitution of
India has certain basic features that cannot be altered or destroyed
through amendments by the parliament
Key among these "basic features", are the fundamental rights granted to individuals by
the constitution.
The basic structure doctrine applies only to constitutional amendments.
The basic features of the Constitution have not been explicitly defined by the Judiciary,
and the claim of any particular feature of the Constitution to be a "basic" feature is
determined by the Court in each case that comes before it.
Basic structure as a new doctrine was laid down by a land mark case , Kesavananda
Bharati case
Which of the above is/are the feature[s] of Basic Structure of the Constitution?
IASbabas Prelims 60 Day Plan Day 13 2017
a) 1, 2 ,3 and 5
b) 2,3 and 5 only
c) 2, 3, 4 and 5
d) 1, 2, 3, 4 and 5
1. To promote equal justice and to provide free legal aid to the poor
2. To secure for all citizens a uniform civil code throughout the country
3. To protect and improve the environment and to safeguard forests and wild life
IASbabas Prelims 60 Day Plan Day 13 2017
Which of the following Directive Principles were added to the original list by 42nd
Amendment Act of 1976?
a) 1 only
b) 1 and 3 only
c) 2 and 3 only
d) 1,2 and 3
42nd Amendment Act of 1976 added following Directive Principles to the original list:
Q.5) With regard to President of India, which of the following statement is incorrect?
The President holds office for a term of five years from the date on which he enters upon his
office. However, he can resign from his office at any time by addressing the resignation letter
to the Vice President. Further, he can also be removed from the office before completion of his
term by the process of impeachment.
IASbabas Prelims 60 Day Plan Day 13 2017
The President can hold office beyond his term of five years until his successor assumes charge.
He is also eligible for re-election to that office. He may be elected for any number of terms.
However, in USA, a person cannot be elected to the office of the President more than twice.
A person to be eligible for election as President should fulfil the following qualifications:
4. He should not hold any office of profit under the Union government or any state government
or any local authority or any other public authority. A sitting President or Vice-President of the
Union, the Governor of any state and a minister of the Union or any state is not deemed to hold
any office of profit and hence qualified as a presidential candidate. Further, the nomination of a
candidate for election to the office of President must be subscribed by at least 50 electors as
proposers and 50 electors as seconders.
The President has veto power with respect to state legislation also. A bill passed by a state
legislature can become an act only if it receives the assent of the governor or the President (in
case the bill is reserved for the consideration of the President).
When a bill, passed by a state legislature, is presented to the governor for his assent, he has
four alternatives (under Article 200 of the Constitution):
3. He may return the bill (if it is not a money bill) for reconsideration of the state legislature
Q.6) In case of vacancy (caused by the expiration of the term of the sitting President) in the
Presidents office, which of the following statements is correct?
1. An election to fill the vacancy must be held before the expiration of the term.
IASbabas Prelims 60 Day Plan Day 13 2017
2. In case of any delay in conducting the election of new President, the Vice-President
discharges the functions of the President.
a) 1 Only
b) 2 Only
c) Both 1 and 2
d) Neither 1 nor 2
When the vacancy in the Presidents office is going to be caused by the expiration of the
term of the sitting President, an election to fill the vacancy must be held before the
expiration of the term.
In case of any delay in conducting the election of new President by any reason, the
outgoing President continues to hold office (beyond his term of five years) until his
successor assumes charge. This is provided by the Constitution in order to prevent an
interregnum. In this situation, the Vice-President does not get the opportunity to act
as President or to discharge the functions of the President.
If the office falls vacant by resignation, removal, death or otherwise, then election to
fill the vacancy should be held within six months from the date of the occurrence of
such a vacancy. The newly elected President remains in office for a full term of five years
from the date he assumes charge of his office.
When a vacancy occurs in the office of the President due to his resignation, removal,
death or otherwise, the Vice-President acts as the President until a new President is
elected. Further, when the sitting President is unable to discharge his functions due to
absence, illness or any other cause, the Vice-President discharges his functions until
the President resumes his office.
In case the office of Vice-President is vacant, the Chief Justice of India (or if his office is
also vacant, the senior most judge of the Supreme Court available) acts as the President
or discharges the functions of the President.
Q.7) With regard to procedure for amendment of the Constitution as per Article 368, consider
the following statements
a) 1 and 3 only
b) 1 and 2 only
c) 3 only
d) 1 only
Amendment bill can be introduced not only by minister but also by private member
and that too without the prior permission of the president.
Amendment bill is introduced in either houses of the parliament and must be passed
with Special majority in each house separately.
Once Amendment bill duly passed by both the houses of the parliament then the bill is
presented to president for the assent. President must give assent to the bill. President
can neither withhold his assent to the amendment bill nor send it back for
reconsideration.
The basic structure doctrine is an Indian judicial principle that the Constitution of India
has certain basic features that cannot be altered or destroyed through amendments by
the parliament.
By 1973, the basic structure doctrine triumphed in the landmark decision of
Kesavananda Bharati v. State of Kerala. Previously, the Supreme Court had held that the
power of Parliament to amend the Constitution was unfettered. However, in this
landmark ruling, the Court adjudicated that while Parliament has "wide" powers, it did
not have the power to destroy or emasculate the basic elements or fundamental
features of the constitution.
Key among these "basic features", are the fundamental rights granted to individuals by
the constitution.
IASbabas Prelims 60 Day Plan Day 13 2017
The doctrine thus forms the basis of a limited power of the Supreme Court to review
and strike down constitutional amendments enacted by the Parliament which conflict
with or seek to alter this "basic structure" of the Constitution.
The basic features of the Constitution have not been explicitly defined by the Judiciary,
and the claim of any particular feature of the Constitution to be a "basic" feature is
determined by the Court in each case that comes before it.
The basic structure doctrine applies only to constitutional amendments.
The basic structure doctrine does not apply to ordinary Acts of Parliament, which must
itself be in conformity with the Constitution.
98th amendment:
This board will see that sufficient funds are allocated for Development of the region.
There is a need for a broad based National Judicial Appointment Commission (NJAC), for making
recommendations for selection of judges.
The Bill seeks to enable equal participation of Judiciary and Executive, ensure that the
appointments to the higher judiciary are more participatory, transparent and objective.
Article 124 (2) of the Constitution provides that the President will make appointments of
Supreme Court (SC) and High Court(HC) judges after consultation with the Chief Justice
of India and other SC and HC judges as he considers necessary.
The Bill amends Article 124 (2) of the Constitution to provide for a Commission, to be
known as the National Judicial Appointments Commission (NJAC). The NJAC would then
make recommendations to the President for appointments of SC and HC judges.
A new Article, Article 124A provides for the composition of the NJAC. The NJAC would consist
of:
A new Article, Article 124B, provides for the functions of the NJAC which include:
Recommending persons for appointment as Chief Justice of India, Judges of the
Supreme Court, Chief Justices of High Courts and other Judges of High Courts;
Recommending transfer of Chief Justices and other Judges of High Courts from one High
Court to any other High Court; and
Ensuring that the persons recommended are of ability and integrity.
It is intended to operationalizing the May 1974 LBA that pertains to exchange of certain
enclaves of land between both the countries.
Background
The exchange will see Bangladesh take over 111 enclaves (17160 acres) from Indias
possession and India, in turn, receive 51 enclaves (7110 acres) from Bangladesh.
In Assam, India will get 470 acres of land from Bangladesh, while 268 acres will go to
Bangladesh. Enclaves are tiny landlocked territories that each country has within the
borders of the other nation.
The enclaves in Assam, West Bengal, Tripura and Meghalaya come under the bills
ambit.
a) 1 and 3 only
b) 1 and 2 only
c) 2 and 3 only
d) 1, 2 and 3
Council of Ministers are collectively responsible to the Lok Sabha and Ministers are
Individually responsible to President of India.
Council of ministers cannot exist without the Prime Minister. (Each minister need not
resign separately; the resignation of the prime minister amounts to the resignation of
the entire council of ministers.)
Council of Ministers shall not exceed 15 percent of the total number of members of the
House of People.
1. There shall be a Council of Ministers with the Prime Minister at the head to aid and advise
the President who shall, in the exercise of his functions, act in accordance with such advice.
However, the President may require the Council of Ministers to reconsider such advice and the
President shall act in accordance with the advice tendered after such reconsideration.
2. The advice tendered by Ministers to the President shall not be inquired into in any court.
1. The Prime Minister shall be appointed by the President and the other Ministers shall be
appointed by the President on the advice of the Prime Minister.
2. The total number of ministers, including the Prime Minister, in the Council of Ministers
shall not exceed 15% of the total strength of the Lok Sabha. The provision was added by the
91st Amendment Act of 2003.
IASbabas Prelims 60 Day Plan Day 13 2017
3. A member of either house of Parliament belonging to any political party who is disqualified
on the ground of defection shall also be disqualified to be appointed as a minister. This
provision was also added by the 91st Amendment Act of 2003.
4. The ministers shall hold office during the pleasure of the President.
6. The President shall administer the oaths of office and secrecy to a minister.
7. A minister who is not a member of the Parliament (either house) for any period of six
consecutive months shall cease to be a minister.
Source: NCERT Class IX Indian Constitution at Work Chapter 4: Executive & Chapter-20
Council of Ministers - Indian Polity by M. Laxmikanth
a) 1 Only
b) 2 Only
c) Both 1 and 2
d) Neither 1 nor 2
1. They are extra-constitutional in emergence. In other words, they are not mentioned in the
Constitution. However, the Rules of Business provide for their establishment.
2. They are of two typesstanding and ad hoc. The former are of a permanent nature while the
latter are of a temporary nature. The ad hoc committees are constituted from time to time to
deal with special problems. They are disbanded after their task is completed.
IASbabas Prelims 60 Day Plan Day 13 2017
3. They are set up by the Prime Minister according to the exigencies of the time and
requirements of the situation. Hence, their number, nomenclature, and composition varies
from time to time.
4. Their membership varies from three to eight. They usually include only Cabinet Ministers.
However, the non-cabinet Ministers are not debarred from their membership.
5. They not only include the Ministers in charge of subjects covered by them but also include
other senior Ministers.
6. They are mostly headed by the Prime Minister. Sometimes other Cabinet Ministers,
particularly the Home Minister or the Finance Minister, also acts as their Chairman. But, in case
the Prime Minister is a member of a committee, he invariably presides over it.
7. They not only sort out issues and formulate proposals for the consideration of the Cabinet,
but also take decisions. However, the Cabinet can review their decisions.
8. They are an organisational device to reduce the enormous workload of the Cabinet. They also
facilitate in-depth examination of policy issues and effective coordination. They are based on
the principles of division of labour and effective delegation.
1. A resolution to impeach the President must be passed by a majority of not less than
two-third of those present in the house
2. Members of Legislative Councils of the States participate in the election of the President
of India
3. When the offices of both the President and the Vice-President are vacant, Chief Justice
of India performs the function of the office.
a) 1 and 3 only
b) 1 and 2 only
c) 3 only
d) 1, 2 and 3
a) 2 only
b) 2 and 3 only
c) 1 and 3 only
d) 1, 2 and 3
TYPES OF AMENDMENTS:
Article 368 provides for two types of amendments, that is, by a special majority of Parliament
and also through the ratification of half of the states by a simple majority. But, some other
articles provide for the amendment of certain provisions of the Constitution by a simple
majority of Parliament, that is, a majority of the members of each House present and voting
(similar to the ordinary legislative process). Notably, these amendments are not deemed to be
amendments of the Constitution for the purposes of Article 368.
Q.13) Which of the statements is not true with regard to office of the Vice-President of India?
As per article 66, the candidate contesting for election of Vice-President of India should fulfill
the below conditions:
The Vice-President, like the president, is elected not directly by the people but by the method
of indirect election. He is elected by the members of an electoral college consisting of the
members of both Houses of Parliament. Thus, this electoral college is different from the
electoral college for the election of the President in the following two respects:
1. It consists of both elected and nominated members of the Parliament (in the case of
president, only elected members).
IASbabas Prelims 60 Day Plan Day 13 2017
2. It does not include the members of the state legislative assemblies (in the case of President,
the elected members of the state legislative assemblies are included)
1. He acts as the ex-officio Chairman of Rajya Sabha. In this capacity, his powers and functions
are similar to those of the Speaker of Lok Sabha. In this respect, he resembles the American
vice-president who also acts as the Chairman of the Senatethe Upper House of the American
legislature.
2. He acts as President when a vacancy occurs in the office of the President due to his
resignation, removal, death or otherwise. He can act as President only for a maximum period of
six months within which a new President has to be elected. Further, when the sitting President
is unable to discharge his functions due to absence, illness or any other cause, the Vice-
President discharges his functions until the President resumes his office.While acting as
President or discharging the functions of President, the Vice-President does not perform the
duties of the office of the chairman of Rajya Sabha. During this period, those duties are
performed by the Deputy Chairman of Rajya Sabha.
Q.14) In comparison to the offices of President and Vice-President, consider the following
statements
a) 1 only
b) 2 only
c) 1 and 3 only
d) 3 only
IASbabas Prelims 60 Day Plan Day 13 2017
Q.14) Solution (a)
Q.15) In Indian context, which of the following statements is not true with regard to the
Prime Minister
a) The Constitution does not contain any procedure for the selection and appointment of
the Prime Minister
b) As per the constitutional provisions, the President has to appoint the leader of the
majority party in the Lok Sabha as the Prime Minister
c) A person who is not a member of either House of Parliament can be appointed as Prime
Minister for six months
d) Constitutionally, the Prime Minister may be a member of any of the two Houses of
parliament
The Constitution does not contain any specific procedure for the selection and appointment
of the Prime Minister. Article 75 says only that the Prime Minister shall be appointed by the
president. However, this does not imply that the president is free to appoint any one as the
Prime Minister. In accordance with the conventions of the parliamentary system of
government, the President has to appoint the leader of the majority party in the Lok Sabha as
the Prime Minister. But, when no party has a clear majority in the Lok Sabha, then the
President may exercise his personal discretion in the selection and appointment of the Prime
Minister. In such a situation, the President usually appoints the leader of the largest party or
coalition in the Lok Sabha as the Prime Minister and asks him to seek a vote of confidence in
the House within a month.
IASbabas Prelims 60 Day Plan Day 13 2017
In 1997, the Supreme Court held that a person who is not a member of either House of
Parliament can be appointed as Prime Minister for six months, within which, he should
become a member of either House of Parliament; otherwise, he ceases to be the Prime
Minister. Constitutionally, the Prime Minister may be a member of any of the two Houses of
parliament. For example, three Prime Ministers, Indira Gandhi (1966), Deve Gowda (1996) and
Manmohan Singh (2004), were members of the Rajya Sabha. In Britain, on the other hand, the
Prime Minister should definitely be a member of the Lower House (House of Commons).
Q.16) In India, the Prime Minister is the chairman of which of the following
institution/organizations?
a) 1, 2 and 5 only
b) 1, 3 and 4 only
c) 1, 2, 3 and 5 only
d) 1, 2, 3, 4 and 5
1. NITI Aayog
Source: Chapter-19 Prime Minister Indian Polity by M. Laxmikanth & Current Affairs
In order to protect and increase wild tiger population, nearly one million acres of protected
habitat in India and Bhutan will be covered under a new private conservation efforts.
The 'Project C.A.T - Conserving acres for Tigers' by Discovery Communications and NGO World
Wildlife Fund (WWF) aims to conserve the wild tiger population, which has dropped by 96 per
cent in the last century alone to only 4,000 left in the wild due to habitat loss and pervasive
poaching.
IASbabas Prelims 60 Day Plan Day 13 2017
This transboundary partnership will allow rangers to more closely monitor tiger health and
other key scientific data, take additional anti-poaching safeguards, and maintain land and
corridors to improve movement of all wild animals.
Discovery will also leverage its formidable creative capabilities and global multiplatform
distribution of channels to 3 billion cumulative worldwide viewers to support WWF's Tx2 effort,
which began in 2010, to double the population of wild tigers across the world by 2022.
Source: http://timesofindia.indiatimes.com/home/environment/flora-fauna/Project-to-
increase-wild-tiger-population-in-India-Bhutan/articleshow/54960351.cms
a) 1 and 2
b) 2 and 3
c) 1 and 3
d) All of the above
This species is confined to the Sundaic lowlands, where it is known from south Tenasserim,
Myanmar, peninsular Thailand, Sabah, Sarawak and Peninsular Malaysia, Singapore, Kalimantan
and Sumatra, Indonesia and Brunei (BirdLife International 2001). It is generally scarce and
occurs at low densities even in optimal habitat.
Native:
Regionally extinct:
Singapore
IASbabas Prelims 60 Day Plan Day 13 2017
Poachers arent interested in their brilliant plumage or large bills, but a helmet-like block of
reddish-gold keratin at the front of the skulls known as a casque. Its this soft, ivory-like
substance thats carved by craftsmen in China into luxury ornaments, statues and jewellery -
trendy top-shelf trinkets that have soared in value as so-called red ivory has grown more
prestigious.
In addition to hunting the species for its casque, it is also targeted for its feathers. Although this
trade is small, it exerts an additional pressure which is also likely to contribute to population
declines.
Helmeted hornbills had been traditionally hunted in the past by Borneos indigenous tribes, but
never at levels that posed any conservation risk.
In West Kalimantan, the extinction of this iconic bird would also have far-reaching cultural
ramifications. The helmeted hornbill is revered by ancient forest-dwelling Dayak communities,
and its image adorns flags and sigils across the province.
By the close of 2015, the species had progressed from vulnerable to critically endangered -
leapfrogging two threat levels to the highest possible risk category on the International Union
for Conservation of Natures red list.
The commercial trade of helmeted hornbills is prohibited by law in China and across its habitat
zones in Southeast Asia: Thailand and Myanmar, as well as Malaysia, Brunei and Indonesia - the
three countries that share Borneo island.
Source: http://www.thehindu.com/sci-tech/energy-and-environment/The-helmeted-
hornbill-is-being-hunted-to-extinction/article16073967.ece
Q.19) Borneo Island is politically divided among which of the following countries?
1. Philippines
2. Malaysia
3. Brunei
4. Indonesia
a) 1, 2 and 4
b) 2 and 4
c) 2, 3 and 4
IASbabas Prelims 60 Day Plan Day 13 2017
d) All of the above
It is the third-largest island in the world and the largest island in Asia. At the geographic centre
of Maritime Southeast Asia, in relation to major Indonesian islands, it is located north of Java,
west of Sulawesi, and east of Sumatra.
The island is politically divided among three countries: Malaysia and Brunei in the north, and
Indonesia to the south. Approximately 73% of the island is Indonesian territory. In the north,
the East Malaysian states of Sabah and Sarawak make up about 26% of the island. Additionally,
the Malaysian federal territory of Labuan is situated on a small island just off the coast of
Borneo. The sovereign state of Brunei, located on the north coast, comprises about 1% of
Borneo's land area. Antipodal to an area of Amazon rainforest, Borneo is itself home to one of
the oldest rainforests in the world, and to Bornean orangutans.
Source: http://www.thehindu.com/sci-tech/energy-and-environment/The-helmeted-
hornbill-is-being-hunted-to-extinction/article16073967.ece
The Global Hunger Index (GHI) is a multidimensional statistical tool used to describe the state of
countries hunger situation. The GHI measures progress and failures in the global fight against
hunger. The GHI is updated once a year.
India was placed behind Nepal, Sri Lanka, Bangladesh, among others, but ahead of Pakistan and
three other Asian countries.
India has improved its score on various parameters over the past few years, two out of five
children below five years of age are stunted in India. Stunting measures chronic malnutrition
and affected childrens height would be considerably below the average for their age.
India was still rated with serious hunger levels in the 2016 Index.
India had the lowest rank among BRICS nations, with Brazil in the top 16, Russia at 24, China at
29 and South Africa at 51.
Globally, the Central African Republic, Chad, and Zambia were estimated to have the highest
levels of hunger. For the developing world, the GHI score is estimated to be 21.3 which,
according to the report, is at the lower end of the serious category.
Source: http://www.business-standard.com/article/current-affairs/stomach-this-india-ranks-
dismal-97-in-global-hunger-index-116101100619_1.html
Q.21) Consider the following statements about Janani Suraksha Yojana (JSY)
a) 1 and 2
b) 2 and 3
c) 1 and 3
d) All of the above
The Yojana has identified ASHA, the accredited social health activist as an effective link
between the Government and the poor pregnant women in l0 low performing states, namely
the 8 EAG states and Assam and J&K and the remaining NE States. In other eligible states and
UTs, wherever, AWW (Anganwadi workers) and TBAs or ASHA like activist has been engaged in
this purpose, she can be associated with this Yojana for providing the services.
The scheme focuses on the poor pregnant woman with special dispensation for states
having low institutional delivery rates namely the states of Uttar Pradesh, Uttaranchal,
Bihar, Jharkhand, Madhya Pradesh, Chhattisgarh, Assam, Rajasthan, Orissa and Jammu
and Kashmir. While these states have been named as Low Performing States (LPS), the
remaining states have been named as High performing States (HPS).
Tracking Each Pregnancy: Each beneficiary registered under this Yojana should have a
JSY card along with a MCH card. ASHA/AWW/ any other identified link worker under the
overall supervision of the ANM and the MO, PHC should mandatorily prepare a micro-
birth plan. This will effectively help in monitoring Antenatal Check-up, and the post-
delivery care.
Eligibility for Cash Assistance: BPL Certification This is required in all HPS states.
However, where BPL cards have not yet been issued or have not been updated,
States/UTs would formulate a simple criterion for certification of poor and needy status
of the expectant mothers family by empowering the gram pradhan or ward member.
Source: http://www.thehindu.com/data/Janani-Suraksha-Yojana-pays-dividends-
Study/article15477844.ece
Q.22) Which of the following Ministries deals with the management of Haj Pilgrimage and
including administration of the Haj Committee Act, 1959?
Source: http://www.thehindu.com/news/national/minority-affairs-ministry-takes-charge-of-
haj-pilgrimage/article9176608.ece
The BRICS nations have adopted the New Delhi Declaration on Education to ensure inclusive
and equitable quality education and promote life-long learning opportunities for all.
Source: http://indiatoday.intoday.in/education/story/brics-new-delhi-declaration-on-
education/1/778855.html
IASbabas Prelims 60 Day Plan Day 14 2017
TOPICS:
PRELIMS MCQs:
Q.1) The speaker of the Lok Sabha derives his powers and duties from which of the following
sources?
a) 2 only
b) 1 and 2 only
c) 2 and 3 only
d) All the above
The Speaker of the Lok Sabha derives his powers and duties from three sources, that is, the
Constitution of India, the Rules of Procedure and Conduct of Business of Lok Sabha, and
Parliamentary Conventions (residuary powers that are unwritten or unspecified in the Rules)
The Speaker is the head of the Lok Sabha, and its representative.
He is the guardian of powers and privileges of the members, the House as a whole and
its committees.
He is the principal spokesman of the House, and his decision in all Parliamentary matters
is final. He is thus much more than merely the presiding officer of the Lok Sabha. In
these capacities, he is vested with vast, varied and vital responsibilities and enjoys great
honour, high dignity and supreme authority within the House.
a) 1 only
b) 2 only
c) Both 1 and 2
d) Neither 1 nor 2
Financial bills are those bills that deal with fiscal matters, that is, revenue or
expenditure. However, the Constitution uses the term financial bill in a technical sense.
Financial bills are of three kinds:
1. Money billsArticle 110
2. Financial bills (I)Article 117 (1)
3. Financial bills (II)Article 117 (3)
This classification implies that money bills are simply a species of financial bills. Hence,
all money bills are financial bills but all financial bills are not money bills. Only those
financial bills are money bills which contain exclusively those matter,which are
mentioned in Article 110 of the Constitution.
These are also certified by the Speaker of Lok Sabha as money bills. The financial bills (I)
and (II), on the other hand, have been dealt with in Article 117 of the Constitution.
Hence statement 1 is incorrect.
The Constitution lays down a special procedure for the passing of money bills in the Parliament.
A money bill can only be introduced in the Lok Sabha and that too on the recommendation of
the President. Every such bill is considered to be a government bill and can be introduced only
by a minister. Hence statement 2 is correct.
1. The joint sitting is governed by the Rules of Procedure of Lok Sabha and not of Rajya
Sabha
2. Chairman of Rajya Sabha presides over a joint sitting in the absence of Speaker.
3. Joint sitting is applicable to money bills
a) 1 only
b) 2 only
c) 1 and 3 only
d) All the above
The Speaker of Lok Sabha presides over a joint sitting of the two Houses and the
Deputy Speaker, in his absence.
If the Deputy Speaker is also absent from a joint sitting, the Deputy Chairman of Rajya
Sabha presides. If he is also absent, such other person as may be determined by the
members present at the joint sitting, presides over the meeting.
It is clear that the Chairman of Rajya Sabha does not preside over a joint sitting as he is
not a member of either House of Parliament.
Joint sitting is applicable to ordinary bills or financial bills only and not to money bills
or Constitutional amendment bills. In the case of a money bill, the Lok Sabha has
overriding powers, while a Constitutional amendment bill must be passed by each
House separately.
Hence statements 2 and 3 are incorrect.
The quorum to constitute a joint sitting is one-tenth of the total number of members of
the two Houses. The joint sitting is governed by the Rules of Procedure of Lok Sabha
and not of Rajya Sabha. If the bill in dispute is passed by a majority of the total number
of members of both the Houses present and voting in the joint sitting, the bill is deemed
to have been passed by both the Houses. Normally, the Lok Sabha with greater number
wins the battle in a joint sitting.
The Constitution has specified that at a joint sitting, new amendments to the bill cannot
be proposed except in two cases:
IASbabas Prelims 60 Day Plan Day 14 2017
1. those amendments that have caused final disagreement between the Houses; and
2. those amendments that might have become necessary due to the delay in the
passage of the bill.
Since 1950, the provision regarding the joint sitting of the two Houses has been invoked
only thrice. The bills that have been passed at joint sittings are:
Q.4) Who among the following is/are elected by Single transferable vote system
(Proportional representation)?
a) 2 only
b) 1, 2 and 4
c) 2 and 3 only
d) 1, 2, 3 and 4
Though the Constitution has adopted the system of proportional representation in the
case of Rajya Sabha, it has not preferred the same system as in the case of Lok Sabha.
Instead, it has adopted the system of territorial representation for the election of
members to the Lok Sabha.
The system of proportional representation aims at removing the defects of territorial
representation. Under this system, all sections of the people get representation in
IASbabas Prelims 60 Day Plan Day 14 2017
proportion to their number. Even the smallest section of the population gets its due
share of representation in the legislature.
There are two kinds of proportional representation, namely, single transferable vote
system and list system. In India, the first kind is adopted for the election of members to
the Rajya Sabha and state legislative council and for electing the President and the
Vice-President.
a) 1 only
b) 2 only
c) Both 1 and 2
d) Neither 1 nor 2
Unlike the question hour, the zero hour is not mentioned in the Rules of Procedure.
Thus it is an informal device available to the members of the Parliament to raise matters
without any prior notice.
IASbabas Prelims 60 Day Plan Day 14 2017
The zero hour starts immediately after the question hour and lasts until the agenda for
the day (ie, regular business of the House) is taken up. In other words, the time gap
between the question hour and the agenda is known as zero hour.
It is an Indian innovation in the field of parliamentary procedures and has been in
existence since 1962.
Q.6) Who among the following issues notification for Prorogation of a session of the
Parliament?
a) Speaker
b) President
c) Vice-President
d) Chairman
The presiding officer (Speaker or Chairman) declares the House adjourned sine die,
when the business of a session is completed. Hence options a) and d) are incorrect.
Within the next few days, the President issues a notification for prorogation of the
session. However, the President can also prorogue the House while in session.
Summoning
The president from time to time summons each House of Parliament to meet. But, the
maximum gap between two sessions of Parliament cannot be more than six months. In other
words, the Parliament should meet at least twice a year. There are usually three sessions in a
year, viz,
A session of Parliament is the period spanning between the first sitting of a House and its
prorogation (or dissolution in the case of the Lok Sabha). During a session, the House meets
IASbabas Prelims 60 Day Plan Day 14 2017
everyday to transact business. The period spanning between the prorogation of a House and its
reassembly in a new session is called recess.
Adjournment :
3. It does not affect the bills or any other business pending before the House and the same can
be resumed when the House meets again.
Prorogation :
3. It also does not affect the bills or any other business pending before the House. However, all
pending notices (other than those for introducing bills) lapse on prorogation and fresh notices
have to be given for the next session. In Britain, prorogation brings to an end all bills or any
other business pending before the House.
Q.7) In India, no discussion on a matter of general public importance can take place except on
a motion made with the consent of the presiding officer. With regard to resolutions and
motions, consider the following statements
a) 1 only
b) 2 only
c) Both 1 and 2
d) Neither 1 nor 2
IASbabas Prelims 60 Day Plan Day 14 2017
All resolutions come in the category of substantive motions, that is to say, every
resolution is a particular type of motion.
All motions need not necessarily be substantive.
Further, all motions are not necessarily put to vote of the House, whereas all the
resolutions are required to be voted upon.
The House expresses its decisions or opinions on various issues through the adoption or
rejection of motions moved by either ministers or private members.
The motions moved by the members to raise discussions on various matters fall into three
principal categories
3. Subsidiary Motion: It is a motion that, by itself, has no meaning and cannot state the
decision of the House without reference to the original motion or proceedings of the House. It
is divided into three sub-categories:
(a) Ancillary Motion: It is used as the regular way of proceeding with various kinds of business.
(b) Superseding Motion: It is moved in the course of debate on another issue and seeks to
supersede that issue.
(c) Amendment: It seeks to modify or substitute only a part of the original motion.
Q.8) The Excess grant must be approved by which of the following committee before being
submitted for voting?
IASbabas Prelims 60 Day Plan Day 14 2017
Excess Grant: It is granted when money has been spent on any service during a financial
year in excess of the amount granted for that service in the budget for that year. It is
voted by the Lok Sabha after the financial year.
Before the demands for excess grants are submitted to the Lok Sabha for voting, they
must be approved by the Public Accounts Committee of Parliament.
This committee was setup first in 1921 under the provisions of the Government of India Act of
1919 and has since been in existence. At present, it consists of 22 members (15 from the Lok
Sabha and 7 from the Rajya Sabha). The members are elected by the Parliament every year
from amongst its members according to the principle of proportional representation by means
of the single transferable vote. Thus, all parties get due representation in it. The term of office
of the members is one year. A minister cannot be elected as a member of the committee. The
chairman of the committee is appointed by the Speaker from amongst its members. Until
196667, the chairman of the committee belonged to the ruling party. However, since 1967 a
convention has developed whereby the chairman of the committee is selected invariably from
the Opposition.
The function of the committee is to examine the annual audit reports of the comptroller and
auditor general of India (CAG), which are laid before the Parliament by the president. The CAG
submits three audit reports to the president, namely, audit report on appropriation account,
finance accounts and audit report on public undertakings.
The committee examines public expenditure not only from legal and formal point of to discover
technical irregularities but also from the point of view of economy, prudence, wisdom and
propriety to bring out the cases of waste, loss, corruption, extravagance, inefficiency and
nugatory expenses
1. To examine the appropriation accounts and the finance accounts of the Union govt. and any
other accounts laid before the Lok Sabha. The appropriation accounts compare the actual
expenditure with the expenditure sanctioned by the Parliament through the appropriation
act,while the finance accounts shows the annual receipts and disbursements of the union
government.
2. In scrutinising the appropriation accounts and the audit report of CAG on it, the committee
has to satisfy itself that:
(a) the money that has been disbursed was legally available for the purpose;
(b) the expenditure conforms to the authority that governs it; and
(c) every reappropriation has been made in accordance with the related rules
3. To examine the accounts of state corporations, trading concerns and manufacturing products
and the audit report of CAG on them (except those public undertakings which are allotted to
the committee on public undertakings).
4. To examine the accounts of autonomous and semi-autonomous bodies, the audit of which is
conducted by the CAG.
5. To consider the report of the CAG relating to an audit of any receipts or to examine the
accounts of stores and stocks.
6. To examine money spent on any service during a financial year in excess of the amtgranted
by the Lok Sabha for that purpose.
In the fulfilment of the above functions, the committee is assisted by the CAG.
Note: Do read about Vote of Credit& Vote of Account. Understand the difference between
them.
Source: Chapter 22 Parliament - Indian Polity by M. Laxmikanth
a) 1 and 3 only
b) 1, 3 and 4 only
c) 2 and 4 only
d) 1, 2, 3 and 4
Public Account of India All other public money (other than those which are credited to
the Consolidated Fund of India) received by or on behalf of the Government of India
shall be credited to the Public Account of India.
This includes provident fund deposits, judicial deposits, savings bank deposits,
departmental deposits, remittances and so on. Hence option 1,3 and 4 are correct.
Consolidated Fund of India: It is a fund to which all receipts are credited and all
payments are debited. In other words, (a) all revenues received by the Government of
India; (b) all loans raised by the Government by the issue of treasury bills, loans or
ways and means of advances; and (c) all money received by the government in
repayment of loans forms the Consolidated Fund of India.
All the legally authorised payments on behalf of the Government of India are made out
of this fund. Hence option 2 is incorrect.
Q.10) In Indian federal system, the Rajya Sabha has been given some special powers that are
not enjoyed by the Lok Sabha. Consider the following statements with regard to powers of
Rajya Sabha
a) 1 only
b) 2 only
c) Both 1and 2 only
d) Neither 1 nor 2
Due to its federal character, the Rajya Sabha has been given two exclusive or special powers
that are not enjoyed by the Lok Sabha:
It can authorise the Parliament to make a law on a subject enumerated in the State List
(Article 249).
It can authorise the Parliament to create new All-India Services common to both the
Centre and states (Article 312).
a) 1 only
b) 2 only
c) Both 1 and 2
d) Neither 1 nor 2
The budget consists of two types of expenditurethe expenditure charged upon the
Consolidated Fund of India and the expenditure made from the Consolidated Fund of
India.
The charged expenditure is non-votable by the Parliament, that is, it can only be
discussed by the Parliament, while the other type has to be voted by the Parliament.
1. Emoluments and allowances of the President and other expenditure relating to his
office.
2. Salaries and allowances of the Chairman and the Deputy Chairman of the Rajya Sabha
and the Speaker and the Deputy Speaker of the Lok Sabha.
3. Salaries, allowances and pensions of the judges of the Supreme Court.
4. Pensions (ONLY) of the judges of high courts. Hence 2nd statement is WRONG!
5. Salary, allowances and pension of the Comptroller and Auditor General of India.
6. Salaries, allowances and pension of the chairman and members of the Union Public
Service Commission.
7. Administrative expenses of the Supreme Court, the office of the Comptroller and
Auditor General of India and the Union Public Service Commission including the salaries,
allowances and pensions of the persons serving in these offices.
8. The debt charges for which the Government of India is liable, including interest,
sinking fund charges and redemption charges and other expenditure relating to the
raising of loans and the service and redemption of debt.
9. Any sum required to satisfy any judgement, decree or award of any court or arbitral
tribunal.
10. Any other expenditure declared by the Parliament to be so charged.
1. The Appropriation Bill becomes Appropriation Act only after it is assented to by the
President
2. The government cannot withdraw money from the Contingency Fund of India till the
enactment of the appropriation bill
3. Amendments cannot be proposed to the Appropriation Bill
a) 2 only
b) 1 and 3 only
c) 2 and 3 only
d) 1, 2 and 3
The Constitution states that no money shall be withdrawn from the Consolidated Fund
of India except under appropriation made by law.
No such amendment can be proposed to the appropriation bill in either house of the
Parliament that will have the effect of varying the amount or altering the destination of
any grant voted, or of varying the amount of any expenditure charged on the
Consolidated Fund of India.
The Appropriation Bill becomes the Appropriation Act after it is assented to by the
President. This act authorises (or legalises) the payments from the Consolidated Fund
of India. This means that the government cannot withdraw money from the
Consolidated Fund of India till the enactment of the appropriation bill.
IASbabas Prelims 60 Day Plan Day 14 2017
a) 1 only
b) 2 only
c) Both 1 and 2
d) Neither 1 nor 2
1. Attorney General
2. Comptroller and Auditor General
3. Union ministers
4. President
5. Vice-President
a) 1, 3 and 4
b) 1, 3, 4 and 5
c) 1 and 3 only
d) 2, 3 and 4 only
Parliamentary privileges are special rights, immunities and exemptions enjoyed by the
two Houses of Parliament, their committees and their members. They are necessary in
order to secure the independence and effectiveness of their actions.
The Constitution has also extended the privileges of the state legislature (or Parliament)
to those persons who are entitled to speak and take part in the proceedings of a
IASbabas Prelims 60 Day Plan Day 14 2017
House of the state legislature or any of its committees. These include advocate-general
of the state and state ministers (Union Ministers)
It must be clarified here that the privileges of the state legislature do not extend to the
governor (President) who is also an integral part of the state legislature (Parliament).
Vice President and CAG being not a part of the Parliament do not enjoy Parliamentary
privileges.
Article 105(1) guarantees freedom of speech in Parliament subject of course to the rules
and Standing Orders regulating the procedure of Parliament. What makes Article 105(1)
effective and much more than the right of every citizen to free speech guaranteed by
Article 19(1)(a), is the immunity from the process of the courts in respect of anything
said in the House.
1. The Speaker of Lok Sabha is the President of all the Parliamentary Forums
2. Each Forum consists of not more than 31 members, excluding the President and ex-
officio Vice-Presidents
3. Members of these forums are elected by the Speaker/Chairman from amongst the
members of the Parliament
a) 2 only
b) 1 and 2 only
c) 1 only
d) 1 and 3 only
(i) To provide a platform to the members to have interactions with the ministers concerned,
experts and key officials from the nodal ministries with a view to have a focused and
meaningful discussion on critical issues with a result-oriented approach for speeding up the
implementation process
(ii) To sensitise members about the key areas of concern and also about the ground level
situation and equip them with the latest information, knowledge, technical know-how and
valuable inputs from experts both from the country and abroad for enabling them to raise
these issues effectively on the Floor of the House and in the meetings of the Departmentally-
Related Standing Committees (DRSCs); and
(iii) To prepare a data-base through collection of data on critical issues from ministries
concerned, reliable NGOs, newspapers, United Nations, Internet, etc. and circulation thereof to
the members so that they can meaningfully participate in the discussions of the forums and
seek clarifications from experts or officials from the Ministry present in the meetings.
IASbabas Prelims 60 Day Plan Day 14 2017
It has been mandated that the Parliamentary Fora will not interfere with or encroach upon the
jurisdiction of the Departmentally-Related Standing Committees of the Ministry/Department
concerned.
The Speaker of Lok Sabha is the President of all the Forums except the Parliamentary Forum
on Population and Public Health wherein the Chairman of Rajya Sabha is the President and
the Speaker is the Co-President. The Deputy Chairman of Rajya Sabha, the Deputy Speaker of
Lok Sabha, the concerned Ministers and the Chairmen of Departmentally-Related Standing
Committees are the ex-officio Vice-Presidents of the respective Forums.
Each Forum consists of not more than 31 members (excluding the President and ex-officio
Vice-Presidents) out of whom not more than 21 are from the Lok Sabha and not more than 10
are from the Rajya Sabha.
Members (other than the President and Vice-Presidents) of these forums are nominated by the
Speaker/Chairman from amongst the leaders of various political parties/groups or their
nominees, who have special knowledge/keen interest in the subject.
The duration of the office of members of the forum is co-terminus with their membership in
the respective Houses. A member may also resign from the forum by writing to the
Speaker/Chairman.
The President of the forum appoints a member-convener for each forum to conduct regular,
approved programmes/meetings of the forum in consultation with the President. The meetings
of the forums are held from time to time, as may be necessary, during Parliament sessions.
1. It is an organization that studies and practices more natural and sustainable approaches
to learning and development, most of which differ radically from typical approaches to
education or fixing poverty.
2. It bagged the UN climate award 2016 for clean energy project
a) Only 1
IASbabas Prelims 60 Day Plan Day 14 2017
b) Only 2
c) Both 1 and 2
d) Neither 1 nor 2
An Indian NGO, Swayam Shikshan Prayog, has bagged the UN climate award this year. The NGO,
which trains women to become clean energy entrepreneurs across Maharashtra and Bihar
To enable women as leaders and entrepreneurs, through social enterprises and initiatives that
offer a range of skill-building, livelihoods, agricultural and health-enhancing opportunities to
rural women, youth and communities.
It equips rural women through self-learning avenues and foster them as entrepreneurs, leaders
and change makers in their communities. It enable grassroots women to set up social ventures
that serve their communities, and provide access to finance, skills and markets to grow their
enterprises.
SSP began its work as a consultant to the government in 1993, to drive rehabilitation efforts
across 1,300 villages in the earthquake-struck region of Latur, Maharashtra. It was legally
registered in 1998, and built on its women-led approach to drive resilience and development
across various disaster-prone regions for a decade. It developed a movement of over 1 lakh
women in self-help groups (SHGs), to increase access to finance and livelihoods.
In 2006, SSP co-created a grassroots business model with BP Energy, to develop rural women
entrepreneurs to sell clean cookstoves. It provided them access to finance, skills training and
rural distribution platforms by setting up various for-profit enterprises (SURE, SSEN, and SSK). It
went on to train community women leaders and entrepreneurs in the areas of preventive
health and sustainable agriculture.
UNFCCC, the nodal UN climate body, has applauded this project for building a rural distribution
network of 1,100 women entrepreneurs facilitating access to clean energy, water and
sanitation products and services in several communities.
Source: http://www.thehindu.com/news/national/Indian-NGO-bags-UN-climate-award-for-
clean-energy-project/article15422329.ece
IASbabas Prelims 60 Day Plan Day 14 2017
Q.17) Consider the following statements about Modified Assured Career Progression Scheme
1. It was originally introduced in 2008 following the recommendations of the Sixth Pay
Commission
2. It would ensure one minimum promotion for every 10 years till 30 years of service for
Central employees
a) Only 1
b) Only 2
c) Both 1 and 2
d) Neither 1 nor 2
The Union government has accepted the recommendation of the 7th Pay Commission on
Modified Assured Career Progression (MACP) for Central employees which would ensure one
minimum promotion for every 10 years till 30 years of service.
There shall be three financial upgradations under the assured promotion scheme counted from
the direct entry grade on completion of 10, 20 and 30 years services respectively or 10 years of
continuous service in the same level in Pay Matrix, whichever is earlier.
The scheme envisages mere placement in the immediate next higher level in the Pay Matrix.
Thus in certain cases where regular promotion is not between two successive levels in the Pay
Matrix, it will be different than what is available at the time of regular promotion. In such cases,
the higher level in the Pay Matrix attached to the next promotion post in the hierarchy of the
concerned cadre/organisation will be given only at the time of regular promotion.
The Modified Assured Career Progression Scheme was originally introduced in 2008 following
the recommendations of the Sixth Pay Commission.
Source: http://www.thehindu.com/news/national/Centre-accepts-pay-panel-
recommendation-on-promotion/article15424033.ece
IASbabas Prelims 60 Day Plan Day 14 2017
a) Tawang
b) Dharmashala
c) Nalanda
d) Sarnath
Source: http://pib.nic.in/newsite/PrintRelease.aspx?relid=151409
Q.19) International Court of Justice rejected nuclear disarmament cases filed by Marshall
Islands against Britain, India and Pakistan, saying it did not have jurisdiction. Marshall Islands
is located in which of the following oceans?
a) Pacific Ocean
b) Atlantic Ocean
c) Indian Ocean
d) Arctic Ocean
It is an island country located near the equator in the Pacific Ocean, slightly west of the
International Date Line.
Source: http://www.thehindu.com/news/national/UN-court-rejects-disarmament-case-
against-India/article15471059.ece
Q.20) The worlds largest marine protected area (MPA) has been established in
a) Ross Sea
b) Weddell Sea
c) Beaufort Sea
d) Greenland Sea
IASbabas Prelims 60 Day Plan Day 14 2017
The Ross Sea lies 3500 km south of New Zealand next to the Antarctic continent. It is one of two
areas around Antarctica with a wide continental shelf (the other being the Weddell Sea on the
opposite side of Antarctica).
1.55 million sq.km area of the Ross Sea will have special protection from human activities such
as commercial fishing.
Source: www.thehindu.com/sci-tech/energy-and-environment/Countries-agree-to-worlds-
largest-marine-reserve-in-Antarctica/article16084438.ece
a) Inflation
b) Balance of Payment
c) Taxation
d) Employment
A supposed relationship between economic activity and the rate of taxation which suggests
that there is an optimum tax rate which maximizes tax revenue
It illustrates the concept of taxable income elasticity i.e., taxable income will change in
response to changes in the rate of taxation. The Laffer curve is typically represented as a graph
which starts at 0% tax with zero revenue, rises to a maximum rate of revenue at an
intermediate rate of taxation, and then falls again to zero revenue at a 100% tax rate.
Source: http://www.thehindu.com/opinion/op-ed/Ways-out-of-the-GST-
maze/article16083450.ece
The gas pipeline project aims to provide piped cooking gas to residents of Varanasi within two
years and, in another year after that, cater to millions of people in states like Bihar, Jharkhand,
West Bengal and Odisha. From Varanasis perspective, an 800-km long MDPI pipeline will be
laid and 50,000 households and 20,000 vehicles will get PNG and CNG gas respectively. The
government estimates that around 5 lakh gas cylinders will be sent at rural areas annually.
According to GAIL, with the Urja Ganga project, 20 lakh households will get PNG connections.
The project is said to be a major step towards collective growth and development of the
Eastern region of India. GAIL has built a network of trunk pipelines covering the length of
around 11,000 km. With Urja Ganga project, this number will further increase by 2540 km.
Work on the 2540-km long Jagdishpur-Haldia and Bokaro-Dhamra Natural Gas pipeline project
will begin and will be completed between 2018 and 2020.
Source: http://www.financialexpress.com/economy/urja-ganga-gas-pipeline-project-
varanasi-eastern-india-details-highlights/428055/
a) 1, 2 and 4
b) 1, 2 and 3
c) 1 and 2
d) All of the above
The Foreign exchange reserves of India are mainly composed of US dollar in the forms of US
government bonds and institutional bonds.
Source: http://economictimes.indiatimes.com/news/economy/finance/indias-forex-
reserves-rise-1-billion-to-367-14-billion/articleshow/55119175.cms
IASbabas Prelims 60 Day Plan Day 15 2017
DAY 15
SUBJECT GEOGRAPHY
TOPICS:
PRELIMS MCQs:
Q.1) Which among the following includes the elements that influence the making of
climates?
1. Temperature
2. Precipitation
3. Humidity
4. Air pressure
5. Winds
a) 1, 2 and 3 only
b) 1, 2, 3 and 4 only
c) 1, 2, 3 and 5 only
d) 1, 2, 3, 4 and 5
All the five Temperature, Precipitation, Humidity, Air pressure and Winds are elements
that influence the making of climates.
These climatic elements individually and in combination with each other vary from place to
place and from season to season. These variations ultimately produce climatic variations.
1. The ideal conditions for formation of dew are clear sky, calm air, high relative
humidity, and cold and long nights.
2. For the formation of dew, it is necessary that the dew point is below the freezing
point.
a) 1 only
b) 2 only
c) Both 1 and 2
d) None
Dew is the moisture that forms as a result of condensation. Condensation is the process a
material undergoes as it changes from a gas to a liquid. Dew is the result of water changing
from a vapor to a liquid.
Dew forms as temperatures drop and objects cool down. If the object becomes cool
enough, the air around the object will also cool. Colder air is less able to hold water vapor
than warm air. This forces water vapor in the air around cooling objects to condense. When
condensation happens, small water droplets formdew.
The temperature at which dew forms is called the dew point. The dew point varies widely,
depending on location, weather, and time of day.
The ideal conditions for its formation are clear sky, calm air, high relative humidity, and cold
and long nights. However, for the formation of dew, it is necessary that the dew point is
above the freezing point (not below). Hence, statement (2) is wrong.
a) 1 only
b) 2 only
c) Both 1 and 2
d) None
IASbabas Prelims 60 Day Plan Day 15 2017
Barysphere is associated with inner layer of the earths core (below outer core). Hence,
statement (1) is wrong.
Asthenosphere Plastic-
ii. Lower Upper and lower Semimolten Moderately
mantle are heavy
composed
mainly of Silica
and Magnesium
(SI + MA = SIMA)
C. Core
i. Outer Composed Liquid or in 2220 Km Heavy
mainly of Nickel Plastic State
and Ferrous
(Ni + Fe = Nife)
Q.4) Which of the statements given below is/are correct in regard to Manganese?
a) 1 and 2 only
b) 1, 2 and 3 only
c) 2, 3 and 4 only
d) 1, 2, 3 and 4
Manganese is not found as a free element in nature. It is often found in combination with
iron.
It is primarily used in iron and steel industry. It is the basic raw material for manufacturing
steel alloys.
6 kilograms of manganese is required for manufacturing one tonne of steel.
Manganese is also used in the manufacturing of bleaching powder, insecticides, paints, and
batteries.
India processes second largest reserves in the world after Zimbabwe. India is the world's
fifth largest producer of manganese ore after China, Gabon, South Africa and Australia.
Maharashtra, Madhya Pradesh, Odisha, Andhra Pradesh and Karnataka are the major
manganese producing states.
Maharashtra and Madhya Pradesh together produce more than half of India's manganese.
Q.5) Which among the statements given below is not correct about Chinook/Fohn winds?
Chinook/Fohn winds are strong, dry and warm winds which develops on the leeward side of
the mountains. These winds are formed when descending air (not ascending) becomes
compressed with increased pressure. Hence, statement (b) is wrong.
These winds are experienced in the valleys of the northern Alps, particularly in Switzerland
in spring. The winds helps animal grazing by melting snow and fastens the ripening of
grapes.
It is beneficial to ranchers east of the Rockies (Canada) as it keeps the grasslands clear of
snow during much of the winter.
Q.6) Match List I with List II and select the correct answer using the code given below the
Lists:
List I List II
A-B-C-D
a) 1-2-3-4
b) 1-3-2-4
c) 4-3-2-1
d) 4-2-3-1
Self-explanatory.
Q.7) Select the appropriate climatic zone/type having the below given characteristics:
IASbabas Prelims 60 Day Plan Day 15 2017
a) Laurentian type
b) Mediterranean climate
c) Steppe/Temperate grasslands
d) Coniferous forest Siberian climate
Mediterranean climate
Entirely confined to the western portion of continental masses, between 30 and 45
north and south of the equator.
The basic cause of this type of climate is the shifting of the wind belts.
Mediterranean Sea has the greatest extent of this type of winter rain climate, and
gives rise to the name Mediterranean Climate.
The best developed form of this climatic type is found in central Chile.
Clear skies and high temperatures; hot, dry summers and cool, wet winters.
Mean annual precipitation ranges from 35 - 90 cm.
Temperature of warmest month greater than or equal to 10 C.
Temperature of coldest month is less than 18 C but greater than 3 C
Climate is not extreme because of cooling from water bodies.
Q.8) Variations in the length of daytime and night time from season to season are due to
Revolution of the earth on a tilted axis Variation in Day time and Night time
IASbabas Prelims 60 Day Plan Day 15 2017
a) Foehn
b) Loo
c) Mistral
d) Sirocco
Warm Winds include: Foehn or Fohn, Chinook, Zonda, Loo and Sirocco
Cold Winds include: Pampero, Bora, Mistral, Gregale and Tramontane
Q.10) Consider the following statements and choose the incorrect statement?
Winds blow across the Earth from high-pressure systems to low-pressure systems. However,
winds dont travel in a straight line. The actual paths of windsand of ocean currents, which
are pushed by windare partly a result of the Coriolis effect.
Coriolis force is an apparent force that as a result of the earth's rotation deflects moving
objects (as projectiles or air currents) to the right in the northern hemisphere and to the left
in the southern hemisphere.
The Coriolis force is zero at the equator (no cyclones at equator because of zero Coriolis
Force) but it increases with latitude. Hence, statement (1) is incorrect.
Coriolis force at 5 latitude is significant enough to create a storm [cyclonic vortex]. About
65 per cent of cyclonic activity occurs between 10 and 20 latitude.
1. The cyclonic wind movements are anticlockwise in the northern hemisphere and
clockwise in the southern hemisphere.
IASbabas Prelims 60 Day Plan Day 15 2017
2. Tropical cyclones are violent storms that originate over oceans in tropical areas and
move over to the coastal areas.
3. Tropical cyclones are irregular wind movements involving closed circulation of air
around a low pressure center.
a) 2 and 3 only
b) 1 and 2 only
c) 1 and 3 only
d) 1, 2 and 3
Tropical cyclones are violent storms that originate over oceans in tropical areas and move
over to the coastal areas bringing about large scale destruction due to violent winds, very
heavy rainfall (torrential rainfall) and storm surge.
They are irregular wind movements involving closed circulation of air around a low pressure
center. This closed air circulation (whirling motion) is a result of rapid upward movement of
hot air which is subjected to Coriolis force. The low pressure at the center is responsible for
the wind speeds.
The cyclonic wind movements are anticlockwise in the northern hemisphere and clockwise
in the southern hemisphere (This is due to Coriolis force).
1. It is governed by topography.
2. Large changes in mean temperature occur over short distances.
3. Precipitation types and intensity also vary spatially.
The above characteristics are related with which of the following climate type?
Select the correct code:
a) Polar Climates
b) Cold Snow Forest Climates
c) Highland Climates
d) Tundra Climate
Highland climates are governed by topography. In high mountains, large changes in mean
temperature occur over short distances. Precipitation types and intensity also vary spatially
across high lands. There is vertical zonation of layering of climatic types with elevation in the
mountain environment.
Q.13) Arrange the following Iron ores based upon their quality or pure iron content:
1. Limonite
2. Siderite
3. Magnetite
4. Haematite
a) 1-2-3-4
b) 3-4-1-2
c) 3-4-2-1
d) 4-3-2-1
Magnetite: This is the best quality of iron ore and contains 72 per cent pure iron. It
possesses magnetic property and hence is called magnetite. It is found in Andhra Pradesh,
Jharkhand, Goa, Karnataka etc
Haematite: It contains 60 per cent to 70 per cent pure iron and is found in Andhra Pradesh,
Jharkhand, Orissa, Chhattisgarh, Goa etc
Limonite: It contains 40 per cent to 60 per cent pure iron. It is of yellow or light brown
colour
Siderite: It contains many impurities and has just 40-50 per cent pure iron. However, due to
presence of lime, it is self fluxing
a) 1 only
IASbabas Prelims 60 Day Plan Day 15 2017
b) 2 only
c) 1 and 2 only
d) 1, 2 and 3
Mesas and Buttas are associated with arid areas, whereas Dissected plateaux are associated
with humid areas.
Peneplains are associated with humid conditions, whereas Pediplains are associated with
arid and semi-arid conditions.
Q.15) Which of the statements given below is/are correct in regard to Aluminium?
1. Aluminium production starts with the raw material bauxite, a clay like soil type found
in a belt around the equator
2. Alumina is separated from the bauxite by using a hot solution of caustic soda and
lime
3. Three different raw materials are needed to make aluminium, aluminium oxide,
electricity and carbon
4. Aluminium is known for its ductile nature and can be recycled over and over again
with 100 percent efficiency
a) 1 and 2 only
b) 1, 2 and 3 only
c) 2, 3 and 4 only
d) 1, 2, 3 and 4
Q.16) Recently, Indias satellite GSAT-18 was successfully launched. With reference to
GSAT-18, consider the following statements
1. It is a communication-based satellite
2. It is Indias heaviest satellite
3. It was launched by Indias heavy duty launch vehicle GSLV
4. It was built by ISRO
IASbabas Prelims 60 Day Plan Day 15 2017
a) 1, 2 and 4
b) 1, and 2 Only
c) 1, 2 and 3
d) 1, 2, 3 and 4
Source: http://www.thehindu.com/sci-tech/science/communication-satellite-gsat18-
successfully-launched/article9190037.ece
Q.17) Consider the following statements about Polymerase chain reaction (PCR)
a) Only 1
b) 1 and 2
c) 1 and 3
d) All of the above
PCR is a technique used in molecular biology to amplify a single copy or a few copies of a
piece of DNA across several orders of magnitude, generating thousands to millions of copies
of a particular DNA sequence.
PCR is not generally considered to be a recombinant DNA method, as it does not involve
cutting and pasting DNA, only amplification of existing sequences.
Source: http://www.thehindu.com/news/cities/chennai/gender-identification-with-
feathers-of-birds-by-madras-vet-college/article9202355.ece
a) 1 and 2
b) 2 and 3
c) 1 and 3
d) All of the above
The MPLADS is a Plan Scheme fully funded by Government of India. The annual MPLADS
fund entitlement per MP constituency is Rs. 5 crore. 2.2 Lok Sabha Members can
recommend works within their Constituencies and Elected Members of Rajya Sabha can
recommend works within the State of Election except as provided in paras 2.8 and 2.9.
Nominated Members of both the Rajya Sabha and Lok Sabha can recommend works
anywhere in the country.
The Ministry of Statistics and Programme Implementation has been responsible for the
policy formulation, release of funds and prescribing monitoring mechanism for
implementation of the Scheme. A Department in each State/UT is designated as the Nodal
Department with the overall responsibility of supervision, monitoring and coordination of
the MPLADS implementation with the districts and other Line Departments. The
Government of India informs the State Nodal Department about the MPLADS funds released
IASbabas Prelims 60 Day Plan Day 15 2017
to the District Authorities. The District Authorities report the status of MPLADS
implementation to the Government of India and State Nodal Department.
The district authority is empowered to examine the eligibility of works sanction funds and
select the implementing agencies, prioritize works, supervise overall execution, and monitor
the scheme at the ground level. The district authorities get the works executed through the
line departments, local self-governments or other government agencies. In some cases, the
district authorities get the works executed through reputed non-government organizations.
Read More -
http://www.mplads.gov.in/MPLADS/UploadedFiles/MPLADSGuidelines2016English_638.
pdf
Source: http://www.thehindu.com/opinion/editorial/Of-politics-and-
administration/article16070413.ece
Source: http://www.thehindu.com/todays-paper/tp-international/twothirds-of-wild-
animals-may-go-extinct-by-2020-warns-report/article9278553.ece
Q.20) The folk art of Alpana is from which of the following states?
a) Rajasthan
b) Maharashtra
c) West Bengal
d) Assam
Source: http://www.thehindu.com/news/national/other-states/reviving-a-vanishing-folk-
art-form-in-bengal/article9277490.ece
As defined by the World Health Organization (WHO) and the Food and Agricultural
Organization of the United Nations (FAO), fortification refers to "the practice of deliberately
increasing the content of an essential micronutrient, ie. vitamins and minerals (including
trace elements) in a food irrespective of whether the nutrients were originally in the food
before processing or not, so as to improve the nutritional quality of the food supply and to
provide a public health benefit with minimal risk to health," whereas enrichment is defined
IASbabas Prelims 60 Day Plan Day 15 2017
Source: http://pib.nic.in/newsite/PrintRelease.aspx?relid=151709
Q.22) Indian Roller Bird is the state bird of which of the following states?
1. Andhra Pradesh
2. Odisha
3. Telangana
4. Karnataka
a) 1 and 3
b) 1, 3 and 4
c) Only 2
d) All of the above
The Indian Roller (Coracias benghalensis ) is protected under Schedule IV of the Wildlife
Protection Act, 1972.
This Indian Roller or Neelkanth is a state bird of various states like Karnataka, Telangana,
Andhra Pradesh and Orissa. It is displayed during Dussehera festival in Telangana.
Source: http://www.thehindu.com/todays-paper/tp-national/poachers-go-all-out-to-net-
indian-roller/article7646653.ece
IASbabas Prelims 60 Day Plan Day 16 2017
DAY 16
SUBJECT GEOGRAPHY
TOPICS:
PRELIMS MCQs:
1. Parent Material
2. Time
3. Climate
4. Biological activity
a) 1, 2 and 3 only
b) 2, 3 and 4 only
c) 1, 3 and 4 only
d) 1, 2, 3 and 4
Interactions between these factors produce an infinite variety of soils across the earths
surface.
1. The soils of the young stage are poorly developed and do not have the characteristic
horizons.
2. The soils of the mature stage have a well-developed profile of horizons which
remains constant through time.
a) 1 only
b) 2 only
c) Both 1 and 2
d) None
There are 3 relative stages of soil development, young, mature and old.
The young stage is characterised by the dormant control of the parent material and
invading organisms. The soils of the young stage evolve from recently deposited river
alluvium and glacial till. They are poorly developed and do not have the
characteristic horizons.
The mature stage reflects a greater influence of climate and these soils are
developed over a long period of time. It has a well-developed profile of horizons
which remains constant through time.
The old soils mostly occur in the tropical regions and are one to six million years in
age.
Q.3) Which of the statements given below are correct about desert soils?
1. Desert soils are suitable for cultivation along the flood plains and on the terraces
where their texture is fine.
2. They are usually poor in organic matter.
3. The phosphate and nitrate content of these soils are very poor.
a) 1 and 2 only
b) 1 and 3 only
IASbabas Prelims 60 Day Plan Day 16 2017
c) 2 and 3 only
d) 1, 2 and 3 only
Phosphates and nitrates make these soil fertile wherever moisture is available. There is a
possibility of reclaiming these soils if proper irrigation facilities are available.
In large areas, only the drought resistant and salt tolerant crops such as barley, cotton,
millets, maize and pulses are grown.
1. The soils are covered with a thick brown mantle which inhibits soil growth.
2. They are coarse and alkaline, rich in soluble salts.
3. They are poor in organic matter and have a varying percentage of lime.
4. These soils are derived from the adjoining rocks and the coastal regions.
a) Laterite soil
b) Peaty soil
c) Desert soil
d) Loamy soils
Desert soils posses all the characteristics provided in the statements. Hence (c) is correct
answer.
Desert soils are grey in colour in temperate region and red in hot deserts of the tropics. The
soils are unleached and alkaline but very much lacking in humus because of little vegetation
found.
IASbabas Prelims 60 Day Plan Day 16 2017
Q.5) Which among the following are statements is/are true?
1. Pedocals are soils that are found in humid climates extending from high-latitude
coniferous forests, mid latitude deciduous forest lands to low-climate tropical forests
and grasslands
2. Pedalfers are soils that are found in arid, semi-arid and the sub-humid zones in the
world
3. Pedocals are very rich in calcium and mineral salts
a) 1 and 2 only
b) 2 and 3 only
c) 3 only
d) 1, 2 and 3
Pedalfers are soils that are found in humid climates extending from high-latitude coniferous
forests, mid latitude deciduous forest lands to low-climate tropical forests and grasslands
Pedocals are soils that are found in arid, semi-arid and the sub-humid zones in the world
Pedocals are very rich in calcium and mineral salts
Q.6) Which of the statements given below are true about Chernozems?
a) 1 and 2 only
b) 2 and 3 only
c) 3 only
d) 1, 2 and 3
Chernozems:
These soils are called black earths
IASbabas Prelims 60 Day Plan Day 16 2017
They are rich in humus, rich in calcium, unleached and have a crumby or nut-like
structure
These are fertile soils requiring little fertilizing even when used year after year.
The Steppes of Ukraine, the central part of USA, central Africa, South America and
Australia have these type of soils
1. Subsistence agriculture is one where the farming areas consume all or nearly so of
the products locally grown.
2. Intensive Subsistence agriculture is dominated by wet paddy cultivation involves
limited use of machinery and low labour productivity and farm yard manure used for
fertility retention.
3. Mixed farming found in highly developed parts of the world and farms are
moderate in size.
a) 1 and 2 only
b) 2 and 3 only
c) 1 and 3 only
d) 1, 2 and 3
1. Cropping intensity refers to raising of a number of crops during one agriculture year
by expanding the net area under cultivation.
2. Crop combination refers to the quantum or diversity of crops entering a region in a
given period.
a) 1 only
b) 2 only
c) Both 1 and 2
d) None
Q.9) Match List I with List II and select the correct answer using the code given below the
Lists:
List I List II
(Weathering type) (Landform/Process)
Code:
A-B-C-D
a) 4-3-2-1
b) 2-3-1-4
c) 1-3-2-4
d) 2-1-3-4
Self-explanatory.
Q.10) Which of the statements given below are true about Black soil?
a) 1 and 2 only
b) 1, 2 and 3 only
c) 2, 3 and 4 only
d) 1, 2, 3 and 4
Q.11) Match List I with List II and select the correct answer using the code given below the
Lists:
List I List II
(Farming area) (State)
A. Doab 1. Assam
B. Char 2. Karnataka
C. Maidan 3. Punjab
D. Terai 4. Uttar Pradesh
Code:
A-B-C-D
a) 4-3-2-1
b) 2-3-1-4
c) 3-1-2-4
d) 2-1-3-4
Self-explanatory.
Q.12) We know that all rocks whether igneous or sedimentary may become metamorphic
or changed rocks under great heat and pressure. Consider the below lists and match List I
with List II based upon the alteration they undergo:
List I List II
IASbabas Prelims 60 Day Plan Day 16 2017
A. Clay 1. Slate
B. Sandstone 2. Schist
C. Limestone 3. Marble
D. Granite 4. Gneiss
E. Shale 5. Quartzite
Code:
A-B-C-D-E
a) 1-5-3-4-2
b) 1-2-5-3-4
c) 2-1-5-3-4
d) 2-1-5-4-3
Self-explanatory.
a) 2 and 3 only
b) 1 and 2 only
c) 1 and 3 only
d) 1, 2 and 3
Soil formation is a result of various processes that tend to affect the soil profile (parallel
layers of soil each with distinct physical characteristics). Following are the processes that
result in soil formation:
1. Eluviation: It is the mechanical translocation of clay or other fine particles from upper
layers to lower layers due to downward precipitation. The particles could also translocate in
dissolved form.
IASbabas Prelims 60 Day Plan Day 16 2017
2. Illuviation: It is the accumulation of the washed off material i.e. eluviated material in the
lower layers of the soil profile. Water percolating the soil leaches out soluble salts and
mineral particles, thereby adding them to the lower portions.
3. Leaching: It is the loss of water soluble material like calcium due to percolation of water
down the soil profile.
(Zone) (Soil)
1. Peaty Marshy Soils are soils with large amount of organic matter and considerable
amount of soluble salts.
2. The most humid regions have this type of soil.
3. Since they are not acidic, this type of soil is good for paddy cultivation.
Peaty Marshy Soils are soils with large amount of organic matter and considerable amount
of soluble salts. The most humid regions have this type of soil. They are black, heavy and
highly acidic. Hence statement (3) is incorrect. They are deficient in potash and phosphate.
Most of the peaty soils are under water during the rainy season but as soon the rains cease,
they are put under paddy cultivation.
a) 1 and 2
b) 2 and 3
c) 1 and 3
d) All of the above
Kashmiri Red Stag or Hangul is known for its giant antlers bearing 11 to 16 points.
Only one viable population left today in the wild is largely confined to the Greater Dachigam
Landscape (1,000 sq.km.), encompassing the Dachigam National Park (NP) and adjoining
protected areas.
It is one of three critically endangered species in Jammu and Kashmir. The other two are
markhor, the Tibetan antelope or chiru. It was designated as State Animal of Jammu &
Kashmir in 1980s.
It is listed under Schedule-I of the Indian Wildlife (Protection) Act, 1972 and J&K Wildlife
(Protection) Act, 1978 and has also been listed among the top 15 species of high
conservation priority by the Government of India.
IASbabas Prelims 60 Day Plan Day 16 2017
IUCN (International Union for Conservation of Nature) will declare Kashmiri Red Stag or
Hangul as a 'critically endangered' species.
Earlier it was considered as a subspecies of red deer. Hence, IUCN categorized it as 'Least
Concern' by clubbing with European and other 'red deer' species of the world.
It is the state animal of J&K only.
Source: http://indiatoday.intoday.in/story/kashmir-red-stag-iucn-critically-
endangered/1/787937.html
a) 1 and 2
b) 2 and 3
c) 1 and 3
d) All of the above
Recently National Environmental and Engineering Research Institute (Neeri) and CSIR has
developed Neerdhur, a novel multi-fuel domestic cooking stove.
Apart from wood, other fuel like coal, cow dung and agricultural residue can also be used in
it.
e the
pressure on environment
Neerdhur has been approved and certified by Ministry of New and Renewable Energy
(MNRE) and meets the emission parameters of Bureau of Indian Standards (BIS).
Will help in improvement of womens health in rural areas.
Source: http://timesofindia.indiatimes.com/city/nagpur/Neeris-stove-reduces-smoke-
promises-better-rural-health/articleshow/54665243.cms
Q.18) Consider the following statements about Green train corridors (GTC)
1. They are sections of the railways which will be free of human waste on the tracks
IASbabas Prelims 60 Day Plan Day 16 2017
2. Okha-Kanalus stretch of Western Railway is Indias first GTC
a) Only 1
b) Only 2
c) Both 1 and 2
d) Neither 1 nor 2
Source:
http://economictimes.indiatimes.com/industry/transportation/railways/railways-add-
175-km-long-track-to-green-train-corridors/articleshow/54921234.cms
Q.19) Global Conference on Voter Education titled Voter Education for Inclusive,
Informed and Ethical Participation is being organized by
Source: http://pib.nic.in/newsite/PrintRelease.aspx?relid=151732
Biotech- KISAN (Krishi Innovation Science Application Network) Empowering Small- and
Women- Farmers with Science Implementation
The Scheme is for farmers, developed by and with farmers, it empowers women, impacts
locally, connects globally, is Pan-India, has a hub-and spoke model and stimulates
entrepreneurship and innovation in farmers.
Biotech-KISAN is:
a) Only 1
b) Only 2
c) Both 1 and 2
d) Neither 1 nor 2
The Bishkek Declaration was an agreement signed by 12 countries that are home to this
species namely Afghanistan, Bhutan, China, India, Kazakhstan, the Kyrgyz Republic,
Mongolia, Nepal, Pakistan, Russia, Tajikistan and Uzbekistan. The declaration was initiated
in order to formulate a long-term Global Snow Leopard Conservation Programme by the
IASbabas Prelims 60 Day Plan Day 16 2017
formation of a high-level steering committee to guide programme implementation and
regularly review its progress.
One of the most elusive mountain cats, the snow leopard (Panthera uncial) is categorised as
an endangered species in the International Union of Conservation of Nature (IUCN) Red list.
Source: http://www.thehindu.com/news/national/other-states/Darjeeling-zoo-to-
receive-snow-leopard-from-London/article14397699.ece
In a bid to make Rajasthan child marriage-free, the state government, the United Nations
Population Fund (UNFPA) and United Nations Childrens Fund (UNICEF), launched a district-
level campaign in Dausa under the Sanjha Abhiyan banner of the Rajasthan Government.
Source: http://indianexpress.com/article/india/india-news-india/rajasthan-government-
launches-campaign-against-child-marriage-in-dausa-3103004/
IASbabas Prelims 60 Day Plan Day 17 2017
Q.1) Department of Science and Technology has recently launched Cyber Physical Systems
(CPS) Program. Which of the following statements are correct about it?
a) 1 only
b) 2 only
c) Both 1 and 2
d) Neither 1 nor 2
Department of Science and Technology (DST) has launched Cyber Physical Systems (CPS)
programme dealing with self-driven cars, autonomous unmanned vehicles and aircraft
navigation systems The programme is still at a nascent stage. Rs. 3,000-crore has been has been
conceived for it and it will first take root in some of the Indian Institutes of Technology (IITs).
The thrust of the CPS initiative is to break silos in academia and encourage greater synergy
between the university scientists and industry.
Under it, centres of excellence would be developed at the IITs and universities. Moreover, there
will be dedicated courses on the subject.
CPS is an interdisciplinary field that deals with the deployment of computer-based systems that
do things in the physical world. For Cyber Physical Systems example, self-driven cars produced
by Google and Tesla. It is a mechanism controlled or monitored by computer-based algorithms,
tightly integrated with the internet and its users. It involves transdisciplinary approaches,
merging theory of cybernetics, design and process science and mechatronics. In it, physical and
software components are deeply intertwined, each operating on different spatial and temporal
scales. Both exhibit multiple and distinct behavioural modalities, and interact with each other in
a myriad of ways that change with context. Autonomous unmanned vehicles (UAVs) and
aircraft navigation systems and smart grids (where electricity is optimally distributed on the
IASbabas Prelims 60 Day Plan Day 17 2017
Q.2) Japan has recognized Bitcoins as a legal currency like any other Fiat money. Which of the
following statements regarding Bitcoins are correct?
a) 1 and 2
b) 2 and 3
c) 1 and 3
d) All of the above
Bitcoin is a form of digital currency or virtual currency created and held electronically.
No one controls it that is not regulated by any central bank or government.
They arent printed, like dollars or euros theyre produced by people, and increasingly
businesses, running computers all around the world, using software that solves
mathematical problems.
It is also called a cryptocurrency since it is decentralized and uses cryptography to
prevent double-spending, a significant challenge inherent to digital currencies.
Bitcoin is a distributed peer-to-peer digital currency that functions without the inter-
mediation of any central authority.
It can also be traded on an open market and its exchange rate fluctuates much like a
stock market i.e. based on the demand.
Q.3) Consider the following statements regarding Single Actuator Wave Like Robot (SAW):
1. It is a tiny ingestible snake like robot and can navigate through stomach and intestines.
2. It has been made by 3D printing of individual blocks and interlocking them.
3. It is made for mars research and it can move on the difficult terrain.
4. The robot generates Sound waves to detect objects through SONAR.
a) 2,3 and 4
b) 2 and 3
c) 1 and 2
d) 3 and 4
Scientists have developed a tiny, ingestible 3D-printed snake-like robot called SAW (single
actuator wave-like robot), that can navigate through the small intestines. The snake-like tiny
robot was made from a set of interlocking 3D-printed plastic pieces that look like vertebrae.
The robot moves in a wave-like motion and can travel through the extremely squishy
environment of the small intestine. The external shape of the robot is a 2D projection of a
rotating helix, that result in a continuously moving wave. Its direction can be reversed simply by
reversing the direction of rotation of the motor. In tests, the robot was able to move incredibly
fast and cross a wide array of terrains, from water to rough, rocky soil.
Potential application: It can be used to visualise the digestive system in real time, especially for
colonoscopies.
Q.4) Current Sunway TaihuLight is the fastest Super computer of the world with a top speed
of 124.5 PetaFLOPS. Which of the following countries have developed it?
a) China
b) USA
c) Russia
d) Germany
There is no U.S.-made system that comes close to the performance of China's new system, the
Sunway TaihuLight. Its theoretical peak performance is 124.5 petaflops, according to the latest
biannual release today of the world's Top500 supercomputers. It is the first system to exceed
100 petaflops. A petaflop equals one thousand trillion (one quadrillion) sustained floating-point
operations per second.
IASbabas Prelims 60 Day Plan Day 17 2017
The most important thing about Sunway TaihuLight may be its microprocessors. In the past,
China has relied heavily on U.S. microprocessors in building its supercomputing capacity. The
world's next fastest system, China's Tianhe-2, which has a peak performance of 54.9 petaflops,
uses Intel Xeon processors.
TaihuLight, which is installed at China's National Supercomputing Center in Wuxi, uses ShenWei
CPUs developed by Jiangnan Computing Research Lab in Wuxi. The operating system is a Linux-
based Chinese system called Sunway Raise.
Q.5) National Green Tribunal has recently suspended the Environment clearance of Indian
Neutrino Observatory (INO). Consider the following statements about INO:
a) 1 only
b) 2 and 3
c) 1 and 3
d) All of the above
It will comprise a complex of caverns which will house detector which is 130 metres
long, 26 metres wide and 30 metre high.
Q.6) Government of India has launched Bharat QR code to enable digital payments. Which of
the following statements are correct about QR codes?
a) 1 and 2
b) 2 and 3
c) 1 and 3
d) All of the above
Q.7) The Union Ministry of Electronics and Information Technology (MeitY) has launched
Cyber Swachhta Kendra. The objective of this is:
The Union Ministry of Electronics and Information Technology (MeitY) has launched Cyber
Swachhta Kendra in New Delhi for Botnet cleaning and Malware analysis. It was launched by
Union Minister of Electronics and IT Ravi Shankar Prasad as part of Union Governments
ambitious digital India initiative to create a secure cyber space by detecting botnet infections.
Besides, minister also launched some cyber security tools.
They are
USB Pratirodh: It is a desktop security solution to protect from USB mass storage device
threats.
AppSamvid: It is a desktop solution to protect systems by allowing installation of
genuine applications through white listing. It helps in preventing threats from malicious
applications.
M Kavach : It is an indigenously developed mobile application to address the security
threats in mobiles.
It is being operated by the Indian Computer Emergency Response Team (CERT-In) under
provisions of Section 70B of the Information Technology (IT) Act, 2000.
Q.8) Which of the following statements are correct about the GraphAir technology?
gases that required long hours of operation at high temperatures and extensive vacuum
processing. This production process was costly and was major roadblock in its
commercialisation.
The technology grows graphene film in ambient air with a natural precursor, making its
production faster and simpler. Soybean oil breaks down into a range of carbon building units
when heat is applied. It makes it essential for the synthesis of graphene films.
Q.9) The Maharashtra government has approved a cloud seeding program for 2017 monsoon
to produce sufficient rain. Which of the following compounds are used for Cloud seeding?
a) Sulfuric acid
b) Mercury oxide
c) Nitric Acid
d) Silver Iodide
Maharashtra Government has approved Rs. 250 crore Cloud Seeding Programme during 2017
monsoon season to produce sufficient rain. Under this programme, weather scientists using
aircrafts will spray chemicals (silver iodide) over clouds clouds hovering above Solapur district, a
rain shadow region of Western Ghats in the state.
This will be the first controlled experiment to quantify the extent to which clouds form water
drops large enough to make rain. This programme, coordinated by the Indian Institute of
Tropical Meteorology, will be the first controlled experiment to quantify the extent to which
clouds form water drops large enough to make rain. In this experiment, scientists will fly two
aircraft and spray silver iodide and dry ice on 100 clouds and compare them with 100 unseeded
clouds. Ground radar will be used to track the clouds and verify which ones contributed rain.
Q.10) Which of the following radiations can be used for wireless communication?
1. Radio waves
2. Microwaves
3. Infrared radiations
4. Visible light
IASbabas Prelims 60 Day Plan Day 17 2017
a) 1 and 2
b) 1,2 and 3
c) 2,3 and 4
d) All of the above
Microwaves are used to transmit satellite television and for mobile phones.
Q.11) NASA is building a robotic spacecraft to refuel and repair satellites in the orbit. Which
of the following statements are correct about this spacecraft?
1. The spacecraft will be used to perform in orbit satellite servicing in low earth orbits.
2. The spacecraft has been named Restore L spacecraft.
a) 1 only
b) 2 only
c) Both 1 and 2
d) Neither 1 nor 2
The National Aeronautics and Space Administration (NASA) is building a robotic spacecraft
dubbed as Restore-L Spacecraft to refuel, repair satellites currently in orbit. In this regard,
NASA has awarded a $127 million contract to California-based satellite company Space
Systems/Loral to build Restore-L Spacecraft.
IASbabas Prelims 60 Day Plan Day 17 2017
Space Systems/Loral will provide NASA with spacecraft bus, critical hardware and services for
the Restore-L mission. This mission will be launched in 2020 to perform in-orbit satellite
servicing in low-Earth orbit.
It will help in extending life of existing satellites and reduce operation costs. It can also be used
for cleaning space debris or testing asteroid platforms.
Q.12) Software giant Microsoft India has launched its first full-scale Cybersecurity
Engagement Centre (CSEC) in India. In which of the following cities is it located?
a) Bengaluru
b) New Delhi
c) Hyderabad
d) Chennai
Software giant Microsoft India has launched its first full-scale Cybersecurity Engagement Centre
(CSEC) in New Delhi. It is Microsofts seventh Cybersecurity Centre in the world. It will function
as a satellite to the its Redmond Digital Crimes Unit (DCU).
Microsofts other six cyber security centres are located in Singapore, Tokyo, Beijing, Seoul,
Berlin and Washington DC.
It will help to build a trusted and secure computing environment, a critical enabler for
Indias digital transformation.
It will provide protection across all end-points from sensors to the data centre,
detection of attack using targeted signals, machine learning and behavioural monitoring.
It will eventually help in closing the gap between discovery and action taken to
neutralise the attack. It will enable and empower enterprises to manage modern
security threats effectively.
It will also expand Microsofts public-private partnerships in India. The centre will also
enable customers tap into a pool of resources such as security specialists and
technologies at Microsoft.
Q.13) A relatively small piece of orbital debris can inflict a great deal of damage on satellites
or spacecrafts orbiting in the space. This phenomenon is known as
IASbabas Prelims 60 Day Plan Day 17 2017
a) Kessler Syndrome
b) Kepler Syndrome
c) Newton Syndrome
d) Laplace Syndrome
More than 5 decades of human space exploration since the first Soviet-launched Sputnik
satellite in 1957 has produced this hazardous belt of orbiting debris in the space. There are
estimated to be more than 100 million pieces are circling our planet in the lower orbit, posing a
growing threat to future space exploration. These pieces of debris travel at high speeds. A
relatively small piece of orbital debris can inflict a great deal of damage on satellites or
spacecrafts orbiting in the space. This phenomenon is Kessler Syndrome which describes a self-
sustaining cascading collision of space debris in low earth Orbit.
https://www.youtube.com/watch?v=xgGm5odlIh4
1. Superconductors are the materials with no electrical resistance i.e. theoretically there
are no loses in electricity transmission.
2. To achieve this state, a conductor is superheated upto its critical temperature and
before its melting point the electrons become free to move.
a) 1 only
b) 2 only
c) Both 1 and 2
d) Neither 1 nor 2
Superconductors are materials that conduct electricity with no resistance whatsoever. In order
to achieve superconducting state, the element should have mobile electrons, and these
IASbabas Prelims 60 Day Plan Day 17 2017
Q.15) India has been ranked 138th in the 2016 ICT Development Index out of a total of 175
countries. Which of the following statements are correct about ICT Development Index?
a) 1 only
b) 2 only
c) Both 1 and 2
d) Neither 1 nor 2
India has been ranked 138th in the 2016 ICT Development Index out of a total of 175 countries.
In 2015, India ranked 135th position. The index was released as part of the annual Measuring
the Information Society Report for 2016 of the International Telecommunications Union (ITU).
The ICT Development Index (IDI) rankings compare the state of ICT development across
countries. It is published annually by the UN ITU.
The IDI is based on 11 ICT indicators, grouped in three sub-indices: access, use and skills.
It is standard tool that governments, operators, researchers, development agencies and
others can use to measure the digital divide and compare ICT performance within and
across countries.
Top 5 Countries: South Korea (1st), Iceland (2nd), Denmark (3rd), Switzerland (4th) and
United Kingdom (5th).
Bottom 5 Countries: Niger (175), Chad (174), Guinea-Bissau (173), South Sudan (172)
and Burundi (171).
Q.16) Milk of which of the following mammal contains antimicrobial compounds that is
resistant to the powerful antibiotic vancomycin
a) Tasmanian devil
IASbabas Prelims 60 Day Plan Day 17 2017
b) Bandicoot
c) Wombat
d) Wallaby
Mothers milk from the marsupials known as Tasmanian devils could help the global fight
against increasingly deadly superbugs which resist antibiotics,
Scientists at the University of Sydney found that peptides in the marsupials milk killed resistant
bacteria, including methicillin-resistant golden staph bacteria and enterococcus that is resistant
to the powerful antibiotic vancomycin.
Source: http://www.thehindu.com/todays-paper/tp-national/Milk-from-Tasmanian-devils-
could-fight-superbugs-Study/article15621865.ece
a) Cartagena Protocol
b) Paris Agreement
c) Kyoto Protocol
d) Montreal Protocol
Q.18) Recently a new species of Pika was discovered in Sikkim. Consider the following
statements about Pika
a) Only 1
b) Only 2
c) Both 1 and 2
d) Nether 1 nor 2
Pikas are members of the rabbit family and look like tailless rats.
Keystone species is a species whose addition to or loss from an ecosystem leads to major
changes in occurrence of at least one other species.
Source: http://www.thehindu.com/todays-paper/tp-national/new-species-of-pika-
discovered/article9175091.ece
Contingent Reserve Arrangement (CRA) and National Development Bank (NDB) were
announced as part of BRICS Fortaleza Declaration announced during 6th BRICS summit held in
Fortaleza, Brazil in July 2014
IASbabas Prelims 60 Day Plan Day 17 2017
Source: http://economictimes.indiatimes.com/news/economy/policy/brics-contingent-
reserve-arrangement-operational-arun-jaitley/articleshow/54734524.cms
Q.20) Himansh is a
It is Indian governments initiatives to better study and quantify the Himalayan glacier
responses towards the climate change.
It has been established by the National Centre for Antarctic and Ocean Research (NCAOR),
under the Ministry of Earth Sciences.
Source: http://www.thehindu.com/sci-tech/science/High-altitude-glacial-lab-opens-in-the-
Himalayas/article15477833.ece
a) Only 1
b) Only 2
c) Both 1 and 2
d) Neither 1 nor 2
IASbabas Prelims 60 Day Plan Day 17 2017
It is the economic intelligence agency responsible for enforcing economic laws and fighting
economic crime in India It is under the Department of Revenue, Ministry of Finance.
Source: http://www.business-standard.com/article/companies/ed-to-probe-cyrus-mistry-s-
allegations-against-tata-sons-media-116102900361_1.html
IASbabas Prelims 60 Day Plan Day 18 2017
1. Grassland
2. Wetland
3. Mangroves
4. Estuary
5. Riparian Zone
a) 2, 3 and 4
b) 1, 2, 3 and 4
c) 1, 2, 3, 4 and 5
d) 2, 4 and 5
For e.g. the mangrove forests represent an ecotone between marine and terrestrial
ecosystem. Other examples are grassland (between forest and desert), estuary (between
fresh water and salt water) and river bank or marsh land (between dry and wet).
An ecotone may appear on the ground as a gradual blending of the two communities across
a broad area, or it may manifest itself as a sharp boundary line.
A riparian zone or riparian area is the interface between land and a river or stream.
Q.2) You are trying to protect a National Park/Sanctuary from various pressures including
a dam proposal and widening of a highway. Consider the following regarding this:
1. You can directly approach National Green Tribunal in regard to your case
2. You must have a lawyer to represent you for this case in front of National Green
Tribunal
Which of the above is/are correct w.r.t rules and procedure to approach National Green
Tribunal?
a) 1 Only
b) 2 Only
c) Both
IASbabas Prelims 60 Day Plan Day 18 2017
d) None
Because NGT is not empowered to hear matters pertaining to issues coming under the
ambit of the Wildlife (Protection) Act, 1972, which is applicable in case of National Parks,
Sanctuaries and Tiger Reserves. It would be appropriate to approach either the High Court in
your State or the Supreme Court. Please consult a competent lawyer for advice.
The NGT has the power to hear all civil cases relating to environmental issues and questions
that are linked to the implementation of laws listed in Schedule I of the NGT Act. These
include the following:
This means that any violations pertaining only to these laws, or any order / decision taken
by the Government under these laws can be challenged before the NGT. Importantly, the
NGT has not been vested with powers to hear any matter relating to the Wildlife
(Protection) Act, 1972, the Indian Forest Act, 1927 and various laws enacted by States
relating to forests, tree preservation etc. Therefore, specific and substantial issues related to
these laws cannot be raised before the NGT. You will have to approach the State High Court
or the Supreme Court through a Writ Petition (PIL) or file an Original Suit before an
appropriate Civil Judge of the taluk where the project that you intend to challenge is
located.
Microorganisms are used to leach out the minerals, rather than the traditional methods of
extreme heat or toxic chemicals, which have a deleterious effect on the environment.
In news: http://www.thehindu.com/news/cities/Tiruchirapalli/Bio-mining-helps-in-tackling-
garbage-problem/article16438953.ece
Q.4) Certain special conditions which are necessary prerequisites for the growth and
development of corals. Consider the following in this regard:
1. For the growth and development of corals the surface temperature of the ocean
must be below 20C
2. Shallow continental shelf
3. Corals need saltwater to survive
4. Low sediments
5. If the salinity of the ocean water is very high, the lime content is bound to be low,
which is favourable for the growth of corals
a) 1, 2, 3, 4 and 5
b) 1, 3 and 4 only
c) 2, 3 and 4 only
d) 2, 3 and 5 only
There are certain special conditions which are necessary prerequisites for the growth and
development of corals. That is why the reef building corals and their associates are not
uniformly deposited throughout the tropical warm ocean waters.
For the growth of corals in such large numbers, as is necessary to give rise to coral deposits,
there must be a reasonable balance of favourable conditions. The most important of these
conditions are the following:
1. For the growth and development of corals the surface temperature of the ocean must be
above 20C. Corals cannot live if the temperature of the sea water falls below this.
2. The water must be shallow, with a depth not exceeding 25-30 fathoms. As we know, with
increasing depth, the amount of calcium as well as the temperature of water goes on
decreasing, so that the coral polyps and other sea creatures which live on calcium carbonate
IASbabas Prelims 60 Day Plan Day 18 2017
do not thrive. That is why the reef building corals live in shallow waters and on the surface
of the seas.
3. The water must be normally saline, and, therefore, along the ocean margins where the
water becomes fresh by the inflow of rivers, reef building corals do not grow and develop.
4. Corals need clear water that is free from abundant sediments. That is why coral reefs
cannot develop where rivers enter the sea or where wave-erosion causes muddy coastal
water.
5. There must be adequate food supply to nourish the abundant life of the coral reef. The
most favourable condition for this purpose is the presence of continuously flowing ocean
currents which provide to the stationary reef building organisms the much needed food
supply.
For example, extensive coral reefs are found on the east coasts of Australia, Central
America, and Africa, which are washed by warm ocean currents flowing along them. On the
contrary, corals are found only in scattered patches on the west coasts of these continents.
6. If the salinity of the ocean water is very high, the lime content is bound to be low, which
is not favourable for the growth of corals. Therefore the average salinity ranging from 27%
to 40% is ideal for the proper growth of corals and other reef forming creatures.
7. The most essential prerequisite for the formation of coral reefs is the presence of sub-
marine platforms which must lie near the sea shore, or should be attached to some islands.
The depth of water on such platforms should not exceed 50 fathoms. Remember that the
corals build their permanent colonies only on these platforms.
8. Another point to remember is that corals cannot live for long out of water, and are
therefore, rarely found above the low-tide level. On the other hand, their growth is retarded
at depths much exceeding 25 or 30 fathoms.
9. Corals need clear oxygenated water with sufficient supplies of microscopic life as food.
10. Since food supplies are plentiful on the seaside of a growing reef, the corals tend to
grow rather at a fast rate outwards.
As the reefs grow in size, waves wash much broken corals in the form of boulders and sand
over the crest, thus building up a reef-flat, often with sand dunes, upon which vegetation
may grow. Coconut palms add special beauty to these 'Coral Islands'.
It should be borne in mind that "the reefs and atolls are easily 'drowned' if their upward
growth cannot keep place with any submergence (eustatic or tectonic) that may be in
progress."
IASbabas Prelims 60 Day Plan Day 18 2017
As regards the rate of growth of corals, it depends on the species, the range being 6 to 45
mm per year. However, according to Arthur Holmes, the average rate of reef growth is
about 14 mm a year.
Q.5) With reference to food chains in ecosystems, consider the following statements:
1. A food chain illustrates the order in which a chain of organisms feed upon each
other.
2. Food chains are found within the populations of a species.
3. A food chain illustrates the numbers of each organism which are eaten by others.
a) 1 only
b) 1 and 2 only
c) 1, 2 and 3
d) None
A food chain illustrates the order in which a chain of organisms feed upon each other. (True)
Food chains are found within the populations of a species. (Man wont eat man so, false)
A food chain illustrates the numbers of each organism which are eaten by others (it is
wrong) Note: Food web illustrates the number not the food chain)
Q.6) The ecological footprint is a measure of human demand on the Earth's ecosystems. It
measures-
1. How much of the biological capacity of the planet is demanded by a given human
activity or population
2. Ecological Footprints is calculated for overall activity of a nation or population and
activities such as industrialization etc.
3. The disadvantage of Ecological Footprint is that it cannot be calculated for individual
people or area.
a) 1 and 2
b) Only 2
c) 1 and 3
IASbabas Prelims 60 Day Plan Day 18 2017
d) Only 3
The Ecological Footprint is a resource accounting tool that measures how much biologically
productive land and sea is used by a given population or activity, and compares this to how
much land and sea is available. Productive land and sea areas support human demands for
food, fibre, timber, energy, and space for infrastructure. These areas also absorb the waste
products from the human economy. The Ecological Footprint measures the sum of these
areas, wherever they physically occur on the planet. The Ecological Footprint is used widely
as a management and communication tool by governments, businesses, educational
institutions, and non-governmental organizations.
Ecological Footprint accounts answer a specific research question: how much of the
biological capacity of the planet is demanded by a given human activity or population? To
answer this question, the Ecological Footprint measures the amount of biologically
productive land and water area an individual, a city, a country, a region, or all of humanity
uses to produce the resources it consumes and to absorb the waste it generates with
todays technology and resource management practices. This demand on the biosphere can
be compared to biocapacity, a measure of the amount of biologically productive land and
water available for human use. Biologically productive land includes areas such as cropland,
forest, and fishing grounds, and excludes deserts, glaciers, and the open ocean
Ecological Footprints can be calculated for individual people, groups of people (such as a
nation), and activities (such as manufacturing a product).
a) 1 and 3
b) Only 2
c) Only 1
d) 2 and 3
First statement is true. Coral bleaching not only happens because of warm temperature but
can also happen due to Cold temperature. Not all bleaching events are due to warm water.
In January 2010, cold water temperatures in the Florida Keys caused a coral bleaching event
that resulted in some coral death. Water temperatures dropped 12.06 degrees
Fahrenheit lower than the typical temperatures observed at this time of year. Researchers
will evaluate if this cold-stress event will make corals more susceptible to disease in the
same way that warmer waters impact corals.
When a coral bleaches, it is not dead. Corals can survive a bleaching event, but they are
under more stress and are subject to mortality.
http://www.theaustralian.com.au/news/nation/great-barrier-reef-scientists-
exaggerated-coral-bleaching/news-story/99810c83f5a420727b12ab255256774b
1. Ocean
2. Grasslands
3. Lakes
4. Mangroves
Which one of the following is the correct sequence of ecosystems in the order of
increasing productivity?
Mangroves are high productivity ecosystem. And since it is asked for increasing order, only
option (a) has mangrove in the last.
IASbabas Prelims 60 Day Plan Day 18 2017
1. Lichens
2. Bryophytes
3. Pteridophytes
4. Gymnosperms
5. Angiosperms
a) 1, 4 and 5 only
b) 1, 2, 3, 4 and 5
c) 1, 2 and 4 Only
d) 1, 4 and 5 Only
All the floral groups found in India. Simple examples can be read from class 9 NCERT
Q.10) The migration pattern Amur Falcon, known to be one of the longest distances
undertaken by migratory birds has been one of the interesting fields of research among
scientists and ornithologists. Identify the regions where Amur Falcon travels during its
migration?
1. South Africa
2. China
3. Siberia
4. Rajasthan
5. Nagaland
a) 1, 2 and 5
b) 2, 3, 4 and 5
c) 2, 3 and 5
d) 1, 2, 3, 4 and 5
The bird breeds in south-eastern Siberia and northern China before migrating in large flocks
across India and over the Arabian Sea to winter in southern Africa.
The birds that usually travelled through Maharashtra and Gujarat, were for the first time
spotted in Rajasthan in early May
The entire migration route of Amur falcons from Nagaland to South Africa, Mongolia and
back to Nagaland has been scientifically plotted. Scientists now have confirmed that a
satellite-tagged Amur falcon 'Naga' has travelled through Rajasthan.
The Amur falcon a marathon flier whose migrations stretch from the icy environs of
Siberia to the outcrops of South Africa has been spotted in rare areas across the State
(Spotted in Ballari, Kodagu, and the urban sprawls of Mysuru)
a) 1, 3 and 4
b) 1 Only
c) 1, 2, 3 and 4
d) None of the above
We are concerned about these phenomena because together they mean that even small
concentrations of chemicals in the environment can find their way into organisms in high
enough dosages to cause problems. In order for biomagnification to occur, the pollutant
must be:
long-lived
mobile
soluble in fats
biologically active
IASbabas Prelims 60 Day Plan Day 18 2017
Q.12) As per the notification of Ministry of Environment and Forests, the coastal land up
to 500m from the High Tide Line (HTL) and a stage of 100m along banks of creeks,
estuaries, backwater and rivers subject to tidal fluctuations, is called the Coastal
Regulation Zone(CRZ). Consider the following regarding CRZ
1. The area up to 200 metres from the High Tide Line is earmarked as No Development
Zone
2. Fishing and allied activities are permitted in CRZ-IV
a) 1 Only
b) 2 Only
c) Both
d) None
CRZ-1: these are ecologically sensitive areas these are essential in maintaining ecosystem of
the coast. They lie between low and high tide line. Exploration of natural gas and extraction
of salt are permitted
CRZ-2: these areas form up to the shore line of the coast. Unauthorised structures are not
allowed to construct in this zone.
CRZ-3: rural and urban localities which fall outside the 1 and 2. Only certain activities related
to agriculture even some public facilities are allowed in this zone
CRZ-4: this lies in aquatic area up to territorial limits. Fishing and allied activities are
permitted in this zone. Solid waste should be let off in this zone.
IASbabas Prelims 60 Day Plan Day 18 2017
http://envfor.nic.in/legis/crz/crznew.html
1. Biodiversity Hotspots
2. Community Reserves
3. Zoological Parks
4. Gene Sanctuary
5. Botanical Gardens
6. Sacred Groves
7. Seed banks
a) 2, 4, 5 and 6 only
b) 2, 3, 4 and 7 only
c) 1, 2, 4 and 7 only
d) 3, 5 and 7 only
In-situ conservation, the conservation of species in their natural habitats, is considered the
most appropriate way of conserving biodiversity.
Conserving the areas where populations of species exist naturally is an underlying condition
for the conservation of biodiversity. That's why protected areas form a central element of
any national strategy to conserve biodiversity.
Methods
Biosphere reserves
National parks
Wild sanctuaries
Biodiversity Hotspots
Gene sanctuary
Community reserves
Sacred groves
IASbabas Prelims 60 Day Plan Day 18 2017
Gene banks, e.g. seed banks, sperm and ova banks, field banks;
In vitro plant tissue and microbial culture collections;
Captive breeding of animals and artificial propagation of plants, with possible
reintroduction into the wild; and
Collecting living organisms for zoos, aquaria, and botanic gardens for research and
public awareness.
Q.14) Which among the following can effectively be examples of Secondary Succession?
a) 1, 2 and 3
b) 2, 3 and 4
c) 1, 3 and 4
d) All
The renewal of a forest after a fire: The fire itself destroys a majority of different
types of trees and plant life. Because seeds and roots and other plant and tree parts
remain in and on the soil, gradually the plants and trees begin to grow again and
eventually return to the state of the original ecosystem.
The renewal of a crop after harvesting: A crop is completed harvested when it
becomes ripe. Without new seeds being planted, the crop can regenerate the
following year due to the plants and seeds that remained after harvesting.
IASbabas Prelims 60 Day Plan Day 18 2017
A forest renews after logging: A large amount of trees were chopped down by
loggers in order to create building materials. Over time, trees grow in and the area
returns to its previous state.
Renewal after disease: A plant population can be very negatively affected by a
variety of infectious plant diseases. If the entire population dies, but the soil and
roots remain, it is possible for secondary succession to occur and for the population
of those plants to return.
A flood can ruin farmlands. However, because the soil remains after the waters
recede, over the course of many years a natural secondary succession can occur and
the vegetation that had previously grown there can grow again.
Plants can be very susceptible to attack from pests, particularly if there is an
overpopulation of those pests. When this occurs, the plant population in one area
can be completely destroyed. However, when the pest overpopulation is resolved,
the plants are able to live again and thrive in the soil in which they previously had
lived.
a) 1 only
b) 2 and 3 only
c) 1 and 3 only
d) 1, 2 and 3
Animal Waste like Urea, Uric acid and Death of vegetation add nitrogen in the form of
nitrates directly into soil.
Coal combustion adds nitrogen to atmosphere not to soil directly and from there it falls
back to earth in the form of acid rain and acid rain adds nitrogen to soil.
Q.16) Consider the following statements about Bay of Bengal Initiative for Multi-Sectoral
Technical and Economic Cooperation (BIMSTEC)
IASbabas Prelims 60 Day Plan Day 18 2017
1. All the members of BIMSTEC are members of Indian-Ocean Rim Association (IORA)
2. Nepal and Bhutan have observer status in BIMSTEC
a) Only 1
b) Only 2
c) Both 1 and 2
d) Neither 1 nor 2
The Bay of Bengal Initiative for Multi-Sectoral Technical and Economic Cooperation
(BIMSTEC) is an international organisation involving a group of countries in South Asia and
South East Asia. These are: Bangladesh, India, Myanmar, Sri Lanka, Thailand, Bhutan and
Nepal.
The Indian Ocean Rim Association (IORA) is an international organization with 21 Member
States - Australia, Bangladesh, Comoros, India, Indonesia, Iran, Kenya, Madagascar,
Malaysia, Mauritius, Mozambique, Oman, Seychelles, Singapore, Somalia, South Africa, Sri
Lanka, Tanzania, Thailand, UAE and Yemen.
Nepal was granted observer status by the second Ministerial Meeting in Dhaka in December
1998. Subsequently, full membership has been granted to Nepal and Bhutan in February
2004.
Source: http://pib.nic.in/newsite/PrintRelease.aspx?relid=151717
Q.17) Consider the following statements about African - Asian Rural Development
Organization (AARDO)
a) 1 and 2
b) 2 and 3
c) 1 and 3
IASbabas Prelims 60 Day Plan Day 18 2017
ARDO a non-political body enjoys observer status with various UN and other international
organisations like Food and Agriculture Organization (FAO), International Fund for
Agricultural Development (IFAD), United Nations Conference on Trade and Development
(UNCTAD), United Nations Educational, Scientific and Cultural Organization (UNESCO),
United Nations Development Programme (UNDP), International Cooperative Alliance (ICA),
Centre on Integrated Rural Development for Asia and the Pacific (CIRDAP), etc. It also
promotes collaboration with various international organisations for the economic and social
welfare of the rural community.
Source: http://economictimes.indiatimes.com/news/economy/policy/cabinet-clears-
india-aardo-mou-on-rural-development/articleshow/54693724.cms
Q.18) Which of the following countries left the Commonwealth of Nations recently?
a) Maldives
b) Gambia
c) Ireland
d) Zimbabwe
The Maldives quit Commonwealth over allegation of corruption and deteriorating human
rights.
The country becomes the latest to leave the Commonwealth after Gambia which quit in
October 2013.
IASbabas Prelims 60 Day Plan Day 18 2017
The last two countries to join The Commonwealth are Rwanda and Mozambique.
Source: www.thehindu.com/news/international/Maldives-quits-Commonwealth-against-
unfair-treatment-after-Nasheed-ouster/article16070214.ece
Q.19) Consider the following statements about Zero Defect Zero Effect (ZED) scheme
a) Only 1
b) 2 and 3
c) 1 and 3
d) All of the above
ZED Scheme aims to rate and handhold all MSMEs to deliver top quality products using
clean technology. It will have sector-specific parameters for each industry.
MSME sector is crucial for the economic progress of India and this scheme will help to
match global quality control standards.
The slogan of Zero Defect, Zero Effect (ZED) was first mentioned by PM Narendra Modi in
his Independence Day speech in 2014. It was given for producing high quality manufacturing
products with a minimal negative impact on environment.
Statement 1 - The final certification will be issued by Quality Council of India after reviewing
the site assessment report.
Statement 3 - No it is not mandatory, it is a voluntary scheme which will provide the MSMEs
a roadmap to global competitiveness.
IASbabas Prelims 60 Day Plan Day 18 2017
Quality Council of India, an autonomous body of Ministry of Commerce & Industry, has been
nominated as the National Monitoring & Implementation Unit.
Source: http://pib.nic.in/newsite/PrintRelease.aspx?relid=151762
Q.20) Consider the following statements about Mining Surveillance System (MSS)
a) Only 1
b) Only 2
c) Both 1 and 2
d) Neither 1 nor 2
The system checks 500 meters around the existing mining lease boundary to search
for any unusual activity relating to illegal mining. Any discrepancy is flagged off as a
trigger.
Automatic software leveraging image-processing technology will generate automatic
triggers of unauthorised activities. These triggers will be studied at a remote-sensing
control centre of IBM and then transmitted to district-level mining officials for field
verification.
Source: http://pib.nic.in/newsite/PrintRelease.aspx?relid=151684
NITI Aayog launched the first ever Agricultural Marketing & Farm Friendly Reforms Index.
Maharashtra ranks first.
It has a score which can have minimum value 0 implying no reforms and maximum value
100 implying complete reforms in the selected areas. States and UTs have been ranked in
terms of the score of the index.
The state of Maharashtra achieved first rank in implementation of various reforms. The
state has implemented most of the marketing reforms and it offers best environment for
IASbabas Prelims 60 Day Plan Day 18 2017
doing agribusiness among all the states and UTs. Gujarat ranks second with a score of 71.5
out of 100, closely followed by Rajasthan and Madhya Pradesh. Almost two third states
could not reach even halfway mark of reforms score. Major states like U.P., Punjab, West
Bengal, Assam, Jharkhand, Tamil Nadu and J&K are in this group.
Source: http://pib.nic.in/newsite/PrintRelease.aspx?relid=153145
IASbabas Prelims 60 Day Plan Day 19 2017
Q.1) Consider the following statements with reference to the Chamber of Princes.
a) 1 and 2 only
b) 2 and 3 only
c) 1 and 3 only
d) All of the above
The Chamber of Princes was formally formed in February 1921. The authors of Montagu-
Chelmsford report favored the formation of Council of princes and had made definitive
suggestions in that regard.
The idea was earlier put forward by Lytton and later on by Curzon, but it did not find favor
with the Home authorities.
i. 109 states that enjoyed full legislative and jurisdictional powers were represented
directly.
ii. 127 states that enjoyed limited legislative power were represented by the 12
members chosen from among themselves
iii. Remaining 326 states were clubbed as jagirs or estates.
The Chamber of Princes was merely advisory and Consultative body. They had no freedom
to discuss the internal affairs of other states.
The Viceroy and not the Governor-General in Council was the Crowns agent in dealing with
the States. This point was highlighted by the Butler Committee.
Q.2) Consider the following statements with reference to the National Planning
Committee (NPC)
1. The Haripur session of the INC in 1938 set up the National planning Committee.
2. Subhash Chandra Bose was the President of the National Planning Committee.
a) 1 only
b) 2 only
IASbabas Prelims 60 Day Plan Day 19 2017
c) Both 1 and 2
d) Neither 1 nor 2
The INC session of 1938 was held at Haripur and was presided by Subhash Chandra Bose.
The session gave the green signal for setting up the National planning Committee. The
Committee was Headed by Jawaharlal Nehru and had Dr J.C. Kumarppa, Prof Meghnad
Saha, M. Visveswarayya, Prof K.T. Shah, N.M. Joshi, G.L. Nanda, Dr V.K.R.V. Rao, Dr
Gyanchand and V.V. Giri as members.
The committee was to come up with a framework for the development of big industries,
cottage industries and agriculture and was to devise the plans to harmoniously develop all
the sectors of the economy.
a) 1 and 2 only
b) 2 and 3 only
c) 1 and 3 only
d) All of the above
M.N. Roy, while he was in Berlin, devoted much of his time to writing, editing, publishing
books and journals. Here he was able to re-edit, complete and publish (in 1922) his India in
Transition the work on which he had begun about two years ago. In 1922, he also started
publishing a bi-monthly paper titled The Vanguard of Indian Independence, organ of the
migr Communist Party of India, the stated objective of which was to spread socialism in
India. The paper was brought out regularly until 1928. During this period, the title of the
paper was changed several times. Roy and Ellen (under her pen name Santi Devi) wrote
articles calling upon Indian masses and nationalist leaders to adopt more effective lines of
struggle and to align with workers and peasants.
IASbabas Prelims 60 Day Plan Day 19 2017
Indian Federation of Labour is a federation of trade unions in India. IFL was founded in 1941
by M.N. Roy, after a split from the All India Trade Union Congress. In December 1948 IFL
merged into the Hind Mazdoor Sabha.
Q.4) Consider the following statements with reference to Mrs. Annie Besant
a) 1 and 2 only
b) 2 and 3 only
c) 1 and 3 only
d) All of the above
Annie Besant joined the Theosophical Society in May 1889 and became Madame Blavatsky's
devoted pupil and helper. She became a prominent worker in the Society and was elected
President which position she held till her death on 21 September 1933.
She first came to India on 16 November 1893. In October 1913 she spoke at a great public
meeting in Madras recommending that there should be a Standing Committee of the House
of Commons for Indian affairs which would go into the question of how India might attain
freedom.
She founded a weekly newspaper 'Commonweal' in January 1914 for her political work. In
June 1914 she purchased the 'Madras Standard' and renamed it 'New India'. The Home Rule
League was started on 1 September 1916. The main objective was to attain Home-rule for
India within the British Empire (on the lines of autonomous colonies of Ireland, Australia and
New Zealand).
In June 1917, with G. S. Arundale and B. P. Wadia, two of her principal workers, she was
interned at Ooty. Because of the wide protest all over India and abroad, the internment
order was withdrawn, and in August 1917 she was made the President of the Calcutta
Session of the Indian National Congress.
In 1917 she started the Women's Indian Association to which she gave her powerful
support. In 1924 the Association had 51 branches. In 1927 the first All India Women's
Conference was held in Poona and it became a permanent and powerful body.
IASbabas Prelims 60 Day Plan Day 19 2017
Q.5) Consider the following statements with reference to the Karachi Session of INC
(1931).
a) 1 only
b) 2 only
c) Both 1 and 2
d) Neither 1 nor 2
The Karachi session of 1931 was presided over by Sardar Vallabbhai Patel. This session saw
the emergence of Socialistic-radicalism.
The session was famous for the passing of Resolution on Fundamental rights and Economic
policy.
Some important aspects of these resolutions were: Basic civil rights of freedom of speech,
Freedom of Press, Freedom of assembly, Freedom of association, Equality before law
Elections on the basis of Universal Adult Franchise Free and compulsory primary education.
Substantial reduction in rent and taxes. Better conditions for workers including a living
wage, limited hours of work. Protection of women and peasants Government ownership or
control of key industries, mines, and transport and Protection of Minorities.
The Economic Policy included the provisions of protection of domestic industries against
foreign capital. Regulation of currency in national interest. Relief from agricultural
indebtedness and usury and the state ownership of key industries.
a) 1 only
b) 2 only
IASbabas Prelims 60 Day Plan Day 19 2017
c) Both 1 and 2
d) Neither 1 nor 2
The annual session of the Congress was held at Lahore in December 1929. During this
session presided over by Jawaharlal Nehru the Congress passed the Poorna Swaraj
resolution. Moreover, as the government failed to accept the Nehru Report, the Congress
gave a call to launch the Civil Disobedience Movement.
After the Indian National Congress authorized Gandhiji to start Civil Disobedience
Movement, he placed 11 Point Ultimatum to Irwin (31 Jan 1930) for administrative reforms
and stated that if the Ultimatums are ignored, the only way out was civil disobedience.
Gandhiji also informed of his decision to undertake Dandi Satyagraha wherein the laws of
the government would be violated. Breaking the salt laws of the government nonviolently
was the basic activity of civil disobedience. Along with this activity, activities like no tax
campaign, no revenue and no rent (land tax) campaign became very popular in different
parts of India.
The 11 points ultimatum of Gandhiji to Lord Irwin after being ignored by the British
Government made Gandhiji to launch the civil disobedience moment on 12th March 1930
with his famous Dandi March. (From Sabarmati Ashram to Dandi on Gujarat coast). On 6th
April, Gandhiji reached Dandi, picked up a handful of salt and broke the salt law as a symbol
of the Indian peoples refusal to live under British made laws and therefore under British
rule.
Q.7) Consider the following Statements with reference to the Nehru Report of 1928.
a) 1 only
b) 2 only
c) Both 1 and 2
d) Neither 1 nor 2
All the Major political parties had boycotted the Simon Commission (1927). Lord Birkenhead
had then challenged the Indian leaders to prepare a draft constitution. This challenge was
IASbabas Prelims 60 Day Plan Day 19 2017
accepted by the Indian leaders and an All Party Conference was called under the
Chairmanship of M.A. Ansari.
The conference then set up a drafting committee with Motilal Nehru as the President.
Jawaharlal Nehru was the Secretary to the committee. The committee submitted its report
in 1928.
The main points of the Nehru report were as follows: India would be given Dominion status.
This means independence within the British Commonwealth. India will be a federation
which shall have a bicameral legislature at the Centre and Ministry would be responsible to
the legislature. Governor General of India would be the constitutional head of India and will
have the same powers as that of British Crown. There will be no separate electorate. The
draft report also defined the citizenship and fundamental rights.
Q.8) consider the following statements with reference to the Champaran Satyagraha of
1917.
a) 1 and 2 only
b) 2 and 3 only
c) 1 and 3 only
d) All of the above
Champaran Satyagraha was the first Satyagraha started by Gandhiji in india. Gandhiji was
persuaded by Raj Kumar Shukla to come to Champaran to see the inhuman conditions of the
peasants. In Champaran there was an enormous personal increase in the land rent. The
peasants were obliged to grow indigo and this curtailed their freedom of cultivation.
The peasants were compelled to devote the best part of their land for growing particular
crops as desired by the landlord. They were also required to give their best time and energy
to the crops decided by the landlord.
Gandhi arrived in Champaran 10 April 1917 with a team of eminent lawyers: Brajkishore
Prasad, Rajendra Prasad, Anugrah Narayan Sinha. He was ordered to leave Champaran by
the local magistrate. His refusal led to his arrest. The government later developed cold feet
IASbabas Prelims 60 Day Plan Day 19 2017
and can instituted an Enquiry commission with Gandhi as a member. Based on the findings
of the committee champaran agrarian Act was passed.
1. The Meerut Conspiracy case : Arrest of Philip Spratt and Benjamin F. Bradley.
2. The Lahore conspiracy Case : Arrest of Bhagat Singh, Sukhdev and Rajguru.
3. The Alipore Conspiracy case : Arrest of Nalini Gupta and S.A. Dange.
a) 1 and 2 only
b) 2 and 3 only
c) 1 and 3 only
d) All of the above
Meerut Conspiracy case: The Meerut Conspiracy case started with the issuing of arrest
warrants against 31 Communists and other persons on 15th March 1929. The charges were
that these persons conspired to overthrow the British Government of India through strikes
and other militant methods. The arrested included almost all the known
communists including Muzafr Ahamed, S.A.Dange, S.V.Ghate, Dr.G.Adhikari, P.C.Joshi,
S.S.Mirajkar, Shaukat Usmani, Philip Stratt and others.
Lahore Conspiracy Case: On 8th April 1929, Bhagat Singh and Batukeshwar Dutt went to
the Central Legislative Assembly and threw a bomb at government benches and raised
slogans of Long live the Revolution. There were no casualties and probably none was
intended. Bhagat Singh and Dutt surrendered and were taken into custody. Many other
members of the association were arrested later and a bomb workshop unearthed. Except
for Chandra Shekar Azad all the prominent members were arrested and charged with the
murder of the Superintendent of Police of Lahore also.The prisoners were brutally treated in
jail. Jatin Das died after a hunger strike lasting 64 days. Bhagat Singh, Rajguru and Sukhdev
were later sentenced to death. There execution led to massive protests all over the country.
Alipore Conspiracy case: there were raids on the members of the Anushilan Samiti. The
documents such as letters were seized from the house of Arubindo Ghosh and he was
arrested and lodged in Alipore Jail. Khudi Ram Bose was meanwhile arrested. The trial
continued two years and finally Khudi Ram was hanged. Chitranjan Das, one of the
prominent barristers of the day defended and saved Sri Arubindo. Barindra Ghosh and
IASbabas Prelims 60 Day Plan Day 19 2017
Ullaskar Dutt were sentenced to rigorous imprison and they were released in 1920.
Q.10) Who among the following played an important role in the signing of Gandhi-Irwin
pact?
1. Motilal Nehru.
2. Tej Bahadur Sapru.
3. Madan Mohan Malaviya.
4. M. R. Jayakar
5. Maulana Azad.
a) I and 2 only
b) 2 and 4 only
c) 2 and 3 only
d) 3 and 5 only.
Gandhi-irwin Paact was signed on the 5th March 1931, it laid the ground to Gandhis
participation in the Second Round Table conference to be held in London.
The First Round table Conference did not have any representative from the Congress, and
Gandhiji was in jail due to his participation in Civil Disobedience movement.
The Liberal leaders Tej Bahadur Sapru and Jayakar played an important role in bringing into
effect the Gandhi-Irwin pact. Through the parleys of these leaders Gandhiji accepted to
suspend the CDM and take part in the Second Round Table Conference and the Viceroy
agreed to withdraw the ordinance to curb the congress, further more he agreed to free the
political prisoners not charged with acts of Violence.
Q.11) Consider the following statements with respect to the Communal Award of 1932.
a) 1 and 2 only
b) 2 and 3 only
IASbabas Prelims 60 Day Plan Day 19 2017
c) 1 and 3 only
d) All of the above
The Communal Award was made by the British Prime Minister Ramsay MacDonald on 16
August 1932 granting separate electorates in India for the Forward Caste, Lower Caste,
Muslims, Buddhists, Sikhs, Indian Christians, Anglo-Indians, Europeans and Untouchables
(now known as the Dalits) etc. The principle of weightage was also applied.
Special constituencies for women in all the provinces, except the North West Frontier
Province.
The reason behind introduction of this 'Award' was that Ramsay MacDonald considered
himself as 'a friend of the Indians' and thus wanted to resolve the issues in India. The
'Communal Award' was announced after the failure of the Second of the Three Round Table
Conferences (India). The 'award' attracted severe criticism from Mahatma Gandhi.
a) 1 and 2 only
b) 2 and 3 only
c) 1 and 3 only
d) All of the above
The Bombay Government (through its Revenue Department) had, in 1927, enhanced the
land revenue assessment in the Bardoli taluka by a nominal 22 percent, which, when
applied, amounted in some cases to as much as 60 percent enhancement. This translated in
increased land taxes. The Bardoli peasants had immediately made several claims regarding
this modification, the most important of which were that the rate of enhancement was
unjust and that it had been established without full and appropriate investigation. In
addition, they claimed that the tax officials report was inaccurate and thus an increase in
the tax was unwarranted. The local Congress Party organization published a critical report to
show that peasants could not sustain the enhanced assessments and a committee organized
IASbabas Prelims 60 Day Plan Day 19 2017
by the Congress drafted a petition and waited upon the Revenue Member of the State
government early in 1927. Given that the authorities refused to recognize these claims as
legitimate and change the law, the Bardoli peasants decided to organize a campaign aimed
at pressuring the Bombay government to launch an impartial inquiry into the enhancement
of land revenue assessment in Bardoli.
Patel was assisted in his role by a tier of secondary leadership composed of construction
workers (some of other faiths, such as two Muslims) who had worked with Mahatma
Gandhi in South Africa and several women from outside the district (some also of other
faiths, such as a Parsi woman from Bombay). Gandhi, although not directly involved in the
campaign, supported the struggle through his writings in Young India (a weekly journal
published in English by Mahatma Gandhi from 1919 to 1932) and through his visit in Bardoli
two months after the Satyagraha had been launched.
Q.13) In 1939, the Congress ministries in the state legislatures resigned, as a reaction to
Viceroy Linlithgow declared India at war with Germany on 3 September 1939. The Congress
objected strongly to the declaration of war without prior consultation with Indians. The
Congress Working Committee suggested that it would cooperate if there were a central
Indian national government formed, and a commitment made to India's independence after
the war.
The government did not come up with any satisfactory response. The viceroy Linlithgow
could only offer to form a 'consultative committee' for advisory functions. Thus, Linlithgow
IASbabas Prelims 60 Day Plan Day 19 2017
refused the demands of the Congress. On 22 October 1939, all Congress ministries were
called upon to tender their resignations." Both Viceroy Linlithgow and Muhammad Ali
Jinnah were pleased with the resignations. On 2 December 1939, Jinnah put out an appeal,
calling for Indian Muslims to celebrate 22 December 1939 as a "Day of Deliverance" from
Congress.
Q.14) The Azad Hind Government of 1943 was set up in which of the following countries?
a) Germany
b) Japan
c) Singapore
d) Austria
The Azad hind Government or the Provisional Government of Free India, or, more simply,
Free India (Azad Hind), was an Indian provisional government established in occupied
Singapore in 1943 and was supported by Japan and by Nazi Germany.
It was a part of a political movement originating in the 1940s outside of India with the
purpose of allying with Axis powers to free India from British Rule. It was established by
Indian nationalists-in-exile during the latter part of the Second World War in Singapore with
monetary, military and political assistance from Imperial Japan. Founded on 21 October
1943, the government was inspired by the concepts of Subhas Chandra Bose who was also
the leader of the government and the Head of State of this Provisional Indian Government-
in-exile. The government proclaimed authority over Indian civilian and military personnel in
Southeast Asian British colonial territory and prospective authority over Indian territory to
fall to the Japanese forces and the Indian National Army during the Japanese thrust towards
India during the Second World War. The government of Azad Hind had its own currency,
court and civil code, and in the eyes of some Indians its existence gave a greater legitimacy
to the independence struggle against the British
Q.15) Consider the following statements with reference to the Worlis revolt of 1945.
1. It was a tribal uprising against the forest contractors and money lenders.
2. It was a peasant uprising against the land revenue demands of the zamindars.
3. The Kisan Sabha took up their cause and launched a movement in 1945.
a) 1 and 2 only
IASbabas Prelims 60 Day Plan Day 19 2017
b) 2 and 3 only
c) 1 and 3 only
d) All of the above
The Worlis are the tribal peoples who live in western India, in parts of Maharashtra and
Gujrat. The Worli revolt was directed against the Forest contractors, money lenders, and
landlords who oppressed the people with the tacit support of bureaucracy. The Kisan Sabha
took up their cause and launched a struggle in May 1945. The police oppression failed to
terrorize the Worli people.
Important feature of the revolt was the massive women participation. The Worlis came
under the increasing influence of the Communist Party of India
Q.16) India has been ranked 138th in the 2016 ICT Development Index out of a total of 175
countries. It is released by
India has been ranked 138th in the 2016 ICT Development Index out of a total of 175
countries. In 2015, India ranked 135th position.
The index was released as part of the annual Measuring the Information Society Report for
2016 of the International Telecommunications Union (ITU).
The ICT Development Index (IDI) rankings compare the state of ICT development across
countries. It is published annually by the UN ITU. The IDI is based on 11 ICT indicators,
grouped in three sub-indices: access, use and skills. It is standard tool that governments,
operators, researchers, development agencies and others can use to measure the digital
divide and compare ICT performance within and across countries.
Source: http://www.business-standard.com/article/pti-stories/over-half-of-world-s-
population-does-not-use-internet-un-116112201423_1.html
IASbabas Prelims 60 Day Plan Day 19 2017
a) Only 1
b) Only 2
c) Both 1 and 2
d) Neither 1 nor 2
Pusa Arhar 16, a dwarf pigeon pea created by scientists at the Indian Agricultural Research
Institute (IARI), has a maturity time of 120 days, down from the 160-270 days needed by
varieties now in use. It also requires less water and is suitable for mechanized harvesting
with no loss in yields, at about 20 quintals/hectare.
The variety can help India achieve self-sufficiency in pulses in the next 2-3 years.
As crop maturity is synchronous (unlike present varieties which mature unevenly over time),
it can be harvested using combine harvesters. The evenness of the crop means it is (also)
easily amenable to pesticide sprays.
As the new variety is extra-early maturing, the farmers in rain-fed areas will have a wider
window for sowing. Longer duration crops have a shorter window as delayed planting
means sacrificing the next crop.
Source: http://www.livemint.com/Politics/k32rzJB7fMgU9YWALSlGiJ/Will-Pusa-Arhar-16-
solve-Indias-pulse-problem.html
a) 1 and 2
b) 2 and 3
c) 1 and 3
d) All of the above
ICAR-Indian Institute of Soil Science, Bhopal, a research institute under the Natural Resource
Management (NRM) Division of Indian Council of Agricultural Research (ICAR), has
developed Mridaparikshak, a MINILAB that can determine soil health.
Mridaparikshak is a digital mobile quantitative minilab/soil test kit to provide soil testing
service at farmers doorsteps.
Mridaparikshak determines all the important soil parameters i.e. soil pH, EC, organic carbon,
available nitrogen, phosphorus, potassium, sulphur and micronutrients like zinc, boron and
iron.
It also provides crop and soil specific fertilizer recommendations directly to farmers mobile
through SMS.
Mridaparikshak comes with soil sampling tools, GPS, balance, shaker, hot plate, and a Smart
Soil Pro, an instrument for determining the soil parameters and displaying of fertilizer
nutrient recommendations.
It can be operated by young educated farmers/rural youths (11-12 Pass) with short training.
Source:
http://www.iiss.nic.in/news%20and%20event/National%20Seminar%204%20%20and%20
5%20Nov%2016.pdf
a) Sikkim
b) Arunachal Pradesh
c) Uttarakhand
d) Jammu & Kashmir
IASbabas Prelims 60 Day Plan Day 19 2017
Source: http://www.thehindu.com/news/national/ITBP-Chinese-forces-face-off-in-
Demchok-over-water-project/article16435732.ece
Q.20) Consider the following statements about Pradhan Mantri Surakshit Matritva
Abhiyan (PMSMA)
1. It aims to reach out to all pregnant women who are in the 2nd & 3rd trimesters of
pregnancy.
2. A minimum package of antenatal care services will be provided to the beneficiaries
on the 9th day of every month at the Pradhan Mantri Surakshit Matritva Clinics
a) Only 1
b) Only 2
c) Both 1 and 2
d) Neither 1 nor 2
Pradhan Mantri Surakshit Matritva Abhiyan (PMSMA) is a fixed day strategy, every month
across the country during which a range of quality maternal health services are envisaged to
be provided as part of Antenatal Care.
Under the campaign, a minimum package of antenatal care services is to be provided to the
beneficiaries on the 9th day of every month at the Pradhan Mantri Surakshit Matritva Clinics
to ensure that every pregnant woman receives at least one checkup in the 2nd/ 3rd
trimester of pregnancy. If the 9th day of the month is a Sunday / a holiday, then the Clinic
should be organized on the next working day.
The programme aims to reach out to all Pregnant Women who are in the 2nd & 3rd
Trimesters of pregnancy.
Source: http://pib.nic.in/newsite/PrintRelease.aspx?relid=153275
a) World Bank
b) World Economic Forum
IASbabas Prelims 60 Day Plan Day 19 2017
According to the most recent Grant Thornton International Business Report, India improved
its ranking by one spot in a global index of business optimism, ranking second during the
third quarter (July-September 2016). Through the April-June period, India was placed third
on the list.
Source: http://economictimes.indiatimes.com/news/economy/indicators/with-gst-on-
its-way-india-rises-to-second-spot-on-global-biz-optimism-
index/articleshow/55277143.cms?utm_source=contentofinterest&utm_medium=text&ut
m_campaign=cppst
a) 1 and 2
b) 2 and 3
c) 1 and 3
d) All of the above
NASAs MMS has set the Guinness World Record for highest altitude fix of a Global
Positioning System (GPS) signal above the surface of the Earth.
The primary focus for the MMS science team will be one of the most important and least
understood of those processes: Magnetic Reconnection. Magnetic reconnection occurs
when magnetic fields around Earth connect and disconnect, explosively releasing energy.
IASbabas Prelims 60 Day Plan Day 19 2017
Source: http://indianexpress.com/article/technology/science/nasas-mms-creates-new-
guinness-world-record-3739784/
IASbabas Prelims 60 Day Plan Day 20 2017
Q.1) Consider the following statements with reference to the All India Kisan Sabha.
a) 1 only
b) 2 only
c) Both 1 and 2
d) Neither 1 nor 2
The All-India Kisan Sabha was formed in April 1936 at Lucknow with Swami Shajanand as
President and N.G. Ranga as General-Secretary.
The first session of the All-India Kisan Sabha was addressed by Jawaharlal Nehru. Others
participants included Ram Manohar Lohia, Sohan Singh Josh, Indulal Yagnik, Jaya Prakash
Narayan, Acharya Narendra Deva, Kamal Sarkar.
A Kisan Sabha manifesto was finalized and this was adopted by the Congress at Faizabad
session. The Kisan Sabha held its 2nd session along with the Faizpur Congress Session in
1936.
Q.2) Consider the following statements about the Lahore Session of 1916.
a) 1 only
b) 2 only
c) Both 1 and 2
d) Neither 1 nor 2
The 31st Session of the Congress was held at Lucknow in 1916.It was presided over by the
Ambica charan Majumdar who was a prominent lawyer and was actively associated with the
Congress since its birth. After a lapse of about 10 years both the Moderates and Extremists
IASbabas Prelims 60 Day Plan Day 20 2017
were united again which was a good sign for the national movement. In his address the
President declared 'If the United Congress was buried at Sutra it is reborn at Lucknow in the
garden of Wajid Ali Shah. After nearly 10 years of painful separation and wanderings
through the wilderness of misunderstandings the brother had at first met brothers'. In this
session the Congress and the Muslim League came closer to each other and they signed the
historic Lucknow Pact. The Muslim league and the INC held their annual sessions jointly in
1915 and 1916. The Lucknow Pact was significant as they jointly pressurized the
Government for self-rule, Dominion status etc.
Q.3) Consider the following statements with reference to State Peoples Conferences or
Praja Mandals.
a) 1 only
b) 2 only
c) Both 1 and 2
d) Neither 1 nor 2
The launching of the non-cooperation and Khilafat movements stirred the entire Indian
population cutting across the borders of British India. A large number of peoples
organizations in the Princely States were established in Mysore, Hyderabad, Baroda,
Kathiawad, Jamnagar, Indore, Nawanagar, etc. Praja Mandals or States Peoples
Conferences were established. In December 1927, an All India States Peoples Conference
was held and attended by about 700 delegates from different States. Balwantrai Mehta,
Maniklal Kothari and G.R. Abhayankar assumed the leadership of the movement.
The Congress, for the first time at its Nagpur Session in 1920, enunciated its policy towards
the peoples movement in the Princely States. It called upon the Princes to grant full
responsible government in their States. However, it was pointed out that though the people
belonging to the States could enroll themselves as members of the Congress, they could not
initiate political activity in the State in the name of the Congress. They could carry on
political activity in their individual capacity as members of the local Praja Mandals. This
position continued till 1935 though the cooperation between the leaders of the States
Peoples Conferences and the Congress leaders gradually increased. Finally, it was decided
that Congress Committees could be formed in the Indian States but they were not to engage
IASbabas Prelims 60 Day Plan Day 20 2017
in any unparliamentary activity or direct action. This compromise formula helped to bring
about harmony between the Congress and the freedom movement in the States.
From the mid-20s, the Congress started taking keen interest in the states peoples
movements. In 1929, Pandit Jawaharlal Nehru in his presidential address observed, The
Indian States cannot live apart from the rest of India. The only people who have a right to
determine the future of the States must be the people of those States.
Jawaharlal Nehru was elected president of the All India States Peoples Conference in 1939
at its session held in Ludhiana. He exhorted the Praja Mandals to step up the agitations in
the States to uphold the rights and dignities of the people. Important leaders like Ramanand
Tirtha and Ravi Narayan Reddy of Hyderabad, U.N. Debar and Balwantrai Mehta in
Saurashtra and Kathiawad region and Sheikh Abdullah in Kashmir spearheaded the
movement in these States.
During the Quit India movement, Congress formally extended the call for launching a
struggle to the people of the States as well. The constitutional changes, likely to take place
in the near future also underlined the necessity of having an organic relationship between
the Princely States and the Government of India.
Q.4) Which of the following statements about the Indian Councils Act of 1861 is/are
correct?
1. The act restored the legislative powers of the Madras and Bombay Provinces.
2. The act empowered the Viceroy to Issue ordinances.
3. The act created the Bicameral legislatures in the Provinces.
a) 1 and 2 only
b) 2 and 3 only
c) 1 and 3 only
d) All of the above
The Indian Councils Act 1861 was introduced because the British Government wanted to
involve the Indian people with the process of law making. This Act was passed on 1st August
1861. Its main provisions were as under:
1. The Executive Council of the Governor General was extended. It was decided the
members of his council should not be less than 6 and no more than 12 in number. These
IASbabas Prelims 60 Day Plan Day 20 2017
members were called the Additional Members of the Executive Council, and were not given
any significant power with regards to legislation.
2. Governor General was empowered to nominate the Additional Members for a period of 2
years and half of the members must be non-official.
4. Any bill and regulation passed by the Provincial Council could not become law until and
unless Governors and the Governor General gave their assent for the bill and regulation.
5. Under this Act the Governor General was empowered to issue ordinances.
6. Limited powers of legislation were given to the Presidencies of Bengal and Madras and
the Governor General was empowered to create similar councils for the provinces of
Frontier and the Punjab.
Q.5) Consider the following statements with reference to the Cripps Mission of 1942.
a) 1 only
b) 2 only
c) Both 1 and 2
d) Neither 1 nor 2
In March 1942, a mission headed by Stafford Cripps was sent to India with constitutional
proposals to seek Indian support for the war. Jawaharlal Nehru and Maulana Azad were the
official negotiators of Congress.
1. An Indian Union with a dominion status; would be set up; it would be free to decide its
relations with the Commonwealth and free to participate in the United Nations and other
international bodies.
IASbabas Prelims 60 Day Plan Day 20 2017
2. After the end of the war, a constituent assembly would be convened to frame a new
constitution. Members of this assembly would be partly elected by the provincial assemblies
through proportional representation and partly nominated by the princes.
3. The British Government would accept the new constitution subject to two conditions.
(i) any province not willing to join the Union could have a separate constitution and form a
separate Union, and (ii) the new constitution- making body and the British Government
would negotiate a treaty to effect the transfer of power and to safeguard racial and religious
minorities.
a) 1 only
b) 2 only
c) Both 1 and 2
d) Neither 1 nor 2
The Swaraj Party, Swarajaya Party or Swarajya Party or Swarajist Party, established as the
Congress-Khilafat Swarajaya Party, was a political party formed in India in January 1923 after
the Gaya annual conference in December 1922 of the National Congress, which sought
greater self-government and political freedom for the Indian people from the British Raj. It
was inspired by the concept of Swaraj. In Hindi and many other languages of India, swaraj
means "independence" or "self-rule." The two most important leaders were Chittaranjan
Das, who was its president and Motilal Nehru, who was its secretary.
Das and Nehru thought of contesting elections to enter the legislative council with a view to
obstructing a foreign government. Many candidates of the Swaraj Party got elected to the
central legislative assembly and provincial legislative council in the 1923 elections. In these
legislatures they strongly opposed the unjust government policies.
Vallabhai patel and rajendra Prasad belonged to the No Changers camp, they were against
the plan of Council entry.
IASbabas Prelims 60 Day Plan Day 20 2017
Q.7) Consider the following statements with respect to the Mahalwari system.
a) 1 and 2 only
b) 2 and 3 only
c) 1 and 3 only
d) All of the above
The Mahalwari system of land revenue was introduced by Holt MacKenzie and Robert
Merttins Bird. It played a lage part in making the system more flexible. The process of
preparing estimates of produce and rents was simplified too. It also introduced the fixation
of the average rents for different classes of soil. Mahalwari system was introduced in Agra,
Awadh (Oudh), Central parts of India, Punjab, parts of Gangetic valley etc during regime of
Lord Hastings. Mahal refers to an estate with many cultivators. The term Mahal referred to
the fiscal unit / revenue division into which the whole land was divided by Akbar.
Q.8) Consider the Following statement with reference to the Poona Pact
a) 1 only
b) 2 only
c) Both 1 and 2
d) Neither 1 nor 2
The Communal Award of the British Prime Minister Ramsay Macdonald provided separate
electorates for the Sikhs, Muslims, Anglo Indians and the Depressed Classes. Gandhi strongly
opposed the Communal Award on the grounds that it would disintegrate Hindu society.
Ambedkar vehemently criticised the way of handling the problems of the Harijans by
Gandhiji. Gandhi took up a fast unto death in his prison cell in Poona, protesting that
separate electorates were a device which would separate the untouchables from the Hindu
IASbabas Prelims 60 Day Plan Day 20 2017
society forever. He began an indefinite hunger strike at Yerwada Central Jail from 20
September 1932 to protest against this Award. A compromise was reached on 24
September 1932. This was called as Poona pact, as per this pact the Depressed Classes were
to be provided reservation in the general category seats itself.
Q.9) Consider the following statements with reference to the Charles Woods Dispatch of
1854.
a) 1 and 2 only
b) 2 and 3 only
c) 1 and 3 only
d) All of the above
ii. Establishment of D.P.I. office: The Despatch recommended that the existing Board of
Control for Education be abolished and the office of the Director of Public Instructions
should be established in the states.
iv. Medium of instruction: The medium of instruction of education in India would be English.
The Despatch accepted the views of Lord Macaulay.
v. Grant-in-aid system: The despatch proposed the system of grant- in-aid for the Indian
educational institutions in order to encourage the private enterprise for expansion of
education among Indians.
In order to be eligible for grant-in-aid, a school was to fulfill certain conditions such as:
IASbabas Prelims 60 Day Plan Day 20 2017
(c) It must realize a tuition fee, however, small, from the pupils.
vi. Training of teachers: In order to secure properly qualified teachers for schools, the
Despatch suggested the training of teachers in normal schools. To induce men of better
caliber to come to school-service the Despatch recommended sufficient salary for school
teachers.
vii. Women and muslim education: As Indians were two orthodox, the British Government
did not show any interest in education of females. It insisted on a policy of strict neutrality
in religion.
viii. Vocational education: The Despatch put importance on vocational instruction, and to
that end suggested the need of establishing vocational colleges and schools of industry.
ix. Mass education: The Despatch admitted that mass education had totally been neglected
and so far the Government devoted its attention exclusively towards providing means of
education for the higher classes.
The 12 year school scheme was proposed by the Sadler University Commission of 1917-
1919.
Q.10) Consider the following statements with reference to the Faizpur Congress session of
1937.
a) 1 only
b) 2 only
c) Both 1 and 2
d) Neither 1 nor 2
The Fiftieth (50th) Session of the Indian National Congress was held on 27 and 28 December
1936 at Faizpur, a village on the outskirts of Yawal Taluka of Jalgaon District of Bombay
Presidency (Maharashtra). It was, here, for the first time that Congress held its Annual
Session in a backward rural setting. A large number of peasants participated in the session.
The Faizpur Session was important for the Congress which had been raising demands for the
IASbabas Prelims 60 Day Plan Day 20 2017
welfare of the peasants and struggled for them. The Faizpur Session was also important
because it was presided over by Jawaharlal Nehru who was at his peak of influence in the
Congress.
Q.11) Consider the following statements about the Congress Socialist Party.
a) 1 only
b) 2 only
c) Both 1 and 2
d) Neither 1 nor 2
Congress Socialist Party, or (CSP), was a left-wing group within the Congress. It was formed
with Acharya Narendra Deva as President and Jay Prakash Narayan as General Secretary in
1934.
The CSP was not separate from the Congress. Its constitution defined that the members
were all required to be members of the Indian National Congress. Members of communal
organizations or political organizations, whose goals were incompatible with the ones of
CSP, were barred from CSP membership. Ram Manohar Lohia, Minoo Masani, Ashok Mehta
were other prominent members.
Q.12) The Interim Government of 1946 was setup under which of the following plans.
The cabinet Mission plan along with the elections to constituent assembly and provincial
autonomy suggested the formation of Interim Government in which, all the portfolios
including the War member were to be held by the Indian leaders having popular support.
IASbabas Prelims 60 Day Plan Day 20 2017
Q.13) A machine with strong brakes but no engine Jawaharlal Nehru made this remark
in respect to which of the following.
Jawaharlal Nehru made the remark in response to the provisions of the Government of India
Act 1935. The act was most elaborated and the lengthiest of all the British acts concerning
India, But, It had lots of ifs and Buts, which rendered it non-functional.
M.A. Jinnah called the act thoroughly rotten, fundamentally bad and completely
unacceptable.
The GOI Act provided a dual form of government (a "diarchy") for the major provinces. In
each such province, control of some areas of government, the "transferred list", were given
to a Government of ministers answerable to the Provincial Council. The 'transferred list'
included agriculture, supervision of local government, health, and education. The Provincial
Councils were enlarged.
At the same time, all other areas of government (the 'reserved list') remained under the
control of the Viceroy. The 'reserved list' included defence (the military), foreign affairs, and
communications.
It relaxed the central control over the provinces by demarcating and separating the central
and provincial subjects. The central and provincial legislatures were authorised to make
laws on their respective list of subjects. However, the structure of government continued to
be centralised and unitary.
IASbabas Prelims 60 Day Plan Day 20 2017
a) 1-2-3-4
b) 3-2-1-4
c) 3-1-2-4
d) 2-3-1-4
The Salt March is also known as the Dandi March. The 24-day march began from 12 March
1930 and continued till 6 April 1930 as a direct action campaign of tax resistance and
nonviolent protest against the British salt monopoly, and it gained worldwide attention
which gave impetus to the Indian independence movement and started the nationwide Civil
Disobedience Movement.
In the Lahore session of December 1929, Congress passed the Poorna Swaraj resolution. On
Dec 31st Jawaharlal Nehru Unfurled the Tri-colour flag of Congress.
Second Round Table Conference September 7 December 1931, Gandhi attended the
conference as the sole representative of Congress.
1. Article 7 of the Covenant recognises fair wages with equal pay for equal work,
sufficient to provide a decent living for workers and their dependants; safe working
conditions; equal opportunity in the workplace; and sufficient rest and leisure,
including limited working hours and regular, paid holidays
2. India is a party to the covenant
3. The Covenant is monitored by the UN Committee on Economic, Social and Cultural
Rights
a) 1 and 2
b) 2 and 3
c) 1 and 3
d) All of the above
The International Covenant on Economic, Social and Cultural Rights (ICESCR) is a multilateral
treaty adopted by the United Nations General Assembly on 16 December 1966, and in force
from 3 January 1976. It commits its parties to work toward the granting of economic, social,
and cultural rights (ESCR) to the Non-Self-Governing and Trust Territories and individuals,
including labour rights and the right to health, the right to education, and the right to an
adequate standard of living. As of 2015, the Covenant has 164 parties. A further six
countries, including the United States, have signed but not ratified the Covenant.
The ICESCR is part of the International Bill of Human Rights, along with the Universal
Declaration of Human Rights (UDHR) and the International Covenant on Civil and Political
Rights (ICCPR), including the latter's first and second Optional Protocols.
The Covenant is monitored by the UN Committee on Economic, Social and Cultural Rights.
Article 7 of the Covenant - recognises the right of everyone to "just and favourable" working
conditions. These are in turn defined as fair wages with equal pay for equal work, sufficient
to provide a decent living for workers and their dependants; safe working conditions; equal
opportunity in the workplace; and sufficient rest and leisure, including limited working
hours and regular, paid holidays. (Was cited by Supreme Court -
http://www.thehindu.com/news/national/Lesser-wages-for-equal-work-is-violation-of-
human-dignity-SC/article16086879.ece)
Source: http://www.thehindu.com/news/national/Lesser-wages-for-equal-work-is-
violation-of-human-dignity-SC/article16086879.ece
World Bank is all set to rank cities globally on a ease of living index.
The index is being launched keeping in mind that as cities grow and expand, the ease of
living becomes an important parameter.
The index could include categories on social inclusion, cost of living, public transport,
housing, education, health, environment-friendliness, crime/safety, governance and
corruption.
Source: http://www.thehindu.com/news/national/World-Bank-to-rank-cities-on-
%E2%80%98ease-of-living%E2%80%99/article16086210.ece
a) Only 1
b) Only 2
c) Both 1 and 2
d) Neither 1 nor 2
Due to the unique properties of the condensate, Lene Hau showed that light can either be
stopped or slowed down significantly to the velocity of 17 meters per second, resulting in an
extremely high refractive index.
This state was first predicted, generally, in 192425 by Satyendra Nath Bose and Albert
Einstein.
Superfluidity is the characteristic property of a fluid with zero viscosity which therefore
flows without loss of kinetic energy. When stirred a superfluid forms cellular vortices that
continue to rotate indefinitely. Superfluidity occurs in two isotopes of helium (helium-3 and
helium-4) when they are liquified by cooling to cryogenic temperatures. It is also a property
of various other exotic states of matter theorized to exist in astrophysics, high-energy
physics, and theories of quantum gravity. The phenomenon is related to BoseEinstein
condensation, but neither is a specific type of the other: not all Bose-Einstein condensates
can be regarded as superfluids, and not all superfluids are BoseEinstein condensates.
Source: http://www.thehindu.com/sci-tech/science/Paving-the-way-to-backpack-sized-
gravimetry/article16668344.ece
1. It was launched by the Indian Air Force and Indian Army in Jammu & Kashmir
2. It identifies areas and provides students with free coaching and make them
participate in extra-curricular activities
a) Only 1
b) Only 2
c) Both 1 and 2
d) Neither 1 nor 2
Indian Army has launched Operation school chalo in South Kashmir under a local slogan
Chyem ne zaroorat Daulat-o-ubab, Faqat Gochum School te Kitab (I dont need money and
fame, I need books and school. In it, Indian Army will identify areas and provide students
with free coaching and make them participate in extra-curricular activities. Beside this, Army
officers and men will also encourage the parents to send their children to makeshift
dwellings to study.
IASbabas Prelims 60 Day Plan Day 20 2017
Source: http://economictimes.indiatimes.com/news/defence/army-launches-school-
chalo-operation-in-south-kashmir/articleshow/55275160.cms
a) 1 and 2
b) 2 and 3
c) 1 and 3
d) All of the above
WMO is a specialized agency of the United Nations (UN) with 191 Member States and
Territories. It is the UN system's authoritative voice on the state and behaviour of the
Earth's atmosphere, its interaction with the land and oceans, the weather and climate it
produces and the resulting distribution of water resources.
WMO originated from the International Meteorological Organization (IMO), which was
founded in 1873 to facilitate the exchange of weather information across national borders.
Established in 1950, the WMO became a specialized agency of the United Nations in 1951.
Its mandate is in the areas of meteorology (weather and climate), operational hydrology and
related geophysical sciences. Since its establishment, WMO has played a unique and
powerful role in contributing to the safety and welfare of humanity. It has fostered
collaboration between the National Meteorological and Hydrological Services of its
Members and furthered the application of meteorology in many areas.
WMO continues to facilitate free and unrestricted exchange of data and information,
products and services in real- or near-real time on matters relating to safety and security of
society, economic welfare and the protection of the environment. It contributes to policy
formulation in these areas at national and international levels.
IASbabas Prelims 60 Day Plan Day 20 2017
The Organization plays a leading role in international efforts to monitor and protect the
environment through its Programmes. In collaboration with other United Nations agencies
and National Meteorological and Hydrological Services, WMO supports the implementation
of a number of environmental conventions and is instrumental in providing advice and
assessments to governments on related matters. These activities contribute towards
ensuring the sustainable development and well-being of nations.
The WMO and United Nations Environment Programme (UNEP) jointly created
Intergovernmental Panel on Climate Change (IPCC) received the Nobel Peace Prize in 2007
"for their efforts to build up and disseminate greater knowledge about anthropogenic (man-
made) climate change, and to lay the foundations for the measures that are needed to
counteract such change.
IASbabas Prelims 60 Day Plan Day 21 2017
Q.1) You may have heard the term Core Inflation in the news. The computation of Core
Inflation does not include which of the following items.
a) Consumer durables
b) Food and fuel items
c) Clothing and apparels
d) Medical expenses
Core inflation reflects the long-term trend in a particular price level. It is a measure
of inflation that excludes certain items that face volatile price movements because in finding
out the legitimate long run inflation, short-term price volatility and transitory changes in
price must be removed.
In India core inflation in calculated by removing Food and Fuel from the calculations, which
are seasonal, highly volatile and are affected by external factors.
Q.2) Consider the following statements with reference to the Consumer Price Index.
a) 1 only
b) 2 only
c) Both 1 and 2
d) Neither 1 nor 2
Consumer Price Index takes into account the final price paid by the consumer which
includes not only the prices paid at wholesale level, but also distribution costs and the taxes
paid to the government.
Excise duty, Service tax and Value added tax are all subsumed under the CPI as the end
consumer pays for them too.
Q.3) Consider the following statements with respect to the Wholesale Price Index (WPI)
1. Price rise in both the Goods and Services are included in computing WPI
2. It is published by Office of the Economic Adviser, Ministry of Commerce and Industry
IASbabas Prelims 60 Day Plan Day 21 2017
a) 1 and 2 only
b) 2 and 3 only
c) 1 and 3 only
d) All of the above
The WPI is calculated taking into account only the goods that are produced, services are not
included in calculation of WPI. It is calculated by taking into account the prices of a basket of
goods at the wholesale level
It is published by the Office of economic Advisor, Ministry of Commerce and Industry. 2004-
05 is the current base year for calculating WPI at constant prices.
Q.4) Consider the following statements with respect to Inflation measurement in India.
1. RBI has adopted Consumer Price Index (Combined) as the key measure of inflation
2. This was done based on the recommendation of Urjit Patel Committee
a) 1 only
b) 2 only
c) Both 1 and 2
d) Neither 1 nor 2
Both the statements are true, The Central Statistics Office (CSO), Ministry of Statistics and
Programme Implementation has revised the Base Year of the Consumer Price Index (CPI)
from 2010=100 to 2012=100. In this revised series, many methodological changes have been
incorporated, in order to make the indices more robust.
The Reserve Bank of India (RBI) Governor, said that the central bank had adopted the new
Consumer Price Index (CPI) (combined) as the key measure of inflation.
Earlier, RBI had given more weightage to Wholesale Price Index (WPI) than CPI as the key
measure of inflation for all policy purposes.
IASbabas Prelims 60 Day Plan Day 21 2017
The Urijit Patel committee had suggested adopting the CPI (combined) as the key measure
of Inflation.
Q.5) Consider the following statements about the new series of Consumer Price Index
a) 1 only
b) 2 only
c) Both 1 and 2
d) Neither 1 nor 2
The new series of CPI (Rural, Urban and combined) are published by the Central Statistical
Organization and not the RBI. 2012 is the base year for the ccalculation of new indices.
Under the CPI (Urban) food and beverages have 45.86 weightage, followed by housing
(10.07) and fuel and light (6.84).
Q.6) With reference to inflation in India, which of the following statements is correct?
a) Controlling the inflation in India is the responsibility of the Government of India only
b) The Reserve Bank of India has no role in controlling the inflation
c) Decreased money circulation helps in controlling the inflation
d) Increased money circulation helps in controlling the inflation
Inflation Control is primarily the responsibility of the Reserve bank of India. But the actions
of Government have considerable impact on the inflation in the economy.
Decreasing the Money supply will reduce the effective demand as people will have to
prioritize their spending because of less supply of money. This helps in Preventing Inflation.
a) 1 and 2 only
b) 2 and 3 only
c) 1 and 3 only
d) All of the above
Bond Holders are not benefitted by inflation as the real interest they get on the bonds
decreases. Suppose you have a Bond that promises 7% rate of interest, and the inflation is
5%, then the real return on the bond is only 2% and not 7%.
Producers are benefitted by inflation, because of the Price rise their profits will also
increase.
Debtors stand to gain as the real interest they have to pay decreases.
Q.8) Consider the following statements with reference to The Open Market Operations of
RBI
a) 1 only
b) 2 only
c) Both 1 and 2
d) Neither 1 nor 2
Open Market Operations are carried out by the RBI to either increase the Money supply or
to suck up the excess liquidity in the market.
This is done by either selling or purchasing the Government Bonds by the RBI in the
Markets. If RBI sells the Bonds, then the Money supply is lessened by that extent, as the
money will be transferred from public to RBI, hence no longer available in the Market.
IASbabas Prelims 60 Day Plan Day 21 2017
If the RBI buys the Bonds, then the Money supply is increased to that extent, as the out of
circulation money comes into circulation.
a) 1 and 2 only
b) 2 and 3 only
c) 1 and 3 only
d) All of the above
Increase in the Money supply will increase the Net demand for the products. As people will
have more money to spend in their hands, this will cause the Demand Pull inflation.
If the aggregate level of output decreases, then there will be scarcity of goods, which intern
will also cause inflationary pressures, since this inflation is due to the supply side
constraints, it is called as Cost-push Inflation.
a) 1 only
b) 2 and 3 only
c) 3 only
d) None of the above
Both the borrowing from the Public and fro the banks will not lead to increase in the money
supply in the economy, on the contrary, the public will have less money in their hands to
spend.
On the other hand, creating new money increases the supply of Money which helps in
creating aggregate demand. So this is most likely to cause the demand Pull inflation.
Increase in the Interest rates will increase the prices of inputs that are required for the
production, this will either cause a decrease in the aggregate output. If the interest rates are
increased, then to maintain the same profit levels the producer has to sell more goods, this
affects his profit margins.
The rest of the options will cause demand-pull inflation as it will give more money into the
hands of the people.
a) 1 only
b) 2 only
c) Both 1 and 2
d) Neither 1 nor 2
Higher inflation reduces the cost advantage enjoyed by the exporters, they will find local
consumption more profitable than exporting and will divert the goods for local production.
This reduces the exports and also competitive advantage. Decreased exports may lead to
Balance of payment problems if the situation is not rectified.
IASbabas Prelims 60 Day Plan Day 21 2017
Q.13) Consider the following statements with reference to the Philips Curve.
1. It gives the relationship between optimal tax rates and tax collections
2. It gives the relationship between the rate of inflation and rate of employment in the
economy
a) 1 only
b) 2 only
c) Both 1 and 2
d) Neither 1 nor 2
Philips Curve gives the relationship between the rate of inflation and the rate of
Unemployment in the economy.
It says that higher inflation leads to lower rates of Unemployment. Subsequent empirical
investigations have found that higher inflation does not necessarily lead to lower
unemployment rates, it may worsen them, as seen in the case of stagflation.
Q.14) A situation where high inflation and high unemployment co-exist is called as
a) Recession
b) Stagflation
c) Depression
d) Hyper inflation
A stagflation is a situation where high inflation and high unemployment rates co-exist. This
is contrary to the Philips law which says that higher inflation will lead to low unemployment
rates.
a) 1 only
b) 2 only
c) Both 1 and 2
d) Neither 1 nor 2
Higher direct taxes will mean that the people will be left with smaller disposable incomes,
lower disposable incomes do not generate higher demand. This causes the inflation to be
controlled.
Whereas higher indirect taxes will increase the prices of the commodities because the cost
of production will increase. This will lead to Cost-push inflation.
A dedicated Air Quality Information Service for Indian Metropolitan Cites to make India self-
sufficient in providing frontier research based scientific accredited robust Air Quality
Forecasting system.
Indias first air quality checking Mobile App, SAFAR-Air was launched on 17 February 2015 at
the Indian Institute of Tropical Meteorology in Pune, Maharashtra.
SAFAR is an acronym for System of Air Quality Weather Forecasting and Research which
was first launched in Delhi in 2010 during the Commonwealth Games.
It is under the Ministry of Earth Science, Govt. of India.
The app will provide current data and a forecast for air quality in the users current location
through a colour-coded system- green is good, yellow is moderately polluted, orange is
poor, red is very poor and maroon is critical.
Source: http://timesofindia.indiatimes.com/city/mumbai/Fortnight-after-Diwali-Mumbai-
air-quality-hits-very-poor-mark/articleshow/55467096.cms
a) Only 1
b) Only 2
c) Both 1 and 2
d) Neither 1 nor 2
The fishing cat (Prionailurus viverrinus) is a medium-sized wild cat of South and Southeast
Asia. Since 2016, it is listed as Vulnerable on the IUCN Red List.
Fishing cat populations are threatened by destruction of wetlands and declined severely
over the last decade.
Fishing cats live foremost in the vicinity of wetlands, along rivers, streams, oxbow lakes, in
swamps and mangroves.
Source: http://www.thehindu.com/news/national/andhra-pradesh/Fishing-cat-found-
moving-in-mangrove/article16086026.ece
a) Nepal
b) Bangladesh
c) Bhutan
d) Sri Lanka
The joint exercise SAMPRITI 2016 will simulate a scenario where both nations are working
together in a Counter Insurgency and Counter Terrorism environment under the UN
Charter.
Source: http://pib.nic.in/newsite/PrintRelease.aspx?relid=153191
Q.19) Azerbijan shares its border with which of the following countries?
a) Iran
b) Armenia
c) Georgia
d) Serbia
a) 1, 2 and 3
b) 2 and 3
c) 2, 3 and 4
d) All of the above
It is a country in the South Caucasus region, situated at the crossroads of Southwest Asia
and Southeastern Europe. It is bound by the Caspian Sea to the east, Russia to the north,
Georgia to the northwest, Armenia to the west and Iran to the south. The exclave of
Nakhchivan is bound by Armenia to the north and east, Iran to the south and west, while
having an 11 km border with Turkey in the north west.
IASbabas Prelims 60 Day Plan Day 21 2017
Source: http://pib.nic.in/newsite/PrintRelease.aspx?relid=153267
a) Diamonds
b) Gold
c) Crude Oil
d) None of the above
The Kimberley Process (KP) is an international initiative to stem the trade in conflict
diamonds. In 2002, the KPCS was launched by a coalition of governments, civil society and
the diamond industry in response to the role of diamonds in funding some of the most
devastating civil wars in Africa.
India has been elected as the Vice Chair for 2018 and Chair for 2019 of the Kimberley
Process Certification Scheme (KPCS).
Source: http://www.business-standard.com/article/news-ians/india-elected-vice-chair-of-
kpcs-2018-116111900835_1.html
IAsbabas Prelims 60 Day Plan Day 22 2017
Q.1) Consider the following statements with reference to the recently launched India Post
Payment Bank (IPPB)
a) 1 only
b) 2 only
c) Both 1 and 2
d) Neither 1 nor 2
IPPB is a payment bank and as per the RBIs regulations the Payment banks cannot accept
Term deposits, they can accept demand deposits only.
Apart from amounts maintained as Cash Reserve Ratio (CRR) with the Reserve Bank on its
outside demand and time liabilities, it will be required to invest minimum 75 per cent of its
"demand deposit balances" in Statutory Liquidity Ratio (SLR) eligible Government
securities/treasury bills with maturity up to one year and hold maximum 25 per cent in
current and time/fixed deposits with other scheduled commercial banks for operational
purposes and liquidity management.
Source: https://www.rbi.org.in/Scripts/BS_PressReleaseDisplay.aspx?prid=32615
Q.2) Consider the following statements with reference to the Small Finance Banks
a) 1 and 2 only
b) 2 and 3 only
IAsbabas Prelims 60 Day Plan Day 22 2017
c) 1 and 3 only
d) All of the above
The objectives of setting up of small finance banks will be to further financial inclusion by (a)
provision of savings vehicles, and (ii) supply of credit to small business units; small and
marginal farmers; micro and small industries; and other unorganized sector entities,
through high technology-low cost operations
The small finance bank shall primarily undertake basic banking activities of acceptance of
deposits and lending to unserved and underserved sections including small business units,
small and marginal farmers, micro and small industries and unorganised sector entities.
The minimum paid-up equity capital for small finance banks shall be Rs. 100 crore.
The small finance bank will be subject to all prudential norms and regulations of RBI as
applicable to existing commercial banks including requirement of maintenance of Cash
Reserve Ratio (CRR) and Statutory Liquidity Ratio (SLR). No forbearance would be provided
for complying with the statutory provisions.
The small finance banks will be required to extend 75 per cent of its Adjusted Net Bank
Credit (ANBC) to the sectors eligible for classification as priority sector lending (PSL) by the
Reserve Bank.
At least 50 per cent of its loan portfolio should constitute loans and advances of upto Rs. 25
lakh.
If the small finance bank aspires to transit into a universal bank, such transition will not be
automatic, but would be subject to fulfilling minimum paid-up capital / net worth
requirement as applicable to universal banks; its satisfactory track record of performance as
a small finance bank and the outcome of the Reserve Banks due diligence exercise.
Source: https://rbi.org.in/scripts/BS_PressReleaseDisplay.aspx?prid=32614
Q.3) Recently Strategic Debt Restructuring Scheme was in news, consider the following
statements with respect to the SDRC
1. Under this scheme the loan outstanding by a company to the Bank will be converted
into equity
2. Banks will acquire the majority ownership in the debtor company
a) 1 only
IAsbabas Prelims 60 Day Plan Day 22 2017
b) 2 only
c) Both 1 and 2
d) Neither 1 nor 2
Under the strategic restructuring scheme the Forum of lenders to a company can convert
either full or part of the outstanding loan into equity, this changes their nature from
creditors to owners.
The RBI guidelines stipulate that whenever the SDR arrangement is undertaken the forum of
lenders will acquire the majority stake in the Company i.e. 51% majority
Source: https://rbi.org.in/Scripts/NotificationUser.aspx?Id=9767
Q.4) Consider the following statements about the reverse repo rate
1. It is an interest rate at which the Banks deposit their cash with the RBI
2. If reverse repo rate is increased the banks lending rates to customers will also
increase
a) 1 only
b) 2 only
c) Both 1 and 2
d) Neither 1 nor 2
Definition: Reverse repo rate is the rate at which the central bank of a country (Reserve
Bank of India in case of India) borrows money from commercial banks within the country. It
is a monetary policy instrument which can be used to control the money supply in the
country.
Description: An increase in the reverse repo rate will decrease the money supply and vice-
versa, other things remaining constant. An increase in reverse repo rate means that
commercial banks will get more incentives to park their funds with the RBI, thereby
decreasing the supply of money in the market.this also means that lending to the consumers
becomes risky, as they can easily get risk free returns from parking the money with the RBI,
to compensate this the Banks increase their interest rates to earn more profits for taking the
risk
IAsbabas Prelims 60 Day Plan Day 22 2017
Q.5) Consider the following statements with reference to the Cash reserve ratio (CRR)
1. It is a percentage of Banks Net time and Demand Liabilities that are kept with the
RBI
2. CRR deposits earn interest at the rate which is equal to reverse repo rate.
a) 1 only
b) 2 only
c) Both 1 and 2
d) Neither 1 nor 2
CRR is a percentage of a Banks net time and demand liabilities that it has to keep with the
RBI. It is kept in Cash form. The CRR deposits do not earn any interest as long as they are
with the RBI.
Q.6) RBI uses variety of Qualitative and Quantitative tools to control the Credit creation in
the economy, in this regard which of the following is not a Quantitative tool?
E.g. if the Reserve Ratios are increased then Banks have to keep the equivalent money in
either their vault or with the RBI, this takes that much of money out of circulation.
In contrast, by varying the margin requirements, the money supply is not reduced from the
economy, it is only stopped from flowing into one sector or industry.
Q.7) Consider the following statements with reference to the MUDRA Bank
IAsbabas Prelims 60 Day Plan Day 22 2017
1. It will provide long term loans directly to the small scale enterprises
2. It will be the regulator of all Micro Finance Institutions in the country
a) 1 only
b) 2 only
c) Both 1 and 2
d) Neither 1 nor 2
Mudra bank does not provide direct loans to the consumers, it refinances the loans, i.e. it
provides money to the banks, co-operatives etc. which in turn give this money as loan to the
customers.
The Budget proposal to make Mudra Bank as the regulator of the NBFC-MFI was later
withdrawn by the government. As of now, RBI is the regulator of the Micro Finance
institutions in India
Q.8) which of the following statements with respect to Marginal Standing Facility is
incorrect?
a) Under this scheme all the banks can borrow money from RBI for 1 day
b) The minimum loan request has to be 1 crores and multiples there of
c) Banks can borrow 1% of their Net time and Demand Liabilities
d) Interest rate of repo rate plus one percent will be charged by the RBI
The term all the banks will include Schedule Commercial banks, Non-schedule Banks, Co-
operative banks, RRBs etc. Only Scheduled Commercial Banks having Current Account and
SGL Account with Reserve Bank, will be eligible to participate in the MSF Scheme. Under the
facility, the eligible entities can avail overnight, up to one per cent of their respective Net
Demand and Time Liabilities (NDTL) outstanding at the end of the second preceding
fortnight. The rate of interest on amount availed under this facility will be 100 basis points
above the LAF repo rate, or as decided by the Reserve Bank from time to time. Requests will
be received for a minimum amount of Rs. One crore and in multiples of Rs. One crore
thereafter.
Source: https://www.rbi.org.in/scripts/NotificationUser.aspx?Id=6394&Mode=0
IAsbabas Prelims 60 Day Plan Day 22 2017
Q.9) If RBI reduces the CRR requirements, which of the following are most likely to happen?
a) 1 and 2 only
b) 2 and 3 only
c) 1 and 3 only
d) All of the above
If the RBI cuts the Cash Reserve ratio, it means that the banks will no longer have to keep
that much of money with the RBI, instead that money can be used to create more loans and
advances, this increases the money supply in the economy.
If the money supply is increased in the economy, the demand for loans will also come down,
which will force the banks to cut the interest rates they charge on the end Customer.
The side effect of this is that, increased money supply will leave more money in the hands of
the people, which will increase the effective demand, and if the production lags behind, it
will create inflationary situation in the economy
Q.10) The Reserve Bank of India (RBI) acts as a bankers bank. This would imply which of
the following?
a) 1 and 2 only
b) 2 and 3 only
c) 1 and 3 only
d) All of the above
Banker to Banks: Like individual consumers, businesses and organisation of all kinds, banks
need their own mechanism to transfer funds and settle inter-bank transaction-such as
IAsbabas Prelims 60 Day Plan Day 22 2017
borrowing from and lending to other banks-and customer transactions. As the banker to
banks, the Reserve Bank fulfills this role.
Banks are required to maintain a portion of their demand and time liabilities as cash
reserves with the Reserve Bank. For this purpose, they need to maintain accounts with the
Reserve Bank. They also need to keep accounts with the Reserve Bank for settling inter-bank
obligations, such as, clearing transactions of individual bank customers who have their
accounts with different banks or clearing money market transactions between two banks,
buying and selling securities and foreign currencies.
Commercial banks create credit. It is the duty of the RBI to control the credit through the
CRR, bank rate and open market operations. As banker's bank, the RBI facilitates the
clearing of checks between the commercial banks and helps inter-bank transfer of funds. It
can grant financial accommodation to schedule banks. It acts as the lender of the last resort
by providing emergency advances to the banks.
RBI rediscounts Bills of exchange from the Banks that they have discounted from the
consumer. This effectively sets the Bank rate at which the RBI is willing to lend to the banks
Q.11) Recently Monetary Policy Committee (MPC) was in the news, in this regard consider
the following statements
1. It will have six members including the RBI Governor who will be the Chairman
2. All the members of MPC will hold office for a 5 year term
a) 1 only
b) 2 only
c) Both 1 and 2
d) Neither 1 nor 2
The Reserve Bank of India Act, 1934 (RBI Act) has been amended by the Finance Act,
2016, to provide for a statutory and institutionalised framework for a Monetary Policy
Committee, for maintaining price stability, while keeping in mind the objective of growth.
The Monetary Policy Committee would be entrusted with the task of fixing the benchmark
IAsbabas Prelims 60 Day Plan Day 22 2017
policy rate (repo rate) required to contain inflation within the specified target level. A
Committee-based approach for determining the Monetary Policy will add lot of value and
transparency to monetary policy decisions. The meetings of the Monetary Policy Committee
shall be held at least 4 times a year and it shall publish its decisions after each such meeting.
As per the provisions of the RBI Act, out of the six Members of Monetary Policy Committee,
three Members will be from the RBI and the other three Members of MPC will be appointed
by the Central Government.
The Members of the Monetary Policy Committee appointed by the Central Government
shall hold office for a period of four years, with immediate effect or until further orders,
whichever is earlier.
The 3 members from the RBI will be in the committee by virtue of their official position, they
do not get a term of 4 years.
Q.12) Consider the following statements with reference to the Payment Banks
1. They can issue Debit cards but cannot issue Credit Cards
2. They are exempted from the CRR and SLR requirements
3. The main objective of establishing the payment banks is to further Financial Inclusion
a) 1 and 2 only
b) 2 and 3 only
c) 1 and 3 only
d) All of the above
Scope of activities:
5) Distribution of non-risk sharing simple financial products like mutual fund units and
insurance products, etc.
Q.13) Consider the following statements with reference to Sukanya Samruddhi Yojana
1. Main objective of the scheme is to cover the Higher education and Marriage
expenses of the Girl Child
2. The Government will pay interest on the accounts opened under the scheme for 25
years
a) 1 only
b) 2 only
c) Both 1 and 2
d) Neither 1 nor 2
Sukanya Samriddhi Yojana was launched by the Prime Minister Mr. Narendra Modi on 21st
January 2015; under the Beti Bachao, Beti Padhao Campaign. This scheme was launched to
meet the expense of the Girl childs higher education and marriage.
Tenure of the Scheme: Deposits needs to made until 15 years from opening of account.
Deposit under scheme will mature 21 year after opening of the account.
Scheme Tenure is 21 years from date of opening, or when the marriage of the girl child
happens; which ever happens earlier.
Account can be continued after the marriage of girl child, if she wishes to do so; until 21
years from date of opening the account. In case after maturity of the account (21 years) the
account is not closed, balance will not earn any further interest.
IAsbabas Prelims 60 Day Plan Day 22 2017
Q.14) Consider the following statements with reference to the Bharath QR code
1. It will work as a common interface for all the debit/credit card payments in the
country
2. It is managed by the National payments corporation of India
3. It is one of the initiates of the GOI to promote cashless transactions
a) 1 and 2 only
b) 2 and 3 only
c) 1 and 3 only
d) All of the above
What is a QR code?
BharatQR is a common QR code jointly developed by all the four major card payment
companiesNational Payments Corp. of India that runs RuPay cards, MasterCard, Visa and
American Expressunder instructions from the Reserve Bank of India (RBI).
Q.15) Consider the following statements with reference to the Banks Board Bureau
a) 1 only
b) 2 only
c) Both 1 and 2
d) Neither 1 nor 2
IAsbabas Prelims 60 Day Plan Day 22 2017
The BBB will recommend the names for the appointment of Directors for the public sector
banks (it does not appoint, only recommends)
Banks board of bureau will replace existing appointments board. Its members would be
appointed in the next six months to be headed by the RBI governor.
Banks board bureau would also hold bad assets of public sector banks. Under
recapitalisation plans for Public Sector Banks, 13 banks would get Rs 20,058 crore this
financial year. The rest Rs 5,000 crore would be allocated based on efficiency criteria.
Q.16) Ken-Betwa river linking project has become a stage for a unique man-animal
conflict. The environmentalists fear that one of the tiger reserve/national park may get
submerged. Which is the tiger reserve that we are referring to?
a) Panna
b) Pench
c) Kanha
d) Bandhavgarh
An ambitious project to link Ken and Betwa rivers has become a stage for a unique man-
animal conflict. Proponents of the project, led by the Union Water Ministry, say that the
proposed Daudhan dam and the 2.5 km canal the key structures of the project that will
transfer surplus water from the Uttar Pradesh section of the Ken to the Betwa in Madhya
Pradesh are critical to irrigate nearly 7,00,000 hectares in drought-ravaged Bundelkhand.
However, environmentalists say that such a dam will submerge at least 4,000 hectares of
Madhya Pradeshs Panna tiger reserve, whose tigers were almost lost to poaching in 2009
and have only recently been partially replenished. They allege that most districts in Madhya
Pradesh will not actually get the promised water. There are vultures in the region, whose
nests will be threatened by the height of the dam.
IAsbabas Prelims 60 Day Plan Day 22 2017
Source: http://youthdevelopmentindex.org/
a) Only 1
b) Only 2
c) Both 1 and 2
d) Neither 1 nor 2
Government of India will assist in renovation and upgradation of the Ghats adjoining the
Temple area.
IAsbabas Prelims 60 Day Plan Day 22 2017
Pashupatinath temple
Source: http://pib.nic.in/newsite/PrintRelease.aspx?relid=153233
Scientists have detected the presence of water on Psyche, the largest metallic asteroid in
our solar system and the target of a proposed NASA mission.
Change from past- Previous observations of Psyche had shown no evidence of water-rich
minerals on its surface. But recently, new observations from the NASA Infrared Telescope
Facility in Hawaii show evidence of water or hydroxyl on its surface.
Psyche-
Reason: Water-rich minerals detected on Psyche may have been delivered by carbonaceous
asteroids that impacted Psyche in the distant past, according to scientists.
http://www.livemint.com/Science/E6hlhMHBDJ4eDY5UgLMysK/Scientists-detect-water-
on-metallic-asteroid-Psyche.html
a) 1 and 2
b) 2 and 3
c) Only 1
d) All of the above
The Sentinelese (also called the Sentineli or North Sentinel Islanders) are the indigenous
people of North Sentinel Island in the Andaman Islands of India. One of the Andamanese
peoples, they resist contact with the outside world, and are among the last people to
remain virtually untouched and uncontacted by modern civilization.
Source: http://www.thehindu.com/news/cities/kolkata/shompens-men-outnumber-
women/article7827582.ece
IASbabas Prelims 60 Day Plan Day 23 2017
Q.1) Which of the following statements are correct comparisons between Geostationary
and Geosynchronous satellite?
a) 1 only
b) 2 and 3
c) 1 and 3
d) All of the above
Geosynchronous Orbit
There is a difference between the geostationary and geosynchronous orbits. We should
note that while other orbits may be many, there is ONLY ONE Equatorial orbit, i.e. the orbit
which is directly above the earths equator. Sometimes we send a satellite in the space
which though has a period of revolution is equal to period of rotation of earth, but its orbit
is neither equatorial nor Circular. So, this satellite will finish one revolution around the earth
in exactly one day i.e. 23 hours, 56 Minutes and 4.1 seconds, yet it does NOT appear
stationary from the earth. It looks oscillating but NOT stationary and that is why it is called
Geosynchronous.
IASbabas Prelims 60 Day Plan Day 23 2017
Q.2) Which of the following statements are correct about GRAPES 3 experiment located
in Ooty?
The GRAPES-3 experiment is located at Ooty in India. It was started as a collaboration of the
TIFR and the Japanese Osaka City University, and now also includes the Japanese Nagoya
Womens University. It is specially designed to study cosmic rays with an array of air shower
detectors and a large area muon detector. It aims to probe acceleration of cosmic rays in
the four astrophysical settings.
Q.3) Copernicus observation program is the worlds largest single earth observation
programme. Which of the following statements are correct about this project?
a) 1 and 2
b) 2 and 3
c) 1 and 3
IASbabas Prelims 60 Day Plan Day 23 2017
d) All of the above
a) Ammonium Nitrate
b) Polybutadiene Acrylonitrile
c) Kerosene
d) Liquid Hydrogen
Indian Space Research Organisation (ISRO) has successfully ground tested Indias largest
indigenously developed Cryogenic Upper Stage engine for GSLV Mark III. It was tested for
full 10 minutes at ISROs Liquid Propulsion Complex (ILPC) at Mahendragiri in Tirunelveli
district of Tamil Nadu.
The C25 stage is the most powerful upper stage so far developed by ISRO.
It uses Liquid Oxygen and Liquid Hydrogen propellant combination stored at minus
253 degrees centigrade.
The development of C25 cryogenic stage will provide ISRO capability to launch 4 ton
class satellites in Geosynchronous Transfer Orbit (GTO), an altitude where satellites
revolve in sync with Earths rotation.
So far, the cryogenic engine consisting very complex technology has been developed
only by Russia, US, France, China, Japan and India.
Q.5) ISRO made history by launching worlds largest number satellites from one launch
IASbabas Prelims 60 Day Plan Day 23 2017
vehicle. It launched 104 satellites in one go. Which of the following vehicles were used by
ISRO to launch these satellites?
a) PSLV C37
b) GSLV Mark III
c) PSLV C21
d) Ariane 5
Indian Space Research Organisation (ISRO) created history by successfully launching a record
104 satellites in single mission. These satellites were launched on board of Polar Satellite
Launch Vehicle PSLV-C37, on its 39th mission from Satish Dhawan Space Centre, Sriharikota,
Andhra Pradesh.
Of the total 104 satellites, three were Indian and remaining 101 belonged to
international customers.
Indias three satellites included earth-mapping Cartosat-2 satellite (main payload)
and nanosatellites INS-1A and INS-1B.
Foreign Satellites: Of the 101 co-passenger satellites, 96 belong to US and remaining
5 from Israel, Kazakhstan, Netherlands, Switzerland, United Arab Emirates,
respectively. Around 90 small satellites belonged to US-based company Planet Inc.
They are named Doves and their constellation will be used to image the earth at
low cost.
Q.6) ISRO has recently commissioned Hypersonic Wind Tunnel at Vikrams Sarabhai
research center. Which of the following statements are not correct regarding this?
1. Hypersonic Wind Tunnel is used to create zero gravity environment for the
astronauts to train.
2. This is the worlds largest hypersonic wind tunnel ever commissioned.
3. These wind tunnels will be used to study the effects of air flowing past a solid object.
a) 1 and 2
b) 2 only
c) 1 and 3
d) All of the above
IASbabas Prelims 60 Day Plan Day 23 2017
Q.6) Solution (a)
The Indian Space Research Organisation (ISRO) created history by commissioning the
worlds third-largest hypersonic wind tunnel at Vikram Sarabhai Space Centre (VSSC) in
Thiruvananthapuram, Kerala. In this regard, ISRO chairman A S Kiran Kumar commissioned
two facilities a 1-m Hypersonic Wind Tunnel and a 1-m Shock Tunnel.
These facilities are the third largest in terms of size and simulation capability in the world
next only to the ones in the United States (US) and Russia.
These wind tunnels will be used to study the effects of air flowing past a solid object and in
ISROs case, space vehicles. These new facilities will help aerodynamic characterisation of
advanced space transportation systems in a hypersonic environment.
Q.7) ASTROSAT, Indias space Observatory have captured a rare phenomenon of Vampire
Star. Which of the following statements correctly explains the phenomenon of Vampire
Star?
a) The vampire star phenomenon is observed when smaller star sucks material (mass
and energy) out of the bigger companion star, causing its eventual death.
b) The Vampire Star phenomenon is observed when a large Star feasts on a smaller Star
causing its eventual death.
c) The phenomenon is observed when two similar binary stars fuse with each other and
form a larger star.
d) The Vampire Star phenomenon is observed when a star enlarges to engulf its
planets.
The vampire star phenomenon is observed when smaller star sucks material (mass and
energy) out of the bigger companion star, causing its eventual death.
It is also called a blue straggler as small star becomes bigger, hotter and bluer, giving it the
appearance of being young, while the ageing companion burns out and collapses to a stellar
remnant.
a) 1 only
IASbabas Prelims 60 Day Plan Day 23 2017
b) 2 only
c) Both 1 and 2
d) Neither 1 nor 2
Sunspots are temporary phenomena on the photosphere of the Sun that appear as
dark spots compared with surrounding regions. They are areas of reduced surface
temperature caused by concentrations of magnetic field flux that inhibit
convection. Sunspots usually appear in pairs of opposite magnetic polarity.
Sunspots are darker, cooler areas on the surface of the sun in a region called the
photosphere. The photosphere has a temperature of 5,800 degrees Kelvin. Sunspots have
temperatures of about 3,800 degrees K. They look dark only in comparison with the brighter
and hotter regions of the photosphere around them.
Q.9) Which of the following countries have launched Gaofen project, a network of
observation satellites orbiting Earth to provide global surveillance capabilities which is
due to be completed in 2020?
a) USA
b) Russia
c) Israel
d) China
Gaofen (GF, High Resolution) is a series of civilian Earth observation satellites developed
and launched for the China High-definition Earth Observation System (CHEOS), a state-
sponsored programme aimed to develop a near-real time, all-weather, global surveillance
network consisting of satellite, near-space (stratosphere) airships, and aerial observation
platforms. Originally proposed in 2006, the CHEOS programme was officially approved by
the Chinese government in May 2010. As many as 14 satellites equipped with CCD camera,
multi-spectrum imagers and SAR radar are set for launch between 2013 to 2020.
Q.10) China has successfully launched experimental satellite XPNAV 1. Which of the
following statements are correct regarding this satellite?
a) 1 only
b) 2 only
IASbabas Prelims 60 Day Plan Day 23 2017
c) Both 1 and 2
d) Neither 1 nor 2
China has successfully launched experimental X-ray pulsar navigation satellite XPNAV-1. It is
the worlds first pulsar navigation satellite or spacecraft.
The satellite weighs more than 200 kilogrammes and carries two detectors. It will operate in
a Sun-synchronous orbit (500 kilometers) inclined 97 degrees. It will conduct in-orbit
experiments of autonomous spacecraft navigation using pulsar detectors to demonstrate
new technologies. It will also test its detectors functions in responding to the background
noise of the universe, outline pulsar contours, and create a database for pulsar navigation.
Q.11) The Magnetospheric Multistage Mission (MMS) launched by NASA to map magnetic
reconnection'. Which of the following statements correctly explains the process of
magnetic reconnection?
a) A process that occurs as the sun and Earths magnetic fields interact.
b) A process that occurs when the magnetic fields of two or more communication
satellites interact and causes disturbance in transmission.
c) The process of change of earths magnetic field.
d) None of the above.
NASAs MMS mission is an unmanned space mission to study the Earths magnetosphere,
using four identical satellites flying in a tetrahedral or pyramid formation.
The mission launched in March 2015 aims to map magnetic reconnection, a process that
occurs as the sun and Earths magnetic fields interact.
Q.12) With reference to Astrosat, the astronomical observatory launched by India, which
of the following statements is/are correct?
IASbabas Prelims 60 Day Plan Day 23 2017
1. Other than USA and Russia, India is the only country to have launched a similar
observatory into space.
2. Astrosat mission is capable of performing observations in Ultraviolet (UV), optical,
low and high energy X-ray wavebands at the same time.
a) 1 only
b) 2 only
c) Both 1 and 2
d) Neither 1 nor 2
USA, Russia, Japan and Europe have such space observatories. Hence, statement (1) is
wrong.
About ASTROSAT:
It is placed at low earth equatorial orbit at altitude of 650 km. It has ability to observe
celestial bodies like cosmic X-Ray sources and distant stars in different wavelengths
simultaneously. It can observe the universe through ultraviolet, optical, low and high energy
X-ray components of the electromagnetic spectrum. It has mission life of 5 years.
Its successful launch made India member of select elite group of nations comprising US,
Japan, Russia and Europe having its own space observatory.
These all are also functions of repeater to receive, amplify and retransmit the signal.
a) 1 only
b) 2 and 3 only
c) 1 and 3 only
d) 1, 2 and 3
India had created global history by becoming the first Asian nation to reach the Mars orbit in
a space mission. The success is sweeter because this had been done in its maiden attempt.
No other country that has attempted a mission to Mars has succeeded in reaching the
planet on debut.
Marking India's first venture into the interplanetary space, MOM will explore and observe
Mars surface features, morphology, mineralogy and the Martian atmosphere. Further, a
specific search for methane in the Martian atmosphere will provide information about the
possibility or the past existence of life on the planet.
1. Both electromagnetic and gravitational waves travel with the speed of light.
2. Both electromagnetic and gravitational waves can create ripples in space-time.
a) 1 only
b) 2 only
c) Both 1 and
d) Neither 1 nor 2
Gravitational waves are ripples in the curvature of space-time which propagate as waves,
travelling outward from the source. Predicted in 1916 by Albert Einstein on the basis of his
theory of general relativity. Gravitational waves transport energy as gravitational radiation,
a form of radiant energy similar to electromagnetic radiation. The speed of gravitational
waves in the general theory of relativity is equal to the speed of light in vacuum, c.
Electromagnetic waves are synchronized oscillations of electric and magnetic fields that
propagate at the speed of light through a vacuum. Visible light is one type of
electromagnetic radiation; other familiar forms are invisible electromagnetic radiations,
such as radio waves, infrared light and X rays.
Only gravitational waves can create ripples in space-time, electromagnetic waves create
vibrations of electric and magnetic field in space only. Thus, statement 1 is correct and
statement 2 is wrong. Hence, (a) is the correct answer.
IASbabas Prelims 60 Day Plan Day 23 2017
Link:http://www.thehindu.com/sci-tech/science/what-are-gravitational
waves/article5800227.ece
a) Only 1
b) Only 2
c) Both 1 and 2
d) Neither 1 nor 2
Most ODA comes from the 28 members of the DAC, or about $135 billion in 2013. A further
$15.9 billion came from the European Commission and non-DAC countries gave an
additional $9.4 billion. Although development aid rose in 2013 to the highest level ever
recorded, a trend of a falling share of aid going to the neediest sub-Saharan African
countries continued.
Source: http://indianexpress.com/article/india/india-news-india/india-agrees-to-link-
nuclear-testing-to-termination-of-deal-with-japan-4372448/
Q.17) Which of the following statements is correct about Saur Sujala Yojana?
PM launched the Saur Sujala Yojana in Chhattisgarh that would provide solar powered
irrigation pumps to farmers at a subsidized price.
Under the scheme, solar powered irrigation pumps of 3HP and 5HP capacity worth Rs 3.5
lakh and Rs 4.5 lakh respectively would be distributed to the farmers by March 31, 2019.
The beneficiaries would get the pumps at the subsidised price.
Source: http://www.business-standard.com/article/current-affairs/pm-narendra-modi-
launches-saur-sujala-yojana-in-chhattisgarh-116110100699_1.html
a) West Bengal
b) Odisha
c) Both (a) and (b)
d) Arunachal Pradesh
Q.19) Consider the following statements about Pradhan Mantri YUVA Yojana
a) Only 3
b) 1 and 3
c) 2 and 3
d) 1 and 2
Pradhan Mantri YUVA Yojana (Yuva Udyamita Vikas Abhiyan) is a is a centrally sponsored
Scheme on entrepreneurship education and training being implemented by the Ministry of
Skill Development and Entrepreneurship, Government of India.
The scheme spans over five years (2016-17 to 2020-21) with a project cost of Rs. 499.94
crore, and will provide entrepreneurship education and training to over 7 lakh students in 5
years through 3050 Institutes. It will also include easy access to information and mentor
network, credit, incubator and accelerator and advocacy to create a pathway for the youth.
The institutes under the PMYY include 2,200 institutes of higher learning (colleges,
universities, and premier institutes), 500 ITIs, 300 schools and 50 entrepreneurship
development centres through Massive Open Online Courses (MOOCs).
Source: http://www.pradhanmantriyojana.co.in/yuva-yojna-pmmy/
National Aeronautics and Space Administration (Nasa) scientists have developed a new
instrument that may sniff for signs of life on Mars, inspired by a sensing technique used to
monitor the air for life-threatening chemicals and bio-hazards.
Source: http://www.livemint.com/Science/0z6g5WZXsxq923dI8z8ivK/Nasa-develops-
new-instrument-that-could-sniff-out-life-on.html
IASbabas Prelims 60 Day Plan- Day 24 2017
Q.1) Consider the following statements regarding Living Planet Index Report
a) 1 Only
b) 2 Only
c) Both
d) None
The Living Planet Report is published every two years by the World Wide Fund for
Nature since 1998. It is based on the Living Planet Index and ecological
footprint calculations.
The Living Planet Report is the world's leading, science-based analysis on the health of our
only planet and the impact of human activity. Humanity's demands exceed the Earth's
capacity to sustain us. The latest edition of the Living Planet Report was released in October
2016
The Living Planet Database (LPD) currently holds time-series data for over 18,000
populations of more than 3,600 mammal, bird, fish, reptile and amphibian species from
around the world, which are gathered from a variety of sources such as journals, online
databases and government reports.
Using a method developed by Zoological Society of London and WWF, these species
population trends are aggregated to produce indices of the state of biodiversity for
communication and informing policy.
Q.2) Consider the following statements about Rio Declaration on Environment and
Development
www.iasbaba.com Page 1
IASbabas Prelims 60 Day Plan- Day 24 2017
a) 1 and 2
b) 2 and 3
c) 1 and 3
d) 1, 2 and 3
www.iasbaba.com Page 2
IASbabas Prelims 60 Day Plan- Day 24 2017
Some Scholars have regarded the Rio Declaration as Third Generation Human Rights.
http://www.un.org/documents/ga/conf151/aconf15126-1annex1.htm
1. Marine Park
2. National Park
3. Coral reefs
4. Mangroves
5. Sand Dunes
Among the following, entities that fall under Coastal Regulation Zone-I (CRZ)
a) 1,2 and 4
b) 2, 3 and 4
c) 1, 3 and 5
d) 1,2,3, 4 and 5
The CRZ Notification, 2011 clearly lists out the areas that fall within the category
of CRZ-I. It includes:-
(i)Ecologically sensitive areas and the geomorphologic features that play a Primary role in
maintaining the integrity of the coast.
Mangroves, in case mangrove area is more than 1000 square metres, a buffer area of
50 metres shall be provided;
Corals and coral reefs and associated biodiversity;
Sand Dunes;
Mudflats which are biologically active;
National parks, marine parks, sanctuaries, reserve forests, wildlife habitats and other
protected areas under the provisions of Wild Life (Protection) Act, 1972 (53 of 1972),
www.iasbaba.com Page 3
IASbabas Prelims 60 Day Plan- Day 24 2017
the Forest (Conservation) Act, 1980 (69 of 1980)or Environment (Protection) Act,
1986 (29 of 1986); including Biosphere Reserves encompassing;
o Salt Marshes;
o Turtle nesting grounds;
o Horse shoe crabs habitats;
o Sea grass beds;
o Nesting grounds of birds;
o Areas or structures of archaeological importance and heritage sites;
(ii) The area between Low Tide Line and High Tide Line.
Q.4) India is party to the following international agreements which have bearing on
biodiversity?
1. FAO International Treaty on Plant Genetic Resources for food and agriculture
2. World Heritage Convention
3. World Trade Organization
4. UN Convention to Combating Desertification
5. UN Law of the Seas
a) 2 and 4
b) 1, 2 and 4
c) 2, 3, 4 and 5
d) 1, 2, 3, 4 and 5
Article 6 of the treaty on Sustainable Use of Plant Genetic Resources has this clause
pursuing fair agricultural policies that promote use of agricultural biological diversity
UNCCD collaborates closely with Convention on Biological Diversity (CBD). So, UNCCD has a
bearing on Biodiversity
www.iasbaba.com Page 4
IASbabas Prelims 60 Day Plan- Day 24 2017
(2004), the Ramsar Convention on Wetlands (1971), the World Heritage Convention (1972)
and the International Plant Protection Convention (1952).
The Convention on Biological Diversity complements UNCLOS in explicitly providing for the
conservation of biological diversity, the sustainable use of its components and the fair and
equitable sharing of benefits arising out of the use of genetic resources.
1. Food Processing
2. Printing Press
3. Pulp and Paper
4. Sugar
5. Healthcare Establishments
a) 1, 2 and 4
b) 2, 3, 4 and 5
c) 3, 4 and 5
d) 1, 2, 3, 4 and 5
The Ministry of Environment, Forest and Climate Change (MoEFCC) has developed the
criteria of categorization of industrial sectors based on the Pollution Index which is a
function of the emissions (air pollutants), effluents (water pollutants), hazardous wastes
generated and consumption of resources.
Industrial Sectors having Pollution Index score of 60 and above - Red category
Industrial Sectors having Pollution Index score of 41 to 59 Orange category
Industrial Sectors having Pollution Index score of 21 to 40 Green category
Industrial Sectors having Pollution Index score incl.&upto 20 - White category
www.iasbaba.com Page 5
IASbabas Prelims 60 Day Plan- Day 24 2017
Q.7) Scientists have discovered two new species Sholicola and Montecincla in the
Western Ghats. They are-
http://www.thehindu.com/sci-tech/science/Music-to-the-ears-New-species-of-songbird-
found/article17093802.ece
Q.8) Which of the following protected areas are formed as per the guidelines of The
Wildlife (Protection) Act, 1972?
1. National Park
2. Biosphere Reserve
3. Wildlife Sanctuary
4. Conservation Reserves
5. Community Reserves
a) 1, 2, 3 and 4
www.iasbaba.com Page 6
IASbabas Prelims 60 Day Plan- Day 24 2017
b) 1, 3, 4 and 5
c) 1, 2, 3 and 5
d) All
Biosphere Reserves are areas of terrestrial and coastal ecosystems which are internationally
recognized within the framework of the Man and the Biosphere (MAB) programme of the
UNESCO and are not formed according to the guidelines of the Wildlife (protection) Act,
1972 and may have one more national parks or wildlife sanctuaries in it. Under the MAB
programme there is a World Network of Biosphere Reserves (WNBR) and within this
network, exchange of information, experience and personnel is allowed.
1. Green bond is a debt instrument issued by an entity for raising funds from investors
2. The funds generated are used to fund both green and non-green projects
3. A green bond is a tax-exempt bond issued by federally qualified organizations for the
development of brownfield sites.
a) 1 and 3 only
b) 2 and 3 only
c) All the above
d) None of the above
Fund generated from bonds are used only to fund green projects.
A green bond is a tax-exempt bond issued by federally qualified organizations or by
municipalities for the development of brownfield sites. Brownfield sites are areas of land
that are underutilized, have abandoned buildings or are underdeveloped, often containing
low levels of industrial pollution. Green bonds are short for qualified green building and
sustainable design project bonds.
Green bonds are created to encourage sustainability and the development of brownfield
sites. More specifically, green bonds finance projects aimed at energy efficiency, pollution
prevention, sustainable agriculture, fishery and forestry, the protection of aquatic and
www.iasbaba.com Page 7
IASbabas Prelims 60 Day Plan- Day 24 2017
The tax-exempt status makes purchasing a green bond a more attractive investment
compared to a comparable taxable bond, providing a monetary incentive to tackle
prominent social issues such as climate change and a movement to renewable sources of
energy.
Q.10) Which of the following is/are the man-made Ramsar sites of India?
1. Ropar wetland
2. Harike reservoir
3. Bhoj Wetland
4. Ashtamudi
a) 2 only
b) 2 and 3
c) 1, 2 and 3
d) All of the above
The list of Ramsar sites (related to wetland) in India comprises Indian wetlands deemed to
be of "international importance" under the Ramsar Convention. For a full list of all Ramsar
sites worldwide, see the Ramsar list of wetlands of international importance.
According to WWF-India, wetlands are one of the most threatened of all ecosystems in
India. Loss of vegetation, salinization, excessive inundation, water pollution, invasive
species, excessive development and road building, have all damaged the countrys wetlands.
Ashtamudi is a natural backwater in Kollam district in Kerala. Rest are manmade reservoirs.
The Bhoj Wetland consists of two lakes located in the city of Bhopal, the capital of the
central Indian state of Madhya Pradesh. The two lakes are the Bhojtal and the Lower Lake,
which lie to the west of the city center. It is a manmade reservoir. A total of more than
20,000 birds are observed annually. Bhoj Wetland was recognized as a wetland of
international importance under the Ramsar Convention of 1971 in 2002.
Ropar wetland- A human-made wetland of lake and river formed by the 1952 construction
of a barrage for diversion of water from the Sutlej River for drinking and irrigation supplies.
www.iasbaba.com Page 8
IASbabas Prelims 60 Day Plan- Day 24 2017
The site is an important breeding place for the nationally protected Smooth Indian Otter,
Hog Deer, Sambar, and several reptiles, and the endangered Indian Pangolin (Manis
crassicaudata) is thought to be present.
Harike is one of the largest man-made wetlands of northern India which shares its area with
the Tarntaran, Ferozpur and Kapurthala districts of Punjab. It came into existence in 1952
after the construction of barrage near the confluence of rivers Sutlej and Beas. Harike is a
significant abode for the birds migrating from across the international frontiers. The wetland
area is spread over about 41 km2 and supports more than 400 avian species. In addition to
haven for birds, Harike also harbours endangered aquatic mammalian as well as reptilian
fauna like Indus river dolphin, smooth-coated otter and seven species of rare freshwater
turtles. An area of about 86 km2 has been notified as wildlife sanctuary. Considered a
wetland of international importance especially as waterfowl refuge, this site was accorded
the wetland status in 1990 by the Ramsar Convention.
a) 1 and 2
b) 2 and 3
c) 1 and 3
d) 1, 2 and 3
The National Biodiversity Authority (NBA) was established in 2003 to implement Indias
Biological Diversity Act (2002). The NBA is a Statutory, Autonomous Body and it performs
facilitative, regulatory and advisory function for the Government of India on issues of
conservation, sustainable use of biological resources and fair and equitable sharing of
benefits arising out of the use of biological resources.
To regulate access to biological resources of the country to conserve and sustainable use of
biological diversity.
www.iasbaba.com Page 9
IASbabas Prelims 60 Day Plan- Day 24 2017
To secure sharing of benefits with the local people as conservers of biological resources,
holders of knowledge and information relating to the use of biological resources.
Conservation and development of area of importance from the view point of biological
diversity by declaring them as biological diversity heritage sites.
a) 1, 2 and 3
b) 2, 3 and 4
c) 2 and 3
d) All
Durban Declaration and Programme of Action- The DDPA reasserts the principles of
equality and non-discrimination as core human rights, thus transforming victims of
discrimination into rights-holders and States into duty bearers.
Bali Action Plan- The Bali Action Plan did not introduce binding commitments to reduce
greenhouse gas emissions but included the request for developed countries to contribute to
the mitigation of global warming in the context of sustainable development. In addition, the
Bali Action Plan envisaged enhanced actions on adaptation, technology development and on
the provision financial resources, as well as measures against deforestation.
Cancum Agreement
The agreements, reached on December 11 in Cancun, Mexico, at the 2010 United Nations
Climate Change Conference represent key steps forward in capturing plans to reduce
www.iasbaba.com Page 10
IASbabas Prelims 60 Day Plan- Day 24 2017
greenhouse gas emissions and to help developing nations protect themselves from climate
impacts and build their own sustainable futures.
The Copenhagen Agreement is a document that delegates at the 15th session of the
Conference of Parties (COP 15) to the United Nations Framework Convention on Climate
Change.
The Accord
The contiguous zone extends up to 24 nautical miles from the shore. In a contiguous zone
coastal nations can exercise their rights in four areas: customs, taxation, immigration and
pollution.
www.iasbaba.com Page 11
IASbabas Prelims 60 Day Plan- Day 24 2017
In Exclusive Economic Zone (EEZ), which extends up to 200 nautical mile, the coastal nations
have exclusive right to exploit the resources. International vessels are free to cross these
waters without any restriction.
Q.14) TRAFFIC, The Wildlife Trade Monitoring Network is a joint conservation programme
of
TRAFFIC, the wildlife trade monitoring network, is a joint program of WWF and IUCN the
International Union for Conservation of Nature. TRAFFIC works to ensure that trade in wild
plants and animals is not a threat to the conservation of nature. TRAFFIC has gained its
greatest reputation from supporting CITES, the Convention on International Trade in
Endangered Species. Since TRAFFIC was created in 1976 it has helped with the evolution of
this international wildlife trade treaty. TRAFFIC North America (NA) works through its offices
in the United States, Canada and Mexico, and with its programs in Central America and the
Caribbean.
TRAFFIC focuses on leveraging resources, expertise and awareness of the latest globally
urgent species trade issues such as tiger parts, elephant ivory and rhino horn. Large scale
commercial trade in commodities like timber and fisheries products are also addressed and
linked to work on developing rapid results and policy improvements.
1. The policies and objectives of Joint Forest Movement are detailed in the Indian
Forest Act, 1927
2. At least 50% members of the JFM executive committee/management committee
should be women.
3. The presence of at least 50% women members should be a prerequisite for holding
the general body meeting.
www.iasbaba.com Page 12
IASbabas Prelims 60 Day Plan- Day 24 2017
a) 1 and 2
b) Only 2
c) 2 and 3
d) All of the above
Joint Forest Management often abbreviated as JFM is the official and popular term
in India for partnerships in forest movement involving both the state forest departments
and local communities. The policies and objectives of Joint Forest Movement are detailed in
the Indian comprehensive National Forest Policy of 1988 and the Joint Forest Movement
Guidelines of 1990 of the Government of India
The JFM programme in the country was reviewed by Government of India from time to time
in consultation with State Governments, NGOs and other stakeholders in view of several
emerging issues. In order to further strengthen the programme, the State Governments may
take action on the following suggested lines.
i) At present, the JFM committees are being registered under different names in various
States as per the provisions contained in the resolutions. Except in a few States where the
committees are registered under the relevant acts in most of the states there is no legal
back up for these committees. It is therefore, necessary that all the State Governments
register the JFM or village committees under the Societies Registration Act, 1860 to provide
them with legal back up. This may be completed by 31st March, 2000. Completion of such
formation of existing JFM committees may please be reported to this Ministry.
ii) There are different nomenclatures for the JFM committees in different States. It would be
better if these committees are known uniformly as JFM committees (JFMC) in all the states.
Memorandum of Understanding, with clearly defined roles and responsibilities for different
work or areas should be separately assigned and signed between the State Governments
and the committees. All adults of the village should be eligible to become members of the
JFM Committees.
Considering the immense potential and genuine need for womens participation in JFM
programme, following guidelines are suggested for ensuring meaningful participation of
women in JFM.
www.iasbaba.com Page 13
IASbabas Prelims 60 Day Plan- Day 24 2017
i) At least 50% members of the JFM general body should be women. For the general body
meeting, the presence of at least 50% women members should be a prerequisite for holding
the general body meeting.
ii) At least 33% of the membership in the JFM Executive Committee/ Management
Committee should be filled from amongst the women members. The quorum for holding
meeting of such Executive/ Management Committee should be one-third of women
executive members or a minimum of one whichever is more. One of the posts of office
bearer i.e. President/ Vice-President/ Secretary should be filled by a women members of the
Committee.
a) Only 1
b) Only 2
c) Both 1 and 2
d) Neither 1 nor 2
The seas around India abound in vessels of various types ranging from small inland fishing
crafts to larger fishing vessels, coastal crafts carrying personnel and cargo and merchant
ships of all shapes and sizes that carry various types of cargo.
Being aware of the identity of these vessels is imperative to preventing any potential threat
from the sea from impinging on the coastal and offshore security of the country. The 26/11
Mumbai terrorist attack is a case in point. Indian Navy has thus been working towards
achieving complete Maritime Domain Awareness (MDA) along with all other concerned
agencies like the coast guard, customs, ports, fisheries, etc.
www.iasbaba.com Page 14
IASbabas Prelims 60 Day Plan- Day 24 2017
The seas of the entire world are inter-linked and vessels can sail routinely around the globe
in pursuit of cargo. Thus a vessel that is in Indian waters today maybe sailing towards a
distant destination and similarly several vessels set sail from ports around the globe could
be bound for India. Having advance information of the vessel, its destination and planned
itinerary, etc. is thus extremely helpful towards collating an effective MDA as it can then be
properly identified when detected. This information is likely to be available with the country
from whose port it sails. The information is equally relevant for the destination country and
those it passes enroute. Thus mutual exchange of such information, called white shipping
information, is extremely useful for all concerned.
India has signed white shipping agreements with several countries including United States
and Singapore and is seeking similar agreement with more countries as part of its ongoing
effort at developing an effective regional MDA.
Ships would be classified into white (commercial ships), grey (military vessels), and black
(illegal vessels).
Source: http://www.thehindu.com/news/national/Parrikar-in-Dhaka-to-boost-defence-
ties/article16720185.ece
Q.17) Consider the following statements about Market Stabilisation Scheme (MSS)
1. It is a tool used by the Reserve Bank of India to suck out excess liquidity from the
market through issue of securities
2. These securities are not issued to meet governments expenditure
3. MSS bonds have a fixed tenure and earn returns
a) 1 and 2
b) 2 and 3
c) 1 and 3
d) All of the above
Market Stabilisation Scheme or MSS is a tool used by the Reserve Bank of India to suck out
excess liquidity from the market through issue of securities like Treasury Bills, Dated
Securities etc. on behalf of the government. The money raised under MSS is kept in a
separate account called MSS Account and not parked in the government account or utilised
to fund its expenditures.
www.iasbaba.com Page 15
IASbabas Prelims 60 Day Plan- Day 24 2017
The Reserve Bank under Governor YV Reddy initiated the MSS scheme in 2004. To control
the surge of US dollars in the Indian market, RBI started buying US dollars while pumping in
rupee. This eventually led to over-supply of the domestic currency raising inflationary
expectations. MSS was introduced to mop up this excess liquidity.
CRR is a percentage of total deposits the banks are required to set aside with the RBI. It is a
sort of contingency fund and does not earn any interest. An increase in CRR means the
funds available with banks for lending purposes will be that much lower, ultimately limiting
the possibility of a lending rate cut by banks. MSS bonds, on the other hand, have a fixed
tenure and earn returns.
In order to mop up extra liquidity from the system in view of demonetisation, government
and the Reserve Bank today sharply raised the Market Stabilisation Scheme (MSS) ceiling to
Rs. 6 lakh crore from Rs. 30,000 crore.
Source: http://www.thehindu.com/business/Economy/RBI-raises-Market-Stabilisation-
Scheme-limit-to-Rs.-6-lakh-crore-to-manage-liquidity/article16744379.ece
a) Only 1
b) Only 2
c) Both 1 and 2
d) Neither 1 nor 2
www.iasbaba.com Page 16
IASbabas Prelims 60 Day Plan- Day 24 2017
products and services emanating from the Indian Space Programme. In the year 2008, the
Company was awarded MINIRATNA status.
Source: http://www.thehindu.com/business/Industry/Antrix%E2%80%99s-major-
revenues-will-come-from-satcom-business/article16723417.ece
Q.19) Consider the following statements about Climate Vulnerable Forum (CVF)
a) 1 and 2
b) 2 and 3
c) 1 and 3
d) All of the above
The Climate Vulnerable Forum (CVF) is a global partnership of countries that are
disproportionately affected by the consequences of global warming. The forum addresses
the negative effects of global warming as a result of heightened socioeconomic and
environmental vulnerabilities. These countries actively seek a firm and urgent resolution to
the current intensification of climate change, domestically and internationally.
Maldives, 2009-2010
The Forum first met near Male, Maldives in November 2009 just ahead of the major
Copenhagen Summit UN climate change conference (COP15) , where 11 nations gathered
www.iasbaba.com Page 17
IASbabas Prelims 60 Day Plan- Day 24 2017
adopting a first Declaration that expressed alarm at the pace of change witnessed to the
Earth as a result of human-induced changes to the climate.
Kiribati, 2010-2011
The Forums 2nd Chair, Kiribati, hosted the Tarawa Climate Change Conference in November
2010 leading a number of CVF member countries to adopt the Ambo Declaration together
with other governments ahead of the UN Climate Conference at Cancn, Mexico (COP16) . It
was in conjunction with the Cancn conference that the Forum issued its first Climate
Vulnerability Monitor report, entitled, The State of the Climate Crisis, drawing attention to
the body of specialist evidence attesting to the dire socio-economic consequences of
contemporary climatic change.
Bangladesh, 2011-2013
The Forums 3rd Chair, Bangladesh, hosted a major gathering of the Forum at Dhaka,
Bangladesh in November 2011, adopting a second 14-point Declaration of the CVF in an
event inaugurated with the participation of UN Secretary-General, Ban Ki-moon. Another
major Forum event alongside the opening phases of the 67th UN General Assembly in New
York , US in September 2012 saw the launch of the Forums Trust Fund and the 2nd edition
of the Climate Vulnerability Monitor, entitled, A Guide to the Cold Calculus of a Hot
Planet.
Costa Rica assumed the Forums chair in mid-2013. The CVF Costa Rica Action Plan for 2013-
15 was released at the UN Climate Conference at Warsaw, Poland (COP19) in November
2013 detailing joint efforts of Forum members to advance international climate policy find
out more here. Costa Rica also hosted the first CVF regional activity in April 2014.
Philippines, 2015-2016
Philippines assumed the chair of the Forum in January 2015 following the tenure of Costa
Rica. The Philippines hosted the 2015 Forum Meeting at the UN Climate Conference at Paris,
France in November 2015, where the CVF Manila-Paris Declaration and the 2016-2018
Roadmap were adopted.
Ethiopia 2016 -
Ethiopia assumed the Presidency of the Forum in August 2016 following the tenure of
Philippines. Ethiopia was confirmed as incoming chair of the Forum in the Manila-Paris
Declaration of the CVF adopted in November 2015 at the High Level Climate Vulnerable
Forum Meeting at COP21.
www.iasbaba.com Page 18
IASbabas Prelims 60 Day Plan- Day 24 2017
The Climate Vulnerability Monitor (CVM) is an independent global assessment of the effect
of climate change on the worlds populations brought together by panels of key
international authorities. The Monitor was launched in December 2010 in London and
Cancun to coincide with the UN Cancun Summit on climate change (COP-16).
Developed by DARA and the Climate Vulnerable Forum, the report is meant to serve as a
new tool to assess global vulnerability to various effects of climate change within different
nations.
The report distills leading science and research for a clearer explanation of how and where
populations are being affected by climate change today (2010) and in the near future
(2030), while pointing to key actions that reduce these impacts.
DARA and the Climate Vulnerable Forum launched the 2nd edition of the Climate
Vulnerability Monitor on 26 September 2012 at the Asia Society, New York.
In 2015, the twenty member countries in a forum chaired by the Philippines launched the
official bloc of the forum, the 'V20' or 'Vulnerable Twenty', consisting of the top 20 nations
from all over the world that are most affected by the catastrophes rooted from climate
change. The members of the bloc are Afghanistan, Bangladesh, Barbados, Bhutan, Costa
Rica, Ethiopia, Ghana, Kenya, Kiribati, Madagascar, Maldives, Nepal, Philippines, Rwanda,
Saint Lucia, Tanzania, Timor-Leste, Tuvalu, Vanuatu and Vietnam.
Source: http://www.thehindu.com/opinion/op-ed/The-Marrakech-
mandate/article16695548.ece
www.iasbaba.com Page 19
IASbabas Prelims 60 Day Plan- Day 24 2017
On the basis of standardised criteria, the index evaluates and compares the climate
protection performance of 58 countries that together are responsible for about 90% of
global energy-related CO2 emissions.
India has been ranked 20th on Climate Change Performance Index (CCPI) 2017, which
underlined that countries like India are making "great efforts" in the fields of renewables
and energy efficiency.
With the historic Paris Agreement having recently entered into force, the latest CCPI
confirms a boost for renewable energy and positive developments in energy efficiency.
The publication was issued by Germanwatch and Climate Action Network Europe.
Source: http://www.business-standard.com/article/current-affairs/india-ranks-20th-on-
climate-change-performance-index-116111701669_1.html
1. Aedes aegypti
2. Aedes albopictus
3. Culex
a) Only 1
b) 1 and 3
c) 1 and 2
d) All of the above
The Culex species of mosquitoes does not appear to transmit the virus that spreads Zika.
How: The virus does not multiply and instead disappears in the species.
Why important to know: It enables people to target their control strategies so that they are
not wasting time and effort on a mosquito that is not transmitting Zika. Instead, more focus
should be on efforts to control mosquitoes like Aedes aegypti and Aedes albopictus that
infect.
In detail:
www.iasbaba.com Page 20
IASbabas Prelims 60 Day Plan- Day 24 2017
The Aedes aegypti, or yellow fever mosquito, and Aedes albopictus, or Asian tiger
mosquito, as two species that transmit the Zika virus.
The Culex mosquito has a brown body while the Aedes aegypti is black and Aedes
albopictus black and white.
Culex mosquitoes are known to transmit the West Nile virus and Japanese
encephalitis.
Aedes aegypti and Aedes albopictus can live in and around houses in plant trays,
spare containers or gutters.
It is important to know which mosquitoes to target and which mosquitoes not to
target because mosquitoes live in different environments. Some mosquitoes are
found outside and some are more in peoples homes. Hence, researchers need to
know this in order to target their efforts.
Zika virus
Zika virus is a mosquito-borne flavivirus that was first identified in Uganda in 1947 in
monkeys through a network that monitored yellow fever.
Flavivirus is a genus of viruses in the family Flaviviridae. This genus includes the West
Nile virus, dengue virus, tick-borne encephalitis virus, yellow fever virus, Zika virus.
Zika virus disease is caused by a virus transmitted primarily by Aedes mosquitoes,
mainly Aedes aegypti in tropical regions. This is the same mosquito that transmits
dengue, chikungunya and yellow fever.
Symptoms are mild fever, skin rash, conjunctivitis, muscle and joint pain, malaise or
headache.
There is scientific consensus that Zika virus is a cause of microcephaly and Guillain-
Barr syndrome.
The first large outbreak of disease caused by Zika infection was reported from the
Island of Yap (Federated States of Micronesia) in 2007.
There is currently no vaccine available.
Protection against mosquito bites is a key measure to prevent Zika virus infection.
Source: http://www.livemint.com/Science/aqsTtHIHqEmX98vYZdJHFL/Culex-mosquitoes-
not-Zika-virus-carriers-say-Kansas-Univers.html
www.iasbaba.com Page 21
IASbabas Prelims 60 Day Plan- Day 24 2017
a) Only 1
b) Only 2
c) Both 1 and 2
d) Neither 1 nor 2
The Mission Madhumeha has been launched on 28th October, 2016 on the occasion of first
National Ayurveda Day. A protocol on Prevention and Control of Diabetes through
Ayurveda was released for implementing the Mission Madhumeha. The Madhumeha
Assessment Tool (MAT) based on Ayurvedic philosophy has also been developed for the
self-assessment of the people with regards to possibilities of diabetes.
The Anti-diabetic Herbal Composition for Type-2 diabetes mellitus (NBRMAP-DB) trade
name BGR34 has been developed by CSIR, under the category of Ayurvedic Proprietary
Medicine as per Drugs and Cosmetics Act, 1940 (6th amendment) Rules and has been
tested for efficacy and safety in animal models.
Source: http://pib.nic.in/newsite/PrintRelease.aspx?relid=153864
Q.23) The United Nations Economic Commission for Europe (UNECE) Convention on Access
to Information, Public Participation in Decision-Making and Access to Justice in
Environmental Matters is also known as
a) Marpol Convention
b) Aarhus Convention
c) London Convention
d) OSPAR Convention
The most notable feature enshrined in EU legislations based upon this treaty is Water
Framework Directive.
www.iasbaba.com Page 22
IASbabas Prelims 60 Day Plan- Day 24 2017
Aarhus Convention grants the public rights regarding access to information, public
participation and access to justice, in governmental decisionmaking processes on matters
concerning the local, national and Transboundary environment.
www.iasbaba.com Page 23
IASbabas Prelims 60 Day Plan Day 25
Q.1) The President is elected not directly by the people but by members of electoral college
consisting of
a) 1 only
b) 3 only
c) 1, 2 and 3
d) 2 and 3
The President is elected not directly by the people but by members of electoral college
consisting of:
Thus, the nominated members of both of Houses of Parliament, the nominated members of the
state
legislative assemblies, the members (both elected and nominated) of the state legislative
councils (in
case of the bicameral legislature) and the nominated members of the Legislative Assemblies of
Delhi
and Puducherry do not participate in the election of the President. Where an assembly is
dissolved,
the members cease to be qualified to vote in presidential election, even if fresh elections to the
dissolved assembly are not held before the presidential election.
www.iasbaba.com Page 1
IASbabas Prelims 60 Day Plan Day 25
The Constitution provides that there shall be uniformity in the scale of representation of
different
states as well as parity between the states as a whole and the Union at the election of the
President.
Q.2) Consider the following, with reference to the Presidents veto power
1. Suspensive veto of the President can be overridden by re-passage of the bill by a simple
majority of the Parliament
2. The President has no veto power over a Constitutional amendment bill
a) 1 only
b) 2 only
c) Both 1 and 2
d) Neither 1 nor 2
A bill passed by the Parliament can become an act only if it receives the assent of the President.
When such a bill is presented to the President for his assent, he has three alternatives (under
Article
111 of the Constitution):
However, if the bill is passed again by the Parliament with or without amendments and again
presented to the President, the President must give his assent to the bill.
Thus, the President has the veto power over the bills passed by the Parliament10, that is, he
can
www.iasbaba.com Page 2
IASbabas Prelims 60 Day Plan Day 25
withhold his assent to the bills. The object of conferring this power on the President is two-
fold(a)
to prevent hasty and ill-considered legislation by the Parliament; and (b) to prevent a legislation
which may be unconstitutional.
The veto power enjoyed by the executive in modern states can be classified into the following
four
types:
1. Absolute veto, that is, withholding of assent to the bill passed by the legislature.
2. Qualified veto, which can be overridden by the legislature with a higher majority.
3. Suspensive veto, which can be over ridden by the legislature with an ordinary majority.
4. Pocket veto, that is, taking no action on the bill passed by the legislature.
Of the above four, the President of India is vested with threeabsolute veto, suspensive veto
and
pocket veto. There is no qualified veto in the case of Indian President; it is possessed by the
American President.
Suspensive Veto :
The President exercises this veto when he returns a bill for reconsideration of the
Parliament.
However, if the bill is passed again by the Parliament with or without amendments and
again presented to the President, it is obligatory for the President to give his assent to
the bill. This means that the presidential veto is overridden by a re-passage of the bill by
the same ordinary majority (and not a higher majority as required in USA.
As mentioned earlier, the President does not possess this veto in the case of money
bills. The President can either give his assent to a money bill or withhold his assent to a
money bill but cannot return it for the reconsideration of the Parliament. Normally, the
President gives his assent to money bill as it is introduced in the Parliament with his
previous permission.
President has no veto power in respect of a constitutional amendment bill. The 24th
Constitutional Amendment Act of 1971 made it obligatory for the President to give his
assent to a constitutional amendment bill.
www.iasbaba.com Page 3
IASbabas Prelims 60 Day Plan Day 25
1. The first session after each general election and the first session of every fiscal year is
addressed by the President
2. It is discussed in Lok Sabha only
3. A failure to get motion of thanks passed amounts to defeat of government and leads to
collapse of government
a) 1 only
b) 1 and 3 only
c) 1 and 2 only
d) 1, 2 and 3
Motion of Thanks - Presidents address is the speech delivered by the President of India
to both Houses of Parliament assembled together at the commencement of the first
session after each general election and the first session of every fiscal year is addressed
by the president. In this address, the president outlines the policies and programmes of
the government in the preceding year and ensuing year.
This inaugural speech of the president is an occasion available to the members of
Parliament to raise discussions and debates to examine and criticise the government
and administration for its lapses and failures.
The notice of such a motion is received through the Ministry of Parliamentary Affairs
This address of the president, which corresponds to the speech from the Throne in
Britain, is discussed in both the Houses of Parliament on a motion called the Motion of
Thanks.
This motion must be passed in both of the houses.
At the end of the discussion, the motion is put to vote. This motion must be passed in
the House. Otherwise, it amounts to the defeat of the government. This is why, the
Motion of Thanks is deemed to be a no-confidence motion.
www.iasbaba.com Page 4
IASbabas Prelims 60 Day Plan Day 25
This address by the President highlights the legislative and policy activities and
achievements of the government in the previous year. In addition, it gives a broad
indication of the governments agenda for the year ahead. The address is followed by a
motion of thanks that is moved in each House by ruling party MPs. This is followed by a
discussion on the address and concludes with the Prime Minister replying to the points
raised during the discussion.
In the lower house, the motion of thanks has begun today. It began in the upper house
on February 2, 2017. Lok Sabha and Rajya Sabha have allocated two and three days for
the discussion, respectively.
1. A minister who is a member of one House of Parliament does not have the right to
speak and to take part in the proceedings of the other House
2. A minister can vote only in the House of which he/she is a member
a) 1 only
b) 2 only
c) Both 1 and 2
d) Neither 1 nor 2
Usually, the members of Parliament, either Lok Sabha or Rajya Sabha, are appointed as
ministers. A person who is not a member of either House of Parliament can also be
appointed as a minister. But, within six months, he must become a member (either by
election or by nomination) of either House of Parliament, otherwise, he ceases to be a
minister.
www.iasbaba.com Page 5
IASbabas Prelims 60 Day Plan Day 25
A minister who is a member of one House of Parliament has the right to speak and to
take part in the proceedings of the other House also, but he can vote only in the House
of which he is a member. Hence statement 1 is incorrect.
Q.5) Consider the following statements with regard to powers of Prime Minister (PM)
1. The PM can ask a minister to resign or advise the President to dismiss him in case of
difference of opinion.
2. The PM advises the President with regard to summoning and proroguing of the sessions
of the Parliament.
3. The PM can recommend dissolution of the Lok Sabha to President at any time.
a) 1 only
b) 2 and 3 only
c) 1 and 2 only
d) 1, 2 and 3
The Prime Minister enjoys the following powers as head of the Union council of ministers:
1. He recommends persons who can be appointed as ministers by the president. The President
can appoint only those persons as ministers who are recommended by the Prime Minister.
2. He allocates and reshuffles various portfolios among the ministers.
3. He can ask a minister to resign or advise the President to dismiss him in case of difference
of
opinion.
4. He presides over the meeting of council of ministers and influences its decisions.
5. He guides, directs, controls, and coordinates the activities of all the ministers.
www.iasbaba.com Page 6
IASbabas Prelims 60 Day Plan Day 25
6. He can bring about the collapse of the council of ministers by resigning from office.
Since the Prime Minister stands at the head of the council of ministers, the other ministers
cannot
function when the Prime Minister resigns or dies. In other words, the resignation or death of
an
incumbent Prime Minister automatically dissolves the council of ministers and thereby
generates a
vacuum. The resignation or death of any other minister, on the other hand, merely creates a
vacancy
which the Prime Minister may or may not like to fill.
The Prime Minister enjoys the following powers in relation to the President:
1. He is the principal channel of communication between the President and the council of
ministers. It is the duty of the prime minister :
(a) to communicate to the President all decisions of the council of ministers relating to the
administration of the affairs of the Union and proposals for legislation;
(b) to furnish such information relating to the administration of the affairs of the Union and
proposals for legislation as the President may call for; and
(c) if the President so requires, to submit for the consideration of the council of ministers
any matter on which a decision has been taken by a minister but which has not been
considered by the council.
2. He advises the president with regard to the appointment of important officials like attorney
general of India, Comptroller and Auditor General of India, chairman and members of the
UPSC, election commissioners, chairman and members of the finance commission and so on.
In Relation to Parliament :
The Prime Minister is the leader of the Lower House. In this capacity, he enjoys the following
powers:
1. He advises the President with regard to summoning and proroguing of the sessions of the
Parliament.
2. He can recommend dissolution of the Lok Sabha to President at any time.
www.iasbaba.com Page 7
IASbabas Prelims 60 Day Plan Day 25
The right to move a motion for an adjournment of the business of the House is subject to the
following restrictions:
1. It should raise a matter which is definite, factual, urgent and of public importance
3. It should be restricted to a specific matter of recent occurrence and should not be framed in
general terms
5. It should not revive discussion on a matter that has been discussed in the same session;
6. It should not deal with any matter that is under adjudication by court
7. It should not raise any question that can be raised on a distinct motion.
a) 1 and 3 only
b) 1 and 2 only
c) 2 only
d) None of the above
In Cooper case11, (1970), the Supreme Court held that the Presidents satisfaction can
be questioned in a court on the ground of malafide. This means that the decision of the
President to issue an ordinance can be questioned in a court on the ground that the
President has prorogued one House or both Houses of Parliament deliberately with a
view to promulgate an ordinance on a controversial subject, so as to bypass the
parliamentary decision and thereby circumventing the authority of the Parliament. The
38th Constitutional Amendment Act of 1975 made the Presidents satisfaction final and
conclusive and beyond judicial review. But, this provision was deleted by the 44th
Constitutional Amendment Act of 1978. Thus, the Presidents satisfaction is justiciable
on the ground of malafide. Hence statement 2 is incorrect.
The ordinance-making power is the most important legislative power of the President.
It has been vested in him to deal with unforeseen or urgent matters. Hence statement 1
is incorrect.
The President can also withdraw an ordinance at any time. However, his power of
ordinance-making is not a discretionary power, and he can promulgate or withdraw an
ordinance only on the advice of the council of ministers headed by the prime minister.
Hence statement 3 is incorrect.
www.iasbaba.com Page 9
IASbabas Prelims 60 Day Plan Day 25
1. The recruitment and service conditions of the Secretariat are regulated by the presiding
officer of the House
2. Secretary-general of the Secretariat is appointed by the President
a) 1 Only
b) 2 Only
c) Both 1 and 2
d) Neither 1 nor 2
Secretariat of Parliament -Each House of Parliament has separate secretarial staff of its own,
though there can be some posts common to both the Houses. Their recruitment and service
conditions are regulated by Parliament. The secretariat of each House is headed by a secretary-
general. He is a permanent officer and is appointed by the presiding officer of the House.
Q.9) With regard to Attorney General for India, the highest law officer in the country, which
of the following is incorrect?
www.iasbaba.com Page 10
IASbabas Prelims 60 Day Plan Day 25
The Constitution (Article 76) has provided for the office of the Attorney General for India. He
is the highest law officer in the country.
The Attorney General (AG) is appointed by the president. He must be a person who is
qualified to be appointed a judge of the Supreme Court. In other words, he must be a
citizen of India and he must have been a judge of some high court for five years or an
advocate of some high court for ten years or an eminent jurist, in the opinion of the
president.
The term of office of the AG is not fixed by the Constitution. Further, the Constitution
does not contain the procedure and grounds for his removal. He holds office during the
pleasure of the president. This means that he may be removed by the president at any
time. He may also quit his office by submitting his resignation to the president.
Conventionally, he resigns when the government (council of ministers) resigns or is
replaced, as he is appointed on its advice.
Constitutionally, the Prime Minister may be a member of any of the two Houses of
parliament. For example, three Prime Ministers, Indira Gandhi (1966), Deve Gowda
(1996) and Manmohan Singh (2004), were members of the Rajya Sabha.
The term of the Prime Minister is not fixed and he holds office during the pleasure of
the president. However, this does not mean that the president can dismiss the Prime
www.iasbaba.com Page 11
IASbabas Prelims 60 Day Plan Day 25
Minister at any time. So long as the Prime Minister enjoys the majority support in the
Lok Sabha, he cannot be dismissed by the President. However, if he loses the confidence
of the Lok Sabha, he must resign or the President can dismiss him.
The Constitution does not contain any specific procedure for the selection and
appointment of the Prime Minister. Article 75 says only that the Prime Minister shall be
appointed by the president. However, this does not imply that the president is free to
appoint any one as the Prime Minister.
In accordance with the conventions of the parliamentary system of government, the
President has to appoint the leader of the majority party in the Lok Sabha as the Prime
Minister. But, when no party has a clear majority in the Lok Sabha, then the President
may exercise his personal discretion in the selection and appointment of the Prime
Minister.
In such a situation, the President usually appoints the leader of the largest party or
coalition in the Lok Sabha as the Prime Minister and asks him to seek a vote of
confidence in the House within a month.
Source : Council of Ministers (chapter 20) & Prime Minister (chapter 19)- Indian Polity by
M.Laxmikanth.
1. When the Lok Sabha passes a no-confidence motion against the council of ministers, all
the ministers have to resign including those ministers who are from the Rajya Sabha
www.iasbaba.com Page 12
IASbabas Prelims 60 Day Plan Day 25
2. Any exercise of executive power by the President without the aid and advice of the
Council of Ministers is considered to be unconstitutional
a) 1 only
b) 2 only
c) Both 1 and 2
d) Neither 1 nor 2
Article 75 clearly states that the council of ministers is collectively responsible to the
Lok Sabha. This means that all the ministers own joint responsibility to the Lok Sabha
for all their acts of ommission and commission. They work as a team and swim or sink
together. When the Lok Sabha passes a no-confidence motion against the council of
ministers, all the ministers have to resign including those ministers who are from the
Rajya Sabha.
Alternatively, the council of ministers can advise the president to dissolve the Lok Sabha
on the ground that the House does not represent the views of the electorate faithfully
and call for fresh elections. The President may not oblige the council of ministers that
has lost the confidence of the Lok Sabha.
The principle of collective responsibility also means that the Cabinet decisions bind all
cabinet ministers (and other ministers) even if they differed in the cabinet meeting. It is
the duty of every minister to stand by cabinet decisions and support them both within
and outside the Parliament. If any minister disagrees with a cabinet decision and is not
prepared to defend it, he must resign.
In 1971, the Supreme Court held that even after the dissolution of the Lok Sabha, the
council of
ministers does not cease to hold office. Article 74 is mandatory and, therefore, the
president cannot exercise the executive power without the aid and advise of the
council of ministers. Any exercise of executive power without the aid and advice will be
unconstitutional as being violative of Article 74.
Again in 1974, the court held that wherever the Constitution requires the satisfaction of
the President, the satisfaction is not the personal satisfaction of the President but it is
the satisfaction of the council of ministers with whose aid and on whose advice the
President exercises his powers and functions.
www.iasbaba.com Page 13
IASbabas Prelims 60 Day Plan Day 25
Q.12) With regard to Public Accounts Committee, consider the following statements
1. This committee was setup under the provisions of the Government of India Act of 1935.
2. The members are nominated by the Speaker for a term of one year.
3. As per the convention, the chairman of the committee is selected from the Opposition.
a) 3 only
b) 1 only
c) 2 and 3 only
d) None of the above
This committee was setup first in 1921 under the provisions of the Government of India
Act of 1919 and has since been in existence. At present, it consists of 22 members (15
from the Lok Sabha and 7 from the Rajya Sabha). Hence statement 1 is incorrect.
The members are elected by the Parliament every year from amongst its members
according to the principle of proportional representation by means of the single
transferable vote. Thus, all parties get due representation in it. The term of office of the
members is one year.
A minister cannot be elected as a member of the committee. The chairman of the
committee is appointed by the Speaker from amongst its members. Until 196667, the
chairman of the committee belonged to the ruling party. However, since 1967 a
convention has developed whereby the chairman of the committee is selected
invariably from the Opposition.
www.iasbaba.com Page 14
IASbabas Prelims 60 Day Plan Day 25
Q.13) With regard to pardoning power of the President, which of statements given below is
incorrect?
a) The power to pardon is to be exercised by the President on the advice of the Union
Cabinet
b) The President cannot pardon sentences inflicted by court martial
c) The Constitution does not provide for any mechanism to question the legality of
decisions of President
d) None
Article 72 of the Constitution empowers the President to grant pardons to persons who have
been tried and convicted of any offence in all cases where the:
The pardoning power of the President is independent of the Judiciary; it is an executive power.
But, the President while exercising this power, does not sit as a court of appeal. The object of
conferring this power on the President is two-fold: (a) to keep the door open for correcting any
judicial errors in the operation of law; and, (b) to afford relief from a sentence, which the
President regards as unduly harsh.
Under Article 161 of the Constitution, the governor of a state also possesses the pardoning
power. But, the pardoning power of the governor differs from that of the President in following
two respects:
1. The President can pardon sentences inflicted by court martial (military courts) while the
governor cannot.
2. The President can pardon death sentence while governor cannot. Even if a state law
prescribes death sentence, the power to grant pardon lies with the President and not the
governor. However, the governor can suspend, remit or commute a death sentence. In other
www.iasbaba.com Page 15
IASbabas Prelims 60 Day Plan Day 25
words, both the governor and the President have concurrent power in respect of suspension,
remission and commutation of death sentence.
The Constitution does not provide for any mechanism to question the legality of decisions of
President with regard to mercy jurisdiction. But the Supreme Court in Epuru Sudhakar case has
given scope for judicial review of the pardon powers of President and Governors for the
purpose of ruling out any arbitrariness. The court has earlier held that court has retained the
power of judicial review even on a matter which has been vested by the Constitution solely in
the Executive.
1. The petitioner for mercy has no right to an oral hearing by the President.
2. The President can examine the evidence afresh and take a view different from the view taken
by the court.
3. The power is to be exercised by the President on the advice of the union cabinet.
5. The President can afford relief not only from a sentence that he regards as unduly harsh but
also from an evident mistake.
6. There is no need for the Supreme Court to lay down specific guidelines for the exercise of
power by the President.
7. The exercise of power by the President is not subject to judicial review except where the
presidential decision is arbitrary, irrational, mala fide or discriminatory.
8. Where the earlier petition for mercy has been rejected by the President, stay cannot be
obtained by filing another petition.
Q.14) Which of the following statements are correct about Censure motion?
www.iasbaba.com Page 16
IASbabas Prelims 60 Day Plan Day 25
a) 2 only
b) 2 and 3 only
c) 1 and 3 only
d) 1, 2 and 3
In case of censure motion, reasons must be stated for its adoption in the Lok Sabha
while in case of motion of no confidence no such reasons for its adoption are required
to be stated in the Lok Sabha.
Censure motion can be moved against an individual minister or a group of ministers or
the entire council of ministers, whereas No confidence motion can only be moved
against the entire council of ministers
If Censure motion is passed in the Lok Sabha, the council of ministers need not resign
from the office, while in case of motion of no confidence, the council of minister must
resign from office if the motion is passed.
Censure motion is moved for censuring the council of ministers for specific policies and
action and no confidence motion on the otherhand, is moved for ascertaining the
confidence of Lok Sabha in the council of ministers.
a) 1 only
b) 2 only
c) Both 1 and 2
d) Neither 1 nor 2
www.iasbaba.com Page 17
IASbabas Prelims 60 Day Plan Day 25
Bills introduced in the Parliament are of two kinds: public bills and private bills (also known as
government bills and private members bills respectively). Though both are governed by the
same general procedure and pass through the same stages in the House, they differ in various
respects
Public Bill
4. Its rejection by the House amounts to the exp-ression of want of parliamentary confidence in
the government and may lead to its resignation.
Private Bill
4. Its rejection by the House has no implication on the parliamentary confidence in the
government or its resignation.
The bills introduced in the Parliament can also be classified into four categories:
1. Ordinary bills, which are concerned with any matter other than financial subjects.
2. Money bills, which are concerned with the financial matters like taxation, public expenditure,
etc.
www.iasbaba.com Page 18
IASbabas Prelims 60 Day Plan Day 25
3. Financial bills, which are also concerned with financial matters (but are different from money
bills).
4. Constitution amendment bills, which are concerned with the amendment of the provisions of
the Constitution.
Q.16) Which of the following motion can be moved to reduce any Demand for grant?
a) Superseding motion
b) Cut motion
c) Ancillary motion
d) Closure motion
Voting on Demands for Grants - In the light of the reports of the departmental standing
committees, the Lok Sabha takes up voting of demands for grants.The demands are presented
ministrywise. A demand becomes a grant after it has been duly voted.
One, the voting of demands for grants is the exclusive privilege of the Lok Sabha, that is,
the Rajya Sabha has no power of voting the demands.
Second, the voting is confined to the votable part of the budgetthe expenditure
charged on the Consolidated Fund of India is not submitted to the vote (it can only be
discussed).
Motion to reduce any demand for grant are called as cut motion.
Motions :
No discussion on a matter of general public importance can take place except on a motion
made with the consent of the presiding officer. The House expresses its decisions or opinions
on various issues through the adoption or rejection of motions moved by either ministers or
private members.
The motions moved by the members to raise discussions on various matters fall into three
principal categories:
www.iasbaba.com Page 19
IASbabas Prelims 60 Day Plan Day 25
3. Subsidiary Motion: It is a motion that, by itself, has no meaning and cannot state the
decision of the House without reference to the original motion or proceedings of the House. It
is divided into three sub-categories:
(a) Ancillary Motion: It is used as the regular way of proceeding with various kinds of business.
(b) Superseding Motion: It is moved in the course of debate on another issue and seeks to
supersede that issue.
(c) Amendment: It seeks to modify or substitute only a part of the original motion.
Closure Motion It is a motion moved by a member to cut short the debate on a matter before
the House. If the motion is approved by the House, debate is stopped forthwith and the matter
is put to vote. There are four kinds of closure motions15:
(a) Simple Closure: It is one when a member moves that the matter having been sufficiently
discussed be now put to vote.
(b) Closure by Compartments: In this case, the clauses of a bill or a lengthy resolution are
grouped into parts before the commencement of the debate. The debate covers the part as a
whole and the entire part is put to vote.
(c) Kangaroo Closure: Under this type, only important clauses are taken up for debate and
voting and the intervening clauses are skipped over and taken as passed.
(d) Guillotine Closure: It is one when the undiscussed clauses of a bill or a resolution are also
put to vote along with the discussed ones due to want of time (as the time allotted for the
discussion is over).
Calling Attention Motion It is introduced in the Parliament by a member to call the attention of
a minister to a matter of urgent public importance, and to seek an authoritative statement
www.iasbaba.com Page 20
IASbabas Prelims 60 Day Plan Day 25
from him on that matter. Like the zero hour, it is also an Indian innovation in the parliamentary
procedure and has been in existence since 1954. However, unlike the zero hour, it is mentioned
in the Rules of Procedure.
a) 1 and 2
b) 2 and 3
c) 1 and 3
d) All of the above
The idea behind establishment of National Resource Centre on Tribal Livelihood (Vanjeevan) is
to serve as an apex central institution within Ministry of Tribal Affairs to act as research and
technical hub to further socio-economic development of tribal communities through
comprehensive interaction of the 3Es-employment, employability and entrepreneurship. The
resource centre would cater to the development and promotion of sustainable livelihood
avenues in the tribal areas through entrepreneurship and skill up-gradation.
The National Resource Centre on Tribal Livelihood (Vanjeevan) also intends to forge linkages
and build upon the skill building efforts of the other Central Ministries / Departments such as
Ministry of Skill Development and Entrepreneurship, Ministry of Rural Development, Ministry of
Micro-Small and Medium Enterprises etc.
Vanjeevan will be a programme to identify the problems in livelihood issues in select districts of
six states having low HDI of tribal people in the first phase. The states are Assam, Gujarat,
Madhya Pradesh, Rajasthan, Odisha and Telangana. In the second phase the programme will be
www.iasbaba.com Page 21
IASbabas Prelims 60 Day Plan Day 25
Key Activities
b) Profiling of Micro enterprises models (Farm, Non-farm , forestry based and allied on local
geography and resources pattern).
d) Training and capacity development with creation of pool of resource persons and mentoring
organizations.
e) Partnership development
Source: http://pib.nic.in/newsite/PrintRelease.aspx?relid=155295
Q.18) Urban Plus approach was seen in news recently. It is associated with
a) BRICS
b) G4
c) United Nations Security Council (UNSC)
d) None of the above
Asia Pacific countries that account for over 55% of global urban population have resolved to
adopt Urban Plus approach. This was contained in the New Delhi Declaration adopted on the
conclusion of the Asia Pacific Ministerial Conference on Housing and Urban Development
(APMCHUD). It also adopted implementation plan.
This approach will address ways to make urban expansion sustainable and manageable. This
would mean an emphasis on planning for urban and its adjoining peri-urban and rural areas in
an integrated manner.
Source: http://pib.nic.in/newsite/PrintRelease.aspx?relid=155544
www.iasbaba.com Page 22
IASbabas Prelims 60 Day Plan Day 25
1. Under this system both domestic and foreign workers are obliged to seek their
employers permission to change jobs or leave the country
2. All the GCC countries have abolished the kafala system
a) Only 1
b) Only 2
c) Both 1 and 2
d) Neither 1 nor 2
The kafala system is a system used to monitor migrant laborers, working primarily in the
construction and domestic sectors, in Lebanon, Bahrain, Iraq, Jordan, Kuwait, Oman, Qatar,
Saudi Arabia, and the UAE.
Qatar has ended its controversial kafala system, under which foreign workers are obliged to
seek their employers permission to change jobs or leave the country.
Source: www.thehindu.com/news/international/Qatar-abolishes-kafala-labour-
system/article16802505.ece
a) Ross Sea
b) South China Sea
c) Adriatic Sea
d) Celebes Sea
The Peoples Liberation Army (PLA) Navy commemorated the 70th anniversary of the recovery
of the Xisha and Nansha Islands, the Chinese names of Paracel and Spratly islands in the SCS.
www.iasbaba.com Page 23
IASbabas Prelims 60 Day Plan Day 25
Chinas claims over the SCS was contested by Vietnam, the Philippines, Malaysia, Brunei and
Taiwan.
Source: http://www.thehindu.com/news/international/Chinese-navy-marks-70th-year-of-
capture-of-key-islands-in-SCS/article16778533.ece
Q.21) Consider the following statements about Central Electricity Authority (CEA)
a) Only 1
b) 2 and 3
c) 1 and 2
d) All of the above
Bureau of Energy Efficiency develops programs which will increase the conservation and
efficient use of energy in India.
The Central Electricity Authority of India (CEA) is a statutory organisation constituted under
section 3(1) of Electricity Supply Act 1948, which has been superseded by section 70(1) of the
Electricity Act 2003. The CEA advises the government on matters relating to the National
Electricity Policy and formulates short-term and perspective plans for the development of
electricity systems.
Coal based capacity addition will not be required in this period, as a capacity of 50 GW is
already under construction against a requirement of 44 GW.
www.iasbaba.com Page 24
IASbabas Prelims 60 Day Plan Day 25
It said that the renewable energy generation will contribute about 20.3 per cent and 24.2 per
cent of the total energy requirement in 2021-22 and 2026-27, respectively.
Source: http://www.thehindubusinessline.com/economy/cea-coal-power/article9423872.ece
a) Myanmar
b) Nepal
c) Pakistan
d) Bangladesh
Muhuri char is a disputed land between India and Bangladesh. 90% is under Indias control
Muhuri is a transnational river between India and Bangladesh. Rising in Tripura, it flows into
Bangladesh where it merges with the Feni near the latter's mouth to the Bay of Bengal. The
Muhuri is also known as the Little Feni.
Source: http://www.business-standard.com/article/news-ians/india-bangladesh-meeting-
over-disputed-enclave-in-tripura-inconclusive-116112000593_1.html
Husband or relative of husband of a woman subjecting her to cruelty. Whoever, being the
husband or the relative of the husband of a woman, subjects such woman to cruelty shall be
punished with imprisonment for a term which may extend to three years and shall also be liable
to fine.
www.iasbaba.com Page 25
IASbabas Prelims 60 Day Plan Day 25
Source: http://timesofindia.indiatimes.com/india/Adultery-by-itself-isnt-cruelty-to-wife-
says-SC/articleshow/55609961.cms
Q.24) OraQuick is
The WHO-approved OraQuick HIV self-testing is based on HIV antibodies present in oral and
blood samples. The test can detect antibodies developed within three months of getting
infected. It is a screening test, and a positive result should be reconfirmed though a blood-
based test.
The OraQuick self-testing makes diagnosis easier and faster, besides ensuring privacy and
confidentiality, thus encouraging more people to get tested.
Source: http://www.thehindu.com/opinion/editorial/HIV-The-self-test-
option/article16737936.ece
1. Khardung La :: J&K
2. Pir Panjal :: Himachal Pradesh
3. Saser La :: J&K
4. Bom Di La :: Sikkim
a) 1 and 3
b) 1, 2 and 3
c) 1, 3 and 4
d) 1, 2 and 4
www.iasbaba.com Page 26
IASbabas Prelims 60 Day Plan Day 25
1. Khardung La :: J&K
2. Pir Panjal :: J&K
3. Saser La :: J&K
4. Bom Di La :: Arunachal Pradesh
Source: http://www.thehindu.com/todays-paper/tp-national/Pre-historic-camping-site-
found-in-Ladakh/article16757539.ece
www.iasbaba.com Page 27
IASbabas Prelims 60 Day Plan Day 26 (Polity) 2017
TOPICS:
a) 1 only
b) 2 only
c) Both 1 and 2
d) Neither 1 nor 2
1. The Union Territories of Puducherry, Delhi and Chandigarh are provided with a
legislative assembly and a council of ministers headed by a chief minister
IASbabas Prelims 60 Day Plan Day 26 (Polity) 2017
2. The earlier Chief commissioners provinces were constituted as Union Territories by the
7th Constitutional Amendment Act and the States Reorganisation Act
a) 1 only
b) 2 only
c) Both 1 and 2
d) Neither 1 nor 2
The Union Territories of Puducherry (in 1963) and Delhi (in 1992) are provided with a
legislative assembly and a council of ministers headed by a chief minister.
The remaining five union territories do not have such popular political institutions. But,
the establishment of such institutions in the union territories does not diminish the
supreme control of the President and Parliament over them.
During the British Rule, certain areas were constituted as scheduled districts in 1874.
Later, they came to be known as chief commissioners provinces. After independence,
they were placed in the category of Part C and Part D states . In 1956, they were
constituted as the union territories by the 7th Constitutional Amendment Act (1956)
and the States Reorganisation Act (1956). Gradually, some of these union territories
have been elevated to statehood.
Thus, Himachal Pradesh, Manipur, Tripura, Mizoram, Arunachal Pradesh and Goa, which
are states today were formerly union territories. On the other hand, the territories that
were acquired from the Portuguese (Goa, Daman and Diu, and Dadra and Nagar Haveli)
and the French (Puducherry) were constituted as the union territories.
At present, there are seven Union Territories. They are (along with the year of creation):
(1)
Andaman and Nicobar Islands1956, (2) Delhi1956, (3) Lakshadweep1956, (4)
Dadra and
Nagar Haveli1961, (5) Daman and Diu1962, (6) Puducherry1962, and (7)
Chandigarh1966.
1. While appointing the Governor, the President is required to consult the chief minister of
the state concerned.
2. It does not lay down any grounds upon which a governor may be removed by the
President.
a) 1 only
b) 2 only
c) Both 1 and 2
d) Neither 1 nor 2
Appointment of Governor:
The governor is neither directly elected by the people nor indirectly elected by a specially
constituted electoral college as is the case with the president. He is appointed by the president
by warrant under his hand and seal. In a way, he is a nominee of the Central government. But,
as held by the Supreme Court in 1979, the office of governor of a state is not an employment
under the Central government. It is an independent constitutional office and is not under the
control of or subordinate to the Central government.
The American model, where the Governor of a state is directly elected, was dropped and the
Canadian model, where the governor of a province (state) is appointed by the Governor-
General (Centre), was accepted in the Constituent Assembly.
The Constitution lays down only two qualifications for the appointment of a person as a
governor. These are:
Additionally, two conventions (and NOT mentioned in the Constitution) have also developed
in this regard over the years. First, he should be an outsider, that is, he should not belong to the
state where he is appointed, so that he is free from the local politics. Second, while appointing
IASbabas Prelims 60 Day Plan Day 26 (Polity) 2017
the governor, the president is required to consult the chief minister of the state concerned, so
that the smooth functioning of the constitutional machinery in the state is ensured. However,
both the conventions have been violated in some of the cases.
A governor holds office for a term of five years from the date on which he enters upon his
office. However, this term of five years is subject to the pleasure of the President. Further, he
can resign at any time by addressing a resignation letter to the President.
The Supreme Court held that the pleasure of the President is not justifiable. The governor has
no security of tenure and no fixed term of office. He may be removed by the President at any
time.The Constitution does not lay down any grounds upon which a governor may be
removed by the President. Hence, the National Front Government headed by V P Singh (1989)
asked all the governors to resign as they were appointed by the Congress government.
Eventually, some of the governors were replaced and some were allowed to continue. The
same thing was repeated in 1991, when the Congress Government headed by P V Narasimha
Rao changed fourteen governors appointed by the V P Singh and Chandra Sekhar governments.
a) 1, 2 and 5
b) 1, 2 and 3
c) 1, 2, 3 and 5
d) 1, 2, 3, 4 and 5
While most states have unicameral legislature with only legislative assembly, currently,
seven states viz. (BiMARU) Andhra Pradesh, Bihar, Jammu and Kashmir, Karnataka,
Maharashtra, Telangana, and Uttar Pradesh have legislative council. Further,
Parliament has also cleared formation of Legislative Council Rajasthan and Assam.
Source: Chapter-29 State legislature - Indian Polity by M. Laxmikanth & Current Affairs
a) 1 only
b) 2 only
c) Both 1 and 2
d) Neither 1 nor 2
The Governor cannot pardon a death sentence. Even if a state law prescribes for death
sentence, the power to grant pardon lies with the President and not the governor. But,
the governor can suspend, remit or commute a death sentence.
The President may transfer a Governor appointed to one state to another state for the
rest of the term. Further, a Governor whose term has expired may be reappointed in the
same state or any other state. A governor can hold office beyond his term of five years
until his successor assumes charge. The underlying idea is that there must be a
governor in the state and there cannot be an interregnum.
The President can make such provision as he thinks fit for the discharge of the functions
of the governor in any contingency not provided for in the Constitution, for example,
the death of a sitting governor. Thus, the chief justice of the concerned state high court
may be appointed temporarily to discharge the functions of the governor of that state.
Q.6) In Indian context, the Governor has Constitutional discretion in which of the following
cases?
a) 1 and 2 only
b) 1, 2 and 3
c) 1, 2 and 4
d) 1, 2, 3 and 4
The Constitutional position of the governor differs from that of the president in the following
two respects:
1. While the Constitution envisages the possibility of the governor acting at times in his
discretion, no such possibility has been envisaged for the President.
2. After the 42nd Constitutional Amendment (1976), ministerial advice has been made binding
on the President, but no such provision has been made with respect to the governor.
The Constitution makes it clear that if any question arises whether a matter falls within the
governors discretion or not, the decision of the governor is final and the validity of anything
done by him cannot be called in question on the ground that he ought or ought not to have
acted in his discretion.
3. While exercising his functions as the administrator of an adjoining union territory (in case
of additional charge).
4. Determining the amount payable by the Government of Assam, Meghalaya, Tripura and
Mizoram to an autonomous Tribal District Council as royalty accruing from licenses for mineral
exploration.
5. Seeking information from the chief minister with regard to the administrative and legislative
matters of the state.
In addition to the above constitutional discretion (i.e., the express discretion mentioned in the
Constitution), the governor, like the president, also has situational discretion (i.e., the hidden
discretion derived from the exigencies of a prevailing political situation) in the following cases:
1. Appointment of chief minister when no party has a clear-cut majority in the state legislative
assembly or when the chief minister in office dies suddenly and there is no obvious successor.
2. Dismissal of the council of ministers when it cannot prove the confidence of the state
legislative assembly.
3. Dissolution of the state legislative assembly if the council of ministers has lost its majority.
a) 1 only
b) 2 only
c) Both 1 and 2
d) Neither 1 nor 2
The Parliament can abolish a legislative council (where it already exists) or create it
(where it does not exist), if the legislative assembly of the concerned state passes a
resolution to that effect.
Such a specific resolution must be passed by the state assembly by a special majority,
that is, a majority of the total membership of the assembly and a majority of not less
than two-thirds of the members of the assembly present and voting.
This Act of Parliament is not to be deemed as an amendment of the Constitution for
the purposes of Article 368 and is passed like an ordinary piece of legislation (ie, by
simple majority).
1. In the states of Chhattisgarh, Jharkhand, Madhya Pradesh, Bihar and Odisha, there shall
be a Minister in charge of tribal welfare.
2. The Governor can nominate one member to the state legislature council from the
Anglo-Indian Community.
a) 1 only
b) 2 only
c) Both 1 and 2
d) Neither 1 nor 2
In the states of Chhattisgarh, Jharkhand, Madhya Pradesh and Odisha, there shall be a
Minister in charge of tribal welfare who may in addition be in charge of the welfare of
the scheduled castes and backward classes or any other work. The state of Bihar was
excluded from this provision by the 94th Amendment Act of 2006.
Article 166Conduct of Business of the Government of a State -All executive action of
the Government of a State shall be expressed to be taken in the name of the
Governor.
A governor is an integral part of the state legislature. In that capacity, he has the
following legislative powers and functions:
IASbabas Prelims 60 Day Plan Day 26 (Polity) 2017
1. He can summon or prorogue the state legislature and dissolve the state legislative
assembly.
2. He can address the state legislature at the commencement of the first session after
each general election and the first session of each year.
3. He can send messages to the house or houses of the state legislature, with respect to
a bill pending in the legislature or otherwise.
4. He can appoint any member of the State legislative assembly to preside over its
proceedings
when the offices of both the Speaker and the Deputy Speaker fall vacant. Similarly, he
can appoint any member of the state legislature council to preside over its proceedings
when the offices of both Chairman and Deputy Chairman fall vacant.
5. He nominates one-sixth of the members of the state legislative council from
amongst persons
having special knowledge or practical experience in literature, science, art, cooperative
movement and social service.
6. He can nominate one member to the state legislature assembly from the Anglo-
Indian Community.
7. He decides on the question of disqualification of members of the state legislature in
consultation with the Election Commission.
1. The Parliament can make laws on any subject of the three lists (Centre, State,
Concurrent) for all the union territories.
2. The Parliament can make regulations for the peace, progress and good government of
the Andaman and Nicobar Islands and Lakshadweep Islands
a) 1 only
b) 2 only
c) Both 1 and 2
d) Neither 1 nor 2
IASbabas Prelims 60 Day Plan Day 26 (Polity) 2017
The Union Territories of Puducherry (in 1963) and Delhi (in 1992) are provided with a
legislative assembly and a council of ministers headed by a chief minister. The remaining five
union territories do not have such popular political institutions. But, the establishment of such
institutions in the union territories does not diminish the supreme control of the president and
Parliament over them.
The Parliament can make laws on any subject of the three lists (including the State List) for
the union territories. This power of Parliament also extends to Puducherry and Delhi, which
have their own local legislatures. This means that, the legislative power of Parliament for the
union territories on subjects of the State List remain unaffected even after establishing a local
legislature for them. But, the legislative assembly of Puducherry can also make laws on any
subject of the State List and the Concurrent List. Similarly, the legislative assembly of Delhi can
make laws on any subject of the State List (except public order, police and land) and the
Concurrent List.
The President can make regulations for the peace, progress and good government of the
Andaman and Nicobar Islands, Lakshadweep, Dadra and Nagar Haveli, and Daman and Diu. In
the case of Puducherry also, the President can legislate by making regulations but only when
the assembly is suspended or dissolved. A regulation made by the President has the same force
and effect as an act of
Parliament and can also repeal or amend any act of Parliament in relation to these union
territories.
1. When the Governor reserves any bill for the consideration of the President, the
Governor will not have any further role in the enactment of the bill
2. The question of disqualification under the Tenth Schedule is decided by Governor and
the decision is subject to judicial review.
a) 1 only
IASbabas Prelims 60 Day Plan Day 26 (Polity) 2017
b) 2 only
c) Both 1 and 2
d) Neither 1 nor 2
Disqualification on Ground of Defection : The Constitution also lays down that a person
shall be disqualified for being a member of either House of state legislature if he is so
disqualified on the ground of defection under the provisions of the Tenth Schedule.
The question of disqualification under the Tenth Schedule is decided by the Chairman,
in the case of legislative council and, Speaker, in the case of legislative assembly (and
not by the governor). In 1992, the Supreme Court ruled that the decision of
Chairman/Speaker in this regard is subject to judicial review . Hence statement 1 is
incorrect.
On the question whether a member is subject to disqualification, the governors
decision is final. However, he should obtain the opinion of the Election Commission and
act accordingly.
When the Governor reserves a bill for the consideration of the President, the governor
will not have any further role in the enactment of the bill. If the bill is returned by the
President for the reconsideration of the House or Houses and is passed again, the bill
must be presented again for the presidential assent only. If the President gives his
assent to the bill, it becomes an act. This means that the assent of the Governor is no
longer required.
a) Delhi is the only union territory that has a high court of its own.
b) The Governor can resign any time by addressing a resignation letter to the President
c) Constitutional provisions for the administration of union territories do not apply to the
acquired territories
d) Incase of any contingency, the Chief Justice of the concerned state High court may be
appointed temporarily to discharge the functions of the governor of that state
IASbabas Prelims 60 Day Plan Day 26 (Polity) 2017
The Constitution does not contain any separate provisions for the administration of
acquired territories. But, the constitutional provisions for the administration of union
territories also apply to the acquired territories.
The President can make regulations for the peace, progress and good government of the
Andaman and Nicobar Islands, Lakshadweep, Dadra and Nagar Haveli, and Daman and
Diu. In the case of Puducherry also, the President can legislate by making regulations
but only when the assembly is suspended or dissolved.
A regulation made by the President has the same force and effect as an act of
Parliament and can also repeal or amend any act of Parliament in relation to these
union territories
A governor holds office for a term of five years from the date on which he enters upon
his office. However, this term of five years is subject to the pleasure of the President.
Further, he can resign at any time by addressing a resignation letter to the President.
The President can make such provision as he thinks fit for the discharge of the functions
of the
governor in any contingency not provided for in the Constitution, for example, the death
of a sitting governor. Thus, the chief justice of the concerned state high court may be
appointed temporarily to discharge the functions of the governor of that state.
Q.12) With regard to legislative procedure of an Ordinary Bill in the State legislature, which of
the following statements are incorrect?
a) The Constitution provides for the mechanism of joint sitting of both the Houses to
resolve the disagreement between the two Houses over a bill
b) When a bill, which has originated in the council and was sent to the assembly, is
rejected by the assembly, the bill ends and becomes dead
c) The Bill can be introduced either by a minister or a private member
d) The legislative assembly can override the legislative council by passing the bill for the
second time and not vice versa
IASbabas Prelims 60 Day Plan Day 26 (Polity) 2017
The Constitution does not provide for the mechanism of joint sitting of both the
Houses to resolve the disagreement between the two Houses over a bill. On the other
hand, there is a provision for joint sitting of the Lok Sabha and the Rajya Sabha to
resolve a disagreement between the two over an ordinary bill. Hence statement 1 is
incorrect.
Moreover, when a bill, which has originated in the council and was sent to the assembly,
is rejected by the assembly, the bill ends and becomes dead.
It can be introduced in either House of the state legislature. It can be introduced either
by a minister or by private member.
The legislative assembly can override the legislative council by passing the bill for the
second time and not vice versa. When a bill is passed by the assembly for the second
time and transmitted to the legislative council, if the legislative council rejects the bill
again, or porposes amendments that are not acceptable to the legislative assembly, or
does not pass the bill within one month, then the bill is deemed to have been passed by
both the Houses in the form in which it was passed by the legislative assembly for the
second time.
Q.13) Consider the following statements w.r.t special status of Jammu and Kashmir.
1. The President has no power to declare a financial emergency in relation to the state.
2. The preventive detention laws made by the Parliament are not applicable to the state.
3. The Presidents rule is not applicable to the state
a) 1 and 2 only
b) 2 and 3 only
c) 1 only
d) 1, 2 and 3
In pursuance of the provisions of Article 370, the President issued an order called the
Constitution (Application to Jammu and Kashmir) Order, 1950, to specify the Unions
jurisdiction over the state.
In 1952, the Government of India and the State of J&K entered into an agreement at Delhi
regarding their future relationship. In 1954, the Constituent Assembly of J&K approved the
states accession to India as well as the Delhi Agreement. Then, the President issued another
order with the same title, that is, the Constitution (Application to Jammu and Kashmir), Order,
1954. This order superseded the earlier order of 1950 and extended the Unions jurisdiction
over the state. This is the basic order that, as amended and modified from time to time,
regulates the constitutional position of the state and its relationship with the Union. At
present, this is as follows:
1. Jammu and Kashmir is a constituent state of the Indian Union and has its place in Part I and
Schedule I of the Constitution of India (dealing with the Union and its Territory). But its name,
area or boundary cannot be changed by the Union without the consent of its legislature.
2. The State of J & K has its own Constitution and is administered according to that
Constitution. Hence, Part VI of the Constitution of India (dealing with state governments) is not
applicable to this state. The very definition of state under this part does not include the State
of J&K.
3. Parliament can make laws in relation to the state on most of the subjects enumerated in the
Union List and on a good number of subjects enumerated in the Concurrent List.But, the
residuary power belongs to the state legislature except in few matters like prevention of
activities involving terrorist acts, questioning or disrupting the sovereignty and territorial
integrity of India and causing insult to the National Flag, National Anthem and the Constitution
of India. Further, the power to make laws of preventive detention in the state belongs to the
state legislature. This means that the preventive detention laws made by the Parliament are
not applicable to the state.
4. Part III (dealing with Fundamental Rights) is applicable to the state with some exceptions and
conditions. The Fundamental Right to Property is still guaranteed in the state. Also, certain
special rights are granted to the permanent residents of the state with regard to public
employment, acquisition of immovable property, settlement and government scholarships.
5. Part IV (dealing with Directive Principles of State Policy) and Part IVA (dealing with
Fundamental Duties) are not applicable to the state.
IASbabas Prelims 60 Day Plan Day 26 (Polity) 2017
6. A National Emergency declared on the ground of internal disturbance will not have effect in
the state except with the concurrence of the state government.
7. The President has no power to declare a financial emergency in relation to the state.
8. The President has no power to suspend the Constitution of the state on the ground of failure
to comply with the directions given by him.
9. The State Emergency (Presidents Rule) is applicable to the state. However, this emergency
can be imposed in the state on the ground of failure of the constitutional machinery under the
provisions of state Constitution and not Indian Constitution. In fact, two types of Emergencies
can be declared in the state, namely, Presidents Rule under the Indian Constitution and
Governors Rule under the state Constitution. In 1986, the Presidents Rule was imposed in the
state for the first time.
10. International treaty or agreement affecting the disposition of any part of the territory of the
state can be made by the Centre only with the consent of the state legislature.
11. An amendment made to the Constitution of India does not apply to the state unless it is
extended by a presidential order.
12. Official language provisions are applicable to the state only in so far as they relate to the
official language of the Union, the official language of inter-state and Centrestate
communications and the language of the Supreme Court proceedings.
13. The Fifth Schedule (dealing with administration and control of schedule areas and
scheduled tribes) and the Sixth Schedule (dealing with administration of tribal areas) do not
apply to the state.
14. The special leave jurisdiction of the Supreme Court and the jurisdictions of the Election
Commission and the comptroller and auditor general are applicable to the state.
15. The High Court of J&K can issue writs only for the enforcement of the fundamen-tal rights
and not for any other purpose.
16. The provisions of Part II regarding the denial of citizenship rights of migrants to Pakistan are
not applicable to the permanent residents of J&K, who after having so migrated to Pakistan
return to the state for resettlement. Every such person is deemed to be a citizen of India.
Therefore, the two characteristic features of the special relationship between the State of J&K
and the Union of India are: (a) the state has a much greater measure of autonomy and power
IASbabas Prelims 60 Day Plan Day 26 (Polity) 2017
than enjoyed by the other states; and (b) Centres jurisdiction within the state is more limited
than what it has with respect to other states.
Source: Chapter-32 Special status of Jammu & Kashmir - Indian Polity by M. Laxmikanth
Q.14) With regard to Money bill in the State Legislature, consider the following statements
a) 1 only
b) 2 only
c) Both 1 and 2
d) Neither 1 nor 2
A Money Bill cannot be introduced in the legislative council. It can be introduced in the
legislative assembly only and that too on the recommendation of the governor. Every
such bill is considered to be a government bill and can be introduced only by a minister.
When a money bill is reserved for consideration of the President, the president may
either give his assent to the bill or withhold his assent to the bill but cannot return the
bill for reconsideration of the state legislature.
a) 2 and 3 only
b) 1 and 2 only
c) 3 only
d) 1, 2 and 3
Q.16) Consider the following statements about Competition Commission of India (CCI)
1. It is a statutory body
2. The Chairperson and members of CCI are appointed by the Central Government
3. Matters regarding fixation of prices of agricultural products as well as reviewing
progress of activities related to rural development including those concerning small and
marginal farmers are in CCIs competence
a) 1 and 2
b) 2 and 3
c) 1 and 3
d) All of the above
IASbabas Prelims 60 Day Plan Day 26 (Polity) 2017
The Competition Act, 2002, as amended by the Competition (Amendment) Act, 2007, follows
the philosophy of modern competition laws. The Act prohibits anti-competitive agreements,
abuse of dominant position by enterprises and regulates combinations (acquisition, acquiring of
control and M&A), which causes or likely to cause an appreciable adverse effect on competition
within India.
The objectives of the Act are sought to be achieved through the Competition Commission of
India (CCI), which has been established by the Central Government with effect from 14th
October 2003. CCI consists of a Chairperson and 6 Members appointed by the Central
Government. It is the duty of the Commission to eliminate practices having adverse effect on
competition, promote and sustain competition, protect the interests of consumers and ensure
freedom of trade in the markets of India. The Commission is also required to give opinion on
competition issues on a reference received from a statutory authority established under any
law and to undertake competition advocacy, create public awareness and impart training on
competition issues.
Cabinet Committee on Economic Affairs (CCEA) has a mandate to review economic trends on a
continuous basis, as also the problems and prospects, with a view to evolving a consistent and
integrated economic policy framework for the country. It also directs and coordinates all
policies and activities in the economic field including foreign investment that require policy
decisions at the highest level.
Source: http://www.livemint.com/Opinion/GWrGWACHE69WH4Nobl407N/Intellectual-
property-vs-competition-law.html
a) Digital Payments
b) Border Protection
c) Restructuring of Civil Services examination
d) Railway restructuring
IASbabas Prelims 60 Day Plan Day 26 (Polity) 2017
Committee has recommended that a medium term strategy for accelerating growth of Digital
Payments in India with a regulatory regime which is conducive to bridging the Digital divide by
promoting competition, open access & interoperability in payments.
The Report recommends inclusion of financially and socially excluded groups and assimilation
of emerging technologies in the market, while safeguarding security of Digital Transactions and
providing level playing to all stakeholders and new players who will enter this new transaction
space.
It has suggested inter-operability of the payments system between banks and non-banks, up-
gradation of the digital payment infrastructure and institutions and a framework to reward
innovations and for leading efforts in enabling digital payments.
Terms of Reference:
To study and recommend need for charges, if any, in the regulatory mechanism and any
legislation, relevant for the purpose of promotion of payments by digital modes
To study and recommend ways for leveraging Unique Identification Number or any
other proof of identity for authentication of card/digital transactions and setting up of a
Centralised KYC Registry;
To study introduction of single window system of Payment Gateway to accept all types
of Cards/ Digital Payments of Government receipts;
To study feasibility and framing rules for creating a payments history of all Digital
Payments and create necessary linkage between payments transaction history and
credit information;
To study and recommend various measures to incentivize transactions through cards
and digital means.
To study global best practices in payments including initiatives taken by various
Governments/ Government Agencies
To identify market failure (s), if any, along with suitable interventions that may be
implemented to promote payment by card/digital means
To identify regulatory bottlenecks, if any, and suggest changes to promote payment by
card/ digital means
IASbabas Prelims 60 Day Plan Day 26 (Polity) 2017
Source: http://pib.nic.in/newsite/PrintRelease.aspx?relid=155245
1. It will be an online database which will carry information of student attendance and
enrolment, learning outcomes, mid-day meal service and infrastructural facilities among
others.
2. It was launched by Ministry of Human Resource Development
3. It is launched under Shala Asmita Yojana (SAY)
a) 1 and 2
b) Only 2
c) 1 and 3
d) 2 and 3
In pursuit of its efforts to move towards a cashless economy the Government of India has
launched Vittiya Saksharta Abhiyan to promote active participation of youth of Higher
Educational Institutions to motivate all payers and payees to be embrace cashless economic
system for transfer of funds.
The primary aim is to mobilise youth and faculty to promote digital literacy and awareness
among people around them who have been technologically averse.
Source: http://pib.nic.in/newsite/PrintRelease.aspx?relid=154755
Q.19) Consider the following statements about UDAN (Ude Desh ka Aam Naagrik) Scheme
2. Regional Connectivity Fund (RCF) will be created to meet the Viability Gap Funding
(VGF) requirements under the scheme
a) Only 1
b) Only 2
c) Both 1 and 2
d) Neither 1 nor 2
The scheme UDAN envisages providing connectivity to un-served and under-served airports of
the country through revival of existing air-strips and airports. The scheme would be in
operation for a period of 10 years.
UDAN has a unique market-based model to develop regional connectivity. Interested airline
and helicopter operators can start operations on hitherto un-connected routes by submitting
proposals to the Implementing Agency.
The operators could seek a Viability Gap Funding (VGF) apart from getting various concessions.
A Regional Connectivity Fund would be created to meet the viability gap funding requirements
under the scheme. The RCF levy per departure will be applied to certain domestic flights.
The partner State Governments (other than North Eastern States and Union Territories where
contribution will be 10 %) would contribute a 20% share to this fund. For balanced regional
growth, the allocations under the scheme would be equitably spread across the five
geographical regions of the country viz. North, West, South, East and North-east.
The States have a key role under the scheme. The selection of airports where UDAN operations
would start would be done in consultation with State Government and after confirmation of
their concessions. It may be recalled that revival of dysfunctional airports and starting
operations on un-served airports has been a long standing demand of most States and this will
be addressed through UDAN to a large extent.
IASbabas Prelims 60 Day Plan Day 26 (Polity) 2017
The UDAN is likely to a give a major fillip to tourism and employment generation in the
hinterland. Through introduction of helicopters and small aircraft, it is also likely to significantly
reduce travel timings in remote and hilly regions, as well as islands and other areas of the
country.
The State of World Fisheries and Aquaculture (SOFIA) is the flagship publication of the FAO
Fisheries and Aquaculture Department. This premier advocacy document is published every
two years to provide policy-makers, civil society and those whose livelihoods depend on the
sector a comprehensive, objective and global view of capture fisheries and aquaculture,
including associated policy issues.
Source: http://www.thehindu.com/business/Industry/India-reports-fishery-subsidies-aims-
to-protect-%E2%80%98poor%E2%80%99-fisherfolk/article16785645.ece
Q.21) India Achieves 4th rank in Global Wind Power Installed Capacity Index. Which of the
following states has the highest Wind Power installation in India?
a) Tamil Nadu
b) Maharashtra
c) Karnataka
d) Gujarat
India has been ranked 4th in the Global Wind Power Installed Capacity index after China, USA
and Germany with cumulative installed wind power generation capacity of 28,279 MW in 2016.
IASbabas Prelims 60 Day Plan Day 26 (Polity) 2017
India achieved the largest-ever wind power capacity addition of 3,423 MW in 2015-16,
exceeding the target by 43 per cent. During 2016-17, a total of 1,502 MW capacity has been
added till 31 October 2016, making cumulative achievement 28,279 MW.
Wind energy distribution is not uniform across all states in India. Southern and western states
have maximum wind capacity installed due to highest wind density and velocity. Tamil Nadu is
leading state in terms of wind installations with 33% share in overall wind capacity installed in
the country.
Source: http://pib.nic.in/newsite/PrintRelease.aspx?relid=155612
Q.22) China has recently launched a satellite called TanSat. The aim of this satellite is to:
A global carbon dioxide monitoring satellite, TanSat, was launched by China on December 22,
2016, to understand climate change in the country.
The satellite was sent into a sun synchronous orbit about 700 kms above the earth and will
monitor the concentration, distribution and flow of carbon dioxide (CO2) in the atmosphere.
On a three-year mission, TanSat will thoroughly examine global carbon dioxide levels every 16
days, accurate to at least 4 ppm (parts per million).
The new satellite will enable China to obtain emissions data first-hand and share it with
researchers worldwide.
Source: http://indianexpress.com/article/technology/science/china-launches-satellite-to-
monitor-global-carbon-emissions-4439407/
Q.23) With regard to International Criminal Court (ICC), consider the following statements
2. The Roman Statute, 2002 serves as the ICC's foundational and governing document
3. It seeks to replace national courts of a country in crimes related to war and genocide
a) 1 only
b) 2 only
c) 1 and 3 only
d) 1, 2 and 3
The ICC began functioning on 1 July 2002, the date that the Rome Statute entered into force.
The Rome Statute is a multilateral treaty which serves as the ICC's foundational and governing
document. States which become party to the Rome Statute, for example by ratifying it, become
member states of the ICC. Currently, there are 124 states which are party to the Rome Statute
and therefore members of the ICC. However, Burundi, South Africa, and Gambia have given
formal notice that they will withdraw from the Rome Statute.
As a court of last resort, it seeks to complement, not replace, national Courts. Governed by an
international treaty called the Rome Statute, the ICC is the worlds first permanent
international criminal court.
Source: http://www.business-standard.com/article/news-ians/russia-pulls-out-from-the-
international-criminal-court-116111601727_1.html
Thunderstorms in the ionosphere are to blame for loss of GPS signals on low-orbiting
satellites, says a study that used data from the European Space Agencys Swarm mission.
Source: http://indianexpress.com/article/technology/science/why-satellites-sometimes-lose-
their-gps-connection-3730748//
Q.25) Recently Imja Tsho a glacial lake was in news. Where is it located?
Nepal has successfully drained part of a giant glacial lake near Mount Everest, averting risk of a
disastrous flood that could have threatened thousands of lives.
The Nepal government worked together with United Nations Development Programme (UNDP)
to drain the lake.
Source: http://www.thehindu.com/sci-tech/energy-and-environment/Nepal-drains-risky-
glacial-lake-near-Mount-Everest/article16086084.ece
IASbabas 60 Day Plan Day 27 (Geography) 2017
DAY 27
SUBJECT INDIAN GEOGRAPHY
TOPICS:
FOCUS ON:
Indian Geography Cropping (Agriculture)
Cropping Pattern: Example: Rice, Wheat, Pulses, Sugarcane, Millets, Coffee, Tea
etc.
Where(states) are they are grown?; Climatic conditions suitable for their growth
Classification of crops Kharif crops, Rabi and Zaid crops with examples.
Irrigation New irrigation methods like Micro-irrigation, Drip-irrigation their
usage, advantages and disadvantages
Farming Shifting cultivation, Terrace cultivation, Crop Rotation, Mixed and
Multiple farming where are these methods practiced; advantages and
disadvantages.
PRELIMS MCQs:
a) 1 and 2 only
b) 1, 2 and 4 only
c) 1, 3 and 4 only
d) 1, 2, 3 and 4
The agricultural patterns are strictly dependent on the geo-ecological conditions; terrain,
topography, slope and altitude.
While paddy cultivation requires leveled fields, tea plantations perform well in the
undulating topography in which water does not remain standing.
IASbabas 60 Day Plan Day 27 (Geography) 2017
Orchards of coconut are found at low altitudes, preferably closer to the sea level, while the
apple orchards in the tropical and sub-tropical conditions perform well above 1500 metres
above sea level. Moreover, cultivation of crops is rarely done 3500 m above sea-level in the
tropical and sub-tropical latitudes. The highly rarified air, low-pressure, low-temperature
and shortage of oxygen at high altitudes are the serious impediments not only in the
cultivation of crops, but also in keeping dairy cattle. Hence, statement (2) is wrong.
Q.2) Which of the statements given below is/are correct in regard to Jute, considered as
the golden fibre of India?
1. The crop cannot be grown in both moisture stress and water stagnating condition.
2. Warm, humid weather and acidic soil result in good yeild.
3. Jute grows well on the new alluvial soils having higher silt content.
4. Jute is predominantly grown as a rainfed crop.
a) 2 and 3 only
b) 1, 2 and 4 only
c) 2, 3 and 4 only
d) 1, 2, 3 and 4
Jute is considered as the golden fibre of India. It is the commercially available natural fibre
which is utilized mostly as packaging material, nowadays facing a steep competition from
cheap synthetics in packaging sector. Besides the traditional packaging sector, jute has been
used in both textile and non-textile sectors in large and small industries.
Jute is eco-friendly, biodegradable and has much higher CO2 assimilation rate which is
creating an opportunity for the survival and growth of jute industry in the era of
environmental concern. Global production of jute and allied fibres is around 3.0 million
tonnes, 92.5% of which comes from India and Bangladesh alone. India ranks first in area and
production of jute followed by Bangladesh and earns approximately Rs. 1400 crores/annum
through export of jute goods mainly Jute diversified products (JDPs).
Jute requires a warm and humid climate and can be grown within a temperature range of 24
to 37oC and a relative humidity of 57 to 97%. Jute crop thrives well with alternate rains and
sunshine. The crop can grown in both moisture stress and water stagnating condition.
IASbabas 60 Day Plan Day 27 (Geography) 2017
The amount of rainfall and its distribution have marked effects on the growth of crop and
ultimately yield of fibre. In the ideal situation, 120-150 mm pre monsoon rain followed by a
dry period of 30-40 days and 1200 to 1500 mm of precipitation over last 75-80 days is
considered to be the most ambient condition for growth of jute crop.
Jute grows well on the new alluvial soils having higher silt content but can also grow on
various other types of soil. In India, it is grown in mainly colluviums, red and lateritic,
calcareous soils. Acidic to neutral soils are suitable for jute cultivation. Jute cultivation is
mainly concentrated in the eastern and north eastern India.
a) 1 and 2 only
b) 2 and 3 only
c) 1 only
d) 1, 2 and 3
All crops need moisture. Within wide temperature limits, moisture is more important than
any other climatic factor in crop production. There are optimal moisture conditions for crop
development just as there are optimal temperature conditions.
The excessive amount of water in the soil alters various chemical and biological processes,
limiting the amount of oxygen and increasing the formation of compounds that are toxic to
plant roots. Therefore, the excess of water in soil leads to stunted growth of plants.
Drought has devastating consequences on the crops, their yields and production. Soil
drought has been described as a condition in which the amount of water needed for
transpiration and direct evaporation exceeds the amount of water available in the soil.
Drought damages the crops when plants are inadequately supplies with moisture from the
soil.
a) 1, 2, 3 and 4
b) 1, 3 and 4 only
c) 3 and 4 only
d) 4 only
The term forage means the plants used for feeding domestic animals, this includes both
fodder plants and pasture plants.
Fodder crops are the plant species that are cultivated and harvested for feeding the animals
in the form of forage (cut green and fed fresh), silage (preserved under anaerobic condition)
and hay (dehydrated green fodder).
In previous question papers, UPSC had framed questions on above lines, to identify which
crops are used as pulse, fodder and green manure etc.
Q.5) Consider the statements below in regard to Four Water Concept, which was
recently in news:
IASbabas 60 Day Plan Day 27 (Geography) 2017
1. It is an indigenous watershed development technology.
2. It is a low-cost technology that can increase recharge of groundwater by four times
and provide thrice the benefit than conventional models.
3. The concept evolved by integrating the 'Four Waters' - rain water, soil moisture,
ground water and surface water.
a) 1 and 2 only
b) 2 and 3 only
c) 1 only
d) 1, 2 and 3
Four Waters concept was designed and developed by T. Hanumantha Rao, who passed
away recently. He was a well-known irrigation engineering expert and was the brain behind
the Four Waters' concept.
Link:
https://www.google.co.in/search?q=%E2%80%98Four+Waters%27+concept&num=100&so
urce=lnms&tbm=nws&sa=X&ved=0ahUKEwiYy_2Vlb_TAhWKvY8KHXi6CWQQ_AUICygE&biw
=1094&bih=547
Q.6) Consider the below statements about Wheat and choose the incorrect statement:
a) It can be grown not only in the tropical and sub-tropical zones, but also in the
temperate zone and the cold tracts of the far north, beyond even the 60 degree
north altitude.
b) It requires a fairly warm temperature and the period of heat should be
comparatively long.
c) Soils with a clay loam or loam texture, good structure and moderate water holding
capacity are ideal for wheat cultivation.
d) It can be cultivated from sea level to as high as 3300 meters.
Climate requirement:
Wheat is the main cereal crop in India. Wheat crop has wide adaptability. It can be grown
not only in the tropical and sub-tropical zones, but also in the temperate zone and the cold
tracts of the far north, beyond even the 60 degree north altitude.
IASbabas 60 Day Plan Day 27 (Geography) 2017
Wheat can tolerate severe cold and snow and resume growth with the setting in of warm
weather in spring. It can be cultivated from sea level to as high as 3300 meters. The best
wheat are produced in areas favoured with cool, moist weather during the major portion of
the growing period followed by dry, warm weather to enable the grain to ripen properly.
The optimum temperature range for ideal germination of wheat seed is 20-25 C though the
seeds can germinate in the temperature range 3.5 to 35 C. Rains just after sowing hamper
germination and encourage seedling blight. Areas with a warm and damp climate are not
suited for wheat growing.
Wheat requires a fairly warm temperature, but the period of heat should not be long as
grains can ripen quickly. When temperatures are high, too much energy is lost through the
process of transpiration by the plants and the reduced residual energy results in poorer
grain formation and lower yields. Wheat is mainly a rabi (winter) season crop in India.
Hence, statement (d) is wrong.
Soil requirement:
Wheat is grown in a variety of soils of India. Soils with a clay loam or loam texture, good
structure and moderate water holding capacity are ideal for wheat cultivation. Care should
be taken to avoid very porous and excessively drained soils. Soil should be neutral in its
reaction. Heavy soil with good drainage are suitable for wheat cultivation under dry
conditions. These soils absorb and retain rain water well. Heavy soils with poor structure
and poor drainage are not suitable as wheat is sensitive to water logging. Wheat can be
successfully grown on lighter soils provided their water and nutrient holding capacity are
improved.
a) In check method of irrigation, the field is divided into smaller compartments and
water is admitted to each in turn
b) In free flooding irrigation, water is admitted at one corner of a field and is allowed to
spread over the entire area
c) In furrow irrigation water is admitted between the rows of plants in the field
d) None of these.
Gross command area (GCA): This is defined as total area that can be irrigated by a canal
system on the perception that unlimited quantity of water is available.
It is the total area that may theoretically be served by the irrigation system. But this may
include inhibited areas, roads, ponds, uncultivable areas etc which would not be irrigated.
Culturable command area (CCA): This is the actually irrigated area within the GCA.
Cultivable Command Area is that part of Gross Command Area, which is fit for cultivating
crops. So, cultivable area excludes forest and barren land from the Gross Command Area.
What is left is uncultivable area.
Q.9) The prerequisite condition for the formation of artesian well are -
1. Layer of permeable rock lying between two impermeable rock layers so that water
does not get escape.
2. The permeable rock should be exposed at the ground surface, so that rock can soak
rainwater.
3. Structure of rock strata must be synclinal.
a) 1 and 2 only
b) 2 and 3 only
c) 1 and 3 only
d) 1, 2 and 3
IASbabas 60 Day Plan Day 27 (Geography) 2017
The geologic conditions necessary for an artesian well are an inclined aquifer sandwiched
between impervious rock layers above and below that trap water in it. Water enters the
exposed edge of the aquifer at a high elevation and percolates downward through
interconnected pore spaces.
Q.10) With reference to the various multipurpose projects in India, which one among the
following statements is not correct?
Mayurakshi Major Irrigation Project does not involve Odisha. Mayurakshi project is an
irrigation project on the river Mayurakshi in West Bengal.
Q.11) The Tank irrigation is practised mainly in the peninsular region due to which of the
following reasons?
1. The undulating relief and hard rocks make it difficult to dig canals and wells in
peninsular region.
2. There is little percolation of rainwater due to hard rock structure and ground water
is not available in large quantity.
3. The clustered nature of population and agricultural fields also favours tank irrigation
there.
4. There are several streams which become torrential during rainy season and way to
make best use of this water is to impound it by constructing bunds and building
tanks.
a) 1, 2 and 3 only
b) 1, 2 and 4 only
c) 2, 3 and 4 only
d) 1, 2, 3 and 4
IASbabas 60 Day Plan Day 27 (Geography) 2017
Mostly prevalent in uneven and relatively rocky plateau of peninsular India, tanks are a
popular method of irrigation in the states of T.N., Karnataka, Andhra Pradesh and
Maharashtra. The scattered nature (not clustered nature) of population and agricultural
fields also favours tank irrigation.
Q.12) Consider the following pairs and select the correct answer:
a) 3 and 4 only
b) 1 and 3 only
c) 1, 3 and 4 only
d) All of the above
Gobind Vallabh Pant Sagar is a man-made lake situated in southern region of Sonebhadra
named after Bharat Ratna Pandit Govind Ballabh Pant.
Ranjit Sagar Dam - The Ranjit Sagar Dam, also known as the Thein Dam, is part of a
hydroelectric project constructed by the Government of Punjab on the Ravi River in the
state of Punjab.
The Ranapratap Sagar Dam is built on the Chambal River at Rawatbhata in Rajasthan in
India.
a) Cropping pattern : : proportion of area under different crops at a given point of time.
b) Crop concentration : : variation in the density of any crop in a region at a given point
of time.
IASbabas 60 Day Plan Day 27 (Geography) 2017
c) Cropping intensity : : raising of a number of crops during one agriculture year by
expanding the net area under cultivation.
d) Crop combination : : quantum or diversity of crops entering a region in a given
period.
Cropping intensity refers to the raising of a number of crops from the same field during one
agriculture year. This also implies higher productivity per unit of arable land during one
agricultural year.
Q.14) Match List I with List II and select the correct answer using the code given below the
Lists:
List I List II
(Textile industry) (Place)
Code:
ABCD
a) 3412
b) 2143
c) 2413
d) 3142
Self-explanatory.
Q.15) Which of the following methods is/are suitable for soil conservation in hilly region?
Shifting cultivation or slash and burn techniques - clearance of forest causes deforestation
which accelerates soil erosion.
Burning of Jhum land can be considered as one of the worst impact on climate because it
gives scope in high rate of soil erosion and soil erosion structural stability by runoff and
winds. The availability of water in the soil for the crop production also decreases which is
vulnerable to climatic variation.
Q.16) Consider the following statements about Public Financial Management System
(PFMS)
a) Only 1
b) Only 2
c) Both 1 and 2
d) Neither 1 nor 2
The Public Financial Management System (PFMS),earlier known as Central Plan Schemes
Monitoring System (CPSMS), is a web-based online software application developed and
implemented by the Office of Controller General of Accounts (CGA). PFMS was initially
started during 2009 as a Central Sector Scheme of Planning Commission with the objective
of tracking funds released under all Plan schemes of GoI, and real time reporting of
expenditure at all levels of Programme implementation. Subsequently in the year 2013, the
scope was enlarged to cover direct payment to beneficiaries under both Plan and non-Plan
Schemes. The latest enhancement in the functionalities of PFMS commenced in late 2014,
wherein it has been envisaged that digitization of accounts shall be achieved through PFMS
and the additional functionalities would be built into PFMS in different stages. Beginning
IASbabas 60 Day Plan Day 27 (Geography) 2017
with Pay & Accounts Offices payments, the O/o CGA did further value addition by proposing
to bring in more financial activities of the Government of India in the ambit of the project.
The primary objective of PFMS is to facilitate sound Public Financial Management System
for Government of India (GoI) by establishing an efficient fund flow system as well as a
payment cum accounting network. PFMS provides various stakeholders with a real time,
reliable and meaningful management information system and an effective decision support
system, as part of the Digital India initiative of GoI
The enhanced application is envisaged to cater to all Plan and Non Plan payments of GoI, all
tax and non-Tax receipts and also functions such as a comprehensive HRMIS and self-
contained pension as well as GPF modules. Futuristically, all the existing standalone systems
currently catering to various functions in Government of India will be subsumed in PFMS.
The biggest strength of PFMS is its integration with the Core banking system in the Country.
As a result, PFMS has the unique capability to push online payments to almost every
beneficiary/vendor. At present, PFMS interface is having interface in addition to the Core
Banking System (CBS) of all Public Sector Banks, Regional Rural Banks, major private sector
banks, Reserve Bank of India, India post and Cooperative Banks.
Source: http://pib.nic.in/newsite/PrintRelease.aspx?relid=133619
Q.17) Consider the following statements about Pradhan Mantri Fasal Bima Yojana
a) 1 and 2
b) Only 2
c) 1 and 3
d) All of the above
It is open to all farmers but NOT mandatory to anyone. It is optional for loanee as well as
non-loanee farmers. It has so far lowest premium.
IASbabas 60 Day Plan Day 27 (Geography) 2017
The existing premium rates vary between 2.5% and 3.5% for kharif crops and 1.5% for rabi
cropsbut the coverage was capped, meaning farmers could, at best, recover a fraction of
their losses.
The farmers premium has been kept at a maximum of 2 per cent for food grains and up to 5
per cent for annual commercial horticulture crops. For rabi crops, it is 1.5%. The balance
premium will be paid by the government to provide full insured amount to the farmers.
Since there is no upper cap on government subsidy, even if the balance premium is 90
percent, the government will bear it.
This scheme provides full coverage of insurance. While NAIS had full coverage, it was
capped in the modified-NAIS scheme. It also covers the localized risks such as hailstorm,
landslide, inundation etc. Earlier schemes did not cover inundation.
It provides post-harvest coverage. The NAIS did not cover while the modified NAIS covered
only coastal regions.
Ajay Tyagi committee constituted under the Department of Economic Affairs recommended
the creation of statutory body called Financial Data Management Centre
Govt proposed setting up of financial data management centre for managing the repository
of financial regulatory data to ensure stability in the economy.
FDMC Functions -
To establish, operate and maintain the financial system database, collect financial
regulatory data and provide access to it
Standardize data from all financial sector regulators in a single database
To provide analytical support to the FSDC on issues relating to financial stability
IASbabas 60 Day Plan Day 27 (Geography) 2017
Source: http://www.livemint.com/Politics/b6HeuDfIyYcf2hFrkM6EVP/Govt-proposes-
setting-up-financial-data-management-centre.html
a) Only 1
b) Only 2
c) Both 1 and 2
d) Neither 1 nor 2
The Convention is the first international treaty on crimes committed via the Internet and
other computer networks, dealing particularly with infringements of copyright, computer-
related fraud, child pornography, hate crimes, and violations of network security. It also
contains a series of powers and procedures such as the search of computer networks and
lawful interception.
The Convention was signed by Canada, Japan, the United States, and South Africa on 23
November 2001, in Budapest. As of July 2016, the nonCouncil of Europe states that have
ratified the treaty are Australia, Canada, Dominican Republic, Israel, Japan, Mauritius,
Panama, Sri Lanka, and the United States.
Central Government has established Indian Cyber Crime Coordination Center (I4C) at
National Level to deal with all types of cybercrime. The I4C can be utilized for investigation
of Cyber-Crime including Child Pornography and Online Abuse. One of the priorities of I4C
will be to thwart attempts by international cyber gangs to penetrate Indian Networks and
hack them. I4C will act a nodal point in fight against Cyber Crime and also as Early Warning
System for Law Enforcement Agencies. It will also set up a platform for victims to lodge
IASbabas 60 Day Plan Day 27 (Geography) 2017
Cyber Crime complaints. I4C will also provide all necessary assistance to CBI and State police
on Cyber Crime related issue.
a) Only 1
b) Only 2
c) Both 1 and 2
d) Neither 1 nor 2
The Agni V is an intercontinental ballistic missile with a range of 5,500 to 5,800 km.
It has been developed by the Defence Research and Development Organisation under the
Integrated Guided Missile Development Program.
It is the fifth variant in the series of medium to long range Agni missiles. Agni I, II, III have
already been inducted for military use.
The successful induction of Agni V will give India long-range strike capability.
The missile has previously been tested in 2012, 2013 and 2015.
Once the Agni-V is inducted, India will join the super exclusive club of countries with ICBMs
(missiles with a range of over 5,000-5,500km) alongside the US, Russia, China, France and
the UK.
Source: http://www.thehindu.com/news/national/Agni-V-successfully-soars-yet-
again/article16944774.ece
a) India
IASbabas 60 Day Plan Day 27 (Geography) 2017
b) China
c) Bangladesh
d) None of the above
It is endemic to China but now found in India. It could be through spores transported in
ballast water or through natural dispersion.
Source: http://www.thehindu.com/sci-tech/science/Invasive-algae-species-found-in-
Tamil-Nadu/article16789620.ece
1. One China Policy is the diplomatic acknowledgement of the Chinese position that
there is only one China in the world and Taiwan is a part of that China
2. Hong Kong was returned to China in 1997 under One Country, Two Systems
a) Only 1
b) Only 2
c) Both 1 and 2
d) Neither 1 nor 2
"One country, two systems" is a constitutional principle formulated by Deng Xiaoping, the
Paramount Leader of the People's Republic of China (PRC), for the reunification of China
during the early 1980s. He suggested that there would be only one China, but distinct
Chinese regions such as Hong Kong and Macau could retain their own capitalist economic
and political systems, while the rest of China uses the socialist system. Under the principle,
each of the two regions could continue to have its own political system, legal, economic and
financial affairs, including external relations with foreign countries.
Deng Xiaoping proposed to apply the principle to Hong Kong in the negotiation with the
British Prime Minister, Margaret Thatcher over the future of Hong Kong when the lease of
the New Territories (including New Kowloon) of Hong Kong to the United Kingdom was to
expire in 1997. The same principle was proposed in talks with Portugal about Macau.
IASbabas 60 Day Plan Day 27 (Geography) 2017
The principle is that, upon reunification, despite the practice of socialism in mainland China,
both Hong Kong and Macau, which were colonies of the UK and Portugal respectively, can
retain their established system under a high degree of autonomy for at least 50 years after
reunification. What will happen after 2047 (Hong Kong) and 2049 (Macau) has never been
publicly stated.
Source: http://www.business-standard.com/article/pti-stories/china-denies-money-
diplomacy-to-enforce-one-china-policy-116122200812_1.htm
Q.23) The Department of Telecom (DoT) recently launched a portal called Tarang for
a) Mobile radiation
b) Spectrum management
c) INSAT Mobile Satellite Service (INSATMSS)
d) None of the above
The Department of Telecom (DoT) is set to launch a portal Tarang with data on all
mobile towers and transmitters across the country in a month that will allow a user to check
the radiation compliance status.
Source: http://www.thehindu.com/business/Industry/DoT-to-unveil-portal-on-mobile-
radiation-in-a-month/article16895605.ece
Q.24) Which of the following countries forms the part of Greater Mekong Sub region
(GMS)?
a) Vietnam
b) Cambodia
c) Myanmar
d) All of the above
The Greater Mekong Subregion (GMS) is a natural economic area bound together by the
Mekong River, covering 2.6 million square kilometers and a combined population of around
326 million.
IASbabas 60 Day Plan Day 27 (Geography) 2017
The GMS countries are Cambodia, the People's Republic of China (PRC, specifically Yunnan
Province and Guangxi Zhuang Autonomous Region), Lao People's Democratic Republic (Lao
PDR), Myanmar, Thailand, and Viet Nam.
Source:
http://www.livemint.com/Science/CG2CoO6hRmOLZ5EWwcOEQM/Rainbowheaded-
snake-among-163-species-discovered-in-Greate.html
Q.25) Keralas State Board for Wildlife sought Union Governments approval for using deer
antlers in Ayruveda drugs. Consider the following statements in this regard.
1. Deer antlers are mass of solid bone and are shed on regular intervals and regrow
again
2. The Wildlife (Protection) Act, 1972, has included antler in the definition of wildlife
trophy
3. All the three deer varieties found in Kerala, including spotted deer, sambar, and
barking deer, shed their antlers annually and is listed by IUCN as Least Concern
a) 1 and 2
b) 2 and 3
c) 1 and 3
d) All of the above
Antlers
The Wildlife (Protection) Act, 1972, has included antler in the definition of wildlife
trophy.
A wildlife trophy is defined as the whole or any part of any captive animal or wild
animal.
IASbabas 60 Day Plan Day 27 (Geography) 2017
Wildlife and wildlife trophies are considered as owned by the government. The Act
also prescribes imprisonment up to three years and fine of RS. 25,000 for offences
involving wildlife trophies.
All the three deer varieties found in Kerala, including spotted deer (Least Concern), sambar
(Vulnerable), and barking deer (Least Concen), shed their antlers annually.
Source: http://www.thehindu.com/news/national/kerala/Antlers-could-end-up-in-
medicines/article16952009.ece
IASbabas 60 Day Plan Day 28 (Geography) 2017
DAY 28
SUBJECT INDIAN GEOGRAPHY
TOPICS:
FOCUS ON:
Indian Geography: Forests (or Natural Vegetation), Human Geography, Wildlife and
Conservation of flora and fauna, Forests (or Natural Vegetation)
Tropical Evergreen forests, Deciduous, Hill forests Thorn, Littoral and Swamp
forests
Temperature, Rainfall conducive for the growth of these Forests
Regions they occur/grow
Types of trees grown (general awareness)
Economic utilities of these forests
Example: Deciduous Forests found in areas of rainfall 150-200 cm;They are found
majorly in Peninsular India; Trees like teak, sal, sandalwood, rosewood grow here.
They are less dense unlike evergreen forests and are a source of good quality
timber.
Human Geography
FOCUS ON:
Major Tribal groups and States they belong to; their Skills; Festivals they celebrate.
Demographic dividend
Wildlife and Conservation of flora and fauna: This topic overlaps with Environment
PRELIMS MCQs:
Q.1) Identify the state in which when you travel from East to West you tend to find this
pattern of Natural vegetation:
Mangrove Forest Tropical evergreen Forest Tropical Moist deciduous Tropical Dry
deciduous.
a) Odisha
b) Tamil Nadu
c) Kerala
d) West Bengal
IASbabas 60 Day Plan Day 28 (Geography) 2017
Rainfall tend to decrease from East to West of Odisha as Utkal branch of Indian monsoon
from bay of Bengal tend to lose moisture as it heads interior of the state , thus we have
Mangrove Forest->Tropical evergreen Forest->Tropical Moist deciduous->Tropical Dry
deciduous type of natural vegetation..
Q.2) Which of the statements given below is/are correct about Indian Wild Orange or
Citrus Indica, which was recently in news?
a) 1 only
b) 1, 2 and 4 only
c) 2 and 3 only
d) 2, 3 and 4 only
A rare, endemic and endangered species of wild citrus fruit having medicinal properties
which was so far reported only from Meghalaya, has now been found in a remote village of
Manipur.
A team of field biologists and researchers of Centre for Conservation of Nature and
Cultivation of Science (CCNCS), Manipur, reported the species 'citrus indica' commonly
known as Indian Wild Orange from Dailong village in Tamenglong district earlier this month.
"The species is considered to be the most primitive and ancestors of all cultivated citrus
fruits in the world and endemic to north-east India. Earlier the species was reported only
from Nokrek Biosphere Reserve in the Garo Hills of Meghalaya.
IASbabas 60 Day Plan Day 28 (Geography) 2017
Q.3) Compared with other forests, Coniferous forests have little undergrowth. It can be
attributed to which of the following reasons
a) 1, 4 and 5
b) 1, 3, 4 and 5
c) 1, 2 and 3
d) All the above
The lack of leaf litter and the prevailing cold conditions which inhibit the natural decay of
the forest's pine needle carpet - material that is slow to decompose in any case - results in
only a thin underlying layer of topsoil and little or no undergrowth in Coniferous forests.
1. The trees can grow to over 40 metres in the effort to get sunlight
2. These forests have evergreen appearance due to continuous growing season
3. The forest floor is damp and dark. There is little undergrowth because the sunlight
cannot reach ground level.
All are the characteristic features of Equatorial forests. Hence (a) is the correct answer.
IASbabas 60 Day Plan Day 28 (Geography) 2017
Q.5) Identify the state which has Tropical dry evergreen forest as unique type of natural
vegetation?
a) Odisha
b) Tamil Nadu
c) Kerala
d) West Bengal
Tropical dry evergreen forests (TDEFs) occur as patches along the Coromandel coast of
peninsular India. TDEF is the indigenous forest of the coastal seaboard of South East India.
The East Deccan dry evergreen forests are an ecoregion of southeastern India. The
ecoregion includes the coastal region behind the Coromandel Coast on the Bay of Bengal,
between the Eastern Ghats and the sea. It covers eastern Tamil Nadu, part of Puducherry
and south eastern Andhra Pradesh.
Towards the end of September , the low pressure in the northwest begins to disintegrate
and eventually shifts the equitorial region. The cyclonic conditions are replace by anti-
cyclonic ones. As a result, winds start blowing away from the northern region.
The retreating monsoon over Bay of Bengal pick up moisture on their way, which is dropped
over Tamil Nadu and Parts of Karnataka during October November.
Unlike most of the worlds tropical and subtropical dry broadleaf forests, whose trees tend
to lose their leaves during the dry season to conserve moisture, the East Deccan dry
evergreen forests retain their leaves year round.
Tropical dry evergreen forest occur along the coast of Tamil Nadu.
Q.6) The following options lists the tribes with their respective climatic region.
Q.7) Match the following Wildlife Sanctuaries with its associated states
A-B- C-D
a) 1-2- 3-4
b) 2-1- 3-4
c) 4-3- 1-2
d) 3-1- 4-2
a) 1, 4 and 5 only
b) 2 and 3 only
c) 1, 2 and 3 only
d) 1, 3 and 4 only
Kharif Crops
The Kharif crop is the summer crop or monsoon crop in India.
Sown in : beginning of the first rains in July
Harvested : during the rainy (monsoon) season , between April and October
Major Kharif crops: rice, maize, sorghum, pearl millet/bajra, finger millet/ragi
(cereals), arhar (pulses), soyabean, groundnut (oilseeds), cotton etc.
Rabi Crops
The Rabi crop is the spring harvest or winter crop in India .
Sown in: Winter ( Last of October)
Harvested in : Spring (March, April)
Major Rabi crops: wheat, barley, oats (cereals), chickpea/gram (pulses), linseed,
mustard (oilseeds) etc.
Zaid Crop
This crop is grown in some parts of country during March to June.
Major Zaid crops in India are Watermelon, Muskmelon, , bitter gourd, pumpkin,
cucumber etc.
1. These kind of Forests found in areas with Moderate rainfall of 100 to 200 cm per
annum
2. Mean annual temperature of about 27 degree C
IASbabas 60 Day Plan Day 28 (Geography) 2017
3. Average relative humidity of 60 to 70 %
4. This type of forest is found in some parts of Odisha and West Bengal
a) Mediterranean Shrublands
b) Tropical wet evergreen Forests
c) Tropical dry evergreen forests
d) Tropical Moist deciduous Forests
1. Latitude
2. Altitude
3. Continentality
4. Winds and Ocean currents
5. Distance from Sea
6. Aspect
a) 1, 2 and 3 only
b) 2,3 and 4 only
c) 1,2,3,4 and 5 only
d) All
Latitude
IASbabas 60 Day Plan Day 28 (Geography) 2017
Latitude is the distance (measured in degrees) to the north and south of the Equator.
The further away the location is from the equator, the smaller the angle at which the
sun's rays strike the earth.
Away from the equator, the angle of incidence of the sun's ray is less than90deg,
thus solar radiation is spread over a larger area. Therefore less intense heating takes
place, causing temperature to become lower.
The smaller the angle of incidence of the sun's rays, the greater the distance
through the atmosphere the rays have to pass. More heat is absorbed by the
atmosphere or reflected back into outer space, which results in less heat reaching
the Earth's surface.
Temperature thus diminishes from equatorial regions to poles
Altitude
Wind is the horizontal movement of air from a region of high pressure to a region of
low pressure.
Ocean currents modify the temperature of the winds, which blow over them.
Prevailing winds blowing inland from the sea help to moderate the temperature of
adjacent land masses.
A wind, which has blown over a cold ocean current, will have its temperature
lowered. Cold ocean currents originate from the Poles and lower the temperature of
the winds, thus lowering the temperature of the coastal countries they pass through.
Amount of rain in those coastal areas decreases.
A wind, which has blown over a warm ocean current, will have its temperature
raised. Warm ocean currents carry water from the tropics to the poles and raise the
temperature of the winds, thus warming the coasts they pass through. Amount of
rain in those coastal areas increases.
Aspect
IASbabas 60 Day Plan Day 28 (Geography) 2017
Aspect refers to the position of a mountain slope in relation to the direction of the
Sun's rays.
The effect of aspect on temperature is more marked in temperate latitudes. In
tropical latitudes, the angle of the mid day sun is high. The effect of aspect on
temperature is less noticeable.
In the northern hemisphere, the south-facing slopes are warmer than the north-
facing slopes. This is because south-facing slopes receive more direct solar energy
and are protected from polar cold winds.
The sun's rays reach them at a steeper angle, so the heat is distributed over a smaller
area.
In the southern hemisphere, the north-facing slopes are warmer than the south-
facing slopes. This is because north-facing slopes receive more direct solar energy
and are protected from polar cold winds. The sun's rays reach them at a steeper
angle, so heat is distributed over a smaller area.
An inland location is further away from the sea, while a coastal location is close to
the sea.
The land heats up and cools down more quickly than the sea.
During the summer, the air over the land heats up quickly, while the air over the sea
remains relatively cool because the sea gains heat much slower than the land.
Onshore winds blow cooler air towards the land, lowering the summer temperatures
of the areas along the coast.
During winter, the air over the sea remains relatively warm compared to the air over
the land because the sea loses heat slower than the land. The onshore winds blow
warmer air towards the land, raising the winter temperatures of the coastal areas.
As a result of the maritime effect, the coastal areas are likely to experience cooler
summers and warmer winters. Hence, there is a smaller difference between summer
and winter temperatures.
As a result of the continental effect, the sea does not influence the temperatures of
places inland. Instead, these places are likely to experience warmer summers and
colder winters and hence have a large difference between summer and winter
temperatures.
1. It is a transitional type of climate found between the equatorial forests and trade
wind hot deserts.
2. This type of climate is characterized by an alternate hot, rainy season and cool, dry
season.
3. Extreme diurnal range of temperature is another characteristic of this type of climate
4. This landscape is typified by tall grass and short trees
It is a transitional type of climate found between the equatorial forests and trade
wind hot deserts.
This type of climate is characterized by an alternate hot, rainy season and cool, dry
season.
Extreme diurnal range of temperature is another characteristic of this type of climate
This landscape is typified by tall grass and short trees
a) 1 and 2 only
b) 2 and 3 only
c) 1 only
d) 1, 2 and 3
Q.14) Consider the following statements with regard to red panda, classified as
Endangered by the IUCN, which was recently in news:
1. Red panda is a mammal native to the eastern Himalayas and southwestern China.
2. The red panda is endemic to the temperate forests of the Himalayas.
3. It is an herbivorous animal, feeds mainly on bamboo.
4. It is a solitary animal, mainly active from dusk to dawn, and is largely sedentary
during the day.
The red panda also called lesser panda, red bear-cat, and red cat-bear, is a mammal native
to the eastern Himalayas and southwestern China. The red panda is endemic to the
temperate forests of the Himalayas.
It is arboreal, feeds mainly on bamboo, but also eats eggs, birds, and insects. It is a solitary
animal, mainly active from dusk to dawn, and is largely sedentary during the day.
Q.15) Identify the following state/States with such unique pattern of climatic regions:
When we travel from West to East of this particular State, we tend to encounter, Tropical
wet Tropical wet and dry Semi Arid pattern of climatic regions.
a) Karnataka
b) Maharashtra
c) Both a and b
d) Kerala
Q.16) Gaofen project a network of observation satellites orbiting the Earth to provide
global surveillance capabilities is a project of which of the following countries?
a) Russia
b) China
c) Japan
d) Germany
China has launched its first fully- owned overseas satellite ground station near the North
Pole which could enable Beijing to collect satellite data anywhere on the Earth at a speed it
said was the fastest in the world.
The facility, located in Sweden about 200 kms north of the Arctic Circle, would allow China
to collect satellite data anywhere on Earth at speeds that were more than twice as fast as
before.
The new facility would play an important role in Chinas Gaofen project a network of
observation satellites orbiting the Earth to provide global surveillance capabilities which
was due to be completed in 2020.
a) 1 and 2
b) Only 2
c) 1, 2 and 3
d) 2 and 3
Ministry of AYUSH
Organize Swasthya Parikshan Camps, Swasthya Rakshan OPDs and Health and Hygiene
awareness programme
Provide medical aid and incidental support in the adopted villages and colonies.
IASbabas 60 Day Plan Day 28 (Geography) 2017
Document demographic information, hygiene conditions, food habits, seasons, lifestyle etc.,
incidence and prevalence of disease and their relation to the incidence of disease.
Assess health status and propagation of Ayurvedic concept of pathya-apathya and extension
of health care services.
Source: http://pib.nic.in/newsite/PrintRelease.aspx?relid=155555
Q.18) Consider the following statements about Central Sector Scheme for Rehabilitation
of Bonded Labourers, 2016
a) Only 1
b) Only 2
c) Both 1 and 2
d) Neither 1 nor 2
Government has revamped the Central Sponsored Scheme for Rehabilitation of Bonded
Labourers known as Central Sector Scheme for Rehabilitation of Bonded Labourers, 2016.
Bonded Labour Rehabilitation Fund: it will be created at District level by each State with a
permanent corpus of at least Rs. 10 lakh at the disposal of the District Magistrate for
extending immediate help to the released bonded labourers. Funding source: Labour and
employment ministry will release the fund to District National Child Labour Project Society
which in turn will release the fund to implementing agencies including the district
administration. The benefits prescribed above shall be, in addition to, other benefits for
which the beneficiary is entitled to under other such schemes.
Source: http://pib.nic.in/newsite/PrintRelease.aspx?relid=154895
IASbabas 60 Day Plan Day 28 (Geography) 2017
a) Only 1
b) Only 2
c) Both 1 and 2
d) Neither 1 nor 2
The Nai Roshni scheme is a leadership development programme launched by the Ministry of
Minority Affairs in 2012-13.
The scheme has been implemented through NGOs, civil societies and government
institutions.
The scheme also includes Educational Programmes, Health and Hygiene, Swacch Bharat,
Financial Literacy, Life Skills, Legal Rights of Women, Digital Literacy, Advocacy for social and
behavioural change. Minority women can apply for training online as well.
Source: http://pib.nic.in/newsite/PrintRelease.aspx?relid=155365
Kalamkari literally means, Kalam - pen & kari - work, i.e., art work done using a pen.
It is an ancient style of hand painting done on cotton or silk fabric with a tamarind pen using
natural dyes.
The traditional style of Kalamkari flourished in Kalahasti (80 miles north of Chennai) and
Masulipatnam (200 miles east of Hyderabad).
The paintings at that time used to depict Hindu Deities and scenes from Hindu mythology.
Masulipatnam being a Muslim region, the weavers were involved in block printing art.
Source: http://pib.nic.in/newsite/PrintRelease.aspx?relid=155255
India was ranked seventh in the list of countries most impacted by terrorism in 2015,
according to the Global Terrorism Index (GTI), 2016, released by the Institute for Economics
& Peace, a think-tank based in Sydney, Australia.
India is one of six Asian countries ranked in the top ten nations most impacted by terrorism.
Source: http://www.business-standard.com/article/current-affairs/india-ranked-7-
among-countries-hit-most-by-terrorism-116121100014_1.html
Q.22) Which of the following countries signed a treaty to exchange pieces of land along
the Meuse river border?
Belgium and the Netherlands signed a treaty to exchange pieces of land along the Meuse
river border.
The border treaty symbolizes the excellent state of the Belgian-Dutch relations.
This is the second border adjustment between the Netherlands and Belgium in 175 years.
The current border was established in 1843 and followed the deepest point of the Meuse
river. Adjustment was necessary because in 1961 the winding banks of the river were
rectified for navigation purposes. As a result, parts of Belgian territory were attached to the
Dutch riverside and vice versa.
Source: http://www.thehindu.com/news/international/Belgium-Netherlands-to-swap-
land-alter-border/article16729159.ece
Open market operations are conducted by the RBI by way of sale or purchase of
government securities (g-secs) to adjust money supply conditions. The central bank sells g-
secs to suck out liquidity from the system and buys back g-secs to infuse liquidity into the
system. These operations are often conducted on a day-to-day basis in a manner that
balances inflation while helping banks continue to lend. The RBI uses OMO along with other
monetary policy tools such as repo rate, cash reserve ratio and statutory liquidity ratio to
adjust the quantum and price of money in the system.
Source: http://www.dnaindia.com/money/report-all-eyes-on-remonetization-in-rbi-
policy-review-today-2280481
IASbabas 60 Day Plan Day 28 (Geography) 2017
Q.24) Consider the following statements regarding Sovereign Gold Bonds (SGBs):
a) 2, 3 and 4
b) 3 and 4
c) 4 only
d) All of the above
SGBs are government securities denominated in grams of gold. They are substitutes for
holding physical gold. Investors have to pay the issue price in cash and the bonds will be
redeemed in cash on maturity. The Bond is issued by Reserve Bank on behalf of Government
of India.
The quantity of gold for which the investor pays is protected, since he receives the ongoing
market price at the time of redemption/ premature redemption. The SGB offers a superior
alternative to holding gold in physical form. The risks and costs of storage are eliminated.
Investors are assured of the market value of gold at the time of maturity and periodical
interest. SGB is free from issues like making charges and purity in the case of gold in
jewellery form. The bonds are held in the books of the RBI or in demat form eliminating risk
of loss of scrip etc.
There may be a risk of capital loss if the market price of gold declines. However, the investor
does not lose in terms of the units of gold which he has paid for.
Q.25) Which of the following was merged with Securities and Exchange Board of India
(SEBI)?
The Forward Markets Commission (FMC) is the chief regulator of commodity futures
markets in India.
On 28 September 2015 the FMC was merged with the Securities and Exchange Board of
India (SEBI).
IASbabas Prelims 60 Day Plan Day 29 2017
(Science & Technology)
Q.1) Consider the following statements regarding Nuclear Recycle Board in India:
1. It is an entity under DRDO and works under the purview of Defence Ministry.
2. It is responsible for the design, construction and operation of nuclear recycle plants
involving reprocessing and waste management.
a) 1 only
b) 2 only
c) Both 1 and 2
d) Neither 1 nor 2
The Nuclear Recycle Board was formed with the approval of the Cabinet during September
2009. Nuclear Recycle Board functions as an entity within Bhabha Atomic Research Centre
(BARC) for execution of projects and operation and maintenance of nuclear recycle facilities
in the back end of Pressurised Heavy Water Reactor (PHWR) fuel cycle. The Board operates
under the purview of BARC Safety Council.
Nuclear Recycle Board is responsible for the design, construction and operation of nuclear
recycle plants involving reprocessing and waste management.
Q.2) Bhabha Atomic Research Center is working on the development of the Advanced
Heavy Water Reactor (AHWR). Which of the following statements are correct about
AHWR?
a) 1 and 2
b) 2 and 3
c) 1 and 3
d) All of the above
IASbabas Prelims 60 Day Plan Day 29 2017
(Science & Technology)
Q.2) Solution (b)
AHWR300-LEU is a 300 MWe, vertical, pressure tube type, boiling light water cooled, and
heavy water moderated reactor. The reactor incorporates a number of passive safety
features and is associated with a fuel cycle having reduced environmental impact.
AHWR300-LEU possesses several features, which are likely to reduce its capital and
operating costs.
It is a demonstration reactor which will pave way for third stage of Indias Nuclear Program
which will use Thorium Uranium cycle.
Q.3) Bhartiya Nabhikiya Vidhyut Nigam Ltd. (BHAVANI), a PSU of Department of Atomic
Energy, is constructing a Prototype Fast Breeder Reactor (PFBR) in Kalpakkam. Which of
the following statements are correct about FBRs?
a) 1 only
b) 2 only
c) Both 1 and 2
d) Neither 1 nor 2
A breeder reactor is a nuclear reactor that generates more fissile material than it consumes.
These devices achieve this because their neutron economy is high enough to breed more
fissile fuel than they use from fertile material, such as uranium-238 or thorium-232.
The Prototype Fast Breeder Reactor (PFBR) is a 500 MWe fast breeder nuclear reactor
presently being constructed at the Madras Atomic Power Station in Kalpakkam, India. The
Indira Gandhi Centre for Atomic Research (IGCAR) is responsible for the design of this
reactor. As of 2007 the reactor was expected to begin functioning in 2010 but now it is not
expected to achieve first criticality before October 2017. The Kalpakkam PFBR is using
uranium-238 not thorium, to breed new fissile material, in a sodium-cooled fast reactor
IASbabas Prelims 60 Day Plan Day 29 2017
(Science & Technology)
design. The power island of this project is being engineered by Bharat Heavy Electricals
Limited, largest power equipment utility of India.
1. India is the first country to sign a Civil Nuclear deal with Japan.
2. The termination clause in India Japan Nuclear deal is binding on India.
a) 1 only
b) 2 only
c) Both 1 and 2
d) Neither 1 nor 2
India is the first non NPT country (and not the first country) to sign a civil Nuclear deal with
Japan.
The termination clause in non binding on India. According to it, Japan can terminate the
deal, with immediate effect, if India does a nuclear test.
Q.5) Hall of nuclear power Atoms serving the nation is a permanent exhibition which
covers various aspects of Nuclear Energy. In which of the following cities has it been
established?
a) Kolkata
b) Kudankulam
c) New Delhi
d) Jaitapur
In the diamond jubilee year of the Bhabha Atomic Research Centre (BARC), a 'hall of nuclear
power' has been set up in the capital with an aim to dispel "misgivings" related to atomic
energy as well as to spread awareness about it.
IASbabas Prelims 60 Day Plan Day 29 2017
(Science & Technology)
It covers various aspects of Nuclear Energy with prime focus on nuclear power plant safety
and applications for human welfare, which include nuclear medicine, food irradiation, as
well as several other day to day applications.
Q.6) A multipurpose fast research reactor project, named as MBIR, is coming up in Russia.
Which of the following statements are correct about MBIR?
a) 1 only
b) 2 only
c) Both 1 and 2
d) Neither 1 nor 2
The multipurpose fast research reactor project, known by the Russian acronym MBIR, is
coming up as the International Research Center in Dimitrovgrad located in the Ulyanovsk
region.
The purpose of the programme is the creation of a new technological platform for nuclear
energy, which will be based on the closed fuel cycle with fast neutron reactors
A fast neutron reactor, also known simply as a fast reactor, is a type in which the nuclear
fission chain reaction is sustained by fast neutrons. Such a reactor needs no neutron
moderator like normal water, which serves such a purpose in thermal reactors.
A moderator is a material used in a nuclear reactor to slow down the neutrons produced
from fission. By slowing the neutrons down the probability of a neutron interacting with
Uranium-235 nuclei is greatly increased thereby maintaining the chain reaction.
Nuclei with low mass numbers are most effective for this purpose, so the moderator is
always a low-mass-number material. In a fast reactor there is no moderator, only fuel and
coolant. The moderation of neutrons is undesirable in fast reactors. Commonly used
moderators include regular (light) water (roughly 75% of the worlds reactors), solid
graphite (20% of reactors) and heavy water (5% of reactors). Beryllium and beryllium oxide
(BeO) have been used occasionally, but they are very costly.
Q.8) Which part of the thermal power plant does the same function as that of the reactor
in Nuclear power plant?
a) Turbine
b) Boiler
c) Furnace
d) Cooling tubes
The role of the reactor is to generate the heat. This heat is absorbed by a coolant (usually
water) which heats up to rotate the turbine.
The same function is performed by the furnace in a thermal power plant. Where coal or gas
is burnt to generate heat, which is used to generate steam in the boiler.
a) 1 and 2
b) 2 and 3
c) 1 and 3
d) All of the above
The name CERN is derived from the acronym for the French "Conseil Europen pour la
Recherche Nuclaire", or European Council for Nuclear Research.
In recognition of these substantial contributions, India was granted Observer status to the
CERN Council in 2002. Recently it has been upgraded to Associate member of CERN.
The 27-kilometre LHC is the world's largest particle accelerator. It collides protons or lead
ions at energies approaching the speed of light.
https://home.cern/about
a) 1 only
b) 2 only
c) Both 1 and 2
d) Neither 1 nor 2
Cobalt-60, 60. Co. , is a synthetic radioactive isotope of cobalt with a half-life of 5.2714
years. It is produced artificially in nuclear reactors. Deliberate industrial production depends
IASbabas Prelims 60 Day Plan Day 29 2017
(Science & Technology)
on neutron activation of bulk samples of the monoisotopic and mononuclidic cobalt isotope
59.
Cobalt therapy or cobalt-60 therapy is the medical use of gamma rays from the radioisotope
cobalt-60 to treat conditions such as cancer.
Q.11) Which of the following correctly explains the term Nuclear Transmutation?
a) Bonds are broken between the elements as they are hit by neutrons.
b) Kinetic energy of neutrons is converted into heat energy
c) Mass gets converted into energy
d) All of the above
The energy generated can be calculated with the help of Einsteins famous equation
E = MC2 , where
E = Energy generated
M = Mass converted
C = Speed of light
IASbabas Prelims 60 Day Plan Day 29 2017
(Science & Technology)
In case of nuclear fusion, two small nuclei are fused to form a heavier element.
Since radioactive materials are not playing any role, radioactive waste is not generated.
nuclear fusion is a reaction in which two or more atomic nuclei come close enough to form
one or more different atomic nuclei and subatomic particles (neutrons or protons). The
difference in mass between the products and reactants is manifested as the release of large
amounts of energy.
a) 1 only
b) 1 and 3
c) 2 and 3
d) All of the above
The ITER fusion reactor has been designed to produce 500 megawatts of output power for
around twenty minutes while needing 50 megawatts to operate.
The project is funded and run by seven member entitiesthe European Union, India, Japan,
China, Russia, South Korea, and the United States. The EU, as host party for the ITER
complex, is contributing about 45 percent of the cost, with the other six parties contributing
approximately 9 percent each.
Q.15) Jaitapur Nuclear Power Project is a proposed 9900 MW power project of Nuclear
Power Corporation of India (NPCIL). Consider the following statements regarding this:
1. If built, it would be the largest nuclear power generating station in the world by net
electrical power rating.
2. It is being built with the help of a Russian firm, Areva.
3. The complete farm will have ten Voda Voda nuclear reactors.
a) 1 only
b) 1 and 2
c) 2 and 3
d) All of the above
French state-controlled nuclear engineering firm Areva S.A. and Indian state-owned nuclear
operator Nuclear Power Corporation of India signed the agreement, valued about $9.3
billion.
a) 1 and 2
b) Only 2
c) 1 and 3
d) 2 and 3
The Union Ministry of Electronics and Information Technology (MeitY) has launched a TV
channel named DigiShala to promote cashless transactions. The channel was launched as
part of the Digidhan campaign which aims to spread awareness about digital transactions.
DigiShala is dedicated 24*7 and 365 days free-to-air TV channel to inform citizens about
digital payment ecosystem, benefits and processes. It is a satellite channel managed by
Doordarshan (DD). It will be broadcasted nationally on DD Free Dish DTH service.
Through it, citizens will be informed and educated about various digital payment options
through step by step demos of digital payments using e-wallets, UPI, USSD, Aadhaar-
enabled payment systems and cards. It will impart information and education, especially in
rural and semi-urban areas related to Digital Payment ecosystem, its tools, processes and
IASbabas Prelims 60 Day Plan Day 29 2017
(Science & Technology)
benefits. The services on the channel initially will be available in Hindi and English and later
in local languages as well. It is not mandatory for the service provider and direct-to-home
(DTH) airing it and customers may opt for it.
Source: http://www.business-standard.com/article/economy-policy/digishala-the-new-
24-7-gov-tv-channel-for-cashless-lessons-116120901354_1.html
Q.17) The Kuril Islands dispute is concerned with which of the following countries?
The Kuril Island dispute between Russia and Japan is mainly on four islands viz. Iturup,
Kunashir, Shikotan and Habomai rocks which are currently in possession of Russia and
claimed by Japan.
Source: http://www.thehindu.com/news/international/Medvedevs-Kuril-islands-visit-
angers-Japan/article15672357.ece
1. It is a landlocked country
2. It is bordered by Kazakhstan, Uzbekistan, Tajikistan, Mongolia and China
a) Only 1
b) Only 2
c) Both 1 and 2
d) Neither 1 nor 2
IASbabas Prelims 60 Day Plan Day 29 2017
(Science & Technology)
Q.18) Solution (a)
1. It is a statutory body
2. It is functioning as the Central Counter Terrorism Law Enforcement Agency in India
3. Central Government can issue directions to Suo-Moto take over the investigation of
a case
a) 1 and 2
b) 2 and 3
c) 1 and 3
d) All of the above
A State Government may request the Central Government to hand over the investigation of
a case to the NIA, provided there is reasonable ground to suggest the commission of a
scheduled offence under the NIA Act in the said crime.
Central Government can issue directions to Suo-Moto take over the investigation of a case -
provided under section 6(5) of the NIA Act, 2008.
State government shall extend all assistance and co-operation to the Agency for
investigation of the scheduled offences under section 9 of the NIA Act.
Q.20) Consider the following statements about Inner Line Permit (ILP)
1. It is required for Indian citizens to enter Arunachal Pradesh, Nagaland and Mizoram
and Manipur
2. It is issued under the Bengal Eastern Frontier Regulation, 1873
a) Only 1
b) Only 2
c) Both 1 and 2
d) Neither 1 nor 2
Inner Line Permit is required for Indian citizens to enter Arunachal Pradesh, Nagaland and
Mizoram. Inner Line Permit is issued under the Bengal Eastern Frontier Regulation, 1873.
Indian citizens also need a Nagaland-issued ILP if entering Manipur by road via Dimapur or
Kohima. However, demands by the Government of Manipur for the introduction of the
provision of an Inner Line Permit system to restrict entry of outsiders into the state were
refused.
An ILP was previously required for certain parts of the Leh district in Jammu and Kashmir.
This requirement was abolished by a circular issued by district magistrate which took effect
from 1 May 2014, although foreign nationals are required to get Protected Area Permit for
this region.
IASbabas Prelims 60 Day Plan Day 29 2017
(Science & Technology)
There are also ongoing demands for the introduction of ILP in Assam, Meghalaya and
Manipur to regulate entry of outsiders into the state.
a) 1 and 2
b) 2 and 3
c) 1 and 3
d) All of the above
The Government of India, Ministry of Tribal Affairs has launched Vanbandhu Kalyan Yojana
(VKY) for welfare of Tribals. VKY aims at creating enabling environment for need based and
outcome oriented holistic development of the tribal people. This process envisages to
ensure that all the intended benefits of goods and services under various
programmes/schemes of Central as well as State Governments actually reach the target
groups by convergence of resources through appropriate institutional mechanism.
It covers all tribal people and all areas with tribal population across the country.
Under the scheme maximum selling price for MFP is being implemented in schedule V
States. Web based portal has also been developed which indicate current price of MFPs on
real time basis across different mandis of the States.
Source: http://www.thehindu.com/news/national/Status-of-tribal-development-
remains-poor-Ministry-report/article16951999.ece
The Oslo Accords are a set of agreements between the government of Israel and the
Palestine Liberation Organization (PLO): the Oslo I Accord signed in Washington, D.C., in
1993 and the Oslo II Accord signed in Taba in 1995. The Oslo Accords marked the start of the
Oslo process, a peace process that is aimed at achieving a peace-treaty based on the United
Nations Security Council Resolution 242 and 338, and to fulfil the "right of the Palestinian
people to self-determination." The Oslo process started after secret negotiations in Oslo,
resulting in the recognition by the PLO of the State of Israel and the recognition by Israel of
the PLO as the representative of the Palestinian people and as a partner in negotiations.
The Oslo Accords created the Palestinian Authority, whose functions are the limited self-
governance over parts of the West Bank and Gaza Strip; and, it acknowledged that the PLO
is now Israel's partner in permanent status negotiations about the remaining issues. The
most important issues are the borders of Israel and Palestine, the Israeli settlements, the
status of Jerusalem, the question of Israel's military presence in and control over the
remaining territories after the recognition of the Palestinian autonomy by Israel, and the
Palestinian right of return. The Oslo Accords, however, did not create a Palestinian state.
Source: http://www.thehindu.com/opinion/lead/Tel-Aviv-on-
tenterhooks/article16955092.ece
a) 1 and 2
b) 2 and 3
c) 1 and 3
d) All of the above
IASbabas Prelims 60 Day Plan Day 29 2017
(Science & Technology)
Q.23) Solution (a)
Polavaram Project is a multi-purpose irrigation project which has been accorded national
project status by the central government. This dam across the Godavari River is under
construction located in West Godavari District and East Godavari District in Andhra Pradesh
state and its reservoir spreads in parts of Chhattisgarh and Odisha States also.
National River-Linking Project, which works under the aegis of the Indian Ministry of Water
Resources, was designed to overcome the deficit in water in the country. As a part of this
plan, surplus water from the Himalayan rivers is to be transferred to the peninsular rivers of
India. In this project's case, the Godavari river basin is considered as a surplus one, while the
Krishna River basin is considered to be a deficit one. As of 2008, 644 tmcft of underutilised
water from Godavari River flowed into the Bay of Bengal. Based on the estimated water
requirements in 2025, the Study recommended that sizeable surplus water was to be
transferred from the Godavari River basin to the Krishna River basin.
Source: http://www.thehindu.com/news/national/Polavaram-project-gets-Central-
funds/article16946581.ece
Source: http://www.thehindu.com/todays-paper/tp-national/Dudhwa-rhino-
rehabilitation-in-limbo-over-sedation-drug/article16944312.ece
Q.25) X country has dragged India to the World Trade Organisation (WTO) against certain
measures taken by India on imports of iron and steel products. Which is that country?
India has imposed minimum import price (MIP) on imports of certain iron and steel
products.
Japan has dragged India to the World Trade Organisation (WTO) against certain measures
taken by New Delhi on imports of iron and steel products.
India has imposed MIP as growing imports from steel surplus countries like China, Japan and
Korea with predatory prices have been a major concern for the domestic industry since
September 2014.
India has also imposed anti-dumping duties on certain steel products to guard domestic
players from cheap imports.
Source: http://www.thehindu.com/business/Industry/Japan-drags-India-to-WTO-against-
steps-on-iron-steel-imports/article16934465.ece
IASbabas 60 Day Plan- Environment [Day 30] 2017
Pattiseema Lift Irrigation Scheme which has interlinked rivers Godavari and Krishna in West
Godavari district of Andhra Pradesh, has now got national level acknowledgment.
It is registered in Limca Book of Records for integrating Krishna and Godavari rivers within a
year time.
1. Seaweeds are rich in vitamins and minerals and are consumed as food in various
parts of the world
2. They are also used for the production of phytochemicals, viz., agar, carrageenan and
alginate.
3. Seaweed can be potentially used as organic fertilizers
a) 1 and 2
b) 2 and 3
c) Only 1
d) 1, 2 and 3
www.iasbaba.com Page 1
IASbabas 60 Day Plan- Environment [Day 30] 2017
In India, seaweeds are used as raw materials for the production of agar, alginate and liquid
seaweed fertilizer (LSF).
There are a few agar industries, algin industries and LSF industries situated at different
places in the maritime states of Tamil Nadu, Karnataka, Andhra Pradesh and Gujarat.
The red algae Gelidiella acerosa, Gracilaria edulis, G. crassa, G. foliifera and G. verrucosa are
used for agar manufacture and brown algae Sargassum spp., Turbinaria spp. and Cystoseira
trinodis for the production of alginates and liquid seaweed fertilizer.
The quantity of seaweeds exploited is inadequate to meet the raw material requirement of
Indian seaweed industries.
http://www.thehindu.com/todays-paper/tp-national/tp-andhrapradesh/seaweed-
cultivation-will-be-encouraged/article17791027.ece
Q.3) Consider the following about applications of Super Absorbent Polymers (SAP) in
Agriculture
1. SAPs are polluting and non-biodegradable and dont helps in reducing irrigation
frequency and water consumption.
2. They improves soil quality and resists drought stress
3. SAPs can reduce overuse of fertilizers and pesticides in fields.
4. SAPs act as soil matter flocculants.
a) 1, 3 and 4 Only
b) 2, 3 and 4 Only
c) 2 and 3 Only
d) 1, 2, 3 and 4
Super Absorbent Polymers, also known as SAP, hydrogel, absorbent polymers, absorbent
gels, super soakers, super slurpers, water gel, is a new type of macro molecular synthetic
water absorbing polymer material.
It has a water uptake potential as high as 100,000% of its own weight in a short period of
time by osmosis and form granules in soil to enhance soil properties.
www.iasbaba.com Page 2
IASbabas 60 Day Plan- Environment [Day 30] 2017
SAPs are generally white sugar-like hygroscopic materials that swell in water to form a clear
gel made of separate individual particles and can retain moisture even under pressure
without risk of conflagration or rupturing/blasting.
Super Absorbent Polymers used in agriculture are mostly prepared from acrylic acids and a
cross-linking agent like potassium by solution or suspension polymerization. The polymer so
formed is called a polyacrylate whose swelling capacity and gel modulus depends greatly on
the quantity and type of cross-linker used.
The three most common soil conditions that hinder plant growth and crop yield are low
water retention capability, high evapo-transpiration rate and soil moisture leaching. Apart
from these, factors like unforeseen drought conditions, degradation and salination, overuse
of synthetic fertilizers and pesticides and improper irrigation practices severely affect soil
and plants, often rendering permanent damage to soil biota as well.
www.iasbaba.com Page 3
IASbabas 60 Day Plan- Environment [Day 30] 2017
SAPs form a consistent cyclic process of absorption and release of water; the water
so released can provide optimum moisture for quick germination and seedling
maturation. Thus it reduces seedling mortality by several folds in nurseries.
In cold regions, death during germination and maturation is common due to
moisture freezing in & around plant root tissue. Absorbed moisture in hydrogels
does not freeze and makes easy accessibility to plants. It also regulates seedling
growth temperature preventing death by freezing.
SAPs can help save water and labor by reducing irrigation frequency, help overcome
drought conditions and act as soil conditioners, prevent leaching in sandy soils,
runoffs in mountainous and sloping fields, improve virescence efficiency and restore
soil biota.
SAPs can reduce overuse of fertilizers and pesticides in fields. The chemicals so
absorbed with water are slowly released thus extending the operational life and
uptake efficacy by root systems.
SAPs act as soil matter flocculants. They closely bind loose soil thus forming loams
that can help better root latching. Simultaneously, the repeated absorb-release
mechanism prevents over compaction of soil minerals and provides space for
aeration and development of soil edaphon.
It has a wide area of application ranging from agriculture, forestry, industrial
planting, municipal gardening, drought management, water conservation, It helps
reduce soil erosion by surface run-offs, fertilizer and pesticide leaching to ground
water, reducing cost of water and irrigation and success rate at growth and high
yields of crops.
Q.4) Plant nutrition is a term that takes into account the interrelationships of mineral
elements in the soil or soilless solution as well as their role in plant growth. Consider the
following
1. Nickel
2. Molybdenum
3. Copper
4. Boron
5. Sulphur
6. Silicon
7. Vanadium
Which of the given above has a role of essential and beneficial mineral nutrients that are
crucial for plant nutrition and growth?
a) 2, 3, 5 and 6
b) 1, 2, 3, 4, 5 and 7
www.iasbaba.com Page 4
IASbabas 60 Day Plan- Environment [Day 30] 2017
c) 2, 3, 4, 5, 6 and 7
d) 1, 2, 3, 4, 5, 6 and 7
The following is a brief guideline of the role of essential and beneficial mineral nutrients that
are crucial for growth. Eliminate any one of these elements, and plants will display
abnormalities of growth, deficiency symptoms, or may not reproduce normally.
Macronutrients
Sulfur is a structural component of amino acids, proteins, vitamins and enzymes and is
essential to produce chlorophyll. It imparts flavor to many vegetables. Deficiencies show as
light green leaves. Sulfur is readily lost by leaching from soils and should be applied with a
nutrient formula. Some water supplies may contain Sulfur.
www.iasbaba.com Page 5
IASbabas 60 Day Plan- Environment [Day 30] 2017
Micronutrients
Iron is necessary for many enzyme functions and as a catalyst for the synthesis of
chlorophyll. It is essential for the young growing parts of plants. Deficiencies are pale leaf
color of young leaves followed by yellowing of leaves and large veins. Iron is lost by leaching
and is held in the lower portions of the soil structure. Under conditions of high pH (alkaline)
iron is rendered unavailable to plants. When soils are alkaline, iron may be abundant but
unavailable. Applications of an acid nutrient formula containing iron chelates, held in
soluble form, should correct the problem.
Boron is necessary for cell wall formation, membrane integrity, calcium uptake and may aid
in the translocation of sugars. Boron affects at least 16 functions in plants. These functions
include flowering, pollen germination, fruiting, cell division, water relationships and the
movement of hormones. Boron must be available throughout the life of the plant. It is not
translocated and is easily leached from soils. Deficiencies kill terminal buds leaving a rosette
effect on the plant. Leaves are thick, curled and brittle. Fruits, tubers and roots are
discolored, cracked and flecked with brown spots.
www.iasbaba.com Page 6
IASbabas 60 Day Plan- Environment [Day 30] 2017
leaves develop brown spots. Copper is bound tightly in organic matter and may be deficient
in highly organic soils. It is not readily lost from soil but may often be unavailable. Too much
copper can cause toxicity.
Chlorine is involved in osmosis (movement of water or solutes in cells), the ionic balance
necessary for plants to take up mineral elements and in photosynthesis. Deficiency
symptoms include wilting, stubby roots, chlorosis (yellowing) and bronzing. Odors in some
plants may be decreased. Chloride, the ionic form of chlorine used by plants, is usually
found in soluble forms and is lost by leaching. Some plants may show signs of toxicity if
levels are too high.
Nickel has just recently won the status as an essential trace element for plants according to
the Agricultural Research Service Plant, Soil and Nutrition Laboratory in Ithaca, NY. It is
required for the enzyme urease to break down urea to liberate the nitrogen into a usable
form for plants. Nickel is required for iron absorption. Seeds need nickel in order to
germinate. Plants grown without additional nickel will gradually reach a deficient level at
about the time they mature and begin reproductive growth. If nickel is deficient plants may
fail to produce viable seeds.
Cobalt is required for nitrogen fixation in legumes and in root nodules of nonlegumes. The
demand for cobalt is much higher for nitrogen fixation than for ammonium nutrition.
Deficient levels could result in nitrogen deficiency symptoms.
Silicon is found as a component of cell walls. Plants with supplies of soluble silicon produce
stronger, tougher cell walls making them a mechanical barrier to piercing and sucking
insects. This significantly enhances plant heat and drought tolerance. Foliar sprays of silicon
have also shown benefits reducing populations of aphids on field crops. Tests have also
found that silicon can be deposited by the plants at the site of infection by fungus to combat
the penetration of the cell walls by the attacking fungus. Improved leaf erectness, stem
strength and prevention or depression of iron and manganese toxicity have all been noted
as effects from silicon. Silicon has not been determined essential for all plants but may be
beneficial for many.
Vanadium may be required by some plants, but at very low concentrations. It may also be
substituting for molybdenum.
www.iasbaba.com Page 7
IASbabas 60 Day Plan- Environment [Day 30] 2017
P - energy transfer
Mg - chlorophyll molecule
S - proteins
MOBILE IMMOBILE
N Ca
P S
K B
Mg Fe
Cu
Mn
Zn
1. Apple
2. Banana
3. Buckwheat
4. Coffee
5. Mango
6. Carrot
7. Soybeans
a) 1, 2, 3, 4, 5 and 6
b) 2, 3, 6 and 7
c) 1, 3, 4, 5 and 6
d) 1, 3, 5, 6 and 7
www.iasbaba.com Page 8
IASbabas 60 Day Plan- Environment [Day 30] 2017
Other staple food crops, like bananas and plantains, are sterile and propagated from
cuttings, requiring no pollination of any form, ever.
Name Pollinators
Fagopyrum
Buckwheat Honey bees, Solitary bees
esculentum
Mango Mangifera indica Honey bees, Stingless bees, Flies, Ants, Wasps
Hint- http://www.thehindu.com/life-and-style/food/new-buzz-in-town/article17846880.ece
http://www.thehindu.com/todays-paper/tp-national/tp-kerala/mango-farmers-rue-fall-in-
price/article18263086.ece
http://www.thehindu.com/opinion/columns/Decline-of-pollinators-threatens-food-
supply/article14181840.ece
www.iasbaba.com Page 9
IASbabas 60 Day Plan- Environment [Day 30] 2017
a) 1 and 3
b) 2 Only
c) 1 Only
d) None
Parthenium hysterophorus L., commonly known as carrot weed, white top or congress grass
in India, is a herbaceous, erect and annual plant belonging to the family Asteracae
(compositae).
It is most popularly known as gajar ghas due to its appearance like carrot plant. The origin of
Parthenium is considered to be from Mexico, America, Trinidad to and Argentina. After
noticeable occurrence of Parthenium in Pune (Maharashtra) in 1956, it has spread like a wild
fire throughout India. At present it has invaded about 35 million hectares of land in India. It
is a nuisance on road sides and railway tracks, vacant lands, wastelands, industrial areas, on
the sides of open drainage system and irrigation canals besides invading agricultural crop.
Parthenium also infest every type of crop, orchards, plantations and forest. It severely
reduces the crop productivity besides loss to biodiversity and environment.
http://www.thehindu.com/news/cities/Madurai/Efforts-needed-to-eradicate-
parthenium/article14574270.ece
http://timesofindia.indiatimes.com/citizen-reporter/stories/they-came-across-an-
important-thing-is-this-morning-tee/crshow/57413527.cms
www.iasbaba.com Page 10
IASbabas 60 Day Plan- Environment [Day 30] 2017
Readiness for Investment in Sustainable Energy (RISE), developed by the World Bank Group,
is a suite of indicators that assesses the legal and regulatory environment for investment in
sustainable energy. It establishes a framework for better depicting the national enabling
environment to attract investment into sustainable energy.
RISE is aimed at policymakers who focus on actions within their control. Creating this
environment is directed by policymakersthe primary constituency RISE aims to influence.
RISE supports the achievement of the objectives of the Sustainable Energy for All (SE4ALL)
initiative.
Q.8) The decisions on REDD+ enumerate some "eligible activities" that developing
countries may implement to reduce emissions and enhance removals of greenhouse
gases. Identify the activities
a) 1, 2, 3 and 5
b) 1, 3 and 5
c) 1, 2, 3, 4 and 5
d) 2, 3, 4 and 5
What is REDD+?
www.iasbaba.com Page 11
IASbabas 60 Day Plan- Environment [Day 30] 2017
Reducing emissions from deforestation and forest degradation in developing countries and
the role of conservation, sustainable management of forests and enhancement of forest
carbon stocks in developing countries (REDD+) encourages developing countries to
contribute to climate change mitigation in the forest sector through the following activities:
a) 1 and 2
b) 2 and 3
c) 3 only
d) None of the above
The National Mission on Sustainable Habitat is one of the missions under National Action
Plan on Climate Change and aims to make cities sustainable through improvements in
energy efficiency in buildings, management of solid waste & shift to public transport.
www.iasbaba.com Page 12
IASbabas 60 Day Plan- Environment [Day 30] 2017
a) 1, 2 and 3
b) 1 and 2
c) 2 and 3
d) None
They are common in wetlands, where they are responsible for marsh gas, and in the
digestive tracts of animals such as ruminants and humans, where they are responsible for
the methane content of belching in ruminants and flatulence in humans.
Q.11) Match the following list of bioactive substances and their roles
www.iasbaba.com Page 13
IASbabas 60 Day Plan- Environment [Day 30] 2017
The World Sustainable Development Summit (WSDS), TERI's flagship event, has been
conceptualized as a single platform to accelerate action towards sustainable development
and especially climate change. The WSDS series seeks to bring together the finest minds and
leading thinkers of the world to focus attention on the challenge of sustainable
development and has emerged as a landmark event addressing issues pertinent to the
future of humanity.
WSDS builds on the 15 years legacy of the Delhi Sustainable Development Summit (DSDS)
which was the leading forum for discussing sustainable development issues. The DSDS held
under the aegis of the Ministry of Environment, Forest and Climate Change with support
from the Ministry of External Affairs, Government of India was an epitome of Track 2
diplomacy .
With an aim of expanding the scope and reach of the Summit to the global community,
DSDS has now transitioned to WSDS.
WSDS 2016 was held in New Delhi from October 5-8, 2016 under the broad rubric of
'Beyond 2015: People, Planet & Progress', and it broadly focused on actions, on accelerated
implementation of SDGs and NDCs.
a) 2014
b) 2015
c) 2016
d) 2017
www.iasbaba.com Page 14
IASbabas 60 Day Plan- Environment [Day 30] 2017
http://media.unwto.org/press-release/2017-01-03/2017-international-year-sustainable-
tourism-development
Q.14) National Mission for Sustaining the Himalayan Ecosystem comes under
http://knowledgeportal-nmshe.in/NAPCC.aspx
Q.15) The United Nations General Assembly formally adopted the "universal, integrated
and transformative" 2030 Agenda for Sustainable Development, a set of 17 Sustainable
Development Goals (SDGs). Identify the correct goals
1. Gender Inequality
2. Affordable and clean energy
3. Combat desertification
4. End of nuclear energy
5. Clean water and sanitation
6. Food security
a) 1, 2, 3, 4, 5 and 6
b) 2, 3, 5 and 6
c) 1, 2, 3, 5 and 6
d) 1, 2, 3, 4 and 5
Goal 1: No Poverty
Goal 2: Zero Hunger
Goal 3: Good Health and Well-being
Goal 4: Quality Education
Goal 5: Gender Equality
Goal 6: Clean Water and Sanitation
www.iasbaba.com Page 15
IASbabas 60 Day Plan- Environment [Day 30] 2017
1. Northern Lights are result of collisions between gaseous particles in the Earth's
atmosphere with charged particles released from the sun's atmosphere
2. Northern Lights are also called Aurora australis
a) Only 1
b) Only 2
c) Both 1 and 2
d) Neither 1 nor 2
The bright dancing lights of the aurora are actually collisions between electrically charged
particles from the sun that enter the earth's atmosphere. The lights are seen above the
magnetic poles of the northern and southern hemispheres. They are known as 'Aurora
borealis' in the north and 'Aurora australis' in the south.
Auroral displays appear in many colours although pale green and pink are the most
common. Shades of red, yellow, green, blue, and violet have been reported. The lights
appear in many forms from patches or scattered clouds of light to streamers, arcs, rippling
curtains or shooting rays that light up the sky with an eerie glow.
The Northern Lights are actually the result of collisions between gaseous particles in the
Earth's atmosphere with charged particles released from the sun's atmosphere. Variations
in colour are due to the type of gas particles that are colliding. The most common auroral
www.iasbaba.com Page 16
IASbabas 60 Day Plan- Environment [Day 30] 2017
'Aurora borealis', the lights of the northern hemisphere, means 'dawn of the north'. 'Aurora
australis' means 'dawn of the south'.
Source: http://www.thehindu.com/sci-tech/energy-and-environment/NASA-satellite-
spots-stunning-view-of-northern-lights/article16958151.ece
The Saudi-brokered Arab Peace Initiative, which was endorsed by the Arab League's 22
members during the March 2002 Beirut summit, outlined comprehensive steps to ending
the Palestinian-Israeli conflict.
Arab leaders collectively offered Israel recognition of its right to exist and a normalisation of
diplomatic ties in exchange for its complete withdrawal from Arab lands captured since
1967.
The plan, first floated by King Abdullah, then crown prince of Saudi Arabia, called for the
restoration of a Palestinian state with east Jerusalem as its capital and a "fair solution" for
the 3.8 million Palestinian refugees, including but not limited to the Syrian Golan Heights
and Israeli-occupied territory in southern Lebanon.
The Saudi plan is based on UN resolutions 242 and 338 which collectively called for Israeli
withdrawal in exchange for peaceful ties with its Arab neighbours and the "respect for the
right of every state in the area to live in peace within secure and recognised boundaries".
www.iasbaba.com Page 17
IASbabas 60 Day Plan- Environment [Day 30] 2017
Source: http://www.livemint.com/Opinion/jkAIxh8lvmRV0MmKS683VP/Abdullah-Gul--A-
Middle-East-peace-agenda-for-2017.html
a) Sri Lanka
b) Afghanistan
c) Iraq
d) Somalia
The U.S. government used the term "Operation Enduring Freedom Afghanistan" to
officially describe the War in Afghanistan, from the period between October 2001 and
December 2014. Continued operations in Afghanistan by the United States' military forces,
both non-combat and combat, now occur under the name Operation Freedom's Sentinel.
Resolute Support or Operation Resolute Support is a NATO-led train, advise, and assist
mission consisting of over 13,000 troops in the Islamic Republic of Afghanistan, which began
on January 1, 2015.
Source: http://www.thehindu.com/todays-paper/tp-opinion/Afghanistan-India-and-
Trump/article16952231.ece
Q.19) 'Atal-Amrit Abhiyan' health insurance scheme which will provide coverage against
several critical illnesses was lauched by which of the following states?
a) Rajasthan
www.iasbaba.com Page 18
IASbabas 60 Day Plan- Environment [Day 30] 2017
b) Maharashtra
c) Assam
d) Uttar Pradesh
Assam rolled out 'Atal-Amrit Abhiyan' health insurance scheme which will provide coverage
against several critical illnesses.
Both Below Poverty Line (BPL) and Above Poverty Line (APL) families, with annual income
below Rs 5 lakh, are eligible for the scheme.
The scheme will offer coverage against 437 illnesses in six disease groups, namely,
cardiovascular, cancer, kidney, neo-natal, neurological conditions and burns.
A separate society will also be set up under the Department of Health and Family Welfare
which will be responsible for administration of the scheme.
Source: http://economictimes.indiatimes.com/news/politics-and-nation/assam-launches-
health-insurance-scheme-atal-amrit-abhiyan/articleshow/56170511.cms
Source: http://economictimes.indiatimes.com/news/defence/china-tests-latest-stealth-
fighter-aircraft-fc-31-its-answer-to-us-f-35-jets/articleshow/56180243.cms
a) Odisha
b) Tamil Nadu
www.iasbaba.com Page 19
IASbabas 60 Day Plan- Environment [Day 30] 2017
c) Kerala
d) Andhra Pradesh
It is a dance-drama performance art, with its roots in the ancient Hindu Sanskrit text of
Natya Shastra. Like all major classical dances of India, it was also developed as a religious art
linked to traveling bards, temples and spiritual beliefs.
The Kuchipudi style was conceived by Siddhendra Yogi, a talented Vaishnava poet of 17th
century.
It begins with an invocation to Lord Ganesha followed by nritta (non-narrative and abstract
dancing); shabdam (narrative dancing) and natya.
The dance is accompanied by song which is typically Carnatic music. The singer is
accompanied by musical instruments like mridangam, violin, flute and tambura.
Like other classical dances, Kuchipudi also comprises pure dance, mime and histrionics but it
is the use of speech that distinguishes Kuchipudis presentation as dance drama.
Source: http://www.thehindu.com/news/cities/Vijayawada/6117-Kuchipudi-dancers-put-
A.P.-in-Guinness-book/article16943005.ece
Chillai-Kalan, is the 40-day period of harsh winter. Chillai-Kalan begins from December 21
and ends on January 31 next year.
www.iasbaba.com Page 20
IASbabas 60 Day Plan- Environment [Day 30] 2017
Source: www.thehindu.com/todays-paper/tp-national/Kashmir-braces-for-Chillai-
Kalan/article16908581.ece
Q.23) Consider the following statements about Smart Anti-Airfield Weapon (SAAW)
a) Only 1
b) Only 2
c) Both 1 and 2
d) Neither 1 nor 2
SAAW, an indigenously designed and developed 120 kg. class smart weapon, developed by
DRDO, is capable of engaging ground targets with high precision up to a range of 100 kms.
The light weight high precision guided bomb is one of the world class weapons systems.
Source: http://pib.nic.in/newsite/PrintRelease.aspx?relid=155817
a) Only 1
b) Only 2
c) Both 1 and 2
d) Neither 1 nor 2
Border types can be classified into soft and hard borders. Soft borders include open and
regulated and controlled frontiers. Hard borders/fortified borders, include wire fenced
borders; wire fenced and walled borders; walled borders; and militarized borders.
www.iasbaba.com Page 21
IASbabas 60 Day Plan- Environment [Day 30] 2017
Controlled Borders
http://www.siue.edu/GEOGRAPHY/ONLINE/Vogeler/ControlledBorders.htm
Open Borders
http://www.siue.edu/GEOGRAPHY/ONLINE/Vogeler/OpenBorders.htm
Source: http://www.thehindu.com/news/international/%E2%80%98Soft-border-
approach-can-bring-India-into-CPEC%E2%80%99/article16943237.ece
a) Only 1
b) Only 2
c) Both 1 and 2
d) Neither 1 nor 2
Scientists record live video of underwater volcanic eruption which had erupted last year
(2015) and twice in the past in 1998 and 2011. Axial Seamount is the worlds most active
and studied underwater volcano, located at 300 miles from Oregon coast, rising from Juan
de Fuca Ridge. Axial Seamount is caused by the movement of two tectonic plates in the
region that are gradually spreading apart (divergent plate margin). The convergent plate
margin is located between the North American Plate and Juan de Fuca Plate. Axial
Seamount is the youngest volcano and current eruptive center of the Cobb-Eickelberg
Seamount chain.
Source: http://indianexpress.com/article/technology/science/scientists-record-live-
video-of-underwater-volcanic-eruption-4431497/
www.iasbaba.com Page 22
IASbabas 60 Day Plan- Environment [Day 30] 2017
www.iasbaba.com Page 23
IASbabas Prelims 60 Day Plan Day 31 2017
(History)
Q.1) Consider the following statements with reference to the Third battle of Panipat
a) 1 only
b) 2 only
c) Both 1 and 2
d) Neither 1 or 2
The Third Battle of Panipat fought on January 14, 1761 between the Marathas and forces of
the Afghan ruler Ahmad Shah Abdali and his allies.
The battle started in the wee hours on January 14, 1761. Towards the start of the battle the
Marathas pushed back the Rohillas, who were on the Afghan side. But the tide of the battle
soon turned against the Marathas and by the end of the day they were killed, taken prisoner
or fled.
Marathas lacked effective leadership and did not possess enough experience with the guns.
They relied heavily on the Cavalry. This battle crushed the Maratha dream of ruling over the
whole of India.
Q.2) Consider the following statements with reference to the Battle of Karnal
a) 1 and 4 only
b) 2 and 4 only
c) 3 and 4 only
d) None of the above
IASbabas Prelims 60 Day Plan Day 31 2017
(History)
Q.2) Solution (b)
The Battle of Karnal was fought between the armies of Nader Shah and Mughal Emperor
Muhammad Shah. Inspite of having a large army, Muhammad Shah was defeated in the
battle. The incident took place on 24th February, 1739 near Karnal, Haryana, India.
Despite opposition by others, Saadar Khan led a premature attack without adequate
preparations. Khan Dauran rode forth in his support. Thereupon the emperor and the rest of
the Indian army also got ready for an attack. But it had neither a plan of action nor an
agreed leader.
An experienced and gifted general like Nadir shah, therefore, took no time in wasting it. The
battle of Karnal lasted only for three hours. Saadat Khan fell a prisoner while Khan Dauran
was mortally wounded and soon died.
1. These were low interest loans granted by Mughal king to the artisans
2. These were the low interest loans given by the Marathas to the farmers
3. The aim of the loans was to protect the cultivators from the money lenders
a) 1 and 2 only
b) 2 and 3 only
c) 1 and 3 only
d) All of the above.
Agricultural taxes were the main source of revenue for the Maratha administration, as such
they took various measures to expand and promote agriculture.
Waste or barren land that was brought into cultivation was given to the cultivator as the
Inam land, and Tagai loans were given to to the farmers, these were the low interest loans
given by the state to the cultivators, the primary aim was to protect the farmers from the
clutches of the moneylenders.
IASbabas Prelims 60 Day Plan Day 31 2017
(History)
Q.4) Consider the following statements regarding Chauth and Sardeshmukhi
a) 1 only
b) 2 only
c) Both 1 and 2
d) Neither 1 or 2
Chauth and Sardeshmukhi were the taxes levied on the lands that were outside the
Swarajya (own kingdom) land, these were levied on the lands in Mughal empire and Deccan
kingdom surrounding the Maratha kingdom. The Chauth was of the tax on taxes given to
the Mughal Emperor and Sardeshmukhi was additional 10% on the Chauth.
This system of taxation was started by Shivaji as system of protection against Maratha raids
on these lands.
Q.5) Consider the following statements with reference to the Third Carnatic War
a) 1 and 2 only
b) 2 and 3 only
c) 1 and 3 only
d) All of the above.
In 1756 there broke out Seven Years War in Europe and before long the two nations began
to fight in India also. In India it was considered the Third Carnatic War. The French
government sent a powerful army under the command of Count de Lally. He was forced to
IASbabas Prelims 60 Day Plan Day 31 2017
(History)
retire to Pondicherry and was defeated by Sir Eyre Coote in the battle of Wandiwash in
1760.
The French lost the possession to the British and also other territories. The war came to an
end with the Signing of the Treaty of Paris. The acquired territories of both the sides were
restored, but the French lost their influence in India for ever.
Q.6) consider the following statements with reference to the Royal Firman of 1765
a) 1 only
b) 2 only
c) Both 1 and 2
d) Neither 1 or 2
The royal Firman of 1765 was issued by the King Shah Alam II. He did this after his defeat in
the Battle of Buxar of 1764.
As per this firman, the Company acquired the Diwani rights (Revenue Collection Rights) in
Bihar, Bengal and Orissa. This Firman led to the infamous Dual system of Government of
Robert Clive. Under this the Company was to have Diwani rights and the Nizami rights
(Justice and law and order) were to remain with the Indians.
Q.7) Which of the Following was the reason for the Kol Uprising of 1831?
Q.8) Consider the following statements with reference to the Third Anglo-Mysore War
1. It was fought between the Tippu Sultan, Marathas and Nizam on one side and the
British on the other
2. The attack on Travancore by Tippu became the immediate cause of the war
3. The war ended with the signing of the Treaty of Mangalore
a) 1 and 2 only
b) 2 and 3 only
c) 1 and 3 only
d) None of the above
The Third Anlo-Mysore war was fought between the troops of Tippu Sultan on one side and
the Combined armies of the Marathas, The British and the Nawab on the other.
Tippus campaign against the Raja of Travancore became the immediate cause for the war,
and the British quickly took the Rajas side.
Tippu was defeated and the war ended with the Signing of Treaty of Srirangapatna. The
terms of the treaty laid the seeds of Fourth Anglo-Mysore War
a) 4-1-2-3
b) 4-1-3-2
c) 1-4-2-3
d) 1-4-3-2
The First Carnatic War (17461748): Fought between the French and the British. The first of
the 3 war fought between the French and the British.
The Fourth AngloMysore War (17981799): Fought between the British troops and the
army of Tippu Sultan
The Third Anglo-Maratha War (18171818): Fought between the British and the Marathas,
led to decisive defeat of the Marathas.
The Second Anglo Sikh war (1848-49): It resulted in the subjugation of the Sikh Empire, and
the annexation of the Punjab and what subsequently became the North-West Frontier
Province, by the East India Company.
Q.10) Which of the Following states was not annexed through the Doctrine of Lapse Policy
a) Satara
b) Udaipur
c) Awadh
d) Nagpur
Awadh (1856) was annexed on the pretext of Mis-governance/ Misrule by the Nawab, while
the rest of the above states were annexed under the policy of Doctrine of Lapse devised by
Lord Dalhousie.
Q.11) Consider the following statements with reference to the Madras labour Union
IASbabas Prelims 60 Day Plan Day 31 2017
(History)
1. It was one of the first trade union to be organized on modern lines
2. B. P. Wadia was the first president of MLU
a) 1 only
b) 2 only
c) Both 1 and 2
d) Neither 1 or 2
Q.12) The Durand commission to demarcate boundary between India and Afghanistan
was setup by
a) Lord Lytton
b) Lord Lansdowne
c) Lord Curzon
d) Lord Dufferin
Durand Commission (1893) was formed during the term of Lord Lansdowne, his term is also
associated with the passing of second factory Act and the passing of Indian Councils Act
1892.
The Durand line was to Demarcate India and Afghanistan (Present Pakistan and
Afghanistan).
Q.13) Consider the following statements with reference to the annexation of Kingdom of
Mysore
a) 1 only
b) 2 only
c) Both 1 and 2
d) Neither 1 or 2
Mysore was annexed by the Governor General William Bentinck on the grounds of mal-
administration in 1831.it was under the british rule for 50 years. It was handed over to King
Chamaraja Wodeyar by the British in 1881.
Q.14) Consider the following statements with reference to the office of Law Member
a) 1 and 2 only
b) 2 and 3 only
c) 1 and 3 only
d) All of the above
The Office of the Law member was created by the Charter Act of 1833. The act expanded
the Executive Council of the Governor general by adding the Fourth member. He was to aid
the Governor general in the framing of the laws. Macaulay became the first law member.
The Law member was not entitled to Vote in the Executive council but was permitted to sit
in the proceedings.
IASbabas Prelims 60 Day Plan Day 31 2017
(History)
Q.15) Consider the following statements with reference to the provisions of the
Rajagopalachari formula
a) 1 and 2 only
b) 2 and 3 only
c) 1 and 3 only
d) All of the above
1. The Muslim league endorsing the congress demand for full independence and
cooperating with it in forming an interim government in the transition period.
2. A plebiscite was to be held in NWFP and NEFP where the people were to decide
whether to join which state
3. In the event of separation, agreements would be made for defense, communication
and other purposes.
4. The terms would be binding only in case of transfer by Britain of full power and
responsibility for full governance of India.
Equal representation of Congress and the Muslim League was part of the Liaqat-Desai Pact.
This had far reacing consequences for the country.
INS Khanderi, second of the Scorpene class submarine, was launched in Mumbai.
INS Khanderi is designed to operate in all theatres, including the tropics. All means and
communications are provided to ensure interoperability with other components of a Naval
Task Force.
It includes superior stealth and the ability to launch a crippling attack on the enemy using
precision guided weapons. The stealth features are expected to give the submarine an
invulnerability, unmatched by many submarines.
INS Khanderi can undertake diverse types of missions including anti-surface warfare, anti-
submarine warfare, intelligence gathering, mine laying, area surveillance and others. The
features of this submarine also include the ability to launch attacks with torpedoes, as well
as tube-launched anti-ship missiles, whilst underwater or on surface.
Khanderi is the second of the six submarines being built at MDL in collaboration with M/s
DCNS of France, as part of Project 75 of Indian Navy. The first one is Kalvari which is
currently completing sea trials.
INS Khanderi is named after the Island fort of Maratha forces, which played a vital role in
ensuring their supremacy at sea in the late 17th century.
Source: http://indianexpress.com/article/india/scorpene-kalvari-class-submarine-ins-
khanderi-launched-mumbai-4470239/
Q.17) Which of the following are members of the Quartet on the Middle East
1. United Nations
2. Israel
3. Russia
4. United States of America
5. European Union
6. Palestine
a) 1, 3, 4 and 5
b) 1, 2, 4 and 6
c) 1, 2 and 6
d) 2, 3, 4 and 6
IASbabas Prelims 60 Day Plan Day 31 2017
(History)
The Quartet on the Middle East or Middle East Quartet, sometimes called the Diplomatic
Quartet or Madrid Quartet or simply the Quartet, is a foursome of nations and international
and supranational entities involved in mediating the IsraeliPalestinian peace process. The
Quartet are the United Nations, the United States, the European Union, and Russia. The
group was established in Madrid in 2002, recalling Madrid Conference of 1991, as a result of
the escalating conflict in the Middle East.
Q.18) Consider the following statements about Directorate General of Foreign Trade
(DGFT)
a) Only 1
b) Only 2
c) Both 1 and 2
d) Neither 1 nor 2
The Directorate General of foreign Trade (DGFT) is the agency of the Ministry of Commerce
and Industry of the Government of India, responsible for execution of the import and export
Policies of India. It was earlier known as Chief Controller of Imports & Exports (CCI&E) till
1991. DGFT plays a very important role in the development of trading relations with various
other nations and thus help in improving not only the economic growth but also provides a
certain impetus needed in the trade industry. For promoting exports and imports DGFT
establish its regional offices across the country.
a) World Bank
b) World Economic Forum
c) United Nations Development Programme
d) None of the above
India ranked 60th among the 79 developing countries in 2017 Inclusive Development Index
(IDI) released in World Economic Forums (WEF) Inclusive Growth and Development
Report.
The index is based on 12 performance indicators and countries are ranked on IDI scores
based on a scale of 1-7. It has three pillars Growth and Development, Inclusion and
Intergenerational Equity, and Sustainability in order to provide a more complete measure of
economic development than GDP growth alone.
Indias debt-to-GDP ratio is high, that raises some questions about the sustainability of
government spending. Indias labour force participation rate is low, informal economy is
large and many workers are vulnerable to employment situations with little room for social
mobility. India needs more progressive tax system to raise capital for expenditures in
infrastructure, health care, basic services and education.
IASbabas Prelims 60 Day Plan Day 31 2017
(History)
a) Only 1
b) Only 2
c) Both 1 and 2
d) Neither 1 nor 2
An orphan drug is a pharmaceutical agent that has been developed specifically to treat a
rare medical condition, the condition itself being referred to as an orphan disease.
Karnataka became the first state to release a Rare Diseases and Orphan Drugs Policy. It
recommended the implementation of preventive and carrier testing as a means of reducing
morbidity and mortality. Given that over 80% of rare diseases have a genetic basis, it
suggested the use genetic testing to accelerate the identification of the critical genes
involved in rare diseases.
It also recommended that education be used as a tool to combat delayed diagnosis and
treatment and called for the enactment of an orphan drugs statute to allow for tax breaks,
funding and exclusive marketing rights as incentives for orphan drug discovery.
It also highlighted a feature of Indias insurance laws that puts patients suffering from rare
diseases at a particular disadvantage. Private insurance companies treat genetic disorders as
pre-existing conditions and on that ground, exclude them from coverage. Since most rare
diseases are genetic, patients are routinely denied insurance cover.
The Karnataka policy asks that the Insurance Regulatory and Development Authority re-
consider this exclusion and affirmatively require that insurance companies provide basic
coverage of rare diseases at reasonable premiums.
Source: http://www.livemint.com/Opinion/a80dQAFVidRC94jCcoUJIL/Rare-diseases-
orphan-drugs.html
IASbabas Prelims 60 Day Plan Day 31 2017
(History)
Q.21) Which of the following statements about Pradhan Mantri Surakshit Matritva
Abhiyan
a) It has been launched by the Ministry of Health & Family Welfare (MoHFW)
b) It aims to provide assured, comprehensive and quality antenatal care, free of cost,
universally to all pregnant women on the 9th of every month
c) It guarantees a minimum package of antenatal care services to women in their 2nd /
3rd trimesters of pregnancy at designated government health facilities
d) All of the above
The Pradhan Mantri Surakshit Matritva Abhiyan has been launched by the Ministry of Health
& Family Welfare (MoHFW), Government of India. The program aims to provide assured,
comprehensive and quality antenatal care, free of cost, universally to all pregnant women
on the 9th of every month.
Honble Prime Minister of India highlighted the aim and purpose of introduction of the
Pradhan Mantri Surakshit Matritva Abhiyan in the 31st July 2016 episode of Mann Ki Baat.
PMSMA guarantees a minimum package of antenatal care services to women in their 2nd /
3rd trimesters of pregnancy at designated government health facilities.
The programme follows a systematic approach for engagement with private sector which
includes motivating private practitioners to volunteer for the campaign; developing
strategies for generating awareness and appealing to the private sector to participate in the
Abhiyan at government health facilities.
Q.22) The Supreme Court judgement on Sadiq Ali vs Election Commission of India is
concerned with
a) Allotment of a symbol
b) Election speeches
c) Absentee voting
d) NOTA
Source: http://www.thehindu.com/todays-paper/tp-national/Who-will-get-
%E2%80%98cycle%E2%80%99-It%E2%80%99s-poll-panel%E2%80%99s-
call/article16975517.ece
1. Any profit from the sale of a capital asset is deemed as capital gains
2. A capital asset is officially defined as any kind of property held by an assesse,
excluding goods held as stock-in-trade, agricultural land and personal effects
3. Shares and equity mutual funds alone enjoy a special dispensation on capital gains
tax
Which of the following statements is/are correct?
a) 1 and 2
b) 2 and 3
c) 1 and 3
d) All of the above
Any profit from the sale of a capital asset is deemed as capital gains.
A capital asset is officially defined as any kind of property held by an assesse,
excluding goods held as stock-in-trade, agricultural land and personal effects.
Normally if an asset is held for less than 36 months, any gain arising from selling it is
treated as a short-term capital gain (STCG) and taxed in your hands.
This becomes a long-term capital gain (LTCG) if the asset is held for 36 months or
more.
IASbabas Prelims 60 Day Plan Day 31 2017
(History)
Shares and equity mutual funds alone enjoy a special dispensation on capital gains
tax.
In their case, a holding period of 12 months or more qualifies as long-term.
Current tax laws state LTCG arising on the sale of listed equity shares or equity
oriented mutual funds are exempt from tax if you have paid Securities Transaction
Tax (STT) on the sale transaction.
STCG from such shares and funds is also taxable at a flat 15 per cent (plus surcharge
and cess).
The short-term capital loss from financial assets can be set off against any other
capital gain.
Source: http://economictimes.indiatimes.com/news/politics-and-nation/pm-narendra-
modi-hints-at-long-term-capital-gains-tax-in-budget/articleshow/56160784.cms
Q.24) China sent its first freight train to London in what is one of the world's longest train
rides. The train will pass through which of the following counties before it gets to London?
1. Belarus
2. Poland
3. Belgium
4. Kyrgyzstan
5. Russia
Select the correct code
a) 1, 2, 3 and 4
b) 1, 2, 3 and 5
c) 1, 2 and 5
d) 2, 3 and 5
The train left Yiwu West Railway Station in Zhejiang province Sunday and is headed for the
British capital.
The train is hauling household goods, bags, suitcases and garments, among other items, and
will pass through Kazakhstan, Russia, Belarus, Poland, Germany, Belgium and France before
it gets to London.
IASbabas Prelims 60 Day Plan Day 31 2017
(History)
The train is part of Chinese President Xi Jinping's vision for "One Belt, One Road" -- dubbed
by some as the new silk road. It's China's infrastructure initiative, which Xi hopes will
improve China's economic ties with Europe, Asia and the Middle East.
Source: http://indianexpress.com/article/india/chinas-freight-train-to-london-another-
step-in-exploring-ancient-trade-routes-4456591/
a) 1 and 2
b) Only 1
c) 1 and 3
d) 2 and 3
Source: http://economictimes.indiatimes.com/news/environment/global-warming/huge-
antarctic-ice-block-set-to-break-off-scientists/articleshow/56392080.cms
IASbabas Prelims 60 Day Plan Day 32 2017
(History)
Q.1) The Pattini Cult, prevalent in Sangam age was associated with
a) Worship of Ancestors
b) Worship of animals
c) Worship of Goddess Kali
d) Worship of Kannagi
Pattini Cult which was prevanat during the sangam age was associated with the Worship of
Kannagi, the ideal wife, Kannagi was the Wife of Kovalan, their story was the subject of the
Tamil epic Silappadigalam (The jeweled anklet).
Legend has it that Kannagi took revenge on the King of Madurai, for a mistaken death
penalty imposed on her husband Kovalan, by cursing the city with disaster.
Q.2) Which of the following statements about the Fourth Buddhist Council is/are
incorrect?
a) 1 and 3 only
b) 1 only
c) 3 only
d) 2 and 3 only
The Fourth Buddhist Council was held in Kashmir under the patronage of king Kanishka in
1st Century A.D. It was presided over by Vasumitra. Its main purpose was to settle the
difference between all the 18 sects of Buddhism and to compose the commentaries.
It led to the divison of Buddhism into two sects, the Hinayanism and the Mahayanism.
Codification of Sarvastivadinn Doctrines into Mahavibhasa.
IASbabas Prelims 60 Day Plan Day 32 2017
(History)
The whole Council was conducted in Sanskrit instead of Pali as was done earlier. It led to the
spread of Hinayanism to Burma and Sri lanka and Mahayanism to Central Asia, China and
Japan.
Q.3) Consider the following statements with reference to the Buddhist Tripitakas
1. Sutta Pitaka contains the records of the Buddha's teachings and sermons
2. Abhidhhamma Pitaka set out the rules and guidelines for living the monastic life of
the sangha
3. All the Tripitakas were composed at the First Buddhist Council
a) 1 and 2 only
b) 2 and 3 only
c) 1 and 3 only
d) All of the above
The Tripitaka was compiled and arranged in its present form by those Arahants who had
immediate contact with the Buddha.
Immediately after the final passing away of the Buddha, 500 distinguished Arahants held a
convention known as the First Buddhist Council to rehearse the Doctrine taught by the
Buddha. Ananda, the faithful attendant of the Buddha who had the special privilege of
hearing all the discourses the Buddha ever uttered, recited the Dhamma, whilst the Upali
recited the Vinayapitaka, the rules of conduct for the Sangha.
The Tripitaka consists of three sections of the Buddha's Teachings. They are the Discipline
(Vinaya Pitaka), the Discourse (Sutta Pitaka), and Ultimate Doctrine (Abhidhamma Pitaka).
The Sutta Pitaka consists chiefly of discourses delivered by the Buddha Himself on various
occasions.
The Abhidhamma is, to a deep thinker, the most important and interesting, as it contains
the profound philosophy of the Buddha's teaching in contrast to the illuminating but simpler
discourses in the Sutta Pitaka. It was Composed at a later stage than the other two Pitakas,
which were compiled at the First Council itself.
IASbabas Prelims 60 Day Plan Day 32 2017
(History)
Q.4) In Jainism, Posadha means
a) Penance by the Jainas to confess for their sins at the end of the year
b) Fasting by lay Jain members on full and new moon day
c) Initiation ceremony for admitting the new members
d) Punishment given to the violators of the code of conduct
Posadha is a fast kept by the lay followers of the Jainism, this was to be held twice in a
month, once on Full monn and once on new moon. During Posadha, the layman was to lead
life like a monk. He could not go outside the monastery.
Q.5) The Original Pipal tree at Boudh Gaya was cut down by
a) Sasanka
b) MihirKula
c) Pushyamitra Sunga
d) Dhanananda
Pushyamitra Sunga was the founder of Sunga Dynasty of Magadha. Pushyamitra Sunga was
a military general in the Mauryan Army. He became the King by killing King Brihadratha, the
last Mauryan emperor.
Pusyamitra was a staunch Hindu, a champion of Brahmanism and that is why the Buddhist
texts show him as a cruel man.
The tree was again cut down by King Pushyamitra Shunga in the 2nd century BC, and by King
Shashanka in 600 AD.
a) Astronomy
b) Chemistry
c) Medicine
d) Geometry
IASbabas Prelims 60 Day Plan Day 32 2017
(History)
Sulvasutras are sutra texts belonging to the rauta ritual and containing geometry related to
fire-altar construction.
The Shulba Sutras are part of the larger corpus of texts called the Shrauta Sutras, considered
to be appendices to the Vedas. They are the only sources of knowledge of Indian
mathematics from the Vedic period. Unique fire-altar shapes were associated with unique
gifts from the Gods.
Q.7) Which of the following was not a feature of Indus Valley Civilization?
The significant features of Indus Valley civilization are town planning, construction of burnt-
brick houses, ceramics, casting, forging of metals, not one clear example of horse exists in
the Indus excavations and elsewhere in North India before c. 1800 BCE.
Wheat and barley were widely used; the evidence of Rice is found in only few places.
Q.8) The only Harappan city with houses having entry from main street is
a) Chanduraho
b) Lothal
c) Kalibangan
d) Mahenjodaro
Q.9) which of the following Brahmanas describes the eastern sea and the western sea for
the first time
a) Satpatha Brahmana
b) Aithrya Brahmana
c) Gopatha Brahmana
d) Jaiminiya Brahmana
The First ever mention of the Eastern Sea (Bay of Bengal) and the Western Sea (the Arabian
Sea) is found in the Satpatha Brahmana. It is a Bramhana text of Yajurveda.
Q. 10) Mahamastakabhisheka, a great religious event, is associated with and done for
whom of the following?
a) Mahavira
b) Buddha
c) Nataraja
d) Bahubali
The Mahamastabhisheka (or Mahamasthak Abhishek) is an important Jain festival held once
every twelve years in the town of Shravanabelagola in Karnataka state, India. The festival is
held in veneration of an immense 18 meterhigh statue of the Bhagwan (or Saint)
Gomateshwara Bahubali. The anointing last took place in February 2006, and the next
ceremony will occur in 2018.
IASbabas Prelims 60 Day Plan Day 32 2017
(History)
Q.11) Which of the Following pairs are correctly matched:
Symbol : Event
1. Horse : Mahabhinishkramana
2. Bodhi Tree : Parinirvana
3. Wheel : Dharmachakrapravartana
a) 1 and 2 only
b) 2 and 3 only
c) 1 and 3 only
d) All of the above
Symbol Event
Lotus or Bull Birth of Buddha
Horse The Great Renunciation
(Mahabhinishkramana)
Bodhi tree Enlightenment (Nirvana)
Wheel First Sermon (Dhammachakraparivartan
Stupa Death of Buddha (Mahaparinirvana)
Q.12) Consider the following statements with reference to the Pavarana ceremony in
Buddhism
a) 1 only
b) 2 only
c) Both 1 and 2
d) Neither 1 or 2
IASbabas Prelims 60 Day Plan Day 32 2017
(History)
Q.12) Solution (b)
Pavarana is a Buddhist holy day celebrated on Aashvin full moon of the lunar month. It
marks the end of the 3 lunar months of Vassa.
During the three-month rainy season, the Monks live indoors, at the end of rainy season,
Pravarana ceremony takes place where every monk irrespective of rank, accept, if any,
violation of the code of conduct on their part.
Q.13) Consider the following statements about the life of Chandragupta Maurya
a) 1 only
b) 2 only
c) Both 1 and 2
d) Neither 1 or 2
The Maurya Empire was founded in 322 BCE by Chandragupta Maurya, who had overthrown
the Nanda Dynasty and rapidly expanded his power westward across central and western
India in order to take advantage of the disruptions of local powers in the wake of the
withdrawal by Alexander the Great's armies.
According to Jain accounts, Chandragupta abdicated his throne in favour of his son
Bindusara, embraced Jainism, and followed Bhadrabahu and other monks to South India. He
is said to have ended his life at Shravanabelagola (in present-day Karnataka) through
Sallekhana.
Q.14) Consider the following statements with reference to the Sangam literature
Sangam literature comprises some of the oldest extant Tamil literature, and deals with love,
war, governance, trade and bereavement. Much of the Tamil literature belonging to the
Sangam period has been lost. The literature currently available from this period is perhaps
just a fraction of the material produced during this period.
In the Sangam literature, the Tamil language had reached a level of maturity and began to
serve as a powerful and elegant medium of literary expression. Sangam Literature was
patronized by Pandyas who had their capital in Madurai.
Q.15) Tolkappiyam is a
a) Tamil epic
b) Book of sacrifices
c) Eulogy of Pandyas
d) Book on Tamil grammer
Tolkappiam is a work on the grammar of the Tamil language and the earliest extant work of
Tamil literature and linguistics.
Tolkappiyam categorises alphabet into consonants and vowels by analysing the syllables. It
grammatises the use of words and syntaxes and moves into higher modes of language
analysis. The Tolkppiyam formulated thirty phonemes and three dependent sounds for
Tamil.
IASbabas Prelims 60 Day Plan Day 32 2017
(History)
Q.16) Consider the following statements about India INX
a) 1 and 2
b) 2 and 3
c) 1 and 3
d) All of the above
The India INX is located in the country's first International Financial Services Centre (IFSC),
which is located in GIFT City. The India INX is a wholly owned subsidiary of BSE Ltd.
India INX will be the fastest international exchange in the world in terms of order response
time, with a median trade speed of four microseconds. This is better than BSE's domestic
exchange at Mumbai, which has a order response time of six microseconds.
India INX operates for 22 hours a day to allow international investors and Non Residents
Indians to trade from anywhere across the globe. In the first phase it proposes to
commence trading in equity derivatives, currency derivatives, commodity derivatives
including Index and Stocks. Subsequently, depository receipts and bonds would be offered
once the required infrastructure for the same is in place. The technology offerings at India
INX would facilitate co-location of members in its own Data Centre at GIFT IFSC as well as
provide high frequency trading,
India International exchange will compete with other international financial centres such as
Hong Kong, Singapore Dubai, London and New York. Currently RBIs Liberalised Remittance
Scheme does not allow Indian Residents to trade in derivative products traded in exchanges
other than domestic exchanges. So, effectively it will be corporate to hedge from India,
which they are currently doing from outside India. It will also help Indian start-ups to raise
equity from foreign investors by getting listed and provide companies incorporated outside
India a platform to raise money in foreign currencies by issuance and listing of their shares
on the international exchange. The stock brokers, who wish to service the rest of the world,
would set up their offices and companies at Gift City. Foreign individuals, foreign trusts,
foreign institutional investors all of them would be able to trade from India INX.
IASbabas Prelims 60 Day Plan Day 32 2017
(History)
GIFT City, which houses India INX, is part of India but not India. It is like one country, two
systems. It matches Hong Kong that helped China to earn trillions of dollars through
financial transactions. Most people used Hong Kong as gateway to China, similarly, GIFT City
is now be seen as gateway to India.
The exchange being located in IFSC, GIFT City, provides competitive advantage in terms of
tax structure and supportive regulatory framework. These include benefits in security
transaction tax, commodity transaction tax, dividend distribution tax and long-term capital
gain tax waivers and no income tax.
Source: http://www.business-standard.com/article/markets/bse-s-india-inx-10-things-to-
know-about-our-1st-international-exchange-117011000301_1.html
a) Transparency International
b) Amnesty International
c) World Economic Forum
d) World Bank
The parameters for the index ranking are functioning of public institutions like police and
judiciary, press freedom, access to information about public expenditure, stronger
standards of integrity for public official.
Source: http://www.thehindu.com/news/international/India-ranks-76-in-Corruption-
Perception-Index/article14022729.ece
a) Maharashtra
b) Rajasthan
c) Madhya Pradesh
d) Gujarat
IASbabas Prelims 60 Day Plan Day 32 2017
(History)
Source: http://www.business-standard.com/article/government-press-
release/government-to-support-tangaliya-weavers-in-purchasing-looms-textiles-minister-
117012000524_1.html
1. She along with her husband founded the first women's school at Bhide Wada in Pune
2. She opened a care centre called "Balhatya Pratibandhak Griha" for pregnant rape
victims and helped deliver their children
a) Tarabai Shinde
b) Savitribai Phule
c) Ramabai Ranade
d) Lakshmibai Tilak
Savitribai Jyotirao Phule, born on 3 January 1831, was a social reformer and poet. She
played an important role in fighting for women's rights in India during British rule and is
described as "one of the first-generation modern Indian feminists".
Phule along with her husband founded the first women's school at Bhide Wada in Pune in
1848.
IASbabas Prelims 60 Day Plan Day 32 2017
(History)
Born into a family of farmers in Naigaon, Maharashtra, she was married to 12-year-old
Jyotirao Phule at the age of nine.
The practice of child marriage was prevalent in the 19th century and since the mortality rate
was high at the time, many young girls often became widows even before attaining puberty.
Such widows used to shave their heads, wear a simple red sari and live a life of austerity. It
was Savitribai who decided to stand up against this practice and organized a strike against
the barbers in order to persuade them to stop shaving the heads of the widow.
She noticed the plight of women who after falling prey to sexual exploitation, and becoming
pregnant, either committed suicide or killed the newborn due to fear of banishment by the
society. To cater to such women she opened a care center for pregnant rape victims and
helped deliver their children.
The care center was called "Balhatya Pratibandhak Griha" (Infanticide prohibition house).
Phule also worked to abolish discrimination and unfair treatment of people based on caste
and gender.
She found the treatment of the untouchables problematic and opened a well in her house in
1868 so that, people who were refused drinking water by the upper caste can use it.
She died while taking care of the patients suffering during the worldwide Third Pandemic of
the bubonic (bacterial infection) plague.
Source: http://indiatoday.intoday.in/story/who-is-savitribai-phule-what-did-she-do-for-
womens-right-in-india/1/561392.html
1. Gibraltar is a French Overseas Territory located on the southern end of the Iberian
Peninsula
2. The Iberian Peninsula is divided between Portugal, Spain, Algeria and Andorra
3. It is the largest European Peninsula
a) 1 and 3
b) 1 and 2
c) 2 and 3
d) All the statements are wrong
IASbabas Prelims 60 Day Plan Day 32 2017
(History)
The Iberian Peninsula is located in the southwest corner of Europe. The peninsula is
principally divided between Portugal and Spain, comprising most of their territory. It also
includes Andorra and a small part of France along the peninsula's northeastern edge, as well
as Gibraltar on its south coast, a small peninsula which forms an overseas territory of the
United Kingdom. With an area of approximately 582,000 km2 (225,000 sq mi), it is the
second largest European peninsula, after the Scandinavian.
The Gibraltar Arc is a geological region corresponding to an arcuate orogen surrounding the
Alboran Sea, between the Iberian Peninsula and Africa. It consists of the Betic Cordillera
(south Spain), and the Rif (North Morocco).The Gibraltar Arc is located at the western end of
the Mediterranean Alpine belt and formed during the Neogene due to convergence of the
Eurasian and African plates.
Source: http://www.thehindu.com/todays-paper/tp-opinion/Demystifying-
Science/article17111165.ece
a) Bodhgaya
b) Sarnath
c) Gangtok
d) None of the above
IASbabas Prelims 60 Day Plan Day 32 2017
(History)
The Kanishka stupa was a monumental stupa established by the Kushan king Kanishka
during the 2nd century CE in today's Shaji-ki-Dheri on the outskirts of Peshawar, Pakistan.
The magnificent stupa was built during the Kushan era to house Buddhist relics, and would
become perhaps the tallest buildings in the ancient world.
Source: http://www.thehindu.com/news/international/Pakistan-govt-urged-to-ensure-
8th-wonder-tag-to-Kanishka-Stupa/article17101715.ece
News: The Fiscal Responsibility and Budget Management (FRBM) Committee has submitted
its 4 volume report on changes in FRBM Act, 2013
Committee -
The 5 member committee was headed by N.K. Singh, former Revenue and Expenditure
Secretary and former MP.
Its member included RBI Governor Urjit Patel, Chief Economic Advisor Arvind Subramanian,
former Finance Secretary Sumit Bose, and National Institute of Public Finance and Policy
Director Rathin Roy.
Highlights
The panel considered the possibility of replacing absolute fiscal deficit targets with a
target range that may be adjusted in line with the overall credit trends in the economy.
The ambit of the committee included reviewing the working of the FRBM Act over
last 12 years and to suggest the way forward, keeping in view the broad objective of
IASbabas Prelims 60 Day Plan Day 32 2017
(History)
fiscal consolidation and prudence and the changes required in the context of the
uncertainty and volatility in the global economy.
The Committee examined the feasibility of having a fiscal deficit range as the target
instead of the existing fixed numbers.
The Committee also examined the need and feasibility of aligning the fiscal expansion
or contraction with credit contraction or expansion respectively in the economy.
The Report
The first volume of the report addresses the issue of the fiscal roadmap, fiscal policy,
international experience and recommendations therein.
The second volume refers to international experience especially from a lot of
international organisations particularly OECD, the World Bank, ILO.
The third volume deals with Centre-State issues. The fourth volume deals with views
of domain experts both from national and international appropriate for fiscal policy
Source: http://www.thehindu.com/business/N.K.-Singh-panel-submits-report-on-FRBM-
Act/article17083134.ece
The government transferred the role of advising the government on how to utilise the
proceeds from disinvestment from the Department of Investment and Public Asset
Management (DIPAM) to the Department of Economic Affairs.
Development Administration Partnership (DPA) was created in the Ministry of External Affairs
in January 2012 to effectively handle Indias aid projects through the stages of concept,
launch, execution and completion.
DPA has three Divisions. Currently, DPA I deals with project appraisal and lines of credit; DPA
II deals with capacity building schemes, disaster relief, Indian Technical and Economic
Cooperation Programme and DPA III deals with project implementation.
In News - http://indianexpress.com/article/india/india-us-sign-mou-for-collaboration-in-
third-world-countries-4473555/
Q.25) Recently, a place called Aswan was in news. It is located in which of the following
countries?
a) Morocco
b) Egypt
c) Algeria
d) Armenia
Human and animal remains were found in the cemeteries, which were discovered in the
Gabal al-Silsila or Chain of Mountains area 65 km (40.3 miles) north of Aswan.
IASbabas Prelims 60 Day Plan Day 32 2017
(History)
Source: http://www.livemint.com/Science/g2llmmYaTQtsityQDhL8bO/12-ancient-Egyptian-
cemeteries-discovered-near-Aswan.html
IASbabas Prelims 60 Day Plan Day 33 2017
(Economics)
Q.1) Proceeds from which of the following is not credited to the Consolidated Fund of
India
Under Article 266 (1) of the Constitution of India, all revenues ( example tax revenue from
personal income tax, corporate income tax, customs and excise duties as well as non-tax
revenue such as licence fees, dividends and profits from public sector undertakings etc. )
received by the Union government as well as all loans raised by issue of treasury bills,
internal and external loans and all moneys received by the Union Government in repayment
of loans shall form a consolidated fund entitled the 'Consolidated Fund of India' for the
Union Government.
Except the loans that are credited to the Consolidated Fund, government does not have the
liability to pay it back. It is a one-way transaction only.
Proceeds from the Small Savings Scheme, Provident Fund etc, where the government acts as
the custodian of the money is credited to the Public Fund of India established under the Art
266 (2) of the Indian Constitution.
Q.2) Consider the following statements with reference to the National Investment Fund
(NIF)
1. Money raised through the Disinvestment of the PSUs is credited to the NIF
2. All the profits generated by NIF can only be used to fund Social Sector Schemes
a) 1 only
b) 2 only
c) Both 1 and 2
d) Neither 1 or 2
Government decided (17th January 2013) that the disinvestment proceeds, with effect from
the fiscal year 2013-14, will be credited to the existing NIF which is a Public Account under
the Government Accounts and the funds would remain there until withdrawn/invested for
the approved purposes. It was also simultaneously decided that the NIF would be utilized
for the following purposes:
Subscribing to the shares being issued by the CPSE on rights basis, Recapitalization of public
sector banks and public sector insurance companies, Investment by Government in
RRBs/IIFCL/NABARD/Exim Bank, Equity infusion in various Metro projects, Investment in
Indian Railways towards capital expenditure.
Only a part of the NIF corpus can be used to fund the Social Sector Schemes, rest has to be
invested so that the Permanent Corpus is not depleted.
Q.3) Study the table given below and answer the question.
a) Progressive Taxation
b) Regressive Taxation
c) Proportional Taxation
d) None of the above
Regressive Tax: A regressive tax is a tax imposed in such a manner that the tax rate
decreases as the amount subject to taxation increases
IASbabas Prelims 60 Day Plan Day 33 2017
(Economics)
A regressive tax is a tax that takes a larger percentage of income from low-income earners
than from high-income earners. It is in opposition with a progressive tax, which takes a
larger percentage from high-income earners. A regressive tax is generally a tax that is
applied uniformly to all situations, regardless of the payer.
1. In indirect taxes the Incidence and Impact both lie on the same point
2. Indirect taxes are inherently regressive in nature
a) 1 only
b) 2 only
c) Both 1 and 2
d) Neither 1 or 2
Tax incidence is said to "fall" upon the group that ultimately bears the burden of, or
ultimately has to pay, the tax. The term incidence refers to the location of the ultimate or
the direct money burden of the tax as such. It signifies the settlement of the tax burden on
the ultimate tax payer.
The term impact is used to express the immediate result of or original imposition of the tax.
The impact of a tax is on the person on whom it is imposed first. The impact of a tax, as
such, denotes the act of impinging.
Indirect taxes are regressive in nature as people with low incomes have part with larger
amount as tax, while people with higher incomes will find these taxes very small.
E.g. if sales tax (indirect tax) is 10% on a commodity priced 100 rs, then a person earning
200rs will have to part with 5% of his income, while a person earning 500rs will only have to
pay 2% of his income as tax
a) Corporate tax
b) Income tax
c) Custom duties
d) Wealth tax
IASbabas Prelims 60 Day Plan Day 33 2017
(Economics)
Custom duties are an example of indirect taxes. Rest all the options are examples of Direct
taxes. In direct taxes the tax is paid on the first point of Contact, it cannot be moved further.
It has to be paid by the person on whom it is levied.
Q.6) Consider the following statements about taxes levied on imported goods
1. Countervailing duty is levied on goods that are sold below the prices in exporting
country
2. Anti-dumping duty is levied to counterbalance the subsidies provided by the
government of exporting country
a) 1 only
b) 2 only
c) Both 1 and 2
d) Neither 1 or 2
Anti-dumping Duties are levied on the goods that are either sold either, below the cost of
production or below the prices in the home country. This is considered predatory as it tries
to drive-out the products of importing country and create a monopoly. To prevent this from
happening, anti-dumping duty is levied
Q.7) Consider the following statements with reference to the Fourteenth Finance
Commission
1. It has recommended that the Fiscal deficit should be brought to Zero by 2020
IASbabas Prelims 60 Day Plan Day 33 2017
(Economics)
2. It has recommended setting up of an independent Fiscal Council
a) 1 only
b) 2 only
c) Both 1 and 2
d) Neither 1 or 2
The Fourteenth Finance Commission (FFC) had recommended that the Fiscal deficit to be
maintained below 3% for the period of 2015-2020. States will be eligible for a flexibility of
0.25% over this limit. They will be eligible for this flexibility if their debt-GSDP ratio is less
than or equal to 25% in the previous year
An independent fiscal council should be created to evaluate the fiscal policy implications of
budget proposals.
Q.8) Consider the following statements with reference to Financial Sector Legislative
Reforms Commission (FSLRC)
a) 1 and 2 only
b) 2 and 3 only
c) 1 and 3 only
d) All of the above
The Financial Sector Legislative Reforms Commission (FSLRC) has recommended revamping
the legislative framework governing the financial sector by a non-sectoral, principle-based
approach and restructuring existing regulatory agencies and creating new agencies
wherever needed. The FSLRC has given a draft legislation namely Indian Financial Code as
Volume-II of its report. The basic approach of the FSLRC is to provide clear mandate and
powers and mechanism for accountability to financial agencies.
Under the Draft Financial Code recommended by the commission RBI will regulate Banking
and payment System, and the UFA will subsume regulators such as SEBI, FMC and PFRDA
The FSLRC recommended a seven agency structure for the financial sector which are the
Reserve Bank of India (RBI), Unified Financial Agency (UFA), Financial Sector Appellate
Tribunal (FSAT), Resolution Corporation (RC), Financial Redressal Agency (FRA), Financial
Stability and Development Council (FSDC) and Public Debt Management Agency (PDMA).
Q.9) Consider the following statements with reference to the Financial Stability and
Development Council (FSDC)
a) 1 only
b) 2 only
c) Both 1 and 2
d) Neither 1 or 2
Financial Stability and Development Council was setup with a view to strengthen and
institutionalize the mechanism for maintaining financial stability and enhancing inter-
regulatory coordination, Indian Government has setup an apex-level (FSDC), vide its
notification dated 30th December, 2010. (not by the Act of Parliament)
IASbabas Prelims 60 Day Plan Day 33 2017
(Economics)
The Chairman of the FSDC is the Finance Minister of India and its members include the
heads of the financial sector regulatory authorities (i.e, SEBI, IRDA, RBI, PFRDA and FMC),
Finance Secretary and/or Secretary, Department of Economic Affairs (Ministry of Finance),
Secretary, (Department of Financial Services, Ministry of Finance) and the Chief Economic
Adviser.
a) 1 only
b) 2 only
c) Both 1 and 2
d) Neither 1 or 2
Fiscal Consolidation refers to the policies undertaken by Governments (national and sub-
national levels) to reduce their deficits and accumulation of debt stock. In order to run the
country effectively, the government has to use the Finances at its disposal with due
diligence, over dependence on either debt will lead to debt trap which will scareaway the
investors and will harm the countrys reputation. The government must be able to generate
sufficient revenue to run the economy without having to resort to loans.
1. The extent of fiscal deficit is an indication of how far the government is spending
beyond its means.
2. If unaddressed fiscal deficit can lead to lead to debt trap
a) 1 only
b) 2 only
c) Both 1 and 2
d) Neither 1 nor 2
IASbabas Prelims 60 Day Plan Day 33 2017
(Economics)
Fiscal Deficit = Total Expenditure Total Receipts excluding borrowings. The extent of fiscal
deficit is an indication of how far the government is spending beyond its means.
Fiscal deficit indicates the total borrowing requirements of the government. Borrowings not
only involve repayment of principal amount, but also require payment of interest.
Interest payments increase the revenue expenditure, which leads to revenue deficit. It
creates a vicious circle of fiscal deficit and revenue deficit, wherein government takes more
loans to repay the earlier loans. As a result, country is caught in a debt trap.
a) 1 only
b) 2 only
c) Both 1 and 2
d) Neither 1 or 2
The Tobin tax was originally proposed in the early 1970s by James Tobin, an influential
American macroeconomist and recipient of the Nobel prize for economics.
Tobin proposed to reduce this volatility with a small tax for instance 0.1 per cent levied
on every amount exchanged from one currency into another.
Q.13) Consider the following statements with reference to the Goods and Services Tax
a) 1 only
b) 2 only
c) Both 1 and 2
d) Neither 1 or 2
The GST is a Value added Tax (VAT) is proposed to be a comprehensive indirect tax levy on
manufacture, sale and consumption of goods as well as services at the national level. It will
replace all indirect taxes levied on goods and services by the Indian Central and state
governments. It is aimed at being comprehensive for most goods and services.
GST is set to become one of the biggest fiscal reform that our country is going to witness. All
businesses, small or large are going to get impacted because of this paradigm shift in the
indirect tax regime. Policymakers have consistently resonated the benefit of a unified
taxation system in a federal country like India.
There is a long list of benefits, which are being claimed as a result of GST law and one such
benefit is removal of the cascading tax effect. In simple words cascading tax effect means
tax on tax. It is a situation wherein a consumer has to bear the load of tax on tax and
inflationary prices as a result of it.
a) Subsidies
b) Grants given to state governments
c) Loans given to state governments
d) Interest payments on loans taken by the Centre
Revenue expenditure is that expenditure which will be consumed in the given year, they are
not used to create any assets. Revenue expenditure consists of Police and Public
Administration, Salaries and pensions of govt employees, Subsidies provided by the
government, grants given to the state governments, Interest paid and received in a year.
IASbabas Prelims 60 Day Plan Day 33 2017
(Economics)
Loans given to the states are part of Capital expenditure, as it will have to be paid back in
the future.
a) Specific duty
b) Ad-Volerem Tax
c) Countervailing duty
d) Anti-dumping duty
The Latin phrase, ad valorem, means "according to value." In short, all ad valorem taxes are
levied based on the determined value of the item being taxed.
Charge levied as a percentage of value of the item it is imposed on, and not on the item's
quantity, size, weight, or other such factor. Value added tax (VAT) and, generally, import
duties are ad valorem taxes.
If the import duty is on the size of the goods imported, then it is called as specific duty.
Q.16) Consider the following statements about Trade Receivables Discounting System
(TReDS)
a) Only 1
b) Only 2
c) Both 1 and 2
d) Neither 1 nor 2
It is an electronic platform that allows auctioning of trade receivable. The process is also
commonly known as bills discounting, a financier (typically a bank) buying a bill (trade
receivable) from a seller of goods before its due or before the buyer credits the value of the
bill. In other words, a seller gets credit against a bill which is due to him at a later date. The
discount is the interest paid to the financier.
As per RBI TReDS guidelines, only MSMEs can participate as sellers, while banks, non-
banking financial companies and factoring companies are permitted as financiers.
The financiers cant bid below marginal cost of funds-based lending rate (MCLR) rate set by
the RBI. Typically, for buyers with good credit ratings, financiers bid near the MCLR rate. The
spread widens depending upon the buyers' credit rating.
RBI has given license to three entities and they are governed by the Payment And
Settlement Systems Act. These are Receivables Exchange of India (RXIL), which is a joint-
venture between National Stock Exchange and SIDBI; A Treds, a joint-venture between Axis
Bank and Mjunction Services; and Mynd Solution. RXIL was the first one to go live on
January 9.
All the transactions undertaken on the TReDS have to be registered with the Central
Registry of Securitization and Asset Reconstruction and Security Interest of India.
Currently, only banks and certain NBFCs are allowed to be financiers. Experts say even other
participants like high networth individuals should be allowed to act as a financier to expand
the market.
Source: http://www.thehindu.com/business/Economy/%E2%80%98TReDing%E2%80%99-
the-path-to-quick-payments/article17009288.ece
Q.17) Which of the following countries are members of East African Community?
1. Burundi
2. Kenya
3. Angola
4. Nigeria
5. Tanzania
6. Ivory Coast
IASbabas Prelims 60 Day Plan Day 33 2017
(Economics)
Select the correct code:
a) 1, 2 and 5
b) 2, 3, 4 and 6
c) 2, 3, 4 and 5
d) 1, 3 and 6
The work of the EAC is guided by its Treaty which established the Community. It was signed
on 30 November 1999 and entered into force on 7 July 2000 following its ratification by the
original three Partner States - Kenya, Tanzania and Uganda. The Republic of Rwanda and the
Republic of Burundi acceded to the EAC Treaty on 18 June 2007 and became full Members
of the Community with effect from 1 July 2007. The Republic of South Sudan acceeded to
the Treaty on 15 April 2016 and become a full Member on 15 August 2016.
Source: http://www.thehindu.com/todays-paper/tp-opinion/Reaching-out-to-
Africa/article17020813.ece
a) Only 1
b) Only 2
c) Both 1 and 2
d) Neither 1 nor 2
IASbabas Prelims 60 Day Plan Day 33 2017
(Economics)
Q.18) Solution (d)
After the formation of Chhattisgarh State, the major portion of Mahanadi basin now lies in
Chhattisgarh. Presently, only 154 square kilometres (59 sq mi) basin area of Hasdeo River in
Anuppur District lies in Madhya Pradesh.
The Seonath River is the longest tributary of Mahanadi. It rises in an undulating region with
numerous small groups of hills at Kotgal and flows 383 kilometers to join Mahanadi at its
left bank at Khargand.
Brahmani River is NOT a tributary of Mahanadi. Its a seasonal river that flows in Odisha.
Source: http://www.thehindu.com/todays-paper/tp-national/Odisha-rejects-panel-on-
Mahanadi/article17111488.ece
Q.19) Consider the following statements about Financial Intelligence Unit of India (FIU-
IND)
a) Only 1
b) Only 2
c) Both 1 and 2
d) Neither 1 nor 2
Financial Intelligence Unit India (FIU-IND) was set by the Government of India vide O.M.
dated 18th November 2004 as the central national agency responsible for receiving,
processing, analyzing and disseminating information relating to suspect financial
transactions. FIU-IND is also responsible for coordinating and strengthening efforts of
national and international intelligence, investigation and enforcement agencies in pursuing
the global efforts against money laundering and related crimes. FIU-IND is an independent
IASbabas Prelims 60 Day Plan Day 33 2017
(Economics)
body reporting directly to the Economic Intelligence Council (EIC) headed by the Finance
Minister.
The members are inducted from organizations including Central Board of Direct Taxes
(CBDT), Central Board of Excise and Customs (CBEC), Reserve Bank of India (RBI), Securities
Exchange Board of India (SEBI), Department of Legal Affairs and Intelligence agencies
The function of FIU-IND is to receive cash/suspicious transaction reports, analyse them and,
as appropriate, disseminate valuable financial information to intelligence/enforcement
agencies and regulatory authorities. The functions of FIU-IND are:
Collection of Information: Act as the central reception point for receiving Cash
Transaction reports (CTRs), Cross Border Wire Transfer Reports (CBWTRs), Reports
on Purchase or Sale of Immovable Property (IPRs) and Suspicious Transaction
Reports (STRs) from various reporting entities.
Analysis of Information: Analyze received information to uncover patterns of
transactions suggesting suspicion of money laundering and related crimes.
Sharing of Information: Sharing information with national intelligence/law
enforcement agencies, national regulatory authorities and foreign Financial
Intelligence Units.
Act as Central Repository: Establish and maintain national data base on cash
transactions and suspicious transactions on the basis of reports received from
reporting entities.
Coordination: Coordinate and strengthen collection and sharing of financial
intelligence through an effective national, regional and global network to combat
money laundering and related crimes.
Q.20) ________________ has been ranked the most dynamic city in the world in JLL's City
Momentum Index (CMI)
a) Bengaluru
b) Mumbai
c) Pune
d) Hyderabad
Bengaluru has been ranked the most dynamic city in the world, ahead of Silicon Valley and
Boston in the US, in a study by independent property consultancy Jones Lang LaSalle
IASbabas Prelims 60 Day Plan Day 33 2017
(Economics)
(JLL).The ranking is based on a broad range of real estate and socio-economic factors.
Bengaluru's positioning has been particularly driven by ts successful transitioning to higher-
value manufacturing and service-based activities.
The list, which features 30 cities, also includes five other Indian cities including Hyderabad
(5), Pune (13), Chennai (18), Delhi (23) and Mumbai (25).
Source: http://timesofindia.indiatimes.com/trend-tracking/bengaluru-most-dynamic-city-
in-world-jll-study/articleshow/56658813.cms
Q.21) India officially received certificates from World Health Organisation (WHO),
declaring the country yaws and maternal and neonatal tetanus free. What is Yaws?
a) It is a chronic bacterial infection that affects the skin, cartilages and bones.
b) It is a chronic viral infection that affects throat
c) It is a bacterial infection that affects throats
d) None of the above
Yaws is a chronic bacterial infection that affects the skin, cartilages and bones. It is caused
by the spirochete bacterium Treponema pallidum pertenue. It spreads by direct contact
with the fluid from a lesion of an infected person. It mainly affects children below 15 years
of age. It mainly occurs in overcrowded communities with limited access to basic amenities
such as water, sanitation and health care services.
Niryat Bandhu - Hand Holding Scheme for new export import entrepreneurs
IASbabas Prelims 60 Day Plan Day 33 2017
(Economics)
DGFT is implementing the Niryat Bandhu Scheme for mentoring new and potential exporter
on the intricacies of foreign trade through counseling, training and outreach programs.
Considering the strategic significance of small and medium scale enterprises in the
manufacturing sector and in employment generation, MSME clusters have been identified,
based on the export potential of the product and the density of industries in the cluster, for
focused interventions to boost exports.
Outreach activities shall be organized in a structured way with the help of Export Promotion
Councils as industry partners and other willing knowledge partners in academia and
research community to achieve the objective of Niryat Bandhu Scheme. Further, in order to
ensure optimum utilization of resources, efforts would be made to associate all the
stakeholders, including Customs, ECGC, Banks and concerned Ministries.
a) Chumbi Valley
b) Spiti Valley
c) Lohab Valley
d) Nubra Valley
Located in Spiti Valley, one of the most uninhabited parts of the country, Himansh is
considered to be the highest point from where an Indian glacier research facility is
functioning.
Q.24) Recently NASA developed an electric bandage to speed up the healing process of
wounds. It is made up of?
a) Only 3
b) 1 and 2
c) 2 and 3
d) 1, 2 and 3
Source: https://www.unicef.org/media/files/Position_Paper_Ready-to-
use_therapeutic_food_for_children_with_severe_acute_malnutrition__June_2013.pdf
IASbabas Prelims 60 Day Plan Day 34 2017
(Economics)
Q.1) Which of the following is not a Money Market instrument?
a) Treasury Bills
b) Repurchase Agreements
c) Shares of a Company
d) Certificate of Deposit
Money Market is a place for short term lending and Borrowing normally within a year but
terms of upto three years are not uncommon. It deals in short term debt financing and
investments. Investment in money market is done throughout money market instruments.
Money market instrument meets short term necessities of the borrowers and provides
liquidity to the lenders. Money Market Instruments are as follows:
Shares of a company are bought and sold in the capital markets which are instruments of
long term capital.
Q.2) Consider the following statements with reference to the Capital Markets
1. Capital Markets consists of long term borrowing and lending through bonds only
2. Secondary markets in India are regulated by the RBI
a) 1 only
b) 2 only
c) Both 1 and 2
d) Neither 1 nor 2
Capital markets consists of financial instruments that deal with raising the capital for long
term, typically greater than 3 years. Capital is raised both through equity (sales of shares) as
IASbabas Prelims 60 Day Plan Day 34 2017
(Economics)
well as by debt (sale of bonds and debentures). capital markets comprise of Primary
markets, secondary markets and long term finance institutions. Primary and secondary
markets are regulated by SEBI and not RBI
Q.3) Consider the following statements with reference to the Demutualization of the
Stock exchanges
a) 1 only
b) 2 only
c) Both 1 and 2
d) Neither 1 nor 2
Demutualization of Stock exchanges means Changing mutually owned companies into for-
profit company by issuing shares. This is done to separate the ownership of the company
from the management of the company
The Stock Exchanges that are association of persons shall be converted into a for-profit
company limited by shares. The Exchanges that are companies limited by guarantee shall be
re-registered as companies limited by shares. The ownership and management rights and
trading rights associated with membership cards shall be segregated. It shall not be
necessary for a shareholder to be a trading member and vice versa. The membership
cardholders shall become initial shareholders of the Exchange which shall ensure that at
least 51% of its equity shares are held by public other than shareholders having trading
rights within 12 months. No shareholder, who is a trading member, shall have voting rights
(taken together with voting rights held by him and by persons acting in concert with him)
exceeding specified percentage of the voting rights in the Exchange.
a) 1 only
b) 2 only
c) Both 1 and 2
d) Neither 1 nor 2
The primary function of SEBI is regulating the stock markets in India. Until sep 2015, the SEBI
was only regulator of the stock markets and the regulation of trade in Agricultural products
was carried out by Forward Markets Commission. On 28 September 2015 the FMC was
merged with the Securities and Exchange Board of India (SEBI). So, now, SEBI is the
regulator of both the sectors.
Q.5) Consider the following statements with reference to the Foreign Direct Investment
1. FDI inflows are highly volatile as the capital can leave the country overnight
2. FDI is preferable over debt financing as it does not create interest obligations
a) 1 only
b) 2 only
c) Both 1 and 2
d) Neither 1 nor 2
FDI is more stable and less volatile in nature, in contrast the Foreign Portfolio Investments
are highly volatile in nature. They are also called the Hot Money, as they can leave India
overnight to invest in other markets.
FDI being more stable in nature involves investments with long term profits in mind. They
generate profits by locally producing the goods and services, as such are more preferable
over the Debt financing as it creates interest obligations even when the business is not
running well.
IASbabas Prelims 60 Day Plan Day 34 2017
(Economics)
Q.6) Consider the following statements about Foreign Portfolio Investment (FPI)
1. 1 and 2 only
2. 2 and 3 only
3. 1 and 3 only
4. All of the above
Thus, FPI is the passive holding of securities and other financial assets by a foreign firm,
which does not entail management control of the issuing firm. The returns in the case of FPI
are generally in the form of non-voting dividends or interest payments.
Q7) Consider the following statements with reference to the Start-up India Fund
a) 1 only
b) 2 only
c) Both 1 and 2
d) Neither 1 nor 2
The fund does not directly provide finance to the startups, rather it distributes the corpus
among various registered Venture capital funds that will in turn provide the funding to the
startups. This would encompass support at seed stage, early stage and growth stage.
The corpus of FFS is Rs.10,000 crore which shall be built up over the 14th and 15th Finance
Commission cycles subject to progress of the scheme and availability of funds. An amount of
Rs.500 crore has already been provided to the corpus of FFS in 2015-16 and Rs.600 crore
earmarked in the 2016-17.
Q.8) Consider the following statements with reference to the Venture capitalists (VCs)
a) 1 and 2 only
b) 2 and 3 only
c) 1 and 3 only
d) All of the above
Venture capitalists are the mostly the Limited Liability Partnership firms/funds, which raises
fund from different investors. As against Angel investment where the decision of investment
rests with the individual, a Fund/ Portfolio Manager in Venture Capital firms is the one who
hunts for promising deals to get the best returns for their investors money.
VCs have fund/ portfolio managers to manage their investment portfolio. They have a
dedicated and skilled team, which looks out for promising opportunities, and get the deal
closed.
Venture Capitals generally invests in Growth stage (Series A) and forward, when the
company has some proven numbers. As compared to angels they are less risk takers.
IASbabas Prelims 60 Day Plan Day 34 2017
(Economics)
VCs investment can be either as equity or loan or a mix of both. If it is done by equity they
demand a seat in the Board of the company.
Q.9) Consider the following statements with reference to Initial public offering (IPO)
1. IPO is the process through which the shares are sold to the public for the first time
2. Ones the shares are purchased by the general public they can be sold in the Stock
Exchanges
a) 1 only
b) 2 only
c) Both 1 and 2
d) Neither 1 nor 2
An initial public offering, or IPO, is the very first sale of stock issued by a company to the
public. Prior to an IPO the company is considered private, with a relatively small number of
shareholders made up primarily of early investors. The public, on the other hand, consists of
everybody else any individual or institutional investor who wasnt involved in the early
days of the company and who is interested in buying shares of the company.
After the conclusion of IPO, the stocks are listed on the stock exchanges and the people can
buy and sell the shares that they are holding.
Q.10) Consider the following statements with reference to Indian Depository Receipts
(IDRs)
a) 1 only
b) 2 only
c) Both 1 and 2
IASbabas Prelims 60 Day Plan Day 34 2017
(Economics)
d) Neither 1 nor 2
A foreign company can access Indian securities market for raising funds through issue of
Indian Depository Receipts (IDRs). An IDR is an instrument denominated in Indian Rupees in
the form of a depository receipt created by a Domestic Depository (custodian of securities
registered with the Securities and Exchange Board of India) against the underlying equity of
issuing company to enable foreign companies to raise funds from the Indian securities
Markets.
a) 1 only
b) 2 only
c) Both 1 and 2
d) Neither 1 nor 2
The shares of the Indian companies are the underlying assets based on which P-notes are
created, hence they are derivative instruments.
1. In rights issue, the existing shareholders are given the first right to additional shares
2. In rights issue, the existing shareholders are issued additional shares for free
IASbabas Prelims 60 Day Plan Day 34 2017
(Economics)
Which of the following statements is/are correct?
a) 1 only
b) 2 only
c) Both 1 and 2
d) Neither 1 nor 2
If the new shares are offered free of cost, then it becomes a bonus issue.
Q.13) Consider the following statements about Offer For Sale (OFS) issue
a) 1 only
b) 2 only
c) Both 1 and 2
d) Neither 1 nor 2
Unlike a follow-on public offering (FPO), where companies can raise funds by issuing fresh
shares or promoters can sell their existing stakes, or both, the OFS mechanism is used only
when existing shares are put on the block. Only promoters or shareholders holding more
than 10 per cent of the share capital in a company can come up with such an issue.
It makes it easier for promoters of publicly-traded companies to cut their holdings and
comply with the minimum public shareholding norms by June 2013. The method was largely
adopted by listed companies, both state-run and private, to adhere to the Sebi order. Later,
IASbabas Prelims 60 Day Plan Day 34 2017
(Economics)
the government started using this route to divest its shareholding in public sector
enterprises
a) 1 only
b) 2 only
c) Both 1 and 2
d) Neither 1 nor 2
Preference shares have the characteristics of both equity shares and debentures. Like equity
shares, dividend on preference shares is payable only when there are profits and at the
discretion of the Board of Directors
Q.15) Recently the NITI Ayog has suggested Strategic Sale of nearly 22 PSUs, in this
context consider the following statements with reference to Strategic Sale of PSUs
1. In Strategic Scale the government retains the Majority ownership of the company
2. In Strategic Sale the governments stake is sold to the highest bidder
1. 1 only
2. 2 only
IASbabas Prelims 60 Day Plan Day 34 2017
(Economics)
3. Both 1 and 2
4. Neither 1 nor 2
Strategic sale takes place when more than 51% of shares go to the private sector strategic
partner. This means the effective ownership of the company rests with the private partner.
According to the strategic sale guidelines in India, the Strategic Partner, after the
transaction, may hold less percentage of shares than the Government but the control of
management would be with him.
1. Capital gains tax can be defined as a source of custom and excise revenue
2. In case of Shares and equity mutual funds holding period of 36 months or more
qualifies as long-term capital gain
a) Only 1
b) Only 2
c) Both 1 and 2
d) Neither 1 nor 2
Any profit from the sale of a capital asset is deemed as capital gains. A capital asset is
officially defined as any kind of property held by an assessee, excluding goods held as stock-
in-trade, agricultural land and personal effects.
Normally if an asset is held for less than 36 months, any gain arising from selling it is treated
as a short-term capital gain (STCG) and taxed in your hands. This becomes a long-term
capital gain (LTCG) if the asset is held for 36 months or more. Shares and equity mutual
funds alone enjoy a special dispensation on capital gains tax. In their case, a holding period
of 12 months or more qualifies as long-term.
IASbabas Prelims 60 Day Plan Day 34 2017
(Economics)
Current tax laws state LTCG arising on the sale of listed equity shares or equity oriented
mutual funds are exempt from tax if you have paid Securities Transaction Tax (STT) on the
sale transaction. STCG from such shares and funds is also taxable at a flat 15 per cent (plus
surcharge and cess). The short-term capital loss from financial assets can be set off against
any other capital gain.
Q.17) ______________ is the only Indian destination that features in The Association of
British Travel Agents (ABTA) '12 Destinations to Watch for this year'.
a) Kerala
b) Sikkim
c) Meghalaya
d) Rajasthan
Source: http://indiatoday.intoday.in/story/kerala-destination-india-abta-tourism-travel-
liftr/1/848311.html
Q.18) Consider the following statements about Socio Economic and Caste Census 2011
(SECC 2011)
a) Only 1
b) Only 2
c) Both 1 and 2
d) Neither 1 nor 2
IASbabas Prelims 60 Day Plan Day 34 2017
(Economics)
Q.18) Solution (c)
The rural development ministry has taken a decision to use the SECC data in all its
programmes such as MGNREGA, National Food Security Act, and the Deen Dayal Upadhyaya
Grameen Kaushalya Yojana. SECC 2011 was the first-ever caste-based census since 1931
Census of India.
It will adopt the Socio-Economic and Caste Census (SECC) instead of the poverty line-based
method to identify recipients for its pro-poor schemes. The SECC 2011 ranks households
based on their socio-economic status to enable state governments to prepare a list of
families living below the poverty line. It also makes available information regarding the
socio-economic condition and education status of various castes and sections of the
population.
The Ministry of Rural Development will start using SECC 2011 data this year for its National
Social Assistance Programme to pay pension to rural poor and National Rural Livelihood
Mission. Both schemes use BPL data to estimate the number of the poor. BPL data tells us
how many are poor and SECC who are those poor it is a more targeted and scientific
approach in ensuring the right person gets the benefit
Q.19) Consider the following statements about Financial Stability and Development
Council (FSDC)
a) 1 and 2
b) 2 and 3
c) 1 and 3
d) All of the above
IASbabas Prelims 60 Day Plan Day 34 2017
(Economics)
Q.19) Solution (a)
The Central Government had established Financial Stability and Development Council (FSDC)
in December 2010 with the Finance Minister as it Chairman. The idea to create it was first
mooted by the Raghuram Rajan Committee on Financial Sector Reforms in 2008. It is a super
regulatory body for regulating financial sector which is a vital for bringing healthy and
efficient financial system in the economy.
The Chairman of the FSDC is the Finance Minister of India and its members include the
heads of the financial sector regulatory authorities.
A sub-committee of FSDC has also been set up under the chairmanship of Governor RBI. The
Sub-Committee discusses and decides on a range of issues relating to financial sector
development and stability including substantive issues relating to inter-regulatory
coordination.
As a result of the deliberations of the Sub-Committee of the FSDC held on August 16, 2011,
two Technical Groups were set up a Technical Group on Financial Inclusion and Financial
Literacy and an Inter Regulatory Technical Group.
Q.20) Consider the following statements about Pradhan Mantri Kaushal Vikas Yojana
(PMKVY)
a) Only 1
b) Only 2
c) Both 1 and 2
d) Neither 1 nor 2
IASbabas Prelims 60 Day Plan Day 34 2017
(Economics)
Q.20) Solution (b)
PKVY will provide training and certify Indians who are seeking overseas employment in
selected sectors that have high demand in the global labour market in line with
international standards. It will be implemented by the National Skill Development
Corporation (NSDC) through its training partners and in consultation with the Union Ministry
of External Affairs and the Union Skill Development Ministry. It also aims at boosting the
confidence of the Indian youth so that they dont feel like strangers when they land in a
country of their choice for vocation. For this purpose, NSDC will leverage various MoUs it
signed between 2011 and 2015 with different agencies of Germany, Canada, Australia,
Singapore, UK, US, European Union, France, Iran and China.
Source: http://www.business-standard.com/article/news-ani/pm-to-launch-pravasi-
kaushal-vikas-yojana-for-indians-seeking-overseas-employment-117010800581_1.html
Q.21) Consider the following statements about The Credit Guarantee Fund Scheme for
Micro and Small Enterprises (CGS)
1. It was launched to make available collateral-free credit to the micro and small
enterprise sector
2. Only the new enterprises are eligible to be covered under the scheme
3. The Ministry of Micro, Small and Medium Enterprises, and Small Industries
Development Bank of India (SIDBI), established a Trust named Credit Guarantee
Fund Trust for Micro and Small Enterprises (CGTMSE) to implement the Credit
Guarantee Fund Scheme for Micro and Small Enterprises
a) 1 and 2
b) 1 and 3
c) 2 and 3
d) All of the above
IASbabas Prelims 60 Day Plan Day 34 2017
(Economics)
Q.21) Solution (b)
The Credit Guarantee Fund Scheme for Micro and Small Enterprises (CGS) was launched by
the Government of India (GoI) to make available collateral-free credit to the micro and small
enterprise sector. Both the existing and the new enterprises are eligible to be covered under
the scheme. The Ministry of Micro, Small and Medium Enterprises, GoI and Small Industries
Development Bank of India (SIDBI), established a Trust named Credit Guarantee Fund Trust
for Micro and Small Enterprises (CGTMSE) to implement the Credit Guarantee Fund Scheme
for Micro and Small Enterprises.
Source: http://www.thehindu.com/todays-paper/tp-business/Cabinet-boosts-corpus-for-
small-enterprises/article17057511.ece
Agenda 2063 is both a Vision and an Action Plan. It is a call for action to all segments of
African society to work together to build a prosperous and united Africa based on shared
values and a common destiny.
Source: http://www.thehindu.com/todays-paper/tp-opinion/Reaching-out-to-
Africa/article17020813.ece
The objectives of the Scheme are to enhance the manufacturing competitiveness of MSMEs
through the application of various Lean Manufacturing (LM) techniques by;
Reducing waste;
Increasing productivity;
Introducing innovative practices for improving overall competitiveness;
Inculcating good management systems; and
Imbibing a culture of continuous improvement.
The Lean Manufacturing Competitiveness Scheme was started as a pilot phase in 2009 for
100 Mini Clusters (10 or so manufacturing MSME units) in 11th Five Year Plan. National
Productivity Council (NPC) was selected as National Monitoring and Implementing Unit
(NMIU) for facilitating implementation and monitoring of the Scheme.
The Scheme was up-scaled in September, 2013 considering the recommendations of the
evaluation report conducted by Quality Council of India (QCI).
Source: http://www.business-standard.com/article/government-press-
release/interaction-with-beneficiaries-of-up-scaled-revised-2013-lean-manufacturing-
competitiveness-117012001291_1.html
Mission 41K - A plan that would save Rs 41,000 crore over 10 years through an integrated
energy management system
IASbabas Prelims 60 Day Plan Day 34 2017
(Economics)
In this initiative, railways will electrify 24,000 km of rail tracks over the next five years by
doubling the annual rate of electrification from 2,000 km to 4,000 km in the next two years.
Mission Raftaar - Raise average speed of both passenger carrying trains and freight carrying
trains
Source: http://www.business-standard.com/article/economy-policy/suresh-prabhu-
unveils-mission-41k-energy-plan-for-railways-117011800230_1.html
Q.25) Consider the following statements about Transports Internationaux Routiers (TIR)
1. It is the only global customs transit system that provides easy and smooth
movement of goods across borders in sealed compartments or containers under
customs control from the customs office of departure to the customs office of
destination
2. All members of INSTC are signatories to TIR Convention 1975
3. The conventions were adopted under the auspices of the United Nations Economic
Commission for Europe (UNECE)
a) 1 and 2
b) 1 and 3
c) 2 and 3
d) All of the above
India is gearing up to sign the Transports Internationaux Routiers (TIR), or the customs
convention on the international transport of goods.
TIR is the only global customs transit system that provides easy and smooth movement of
goods across borders in sealed compartments or containers under customs control from the
customs office of departure to the customs office of destination.
Since all members of INSTC, except India and Oman, are already signatories to TIR
Convention 1975, custom issues and common documentation issues could be quickly
resolved if India signs the convention and aligns its system with it.
IASbabas Prelims 60 Day Plan Day 34 2017
(Economics)
The TIR system operates with certain parameters secure vehicles or container,
international guarantee chain, TIR carnet, reciprocal recognition of customs controls,
controlled access and TIR IT risk management tools.
These elements guarantee that goods travel across borders with minimum interference en
route and at the same time provide maximum safeguards to customs administration.
The TIR system has a globally accepted electronic control system for integrated transit
operations.
The conventions were adopted under the auspices of the United Nations Economic
Commission for Europe (UNECE)
Source: http://economictimes.indiatimes.com/news/economy/foreign-trade/india-gears-
up-to-ink-pact-for-global-customs-transit-system/articleshow/56578293.cms
IASbabas Prelims 60 Day Plan [Day 35] 2017
http://www.livemint.com/Industry/pcYwlItCLm7MZE88Pn1feO/BRABO-How-India-got-its-
first-Made-in-India-industrial-robo.html
a) Only 1
b) 1 and 2
c) Only 2
d) Only 3
Not all 3D printers use the same technology. There are several ways to print and all those
available are additive, differing mainly in the way layers are build to create the final object.
Some methods use melting or softening material to produce the layers. Selective Laser
Sintering (SLS) and Fused Deposition Modeling (FDM) are the most common technologies
www.iasbaba.com Page 1
IASbabas Prelims 60 Day Plan [Day 35] 2017
using this way of 3D printing. Another method is when we talk about curing a photo-reactive
resin with a UV laser or another similar power source one layer at a time. The most common
technology using this method is called Stereolithography (SLA).
To be more precise: since 2010, the American Society for Testing and Materials (ASTM)
group ASTM F42 Additive Manufacturing, developed a set of standards that classify the
Additive Manufacturing processes into 7 categories according to Standard Terminology for
Additive Manufacturing Technologies. These seven processes are:
Vat Photopolymerisation
Stereolithography (SLA)
Digital Light Processing (DLP)
Continuous Liquid Interface Production (CLIP)
Material Jetting
Binder Jetting
Material Extrusion
Sheet Lamination
Hint- http://www.thehindu.com/sci-tech/science/3d-printing-conquers-
glass/article18195807.ece
http://www.thehindu.com/sci-tech/technology/3d-printing-virtual-reality-used-to-bring-
dinosaur-to-life/article17318487.ece
a) The light from the Sun travels through Earths atmosphere and undergoes scattering
after it reaches the earths surface
www.iasbaba.com Page 2
IASbabas Prelims 60 Day Plan [Day 35] 2017
b) Light of shorter wavelengths, such as violet, blue, green and yellow undergoes
greater scattering than those of longer wavelengths, such as orange and red.
c) The scattering is related to the size and quantity of the scattering particles.
d) The scattering is more in evening than morning
The light from the Sun travels through Earths atmosphere it undergoes scattering before
it reaches us.
The extent of scattering is not uniform for all colours. Light of shorter wavelengths, such as
violet, blue, green and yellow undergoes greater scattering than those of longer
wavelengths, such as orange and red.
Because of the spherical geometry of the earth, the sunlight travels longer distance in the
thick of the earths atmosphere during sunset and sunrise when the Sun is at the horizons
than when the Sun is at the zenith (midday).
Thus, there is more probability for shorter wavelength light to get more scattered than for
the longer wavelength light. Hence, the Sun (and sunrise and sunset) appears reddish
orange during sunset and sunrise.
The scattering is also related to the size and quantity of the scattering particles. During the
night time the atmosphere is cool and the aerial particles and dust particles settle by
morning whereas by evening they get dispersed. Thus, the scattering is more by evening
than in the morning.
Lastly, since the earth is spinning from West to East, relatively we move towards the Sun
during mornings (eastward) and away from the Sun during evenings (westward). There is
one phenomenon, called, Doppler Effect, which adds to this differential.
1. The 2015 Nobel Prize in Chemistry was awarded for the design and synthesis of
molecular machines
2. The most complex molecular machines are proteins found within cells
3. The 2016 Nobel Prize in Chemistry was awarded to Jean-Pierre Sauvage, Sir J. Fraser
Stoddart and Bernard L. Feringa
a) 1 and 3
www.iasbaba.com Page 3
IASbabas Prelims 60 Day Plan [Day 35] 2017
b) 2 and 3
c) 1 and 2
d) 1, 2 and 3
The Nobel Prize in Chemistry 2016 was awarded jointly to Jean-Pierre Sauvage, Sir J. Fraser
Stoddart and Bernard L. Feringa "for the design and synthesis of molecular machines".
Molecular machines can be divided into two broad categories; synthetic and biological.
The most complex molecular machines are proteins found within cells. These include motor
proteins, such as myosin, which is responsible for muscle contraction, kinesin, which moves
cargo inside cells away from the nucleus along microtubules, and dynein, which produces
the axonemal beating of motile cilia and flagella. These proteins and their nanoscale
dynamics are far more complex than any molecular machines that have yet been artificially
constructed.
The Nobel Prize in Chemistry 2015 was awarded jointly to Tomas Lindahl, Paul Modrich and
Aziz Sancar "for mechanistic studies of DNA repair".
a) 1 and 2
b) 1 and 3
c) 2 and 3
d) 1, 2 and 3
A neutron star is the collapsed core of a large (1029 solar masses) star. Neutron stars are
the smallest and densest stars known to exist. Though neutron stars typically have a radius
on the order of 10 km, they can have masses of about twice that of the Sun. They result
www.iasbaba.com Page 4
IASbabas Prelims 60 Day Plan [Day 35] 2017
from the supernova explosion of a massive star, combined with gravitational collapse that
compresses the core past the white dwarf star density to that of atomic nuclei.
Pulsars aren't really stars or at least they aren't "living" stars. Pulsars belong to a family of
objects called neutron stars that form when a star more massive than the sun runs out of
fuel in its core and collapses in on itself. This stellar death typically creates a massive
explosion called a supernova. The neutron star is the dense nugget of material left over
after this explosive death.
All Pulsars are Neutron stars, but not all Neutron stars are Pulsars.
Magnetars are a type of neutron star, like pulsars, but with a magnetic field on serious
steroids.
It is believed that magnetars are a type of neutron star that were made during a Supernova
explosion, similar to that of a pulsar. They are one of the most dense objects in the universe.
It is theorized that the dynamo mechanism may be the reason to their formation. Basically,
if the spin, temperature and the magnetic field of a neutron star are within the right ranges
it can convert the heat and rotational energy into very strong magnetic energy.
Although neutron stars and a magnetar are similar in formation they hold very different
characteristics which sets them apart from each other. For instance magnetars rotate at a
very slower rate, usually once every 8 to 10 seconds as opposed to one or more rotations a
second for neutron stars. Another difference between a magnetar and a neutron star is that
a magnetar emits a steady glow of x-rays with more radiant power than could be supplied
by the rotation of a neutron star. The magnetic fields made by a magnetar are about 1,000
trillion that of the Earths magnetic field and can reach surface temperatures of 18 million
degrees Fahrenheit.
Hint- http://www.thehindu.com/sci-tech/What-are-magnetars/article14640121.ece
http://www.thehindu.com/society/the-titanic-lesson/article17482941.ece
http://www.thehindu.com/sci-tech/science/nasa-observatory-discovers-mysterious-cosmic-
explosion/article17757869.ece
http://www.thehindu.com/sci-tech/science/eclipses-of-binary-star-shed-light-on-orbiting-
exoplanet/article17757774.ece
Q.6) The most common chemicals used for cloud seeding include
1. Dry Ice
2. Silver Iodide
www.iasbaba.com Page 5
IASbabas Prelims 60 Day Plan [Day 35] 2017
3. Liquid Propane
a) Only 2
b) 1 and 2
c) 2 and 3
d) 1, 2 and 3
The most common chemicals used for cloud seeding include silver iodide and dry ice
(frozen carbon dioxide). The expansion of liquid propane into a gas is being used on a
smaller scale. The use of hygroscopic materials, such as salt, is increasing in popularity
because of some promising research results.
Seeding of clouds requires that they contain supercooled liquid waterthat is, liquid water
colder than zero degrees Celsius. Introduction of a substance such as silver iodide, which has
a crystalline structure similar to that of ice, will induce freezing (heterogeneous nucleation).
Dry ice or propane expansion cools the air to such an extent that ice crystals can nucleate
spontaneously from the vapor phase. Unlike seeding with silver iodide, this spontaneous
nucleation does not require any existing droplets or particles because it produces extremely
high vapor supersaturations near the seeding substance. However, the existing droplets are
needed for the ice crystals to grow into large enough particles to precipitate out.
Hint- http://www.thehindu.com/news/national/Clouds-over-Maharashtra-will-have-a-
silver-iodide-lining/article17309167.ece
http://www.thehindu.com/news/national/tamil-nadu/the-cloud-seeding-
myth/article17529580.ece
Q.7) Chlorophyll is vital for photosynthesis, which allows plants to absorb energy
from light. It contains
1. Nitrogen
2. Carbon
3. Potassium
4. Magnesium
5. Chlorine
6. Oxygen
www.iasbaba.com Page 6
IASbabas Prelims 60 Day Plan [Day 35] 2017
a) 1, 2, 3, 5 and 6
b) 1, 2, 4 and 6
c) 2, 3, 4, and 6
d) 1, 2, 3, 4, 5 and 6
Many important natural substances are chelates. In chelates a central metal ion is bonded
to a large organic molecule, a molecule composed of carbon, hydrogen, and other elements
such as oxygen and nitrogen. One such chelate is chlorophyll, the green pigment of plants.
In chlorophyll the central ion is magnesium, and the large organic molecule is a porphyrin.
The porphyrin contains four nitrogen atoms that form bonds to magnesium in a square
planar arrangement. There are several forms of chlorophyll. The structure of one form,
chlorophyll a, is shown.
As you can see from the molecular structure, the "chloro" in chlorophyll does not mean
that it contains the element chlorine. The chloro portion of the word is from the
Greek chloros, which means yellowish green. The name of the element chlorine comes from
the same source. Chlorine is a yellowish green gas.
Q.8) Recently scientists have successfully tweaked the process of photosynthesis to make
it more efficient and increase plant productivity by raising the level of three proteins
involved in the process. The tested plant was a
a) Tea plant
b) Coffee plant
www.iasbaba.com Page 7
IASbabas Prelims 60 Day Plan [Day 35] 2017
c) Tobacco plant
d) Cotton plant
http://www.thehindu.com/sci-tech/science/Tweaking-photosynthesis-for-a-better-crop-
yield/article17004885.ece
Q.9) Newton Fund Research Programme worth up to USD 80 million was recently
announced to jointly address global societal challenges. It is a joint initiative of
http://indianexpress.com/article/india/india-news-india/india-uk-join-forces-for-usd-80m-
newton-fund-research-programme-4364648/
Q.10) Lithium-ion batteries are popular because they have a number of important
advantages over competing technologies. Consider the statements regarding Li-ion
batteries
a) 1 and 2
b) 2 and 3
c) 1 and 3
d) 1, 2 and 3
www.iasbaba.com Page 8
IASbabas Prelims 60 Day Plan [Day 35] 2017
Lithium-ion batteries are popular because they have a number of important advantages
over competing technologies:
They're generally much lighter than other types of rechargeable batteries of the
same size.
The electrodes of a lithium-ion battery are made of lightweight lithium and carbon.
Lithium is also a highly reactive element, meaning that a lot of energy can be stored
in its atomic bonds. This translates into a very high energy density for lithium-ion
batteries. Here is a way to get a perspective on the energy density. A typical lithium-
ion battery can store 150 watt-hours of electricity in 1 kilogram of battery. A NiMH
(nickel-metal hydride) battery pack can store perhaps 100 watt-hours per kilogram,
although 60 to 70 watt-hours might be more typical.
They hold their charge. A lithium-ion battery pack loses only about 5 percent of its
charge per month, compared to a 20 percent loss per month for NiMH batteries.
They have no memory effect, which means that you do not have to completely
discharge them before recharging, as with some other battery chemistries.
Lithium-ion batteries can handle hundreds of charge/discharge cycles.
They start degrading as soon as they leave the factory. They will only last two or
three years from the date of manufacture whether you use them or not.
They are extremely sensitive to high temperatures. Heat causes lithium-ion battery
packs to degrade much faster than they normally would.
If you completely discharge a lithium-ion battery, it is ruined.
A lithium-ion battery pack must have an on-board computer to manage the battery.
This makes them even more expensive than they already are.
There is a small chance that, if a lithium-ion battery pack fails, it will burst into flame.
Hint- http://www.thehindu.com/sci-tech/technology/an-alternative-to-lithium-ion-
batteries/article18278152.ece
http://www.thehindu.com/sci-tech/science/A-novel-electrode-for-lithium-
batteries/article17005579.ece
Q.11) Consider the following statements regarding Liquefied Natural Gas (LNG)
www.iasbaba.com Page 9
IASbabas Prelims 60 Day Plan [Day 35] 2017
a) 1 and 2
b) Only 2
c) 1 and 3
d) 1, 2 and 3
Liquefied natural gas (LNG) is natural gas (predominantly methane, CH4, with some mixture
of ethane C2H6) that has been converted to liquid form for ease of storage or transport. It
takes up about 1/600th the volume of natural gas in the gaseous state. It
is odorless, colorless, non-toxic and non-corrosive
Hazards include flammability after vaporization into a gaseous state, freezing and asphyxia.
Liquefied natural gas, or LNG, is natural gas in its liquid form. When natural gas is cooled to
minus 259 degrees Fahrenheit (-161 degrees Celsius), it becomes a clear, colorless, odorless
liquid. LNG is neither corrosive nor toxic.
LNG achieves a higher reduction in volume than compressed natural gas (CNG) so that the
(volumetric) energy density of LNG is 2.4 times greater than that of CNG or 60 percent that
of diesel fuel.
Hint- http://www.thehindu.com/news/cities/Thiruvananthapuram/First-LNG-driven-bus-
rolls-out/article16440639.ece
http://breakingenergy.com/2014/12/22/how-dangerous-is-lng/
1. Hepatitis A. Blood
2. Alzheimer's B. Liver
3. Cystic Fibrosis C. Lung
4. Ischema D. Brain
www.iasbaba.com Page 10
IASbabas Prelims 60 Day Plan [Day 35] 2017
Ischemia is the medical term for what happens when your heart muscle doesn't get enough
oxygen. Ischemia usually happens because of a shortage of blood and oxygen to
the heart muscle.
Cystic fibrosis (CF) is the most common, fatal genetic disease affecting young Canadians.
Cystic fibrosis mainly affects people's lungs and digestion.
a) 1 and 2
b) 1,2 and 3
c) 2 and 3
d) All of the above
Japanese encephalitis virus (JEV) is a flavivirus related to dengue, yellow fever and West Nile
viruses, and is spread by mosquitoes.
www.iasbaba.com Page 11
IASbabas Prelims 60 Day Plan [Day 35] 2017
Domestic pigs and wild birds are reservoirs of the virus; transmission to humans may cause
severe symptoms. Amongst the most important vectors of this disease are the
mosquitoes Culex tritaeniorhynchus and Culex vishnui. This disease is most prevalent
in Southeast Asia, South Asia and East Asia.
a) 1 only
b) 2 only
c) Both
d) None
The sequence VIBGYOR is in the order of decreasing frequency or increasing wave length.
The light which has lowest frequency (Red) will bend slightly whereas light which has more
frequency will bend more (Violet)
1. Diphtheria
2. Tuberculosis
3. Measles
a) 1 and 3
b) 1 and 2
c) 2 and 3
d) 1, 2 and 3
www.iasbaba.com Page 12
IASbabas Prelims 60 Day Plan [Day 35] 2017
Airborne diseases include any that are caused and transmitted through the air. Some are of
great medical importance. The pathogens transmitted may be any kind of microbe, and they
may be spread in aerosols, dust or liquids. The aerosols might be generated from sources
of infection such as the bodily secretions of an infected animal or person, or biological
wastes such as accumulate in lofts, caves, garbage and the like. Such infected aerosols may
stay suspended in air currents long enough to travel for considerable distances, though the
rate of infection decreases sharply with the distance between the source and the organism
infected.
Airborne pathogens or allergens often cause inflammation in the nose, throat, sinuses and
the lungs. This is caused by the inhalation of these pathogens that affect a person's
respiratory system or even the rest of the body. Sinus congestion, coughing and sore throats
are examples of inflammation of the upper respiratory air way due to these airborne agents.
Air pollution plays a significant role in airborne diseases which is linked
to asthma. Pollutants are said to influence lung function by increasing air way inflammation.
Many common infections can spread by airborne transmission at least in some cases,
including: Anthrax, Chickenpox, Influenza, Measles, Smallpox, Cryptococcosis,
and Tuberculosis.
a) Only 1
b) Only 2
c) Both 1 and 2
d) Neither 1 nor 2
www.iasbaba.com Page 13
IASbabas Prelims 60 Day Plan [Day 35] 2017
The Ministry of Micro, Small and Medium Enterprises (MSME) is implementing the National
Manufacturing Competitiveness Programme (NMCP) to develop global competitiveness
among Indian MSMEs.
Source: http://pib.nic.in/newsite/PrintRelease.aspx?relid=157550
a) Only 1
b) Only 2
c) Both 1 and 2
d) Neither 1 nor 2
ISDS
www.iasbaba.com Page 14
IASbabas Prelims 60 Day Plan [Day 35] 2017
ISDS has been criticized because the United States has never lost any of its ISDS
cases, and that the system is biased to favor American companies and American
trade over other Western countries, and Western countries over the rest of the
world.
India has not signed/ratified ICSID
Source: http://www.thehindu.com/business/India-rejects-attempts-by-EU-Canada-for-
global-investment-agreement/article17083034.ece
a) 1 and 2
b) Only 2
c) 1 and 3
d) 2 and 3
They can accept deposits, offer interest on deposits and can make payments.
Q.19) Consider the following statements about Dam Rehabilitation and Improvement
Project (DRIP)
1. The Implementation Agencies for DRIP are Water Resources Departments and State
Electricity Boards in the participating States and Central Water Commission at
Central Level
www.iasbaba.com Page 15
IASbabas Prelims 60 Day Plan [Day 35] 2017
a) Only 1
b) Only 2
c) Both 1 and 2
d) Neither 1 nor 2
The Dam Rehabilitation and Improvement Project (DRIP) has been taken up with loan
assistance of the World Bank for rehabilitation and improvement of about 250 dams initially
in seven States (namely Jharkhand, Karnataka, Kerala, Madhya Pradesh, Odisha, Tamil Nadu,
and Uttarakhand) and institutional strengthening and project management in Central Water
Commission (CWC) and other Implementing Agencies; the actual number of dams under
DRIP may vary owing to the addition / deletion of dams during implementation. DRIP will be
implemented over a period of six-years starting from 18th April, 2012.
The Implementation Agencies for DRIP are Water Resources Departments and State
Electricity Boards in the participating States and Central Water Commission at Central Level.
State Implementing Agencies will have responsibility for implementation of works of dams
under their charge and responsibility for coordination and management of such works
within a State rests with the concerned State Project Management Unit (SPMU). Overall
project oversight and coordination is to be carried out by Central Project Management Unit
(CPMU) headed by the Project Director with assistance of an Engineering and Management
Consultant.
Source: http://pib.nic.in/newsite/PrintRelease.aspx?relid=157721
www.iasbaba.com Page 16
IASbabas Prelims 60 Day Plan [Day 35] 2017
a) 1 and 2
b) 2 and 3
c) 1 and 3
d) All of the above
Kambala Karnataka
Kambala is an annual buffalo race which is a tradition in the Karnataka's Dakshina Kannada
and Udupi districts' farming community. This area is called Tulunadu (Land of Tulu
Language).
A pair of buffaloes are tied to the plough and one person anchors it, beating the buffaloes
with a stick to run faster. There are two parallel muddy tracks, on which two competing
pairs of buffaloes run. Fastest team wins.
Source: http://timesofindia.indiatimes.com/india/all-you-need-to-know-about-
kambala/listshow/56774237.cms
http://www.ndtv.com/india-news/what-is-jallikattu-1650547
Inspired by the hammer-wielding character, Thor, from Norse mythology and the Marvel
comics universe, the eponymous experiment aims to investigate electrical activity from
thunderstorms. Between 10 and 100 kilometres in the atmosphere, the interaction between
www.iasbaba.com Page 17
IASbabas Prelims 60 Day Plan [Day 35] 2017
charged particles produces a variety of dazzling electric phenomena from blue jets to red
sprites.
The Thor experiment will look at them with a thundercloud imaging system from the
vantage point of the International Space Station. The key aims of the study include
understanding how these discharges influence water vapour levels, cloud formation, and
eventually changes in climate.
a) Only 3
b) 1 and 2
c) 2 and 3
d) 1 and 3
Q.23) Which of the following statements about vote-on-account (VOA) is/are correct?
a) A VOA in the strict sense deals only with the expenditure side of the government's
budget, whereas an interim budget has to include both expenditure and receipts
b) A VOA cannot be for a period longer than six
c) No changes are made to tax and duty structures and no new schemes are announced
in vote-on-account
d) All of the above
www.iasbaba.com Page 18
IASbabas Prelims 60 Day Plan [Day 35] 2017
What is Vote-on-account?
While the words vote-on-account and interim budget are often interchangeably
used, a vote-on-account in the strict sense deals only with the expenditure side of
the government's budget, whereas an interim budget has to include both
expenditure and receipts.
Generally, a vote-on-account is for two or three months, usually till the time it is
replaced by a regular budget.
It cannot be for a period longer than six months as the Constitution stipulates that
the gap between two Parliament sittings cannot be more than six months.
A regular full budget is a complete statement on the financial position of the
government for a full year based on expenditures during the period and proposals
for financing them. Thus, it gives details of how money is to be spent and how it will
be raised by the government.
Why ever have a vote-on-account and not a full-fledged budget?
www.iasbaba.com Page 19
IASbabas Prelims 60 Day Plan [Day 35] 2017
Interim budgets have also been used by governments taking office just before the
financial year begins to get Parliamentary approval for immediate spending, giving
them time to work out a more though-out budget later in the year.
By convention, what are the restrictions on a vote-on-account?
Q.24) Consider the following statements about Varishtha Pension Bima Yojana 2017
a) Only 1
b) Only 2
c) Both 1 and 2
d) Neither 1 nor 2
The scheme will be implemented through Life Insurance Corporation of India (LIC) during
the current financial year to provide social security during old age and protect elderly
persons aged 60 years and above against a future fall in their interest income due to
uncertain market conditions.
The scheme is meant to provide an assured pension based on a guaranteed rate of return of
8% per annum for ten years, with an option to opt for pension on a monthly/quarterly/half-
www.iasbaba.com Page 20
IASbabas Prelims 60 Day Plan [Day 35] 2017
yearly or annual basis. The difference between the return generated by LIC and the assured
return of 8% per annum would be borne by Government of India as subsidy on an annual
basis. VPBY-2017 is proposed to be open for subscription for a period of one year from the
date of launch.
Source: http://www.thehindu.com/business/Economy/Govt-launches-Varishtha-Pension-
Bima-Yojana/article17089241.ece
Q.25) Consider the following statements about Rubber Soil Information System (RubSIS)
a) 1 and 2
b) 2 and 3
c) 1 and 3
d) All of the above
Developed by: Rubber Research Institute of India (RRII) under the Rubber Board in
collaboration with few agencies
Source: http://pib.nic.in/newsite/PrintRelease.aspx?relid=157609
www.iasbaba.com Page 21
IASbabas Prelims 60 Day Plan Day 36 2017
(Environment)
Q.1) Which of the following processes contributes maximum to the power generation in
India?
a) Hydel power
b) Thermal power
c) Nuclear power
d) Solar and wind power
The all India installed power generation capacity by November 2015 is 2,82,023.78 MW
comprising 1,96,204.44 MW thermal 42,623.42 MW hydro, 5.780 MW nuclear and
37,415.53 MW R.E.S.
Q.2) The government of India had launched a very ambitious initiative of Ultra Mega
Power Projects. Which of the following statements are correct about UMPPs?
1. They are large hydel power projects to tap the full potential of unused streams.
2. Each UMPP will have the capacity of generating 10,000 MW of power.
3. Till now 10 UMPPs have already been sanctioned by the Government of India.
Select the code from below:
a) 1 and 2
b) 2 and 3
c) 1 and 3
d) None of the above
The Government of India had launched an initiative for the development of coal-based Ultra
Mega Power Projects (UMPPs), each with a capacity of 4,000 MW. The objective of the
initiative is to ensure cheaper tariffs utilizing economies of scale, catering to the need of a
number of states and to mitigate the risk relating to tie up of land fuel, water and other
statutory clearances, etc. The projects are being awarded to the successful developers on
the basis of
tariff based competitive bidding route employing super critical technology; to tie-up for
necessary inputs and clearances such as provision of site, fuel through captive mining
blocks, water and in-principle environment and forest clearances, project-specific shell
companies (SPVs) are set up as wholly owned subsidiaries of the Power Finance Corporation
Ltd. (PFC) - the nodal agency for these projects. These SPVs, along with the various
clearances, etc. are subsequently transferred to the successful developer. Four UMPPs
namely Sasan in Madhya Pradesh, Mundra in Gujarat, Krishnapatnam in Andhra Pradesh
IASbabas Prelims 60 Day Plan Day 36 2017
(Environment)
and Tilaiya in Jharkhand have already been awarded to successful bidders and are at
different stages of development.
Q.3) Which of the following statements are correct about National Domestic Efficient
Lighting Program?
a) It urged people to use CFLs in place of tube lights and filament bulbs.
b) It urged people to use only 5 star efficiency rating appliances.
c) It promoted rural electrification.
d) None of the above
Domestic Efficient Lighting Programme (DELP) was launched in January 2015 urging the
people to use LED bulbs in place of incandescent bulbs, tube lights and CFL bulbs as they are
more efficient, long lasting and economical in their life cycle duration.
Q.4) The rate of biomass production in an ecosystem is called productivity. Which of the
following statements are correct about it?
1. Net primary productivity (NPP) is less than Gross primary productivity (GPP).
2. Net Primary Productivity (NPP) is the available biomass for the consumption of
heterotrophs.
Select the code from following:
a) 1 only
b) 2 only
c) Both 1 and 2
d) Neither 1 nor 2
Primary production is defined as the amount of biomass or organic matter produced per
unit area over a time period by plants during photosynthesis. It is expressed in terms of
weight (g 2) or energy (kcal m2). The rate of biomass production is called productivity.
It is expressed in terms of g2 yr 1 or (kcal m2) yr1 to compare the productivity of
different ecosystems. It can be divided into gross primary productivity (GPP) and net
primary productivity (NPP). Gross primary productivity of an ecosystem is the rate of
production of organic matter during photosynthesis. A considerable amount of GPP is
utilised by plants
in respiration. Gross primary productivity minus respiration losses (R), is the net primary
productivity (NPP).
IASbabas Prelims 60 Day Plan Day 36 2017
(Environment)
GPP R = NPP
Net primary productivity is the available biomass for the consumption to heterotrophs
(herbiviores and decomposers). Secondary productivity is defined as the rate of formation
of new organic matter by consumers.
Q.5) Which of the following statements correctly explains the process of catabolism?
Q.6) Which of the following factors play an important role in determining the rate of
decomposition of detritus material?
1. Chemical composition of Detritus
2. Temperature
3. Soil moisture
Select the code from below:
a) 1 only
b) 2 and 3
c) 1 and 2
d) All of the above
Q.7) Deep sea Hydrothermal vents were recently discovered by the scientist. Which of
the following statements are correct about Hydrothermal ecosystems?
1. The organisms are not dependent on sunlight for food production.
2. Bacteria create energy using hydrogen sulphide through the process of
chemosynthesis.
3. Only micro organisms can survive in this environment.
Deep-sea hydrothermal vents form as a result of volcanic activity on the ocean floor. Water
seeps through cracks in the Earth's crust, dissolving metals and minerals as it becomes
super-heated from nearby magma.
This water - which can reach temperatures of 400C - eventually rises back through the
ocean floor, erupting as a geyser from a hydrothermal vent. The dissolved minerals and
metals precipitate on contact with the cold sea water, forming a chimney around the vent.
When scientists first discovered these vents in the 1970s, they were amazed to find thriving
communities of shrimp, crabs, giant tubeworms, clams, slugs, anemones, and fish. These
rare geological features turned out to be oases on the otherwise sparsely inhabited ocean
floor, with a biomass equivalent to that of a rainforest.
Instead of sunlight, vent life relies on hydrogen sulfide - more commonly known as rotten
egg gas and toxic to most land-based life.
In a process called chemosynthesis, specialized bacteria create energy from the hydrogen
sulfide present in the mineral-rich water pouring out of the vents. These bacteria form the
bottom level of the food chain in these ecosystems, upon which all other vent animals are
dependent.
Q.8) Based on the source of their food, organisms occupy a special place in the food chain.
This position is called:
IASbabas Prelims 60 Day Plan Day 36 2017
(Environment)
a) Niche
b) Trophic level
c) Horizon
d) Food web
a) 1 only
b) 2 only
c) Both 1 and 2
d) Neither 1 nor 2
In most ecosystems, all the pyramids, of number, of energy and biomass are upright, i.e.,
producers are more in number and biomass than the herbivores, and herbivores are more in
number and biomass than the carnivores. Also energy at a lower trophic level is always
more than at a higher level.
But there are exceptions to this trend. In forests, the number of tress is less than the
number of organisms dependent on each tree(insects, birds etc.)
Even in pyramid of biomass, there are exceptions like sea ecosystem. Very large organisms
are dependent on phytoplanktons.
Pyramid of energy is always upright, can never be inverted, because when energy flows
from a particular trophic level to the next trophic level, some energy is always lost as heat at
IASbabas Prelims 60 Day Plan Day 36 2017
(Environment)
each step. Each bar in the energy pyramid indicates the amount of energy present at each
trophic level in a given time or annually per unit area.
Q.10) Which of the following statements are correct about the process of secondary
succession?
1. Secondary succession is much slower than the primary succession.
2. Secondary succession begins in areas where natural vegetation has been destroyed.
Ecological succession, the process by which the structure of a biological community evolves
over time. Two different types of successionprimary and secondaryhave been
distinguished. Primary succession occurs in essentially lifeless areasregions in which the
soil is incapable of sustaining life as a result of such factors as lava flows, newly formed sand
dunes, or rocks left from a retreating glacier. Secondary succession occurs in areas where a
community that previously existed has been removed; it is typified by smaller-scale
disturbances that do not eliminate all life and nutrients from the environment.
Secondary succession begins in areas where natural biotic communities have been
destroyed such as in abandoned farm lands, burned or cut forests, lands that have been
flooded. Since some soil or sediment is present, succession is faster than primary
succession.
Description of ecological succession usually focuses on changes in vegetation. However,
these vegetational changes in turn affect food and shelter for various types of animals. Thus,
as succession proceeds, the numbers and types of animals and decomposers also change.
At any time during primary or secondary succession, natural or human induced disturbances
(fire, deforestation, etc.), can convert a particular seral stage of succession to an earlier
stage. Also such disturbances create new conditions that encourage some species and
discourage or eliminate other species.
Q.12) The benefits that people obtain from ecosystems are termed as ecosystem services.
Which of the following are the types of Ecosystem services?
1. Provisioning services
2. Regulating services
3. Habitat services
4. Cultural services
Ecosystem services are the direct and indirect contributions of ecosystems to human well-
being.
IASbabas Prelims 60 Day Plan Day 36 2017
(Environment)
Ecosystem services can be categorized in four main types:
Provisioning services are the products obtained from ecosystems such as food, fresh water,
wood, fiber, genetic resources and medicines.
Regulating services are defined as the benefits obtained from the regulation of ecosystem
processes such as climate regulation, natural hazard regulation, water purification and
waste management, pollination or pest control.
Habitat services highlight the importance of ecosystems to provide habitat for migratory
species and to maintain the viability of gene-pools.
Cultural services include non-material benefits that people obtain from ecosystems such as
spiritual enrichment, intellectual development, recreation and aesthetic values.
Q.13) Lichens are one of the most important pioneer species. Which of the following
statements are correct about Lichens?
1. Lichens are not single organisms, but they are formed by symbiotic relationship
between algae and Fungi.
2. Lichens are found only in high alpine areas where moisture is scarce.
Select the code from following:
a) 1 only
b) 2 only
c) Both 1 and 2
d) Neither 1 nor 2
The non-fungal partner contains chlorophyll and is called the photobiont. The fungal
partner may be referred to as the mycobiont. While most lichen partnerships consist of one
mycobiont and one photobiont, that's not universal for there are lichens with more than
one photobiont partner. When looked at microscopically, the fungal partner is seen to be
composed of filamentous cells and each such filament is called a hypha. These hyphae grow
by extension and may branch but keep a constant diameter. Amongst the photobionts there
are those that are also filamentous in structure while others are composed of chains or
clusters of more-or-less globose cells.
IASbabas Prelims 60 Day Plan Day 36 2017
(Environment)
Given that they contain chlorophyll, algae and cyanobacteria can manufacture
carbohydrates with the help of light via the process of photosynthesis. By contrast, fungi do
not make their own carbohydrates. Every fungus needs existing organic matter from which
to obtain carbon. In a lichen some of the carbohydrate produced by the photobiont is of
course used by the photobiont but some is 'harvested' by the mycobiont.
Lichens occur from sea level to high alpine elevations, in many environmental conditions,
and can grow on almost any surface. Lichens are abundant growing on bark, leaves, mosses,
on other lichens, and hanging from branches "living on thin air" (epiphytes) in rain forests
and in temperate woodland. They grow on rock, walls, gravestones, roofs, exposed soil
surfaces, and in the soil as part of a biological soil crust. Different kinds of lichens have
adapted to survive in some of the most extreme environments on Earth: arctic tundra, hot
dry deserts, rocky coasts, and toxic slag heaps. They can even live inside solid rock, growing
between the grains.
Q.14) Bhopal gas tragedy has been one of the most deadliest industrial hazards in the
history of India. Which of the following gas was released causing the disaster?
a) Potassium cyanide
b) Potassium isocyanate
c) Methyl isocyanate
d) Phosgene
The Bhopal disaster or Bhopal gas tragedy was an industrial accident. It happened at a
Union Carbide subsidiary pesticide plant in the city of Bhopal, India. On 3 December 1984,
the plant released 42 tonne of toxic methyl isocyanate (MIC) gas, exposing more than
500,000 people to toxic gases.
A mixture of poisonous gases flooded the city, causing great panic as people woke up with a
burning sensation in their lungs. Thousands died immediately from the effects of the gas.
Many were trampled in the panic that followed. The first official immediate death toll was
2,259. Another estimate is that 8,000 died within two weeks, that an additional 8,000 have
since died from gas-related diseases.
Biochemical oxygen demand (BOD, also called biological oxygen demand) is the amount of
dissolved oxygen needed (i.e., demanded) by aerobic biological organisms to break down
organic material present in a given water sample at certain temperature over a specific time
period. The BOD value is most commonly expressed in milligrams of oxygen consumed per
litre of sample during 5 days of incubation at 20 C and is often used as a surrogate of the
degree of organic pollution of water.
With increase in organic waste the requirement of oxygen by the microbes to decompose it
will also increase.
Q.16) Indias strategic crude oil storages are locate at which of the following
1. Visakhapatnam
2. Mangalore
3. Padur
a) 1 and 2
b) 2 and 3
c) 1 and 3
d) All of the above
Strategic petroleum reserves have become indispensable to safeguard the economy and to
help maintain national security in the event of an energy crisis. These crude oil inventories
(or stockpiles) can be held by the government of a particular country, as well as by private
industry.
Strategic crude oil storages at three locations namely, Visakhapatnam, Mangalore and Padur
(near Udupi)
Source: http://economictimes.indiatimes.com/news/politics-and-nation/india-uae-
strategic-oil-reserves-deal-10-things-to-know/articleshow/56796338.cms
IASbabas Prelims 60 Day Plan Day 36 2017
(Environment)
Q.17) Consider the following statements about Bunkar Mitra
a) Only 1
b) Only 2
c) Both 1 and 2
d) Neither 1 nor 2
Ministry of Textilles.
The helpline provides a single point of contact to handloom weavers across the country for
addressing queries and providing guidance.
Guidance for:
Source: http://pib.nic.in/newsite/PrintRelease.aspx?relid=156130
NDMA was established through the Disaster Management Act enacted by the Government
of India in December 2005. The Prime Minister is the ex-officio chairperson of NDMA. The
agency is responsible for framing policies, laying down guidelines and best-practices and
coordinating with the State Disaster Management Authorities (SDMAs) to ensure a holistic
and distributed approach to disaster management.
National Disaster Management Authority (NDMA), headed by the Prime Minister, and State
Disaster Management Authorities (SDMAs) headed by respective Chief Ministers.
Q.19) Other than Venezuela, which one among the following from South America is a
member of OPEC?
a) Argentina
b) Brazil
c) Ecuador
d) Bolivia
The Organization of the Petroleum Exporting Countries (OPEC) was founded in Baghdad,
Iraq, with the signing of an agreement in September 1960 by five countries namely Islamic
Republic of Iran, Iraq, Kuwait, Saudi Arabia and Venezuela. They were to become the
Founder Members of the Organization.
IASbabas Prelims 60 Day Plan Day 36 2017
(Environment)
These countries were later joined by Qatar (1961), Indonesia (1962), Libya (1962), the
United Arab Emirates (1967), Algeria (1969), Nigeria (1971), Ecuador (1973), Gabon (1975)
and Angola (2007).
Ecuador suspended its membership in December 1992, but rejoined OPEC in October 2007.
Indonesia suspended its membership in January 2009, reactivated it again in January 2016,
but decided to suspend its membership once more at the 171st Meeting of the OPEC
Conference on 30 November 2016. Gabon terminated its membership in January 1995.
However, it rejoined the Organization in July 2016.
This means that, currently, the Organization has a total of 13 Member Countries.
Q.20) Mahamastakabhisheka, a great religious event, is associated with and done for
whom of the following?
a) Bahubali
b) Buddha
c) Mahavir
d) Nataraja
The Mahamastabhisheka (or Mahamasthak Abhishek) is an important Jain festival held once
every twelve years in the town of Shravanabelagola in Karnataka state, India. The festival is
held in veneration of an immense 18 meter high statue of the Bhagwan (or Saint)
Gomateshwara Bahubali. The anointing last took place in February 2006, and the next
ceremony will occur in 2018.
Source: http://www.thehindu.com/news/national/karnataka/Dates-for-
Mahamastabhisheka-at-Shravanabelagola-announced/article16444430.ece
a) Biotechnology
b) Space technology
c) Missile technology
d) Communication technology
IASbabas Prelims 60 Day Plan Day 36 2017
(Environment)
Q.21) Solution (d)
a) World Bank
b) International Labour Organisation
c) World Economic Forum
d) United Nations Development Programme
Source: http://www.livemint.com/Politics/qvL3mTzzJYkuqb5DPN0TwI/Unemployment-
in-India-to-increase-marginally-in-201718-UN.html
a) Only 1
b) Only 2
c) Both 1 and 2
d) Neither 1 nor 2
It is the closest star system to the Solar System at a distance of 4.37 light-years
IASbabas Prelims 60 Day Plan Day 36 2017
(Environment)
Centauri B together with a small and faint red dwarf, Alpha Centauri C (also named Proxima
Centauri.
Source: http://indianexpress.com/article/technology/science/giant-telescope-in-chile-to-
seek-habitable-planets-in-alpha-centauri-4469652/
a) 1 and 2
b) 2 and 3
c) 1 and 3
d) All of the above
Source: http://indianexpress.com/article/technology/tech-news-technology/microsoft-
offers-to-collaborate-with-andhra-pradesh-on-e-governance-cyber-security-4479995/
IASbabas Prelims 60 Day Plan Day 36 2017
(Environment)
Q.25) The 713th death anniversary of Sufi saint Nizamuddin Auliya was recently observed
by reciting qawwals in the Nizamuddin dargah. Consider the following statements
a) Only 1
b) Only 2
c) Both 1 and 2
d) Neither 1 nor 2
The Chisti order is a Sunni Sufi order within the Sufi tradition of Islam. The order believed in
drawing close to God through renunciation of the world and service to humanity.
The Chisti order is the first of the four main Sufi orders Chisti, Qadiriyya, Suhrawardiyya
and Naqshbandi order.
Source: http://www.hindustantimes.com/art-and-culture/meet-the-nizami-bandhu-
qawwali-s-rockstars/story-NlHuHmoLvv2omhiPRbOqHP.html